You are on page 1of 356

Feb 01 - 1

One of the biggest questions facing the art world today is the dilemma over the repatriation of
cultural treasures. Although the subject has not been widely noted by the general public, in recent
decades museums and art dealers have repeatedly faced off against the representatives of nations
and ethnic groups whose cultural legacies have been robbed by the rapacious collecting of these so-
called art experts. Advocates of repatriation have argued that cultural treasures should be returned
to their nations of origin, both because of basic fairness and because the artwork and cultural
artifacts in question are best understood within their local context.

Several prominent museums, most notably the British Museum in London and the Louvre in Paris,
have defended themselves on the grounds that they can better protect and preserve these cultural
treasures than can the developing nations and impoverished ethnic groups that frequently seek their
return. They further argue that more people can see the treasures if they are proudly displayed in a
major museum, as opposed to some poorly funded national museum in a backwater country;
evidently, the quantity of viewers is more important than the relevance of the art and artifacts to the
viewer.

The arguments of the museum curators fall apart in an instance such as the Elgin Marbles. These
majestic marble sculptures, which once graced the Parthenon on the Acropolis in Athens, were
stolen by Lord Elgin in the nineteenth century and given to the British Museum, which holds them to
this day. The people of Athens have built a beautiful, modern museum on the Acropolis to display
the Elgin Marbles and other treasures from the Greek cultural heritage, so there can be no valid
argument that the Greeks are unable to house the sculptures properly. Furthermore, more people
visit the Acropolis every day than visit the British Museum.

1. Of the following, the most appropriate title for the passage above would be:

A. The Elgin Marbles: Timeless Symbols of the Glory That Was Greece
B. The Role of Great Museums in the Preservation of Cultural Artifacts
C. Repatriation of Cultural Treasures: The British Museum’s Dirty Little Secret
D. The Value of Cultural Treasures in Defining National Identity
E. A Curious Curator: Lord Elgin and the Rise of the British Museum

2. The third paragraph plays what role in the passage?

A. It summarizes all the points expressed in the first two paragraphs.


B. It raises new arguments that expand on those previously expressed.
C. It suggests a possible area for useful research in the future.
D. It rejects the arguments expressed in the first paragraph.
E. It provides concrete evidence against arguments expressed in the second paragraph.
3. The situation involving the repatriation of the Elgin Marbles to Athens is most similar
to which of the following?

A. A Native American tribe in Oregon requests that a museum in Chicago return some ceremonial
masks that could help in fundraising efforts to build a proposed museum in Portland.
B. The nation of Peru in South America threatens the nation of Ecuador with military action if
Ecuador does not hand over various gold artifacts of the Inca Empire, which originated in Peru.
C. The National Archeology Museum of Cairo in Egypt requests that the Louvre return
eight mummies from the time of Ramses the Great for the Cairo Museum’s new exhibit
hall dedicated to artifacts from Ramses’ court.
D. The nation of Greece requests the nation of Turkey to provide Greek archeologists with free
access to ancient Greek sites on the Ionian coast of Turkey, and to transfer any cultural artifacts
found there to the National Archeology Museum in Athens.
E. A museum in Baton Rouge, Louisiana, requests that the Texas History Museum in Austin, Texas,
send the original “Lone Star” flag to Baton Rouge for a new exhibit entitled, “Texas: Our Neighbor to
the East.”

Dilemma (n) : tình trạng khó xử

Repatriation (n) : sự hồi hương

Ethnic group (n) : nhóm dân tộc

Rapacious (adj) : tham lam

Advocate (v) : ủng hộ

Prominent (adj) : nổi bật

Backwater country (n) = small town = in the middle of nowhere

Curator (n) : người quản lý, người coi sóc

Majestic (adj) : hùng vĩ, oai nghiêm

Marble (n) : đá hoa, đá cẩm thạch

Sculpture (n) : thuật điêu khắc

Grace (v) : ân sủng, làm vẻ vang

Heritage (n) : gia tài, di sản


Feb 01 – 2

Public health officials are becoming concerned about chlorine, a chemical that is added to many
municipal water supplies to reduce bacterial growth. New evidence shows that the rate of first-
trimester miscarriages is increased significantly when pregnant women drink five or more glasses of
chlorinated water per day. When the chlorine in the water reacts with plant material acids the
chemical trihalomethane is formed. Trihalomethane is believed to be the cause of the miscarriages,
and it is also linked to increased cancer risk in animals. Lead, which has been shown to cause brain
damage in small children, is another main contaminant of urban water systems, as it seeps from old
pipes into the water flowing through them. Many water supplies are also contaminated with toxic
bacteria, cysts, or algae. Increasing concerns about the effects of harmful contaminants have
caused an increase in the number of alternate methods of water filtration that are available to the
general public.

There are six major systems, each with its advantages and considerable drawbacks. Steam
distillation, in which water is boiled and the steam is collected and cooled, produces water without
contaminants. However, energy must be used to heat the water, the resulting water tastes flat, and
some organic compounds may boil with the steam and contaminate the water. Carbon filtration
removes chlorine from water and leaves it with a better taste, but it cannot remove minerals or
heavy metals. In addition, the filter is no longer effective if it becomes clogged, and the carbon can
trap and breed harmful bacteria. The ion exchange method uses charged particles in a filter which
are exchanged with charged particles in the water to remove minerals and toxic metals. This method
has a corrosive effect on pipes, and it does not remove organic molecules. It also increases the level
of sodium, iron, and lead in the water. Ultraviolet light is simple and can kill microorganisms, but it
has no effect on chemicals or minerals in the water. Reverse osmosis removes minerals, toxic heavy
metals, and bacteria by forcing the water through a semipermeable membrane. This method is
extremely slow, wastes large amounts of water, and corrodes the most durable of pipes.

Of the many different types of water filtration systems available, a combination KDF/carbon block
system may be the most effective. The KDF system, which stands for kinetic degradation fluxion,
forces water through a chamber containing a mixture of copper and zinc. In the chamber,
electrochemical reaction that takes place removes
chlorine, chloramine, iron, hydrogen sulfide, and many other harmful substances from the water.
The water is then forced through a solid carbon block filter, which removes more impurities, such as
bacteria and algae, from the water. Many KDF/carbon block filtration systems also contain a ceramic
pre-filter, which removes other bacteria such as Escherichia coli, Streptococcus sp. fecal coliform, as
well as cysts such as Cryptosporidium sp. and Giardia sp. As a by-product of filtration, the
KDF/carbon block system puts small amounts of copper and zinc into the water that are not harmful
to humans.

1. Which of the following best describes the organization of the passage?

A. Several methods are described and ranked in terms of desirability.


B. A general theory is offered and refuted by several counterexamples.
C. A current process is assailed, and several alternatives are presented as possible
replacements.
D. An old solution is revived as a possible remedy to a new problem.
E. A paradox is presented, and several examples are used to reconcile it.
2. Each of the following is a disadvantage of the ion exchange method of water filtration
EXCEPT:

A. higher sodium levels in the water


B. damaged pipes
C. unaffected contaminants of an organic nature
D. dangerous copper and zinc contamination
E. an increase of lead

3. According to the passage, which of the following can be inferred about the KDF
system?

A. It prevents the formation of trihalomethane in the water supply.


B. It would work much better if used in concert with a semipermeable membrane.
C. The electrochemical reaction causes most of the copper and zinc that are originally introduced
into the water to dissolve.
D. Without the additional ceramic pre-filter, it is no more effective than the combined effect of ion
exchange and ultraviolet light.
E. Its ability to remove iron from water is superior to that of ion exchange.

4. According to the passage, the author’s opinion of the best water filtration system
would probably change if which of the following were true?

A. If the KDF system is used indefinitely, the levels of copper and zinc that it introduces
into the water supply will eventually become toxic.
B. All new residential dwellings are fitted with pipes made of durable plastic that lead cannot seep
through.
C. Ultraviolet light is one-tenth as expensive as each of the other techniques mentioned.
D. Subsequent studies have revealed that there is no relation between trihalomethane and cancer.
E. KDF is available only in urban areas with tax bases that withstand the cost of installing such a
system.

Municipal (adj) : thuộc về đô thị

Lead (n) : chì

Algae (n) : rong biển

Filtration (n) : phương pháp lọc

Distillation (n) : trưng cất

Clog (v) : bị tắc, bịt lại

Corrosive (adj) : tính ăn mòn

Membrane (n) : màng

Semipermeable (adj) : bán kết (allowing certain substances to pass through it but not others,
especially allowing the passage of a solvent but not of certain solutes.)
Osmosis (n) : thẩm thấu

Impurity (n) : tạp chất

Formation (n) : sự hình thành

Assail (v) : tấn công, thực hiện

Remedy (n) : phương thuốc

Paradox (n) : nghịch lý

Reconcile (v) : hòa giải


Feb 02 - 1

One often hears that biographies are autobiographies, that the biographer is always writing about
himself. On the contrary, serious biographers seek and welcome the unfamiliar, however
troublesome to account for. Ron Chernow, the author of rich biographies of the American
businessmen J.P. Morgan and John D. Rockefeller, remarks that biographers “like to stub their toes
on hard, uncomfortable facts strewn in their paths.” Such encounters with the unaccountable are
opportunities for breaking out and breaking through, in new directions, to fresh understanding.

One also often hears that biographers must like their subjects. That would of course rule out such
vastly important subjects as Hitler or Stalin. In practice, the biographer must like the subject not as
a person but as a subject. Some are good subjects for the author, some bad. And what makes one
subject better than another for any particular biographer varies dramatically. Some of the reasons
are purely practical. Does the subject need a biography? Are the materials available? How much
time is needed? A biographer’s knowledge and ability also determine the choice. Great scientists are
great subjects, but can one write about their achievements with insight and authority? Personal
idiosyncrasies matter, too. Biographers tend to be attracted to subjects who display particular
personality traits, whether they be ambition, cruelty, ingenuity, or any other characteristic that
separates a potential subject from the multitudes.

In choosing a subject, the biographer’s main question should be, “Can an effective book be made
out of this person’s life?” Day after day for years, the biographer will try to untangle chronology,
compress relationships without distorting them, and keep the main narrative clear while carrying
forward several intricate strands of the subject’s life. What pushes most biographers on in this
endeavor is not necessarily affection for the subject but the feeling that they are writing a good
book.

1. It can be inferred that the author makes which of the following assumptions about
biographies?

A. Their main purpose is to inform readers about key aspects of the subjects’ personalities.
B. Only subjects who share traits with biographers make good subjects for biographies.
C. Compelling biographies cannot be written about ordinary citizens.
D. The biographer’s credibility with readers is a factor in the critical success of a
biography.
E. Practical considerations are most important in the selection of a subject for a biography.

-"What pushes most biographers on in this endeavor is not necessarily affection for the subject but
the feeling that they are writing a good book."
we can infer here that the biographers are concerned about writing a good book. Now how is a book
good? popular opinion is that a book which gains good audience is a good book. SO we can infer
this. This may not be a conclusive argument but after POE only D remains !!
2. The author is primarily concerned with

A. persuading biographers to change their methods


B. refuting some common beliefs about a particular literary genre
C. arguing against continued reform of a historical endeavor
D. refuting an outdated theory of a particular literary genre
E. describing the working methods of certain authors

3. The author mentions Ron Chernow most probably in order to

A. provide a counterexample to a general claim about biography


B. illustrate a questionable assertion regarding biography
C. establish a favorable comparison with an established biographer
D. underscore the importance of research in biography
E. challenge a new approach to biography

Biography (n) : tiểu sử danh nhân

Autobiography (n) : tự truyện

Troublesome (adj) : khó khăn

Strewn (v) : rải rác

Path (n) : con đường công danh

Encounter (v) : gặp gỡ,đụng độ

Vastly (adv) : bao la

Multitudes (n) : đa số

Untangle (v) : gỡ rối

Chronology (n) : niên đại

Distort (v) : xuyên tạc, làm siêu vẹo

Narrative (n) : tường thuật

Intricate (adj) : phức tạp

Endeavor (n) : cố gắng

Affection (n) : tình cảm, lòng thương

Credibility (n) : uy tín

Compelling (adj) : thuyết phục


Feb 02 – 2

The science of color management developed in response to a very specific need: the successful
development of high-volume assembly lines required strict quality control across a number of
parameters, including color. Engineers developed measurement devices, such as colorimeters and
spectrophotometers, to create profiles of various colors. A profile from a particular machine could
then be referenced to a standard color space, or set of definitions for establishing particular colors.
In this way, the profile data for a specific color was “standardized” and could be transferred among
different color creation and calibration machines, allowing, for example, six automobile assembly
lines located in two different states to produce thousands of identically-hued silver sedans over a
period of months.

In the 1990's, color management companies began adapting their color measurement expertise to
the healthcare field. One company, for example, created a cholesterol test that negated the typical
requirements for patients to fast before the test and have blood drawn, making the test much
cheaper and easier to administer. Drops of two different liquids, digitonin and an enzyme substrate,
are placed sequentially on the patient’s palm. First, the digitonin binds to cholesterol present in the
skin; then, the enzyme substrate causes a color-change reaction that is dependent upon the
patient’s cholesterol level. The doctor or technician can quantify the precise level by measuring the
color change with a spectrophotometer. Such novel applications promise to have as far-reaching an
impact on the healthcare industry as the first color measurement products had on early assembly-
line businesses nearly a century ago.

1) The passage is primarily concerned with which of the following?

A) Detailing the analogous application of a technology in two different industries.


B) Debating the historical significance of a particular type of technology.
C) Contrasting two different types of technologies.
D) Explaining the technical aspects of a new technology.
E) Comparing the historical development and success of two different industries.

2) Which of the following can be inferred from the passage?

A) A colorimeter can also be used in the color-based cholesterol test.


B) Color management companies will continue to develop healthcare applications.
C) Color management science would not have been developed had the automotive industry not
needed the technology.
D) Color management products are applicable to industries beyond automotive and healthcare.
E) Before color management technology was developed, it was difficult to produce a
large volume of cars with precisely equal colors.
3) According to the passage, all of the following are accurate EXCEPT

A) a spectrophotometer can measure the change in color of a particular substance or surface


B) color management technologies have been successfully applied to multiple industries
C) a color profile is a collection of definitions used to produce colors
D) it is necessary to apply the digitonin before the enzyme substrate in order for the cholesterol test
to work properly
E) color profiles allow companies to apply standardized metrics across an entire product line

''A profile from a particular machine could then be referenced to a standard color space, or set of
definitions for establishing particular colors.'' - which implies that standard color space is a set of
definitions to which a profile could be referenced to. A color profile is not a collection of definitions.

Parameter (n) : thông số

Colorimeter (n) : máy đo màu

Spectrophotometer (n) : máy đo quang phổ

Reference (v) : tham chiếu

Calibration (n) : hiệu chuẩn

Sequentially (adv) : tuần tự

Palm (n) : lòng bàn tay

Substrate (n) : chất nền

Quantify (v) : định lượng

Precise level (n) : mức độ chính xác

Analogous (n) : tương tự

Metric (n) : số liệu


Feb 03 – 1

From Romania to Germany, from Tallinn to Belgrade, a major historical process—the death of
communism—is taking place. The German Democratic Republic no longer exists as a separate state.
And the former German Democratic Republic will serve as the first measure of the price a post-
Communist society has to pay for entering the normal European orbit. In Yugoslavia we will see
whether the federation can survive without communism. One thing seems common to all these
countries: dictatorship has been defeated and freedom has won, yet the victory of freedom has not
yet meant the triumph of democracy.

Democracy is something more than freedom. Democracy is freedom institutionalized, freedom


submitted to the limits of the law, freedom functioning as an object of compromise between the
major political forces on the scene. We have freedom, but we still have not achieved the democratic
order. That is why this freedom is so fragile. In the years of democratic opposition to communism,
we supposed that the easiest thing would be to introduce changes in the economy. In fact, we
thought that the march from a planned economy to a market economy would take place within the
framework of the bureaucratic system, and that the market within the Communist state would
explode the totalitarian structures. Only then would the time come to build the institutions of a civil
society; and only at the end, with the completion of the market economy and the civil society, would
the time of great political transformations finally arrive.

The opposite happened. First came the big political change, the great shock, which either broke the
monopoly and the principle of Communist Party rule or simply pushed the Communists out of power.
Then came the creation of civil society, whose institutions were created in great pain, and which had
trouble negotiating the empty space of freedom. Only then, as the third moment of change, the final
task was undertaken: that of transforming the totalitarian economy into a normal economy where
different forms of ownership and different economic actors will live one next to the other.

Today we are in a typical moment of transition. No one can say where we are headed. The people
of the democratic opposition have the feeling that we won. We taste the sweetness of our victory
the same way the Communists, only yesterday our prison guards, taste the bitterness of their
defeat. Yet, even as we are conscious of our victory, we feel that we are, in a strange way, losing.
In Bulgaria the Communists have won the parliamentary elections and will govern the country,
without losing their social legitimacy. In Romania the National Salvation Front, largely dominated by
people from the old Communist bureaucracy, has won. In other countries democratic institutions
seem shaky, and the political horizon is cloudy. The masquerade goes on: dozens of groups and
parties are created, each announces similar slogans, each accuses its adversaries of all possible sins,
and each declares itself representative of the national interest. Personal disputes are more important
than disputes over values. Arguments over values are fiercer than arguments over ideas.
1. The author originally thought that the order of events in the transformation of
communist society would be represented by which one of the following?

(A) A great political shock would break the totalitarian monopoly, leaving in its wake a civil society
whose task would be to change the state-controlled market into a free economy.
(B) The transformation of the economy would destroy totalitarianism, after which a new
and different social and political structure would be born.
(C) First the people would freely elect political representatives who would transform the economy,
which would then undermine the totalitarian structure.
(D) The change to a democratic state would necessarily undermine totalitarianism, after which a
new economic order would be created.
(E) The people’s frustration would build until it spontaneously generated violent revolution, which
would sentence society to years of anarchy and regression.

2. Beginning in the second paragraph, the author describes the complicated relationship
between “freedom” and “democracy.” In the author’s view, which one of the following
statements best reflects that relationship?

(A) A country can have freedom without having democracy.


(B) If a country has freedom, it necessarily has democracy.
(C) A country can have democracy without having freedom.
(D) A country can never have democracy if it has freedom.
(E) If a country has democracy, it cannot have freedom.

3. From the passage, a reader could conclude that which one of the following best
describes the author’s attitude toward the events that have taken place in communist
society?

(A) Relieved that at last the democratic order has surfaced.


(B) Clearly wants to return to the old order.
(C) Disappointed with the nature of the democracy that has emerged.
(D) Confident that a free economy will ultimately provide the basis for a true democracy.
(E) Surprised that communism was toppled through political rather than economic means.

This is a tone question. The key to answering this question is found in the closing comments. There
the author states “The masquerade goes on,” referring to nascent democracies. So he has
reservations about the newly emerging democracies. Watch out for (E). Although it is supported by
the passage, it is in a supporting paragraph. The ideas in a concluding paragraph take precedence
over those in a supporting paragraph.
4. A cynic who has observed political systems in various countries would likely interpret
the author’s description of the situation at the end of the passage as

(A) evidence that society is still in the throws of the old totalitarian structure.
(B) a distorted description of the new political system.
(C) a necessary political reality that is a prelude to “democracy.”
(D) a fair description of many democratic political systems.
(E) evidence of the baseness of people.

5. Which one of the following does the author imply may have contributed to the
difficulties involved in creating a new democratic order in eastern Europe?

I. The people who existed under the totalitarian structure have not had the experience of
“negotiating the empty space of freedom.”
II. Mistaking the order in which political, economic, and social restructuring would occur.
III. Excessive self-interest among the new political activists.

(A) I only
(B) II only
(C) I and III only
(D) II and III only
(E) I, II, and III

This is an extension question. Statement I is true.The author implies that the institutions of the new-
born, free society were created in great pain because the people lacked experience. Statement II is
true. Expectations that the market mechanisms would explode totalitarianism and usher in a new
society were dashed, and having to readjust one’s expectations certainly makes a situation more
difficult. Finally, statement III is true. It summarizes the thrust of the passage’s closing lines.

6. By stating “even as we are conscious of our victory, we feel that we are, in a strange
way, losing” the author means that

(A) some of the old governments are still unwilling to grant freedom at the individual level.
(B) some of the new governments are not strong enough to exist as a single federation.
(C) some of the new democratic governments are electing to retain the old political
parties.
(D) no new parties have been created to fill the vacuum created by the victory of freedom.
(E) some of the new governments are reverting to communism.

Federation (n) : liên đoàn

Triumph (n) : chiến thắng

Institutionalized (adj) : thể chế hóa

Submitt (v) : quy phục

Compromise (n) : sự thỏa hiệp

Democratic (adj) : khuynh hướng dân chủ


Order (n) : trật tự

Fragile (adj) : mong manh, yếu ớt

March (n) : cuộc tuần hoành

Bureaucratic (adj) : quan liêu

Explode (v) : phát nổ

Totalitarian (adj) : toàn trị, chuyên chế

Civil society (n) : xã hội dân sự

Undertaken (v) : thực hiện

Parliamentary (adj) : nghị viện

Legitimacy (n) : tính hợp pháp

Shaky (adj) : lung lay

Horizon (n) : chân trời

Masquerade (n) : dạ hội hóa trang

Adversary (n) : đối thủ

Sin (n) : tội lỗi

Dispute (n) : tranh chấp

Fỉercer (adj) : dữ dội hơn

Baseness (n) : căn bản

Nascent (adj) : non trẻ, mới thành lập

Topple (v) : lật đổ

Prelude (n) : việc mở đầu


Feb 03 – 2

Scientists generally credit violent collisions between tectonic plates, the mobile fragments of Earth’s
rocky outer shell, with sculpting the planet’s surface, as, for example, when what is now the Indian
subcontinent collided with Asia, producing the Himalayan Mountains. However, plate tectonics
cannot fully explain certain massive surface features, such as the “superswell” of southern Africa, a
vast plateau over 1,000 miles across and nearly a mile high. Geologic evidence shows that southern
African has been slowly rising for the past 100 million years, yet it has not experienced a tectonic
collision for nearly 400 million years. The explanation may be in Earth’s mantle, the layer of rock
underlying the tectonic plates and extending down over 1,800 miles to the outer edge of Earth’s iron
core.

Since the early twentieth century, geophysicists have understood that the mantle churns and roils
like a thick soup. The relative low density of the hottest rock makes that material buoyant, so it
slowly ascends, while cooler, denser rock sinks until heat escaping the molten core warms it enough
to make it rise again. While this process of convection was known to enable the horizontal
movement of tectonic plates, until recently geophysicists were skeptical of its ability to lift or lower
the planet’s surface vertically. However, recent technological advances have allowed geophysicists to
make three-dimensional “snapshots” of the mantle by measuring vibrations, or seismic waves, set in
motion by earthquakes originating in the planet’s outer shell and recording the time it takes for them
to travel from an earthquake’s epicenter to a particular recording station at the surface. Because
geophysicists know that seismic waves become sluggish in hot, low-density rock, and speed up in
colder, denser regions, they can now infer the temperatures and densities in a given segment of the
interior. By compiling a map of seismic velocities from thousands of earthquakes across the globe,
they can also begin to map temperatures and densities throughout the mantle.These methods have
revealed some unexpectedly immense formations in the deepest parts of the mantle; the largest of
these is a buoyant mass of hot rock directly below Africa’s southern tip. Dispelling researchers’ initial
doubts, computer models have confirmed that this formation is buoyant enough to rise slowly within
the mantle and strong enough to push Africa upward as it rises.

1) In the highlighted text (Himalayan Mountains), the author mentions the Himalayan
Mountains most likely in order to

(A) highlight certain similarities between the southern African superswell and other massive features
on Earth's surface
(B) identify a feature of Earth's surface that predates the origins of the southern African superswell
(C) provide an example of a feature of Earth's surface that can be explained by plate
tectonics
(D) suggest that geophysicists are correct in attributing the sculpting of Earth's surface to violent
collisions between tectonic plates
(E) give an example of a feature of Earth's surface that scientists are unable to explain fully
2) It can be Inferred from the passage that prior to the technological advances referred
to in the highlighted text (recent technological), geophysicist were unable to

(A) understand exactly how the collisions of tectonic plates created the Earth's mountains
(B) establish that mantle material becomes more buoyant as it heats and more sluggish as it cools
(C) start mapping the densities and temperatures of rock throughout the mantle
(D) connect the phenomenon of convection within the mantle to the horizontal movement of
tectonic plates
(E) prove that different types of rock within the mantle have different densities

3) According to the passage, the process of convection mentioned in the highlighted


text (this process of convection) was regarded until recently by geophysicists as

(A) a process unlikely to occur deep within Earth's mantle


(B) a probable explanation for the rising of the southern African plateau
(C) an improbable explanation for the horizontal movement of tectonic plates
(D) unlikely to account for the vertical rising or lowering of Earth's surface
(E) unrelated to the creation of Earth's most massive surface features

4) According to the passage, which of the following pieces of geological evidence makes
plate tectonics an inadequate explanation for the existence of the superswell of
southern Africa?

(A) The depth of the mantle underlying the tectonic plates surrounding southern Africa
(B) The absence of any significant mountain ranges in the vicinity of the superswell
(C) The vast size of the plateau comprising the superswell's most visible feature
(D) The rate at whit the superswell has been rising above sea level over the past 100 million years
(E) The absence of any tectonic collisions in southern Africa for several hundred million
years prior to the origin of the superswell

‘However, plate tectonics cannot fully explain certain massive surface features, such as the
“superswell” of southern Africa, a vast plateau over 1,000 miles across and nearly a mile high.
Geologic evidence shows that southern African has been slowly rising for the past 100 million years,
yet it has not experienced a tectonic collision for nearly 400 million years’

Credit (v) : tin

Collision (n) : va chạm

Tectonic (adj) : kiến tạo

Plate (n) : mảng

Fragment (n) : mảnh vỡ

Sculpt (v) : điêu khắc

Subcontinent (n) : tiểu lục địa

Collide (v) : va chạm


Vast (adj) : rộng lớn

Plateau (n) : cao nguyên

Geologic (adj) : địa chất

Mantle (n) : lớp phủ

Geophysicist (n) : nhà địa vật lý

Buoyant (adj) : có thể nổi được

Ascend (v) : lên cao

Convection (n) : đối lưu

Vibration (n) : rung động

Seismic (adj) : địa chấn

Epicenter (n) : tâm chấn

Sluggish (adj) : chậm chạp

Interior (n) : nội địa, phần trong

Dispell (v) : xua tan, làm cho hết sợ

Superwell (n) : siêu tốc


Feb 04 – 1

Scholars often fail to see that music played an important role in the preservation of African culture in
the United States. They correctly note that slavery stripped some cultural elements from Black
people—their political and economic systems—but they underestimate the significance of music in
sustaining other African cultural values. African music, unlike the music of some other cultures, was
based on a total vision of life in which music was not an isolated social domain. In African culture
music was pervasive, serving not only religion, but all phases of life, including birth, death, work,
and play. The methods that a community devises to perpetuate itself come into being to preserve
aspects of the cultural legacy that that community perceives as essential. Music, like art in general,
was so inextricably a part of African culture that it became a crucial means of preserving the culture
during and after the dislocations of slavery.

1. The primary purpose of the passage is to

(A) analyze the impact that slavery had on African political and economic systems
(B) review the attempt of recent scholarship to study the influence of African music on other music
(C) correct the failure of some scholars to appreciate the significance of music in African
culture
(D) survey the ways by which people attempt to preserve their culture against the effects of
oppression
(E) compare the relative importance of music with that of other art forms in culture

2. In the Passage “isolated social domain” refers to


This reference is made when showing how African music is different from music in other cultures.
(A) African music in relation to contemporary culture as a whole
(B) music as it may be perceived in non-African cultures
(C) a feature of African music that aided in transmitting African cultural values
(D) an aspect of the African cultural legacy
(E) the influence of music on contemporary culture

3. Which of the following statements concerning the function of African music can be
inferred from the passage?

(A) It preserved cultural values because it was thoroughly integrated into the lives of
the people.
(B) It was more important in the development of African religious life than in other areas of culture.
(C) It was developed in response to the loss of political and economic systems.
(D) Its pervasiveness in African culture hindered its effectiveness in minimizing the impact of
slavery.
(E) Its isolation from the economic domains of life enabled it to survive the destructive impact of
slavery.
Tricky question as it is asking two things. Which conclusions the scholars have drawn and which of
them are wrong. Need to keep both things in mind.

4. According to the author, scholars would err in drawing which of the following
conclusions?

I. Slavery stripped the slaves of their political and economic systems. Correct conclusion
II. African music was similar to all other traditions of music in that it originated in a total vision of
life. African music is dissimilar is not concluded and hence they must have drawn this as a
conclusion. Also, the author says the scholars erred in doing so. This is correct.
III. Music was a crucial part of the African cultural legacy. They did not draw this conclusion at all!

(A) I only
(B) II only
(C) I and II only
(D) II and III only
(E) I, II, and III

Strip (v) : tước đoạt

Sustain (v) : duy trì

Isolated social domain (n) : miền xã hội bị cô lập

Pervasive (adj) : phổ biến

Perpetuate (v) : làm cho lâu dài

Devise (v) : nghĩ ra, phát minh

Inextricably (adj) : gắn bó chặt chẽ

Dislocation (n) : sự thoát ra

Contemporary (adj) : đồng thời


Feb 04 – 2

The complex life cycle of the Plasmodium protozoan, the causative agent of malaria, has contributed
to the difficulty of devising effective public health measures to combat the disease. It took scientists
centuries to deconstruct the basic relationship between protozoan, mosquito vector, and human
host. Modern physiologists and epidemiologists are still working out the intricacies of malarial
infection.

The disease is transmitted by the bite of a female Anopheles mosquito infected with the Plasmodium
parasite. Only Anopheles mosquitoes are capable of transmitting the disease, and only females take
blood meals from humans. To become infected with Plasmodium, the female mosquito takes a blood
meal from a human carrying the parasite in his or her blood. Once ingested, the parasite matures in
the mosquito’s gut for approximately a week, after which it migrates to the insect’s salivary glands.
By mixing with the mosquito’s saliva, the parasite facilitates its transmission to a human host when
the mosquito bites that human.

Once in a human’s bloodstream, the parasite travels to the human’s liver. At this initial stage, the
Plasmodium parasite is called a sporozoite. Within the liver, the sporozoite can form 30,000 to
40,000 daughter cells, called merozoites, which are released into the host’s bloodstream at a later
date, sometimes within a week of the initial infection and sometimes as much as several months
later. The merozoites seek out and attach themselves to red blood cells, in which they incubate 8 to
24 daughter cells over the next two days. When the daughter cells are mature, the red blood cell
ruptures and the new parasites are released into the bloodstream to seek out red blood cells of their
own.

Some of the new merozoites become male and female gametocytes; if these gametocytes are
ingested by a mosquito feeding on the host’s blood, they will fertilize in the mosquito’s gut to
produce new sporozoites, and the cycle will continue. The symptoms that we associate with
malaria—a high, recurring fever; joint pain; a swollen spleen are caused by toxins released from the
red blood cells ruptured by merozoites. The human spleen can destroy these infected blood cells,
but the Plasmodium parasite counters this effect by increasing the stickiness of proteins on the
blood cells’ surfaces so that the cells stick to the walls of blood vessels. If the sticky surface proteins
affect a particularly large number of cells, the malaria can transform into a hemorrhagic fever, the
most deadly form of malaria.

A further complicating factor in the natural history of malaria is the many variants of the
Plasmodium protozoan. Scientists now recognize that malaria is caused by at least six different
species: P. falciparum, P. vivax, P. ovale, P. malariae, P. knowesli, and P. semiovale. Of these
species, P. falciparum accounts for the majority of infections and approximately 90 percent of
malarial deaths in the world.
1. The passage is primarily concerned with which of the following?

A. Describing the life cycle of the Plasmodium protozoan as it relates to the disease
malaria
B. Comparing and contrasting the life cycles of the six variants of the Plasmodium protozoan known
to cause malaria
C. Addressing the public health implications of the life cycle of the Plasmodium parasite
D. Providing information on how a person can avoid infection with malaria
E. Describing the life cycle of the Anopheles mosquito as it relates to the transmission of the
Plasmodium protozoan to humans

2 . Which of the following most accurately states the role of the first paragraph in
relation to the passage as a whole?

A. It summarizes two theories, the relative merits of which are debated in the passage.
B. It puts forth an argument that the rest of the passage is devoted to refuting.
C. It introduces a new concept that the rest of the passage expands upon.
D. It frames the background and relevance of the material to follow.
E. It outlines the major themes of each of the four paragraphs to follow.

3. If a mosquito were to bite a person, and that person were later to develop malaria
and die of the disease, it is most likely that the person was infected with which of the
following?

A. Anopheles gambiae
B. Anopheles semiovale
C. Plasmodium malariae
D. Plasmodium vivax
E. Plasmodium falciparum

4. The relationship of a merozoite to a sporozoite is most like which of the following?

A. A mother to a daughter
B. A brother to a sister
C. One of several subsidiaries spun off from a large corporation
D. A computer program to a computer
E. Orange juice to an orange tree

Within the liver, the sporozoite can form 30,000 to 40,000 daughter cells, called merozoites

The passage states, “Within the liver, the sporozoite can form 30,000 to 40,000 daughter cells,
called merozoites, which are released into the host’s blood stream at a later date,” so the
relationship should be that of descent from the latter to the former, and of many from one, which
answer C captures.

Answer A could be correct if the order were reversed. ‘A sporozoite to a merozoite ‘


5.Based on the information given in the passage, which of the following would be most
effective in preventing a person infected with malaria from developing a hemorrhagic
fever?
A. Surgical removal of the spleen
B. A medicine that prevents changes to the surface proteins of red blood cells
C. An effective vaccine against malaria
D. A potent pesticide that reliably kills the Anopheles mosquito without producing any negative
consequences for the environment or for human health
E. A small infusion of a weaker variant of the Plasmodium protozoan that will then compete with the
existing parasitic infection

Causative (adj) : nguyên nhân

Malaria (n) : bệnh sốt rét

Deconstruct (v) : giải cấu trúc

Epidemiologist (n) : nhà dịch tễ học

Physiologist (n) : nhà sinh lý học

Intricacy (n) : phức tạp, rắc rối

Parasite (n) : ký sinh trùng

Salivary (n) : nước bọt

Gland (n) : tuyến

Incubate (v) : ủ bệnh

Rupture (v) : vỡ

Gametocyte (n) : giao tử

Spleen (n) : lá lách

Hemorrhagic (adj) : xuất huyết

Infusion (n) : truyền dịch

Spun off (v) : tách ra


Feb 05 – 1

On August 22, 1939, Adolf Hitler summoned his top military generals to Obersalzberg, where he
delivered a speech explaining his plans for war, first with Poland, then with the rest of Europe.
Despite resistance from those both inside and outside Germany, Hitler felt exceedingly confident
that he could defy the will of the international community and conquer vast amounts of land. In his
speech at Obersalzberg, he laid out numerous factors he believed would contribute to the success of
his war plans.

Chief among Hitler's sources of confidence in Germany's brazen war plans was German military
quickness. Hitler said, "Our strength lies in our quickness." On the advice of Colonel-General von
Brauchitsch, Hitler believed Poland could be captured in a few weeks, an astonishingly short amount
of time given the recent history of trench warfare and the long history of protracted European
military engagements that resulted in minimal land gains and high casualty counts.

Hitler's confidence in the ability of the German military to inflict considerable brutality further
strengthened his determination to pursue an exceedingly ambitious plan of territorial
aggrandizement. He said, "I shall shoot everyone who utters one word of criticism" and noted that
"the goal to be obtained in the war is not that of reaching certain lines but of physically demolishing
the opponent."In this vein, Hitler ordered his military to "be hard, be without mercy, [and] act more
quickly and brutally than others…for it scares the others off." Hitler believed that enemies, not used
to this type of brutality, would surrender quickly.

In addition to speed and brutality, Hitler believed that, in the end, history would overlook his
inhumane conduct. To support this view, which turned out to be anything but prescient, Hitler
invoked a pollyannaish view of Asian leader Genghis Kahn. In Hitler's eyes, Kahn "sent millions of
women and children into death knowingly and with a light heart," yet "history sees in him only the
great founder of States."

Although Hitler brimmed with confidence and experienced initial yet widely-expected success in
Poland and then in Denmark, he overlooked important considerations. In many ways, Hitler made
the same mistake Napoleon Bonaparte made years earlier. Hitler believed he could advance further
and conquer Britain, yet, like Napoleon, Hitler did not adequately foresee the insurmountable barrier
posed by Britain's island status. Despite the damage inflicted at the hands of the German Luftwaffe
during the Battle of Britain (1940), British forces eventually won this important battle. Nevertheless,
Hitler pressed on and, in an even more fateful decision that carried echoes of a Napoleonic tactical
misstep, invaded the USSR where his forces suffered the decisive defeat of World War II at
Stalingrad in 1943. In the end, Hitler's reputation in history proved to be as brutal and decisive as
the battle plans and philosophy he announced at Obersalzberg.
Q.1 According to the passage, Hitler's confidence in his military strategy stemmed from
its:

A) Surprise invasions
B) Emphasis on unconventional warfare
C) Reliance on air supremacy
D) Swift brutality
E) Napoleonic overtones

Q. 2 The author of the passage is primarily concerned with explaining:

A) The logistics of Hitler's war strategy and the mechanics of its failure
B) The philosophy of Hitler's war strategy and the world's reaction
C) Why Hitler believed his war plans would succeed and why they eventually failed
D) Hitler's plans and their failure with an eye to pre-1900 history
E) Explaining the source of Hitler's brutality and the reasons for its failure

Q. 3 Which of the following best characterizes the author's view of the relationship
between Hitler and Napolean?

A) Governed with similar styles


B) Fought military conflicts with similar ideologies
C) In general, shared a legacy as overly ambitious leaders
D) At a high-level, some similarities in military missteps existed
E) Both suffered final defeats by impetuously charging east

Q. 4 According to the passage, what best describes the author's understanding of why
Hitler's military campaign eventually failed?

A) Failed to demoralize opponents


B) Overlooked important tactical and geographic considerations
C) Underestimated international resolve
D) Fell behind technological advancements of European enemies
E) Failed to consolidate initial military and land gains

Q. 5 According to the passage, Hitler's confidence in the war plans announced at


Obersalzberg stemmed from all of the following EXCEPT:

A) The speed of the German military


B) The brutality of the German military
C) The plan to stifle dissent
D) The belief that instilling fear weakened enemies
E) The history of overlooking European military brutalit
Q. 6 According to the passage, why did Hitler believe he could conquer Poland in a few
weeks?
A) The inaction of European neighbors
B) The example of Napoleon
C) The philosophy of Genghis Kahn
D) The counsel of a military general
E) The small size of Poland

On the advice of Colonel-General von Brauchitsch, Hitler believed Poland could be captured in
a few weeks, an astonishingly short amount of time given the recent history of trench warfare and
the long history of protracted European military engagements that resulted in minimal land gains
and high casualty counts.

Q. 7 According to the passage, which of the following represents the chronological


unfolding of events?
A) Generals summoned to Obersalzberg; Invaded Poland; Invaded Denmark; the Battle
of Britain; Battle at Stalingrad
B) Generals summoned to Obersalzberg; Invaded Denmark; Invaded Poland; the Battle of Britain;
Battle at Stalingrad
C) Generals summoned to Obersalzberg; Invaded Denmark; Invaded Poland; Battle at Stalingrad;
the Battle of Britain
D) Generals summoned to Obersalzberg; Invaded Poland; Invaded Denmark; Battle at Stalingrad;
the Battle of Britain
E) Generals summoned to Obersalzberg; the Battle of Britain; Invaded Poland; Invaded Denmark;
Battle at Stalingrad

Summon (v) : triệu tập

Resistance (n) : sự chống lại, kháng cự

Defy (v) : bất chấp, coi thường

Astonishingly (adv) : đáng kinh ngạc

Inflict (v) : gây thiệt hại

Territorial (adj) : lãnh thổ

Aggrandizement (n) : làm rộng thêm

Utter (v) : đọc một chữ

Invoke (v) : viện dẫn, cầu cứu

Pollyannaish (adj) : đa tình

Demolish (v) : phá hủy

Brim with (v) : tràn đầy

Foresee (v) : thấy trước

Insurmountable (adj) : không thể vượt qua


Fateful (adj) : định mệnh

Echo (n/v) : tiếng vang, bắt trước

Counsel (n) : lời tư vấn

Chronological unfolding of events (n) : sự mở ra theo dòng thời gian của các sự kiện
Feb 05 - 2

The pioneers of the teaching of science imagined that its introduction into education would remove
the conventionality, artificiality, and backward-lookingness which were characteristic; of classical
studies, but they were gravely disappointed. So, too, in their time had the humanists thought that
the study of the classical authors in the original would banish at once the dull pedantry and
superstition of mediaeval scholasticism. The professional schoolmaster was a match for both of
them, and has almost managed to make the understanding of chemical reactions as dull and as
dogmatic an affair as the reading of Virgil's Aeneid. The chief claim for the use of science in
education is that it teaches a child something about the actual universe in which he is living, in
making him acquainted with the results of scientific discovery, and at the same time teaches him
how to think logically and inductively by studying scientific method.

A certain limited success has been reached in the first of these aims, but practically none at all in
the second. Those privileged members of the community who have been through a secondary or
public school education may be expected to know something about the elementary physics and
chemistry of a hundred years ago, but they probably know hardly more than any bright boy can pick
up from an interest in wireless or scientific hobbies out of school hours. As to the learning of
scientific method, the whole thing is palpably a farce. Actually, for the convenience of teachers and
the requirements of the examination system, it is necessary that the pupils not only do not learn
scientific method but learn precisely the reverse, that is, to believe exactly what they are told and to
reproduce it when asked, whether it seems nonsense to them or not.

The way in which educated people respond to such quackeries as spiritualism or astrology, not to
say more dangerous ones such as racial theories or currency myths, shows that fifty years of
education in the method of science in Britain or Germany has produced no visible effect whatever.
The only way of learning the method of science is the long and bitter way of personal experience,
and, until the educational or social systems are altered to make this possible, the best we can
expect is the production of a minority of people who are able to acquire some of the techniques of
science and a still smaller minority who are able to use and develop them.

1. The author implies that the professional schoolmaster has

A. no interest in teaching science


B. thwarted attempts to enliven education
C. aided true learning
D. supported the humanists
E. been a pioneer in both science and humanities.

2. The authors apparently believes that secondary and public school education in the
sciences is

A. severely limited in its benefits


B. worse than that in the classics
C. grossly incompetent
D. a stimulus to critical thinking
E. deliberately obscurantist
3. If the author were to study current education in science to see how things have
changed since he wrote the piece, he would probably be most interested in the answer
to which of the following questions?

A. Do students know more about the world about them?


B. Do students spend more time in laboratories?
C. Can students apply their knowledge logically?
D. Have textbooks improved?
E. Do they respect their teachers?

4. All of the following can be inferred from the text EXCEPT

A. at the time of writing, not all children received a secondary school education
B. the author finds chemical reactions interesting
C. science teaching has imparted some knowledge of facts to some children
D. the author believes that many teachers are authoritarian
E. it is relatively easy to learn scientific method

Pioneer (n) : người tiên phong

Conventionality (n) : quy ước, tập tục

Artificiality (n) : nhân tạo, giả bộ

Backward-lookingness (n) : cái nhìn lạc hậu

Gravely (adv) : nghiêm trọng, nặng nề

Humanist thought (n) : tư tưởng nhân văn

Banish (v) : xua đuổi

Dull (adj) : buồn tẻ

Pedantry (n) : nhà sư phạm, thầy giáo

Superstition (n) : mê tín

Mediaeval (adj) : thuộc về thời trung cổ

Scholasticism (n) : kinh viện, triết học

Dogmatic (adj) : giáo điều, độc đoán

Acquainted with (v/adj) : hiểu biết

Privileged (adj) : có đặc quyền

Farce (n) : trò hề

Quackery (n) : thủ thuật

Spiritualism (n) : tâm linh


Astrology (n) : chiêm tinh

Racial (adj) : chủng tộc

Incompetent (adj) : bất tài, không đủ tư cách

Stimulus (n) : chất kích thích, sự khích lệ

Thwarted (adj) : cản trở

Enliven (v) : kích thích, sinh động


Feb 06 – 1

That placebos can cure everything from dandruff to leprosy is well known. They have a long history
of use by witch doctors, faith healers, and even modern physicians, all of whom refuse to admit
their efficacy. Modern distribution techniques can bring this most potent of medicines to the aid of
everyone, not just those lucky enough to receive placebos in a medical testing program.

Every drug tested would prove effective if special steps were not taken to neutralize the placebo
effect. This is why drug tests give half the patients the new medication and half a harmless
substitute. These tests prove the value of placebos because approximately five percent of the
patients taking them are cured even though the placebos are made from substances that have been
carefully selected to be useless.

Most people feel that the lucky patients in a drug test get the experimental drug because the real
drug provides them a chance to be cured. (1) Yet analysis shows that patients getting the placebo
may be the lucky ones because they may be cured without risking any adverse effects the new drug
may have. Furthermore, the drug may well be found worthless and to have severe side effects. No
harmful side effects result from placebos.

Placebos regularly cure more than five percent of the patients and would cure considerably more if
the doubts associated with the tests were eliminated. Cures are principally due to the patient’s
faith, (2) yet the patient must have doubts knowing that he may or may not be given the new
drug, which itself may or may not prove to be an effective drug. Since he knows the probability of
being given the true drug is about fifty percent, the placebo cure rate would be more than doubled
by removing these doubts if cures are directly related to faith.

The actual curing power of placebos probably stems from the faith of the patient in the treatment.
This suggests that cure rates in the ten percent range could be expected if patients are given
placebos under the guise of a proven cure, even when patients know their problems are incurable. It
may take a while to reach the ten percent level of cure because any newly established program will
not have cultivated the word-of-mouth advertising needed to insure its success. One person saying
“I was told that my problem was beyond medical help, but they cured me,” can direct countless
people to the treatment with the required degree of faith. Furthermore, when only terminal illnesses
are treated, those not cured tell no one of the failure.

Unfortunately, placebo treatment centers cannot operate as nonprofit businesses. The nonprofit idea
was ruled out upon learning that the first rule of public medicine is never to give free medicine.
Public health services know that medicine not paid for by patients is often not taken or not effective
because the recipient feels the medicine is worth just what it cost him. (3) Even though the
patients would not know they were taking sugar pills, the placebos cost so little that the patients
would have no faith in the treatment. Therefore, though it is against higher principles, treatment
centers must charge high fees for placebo treatments. This sacrifice of principles, however, is a
small price to pay for the greater good of the patients.
1. Which one of the following best expresses the main idea of the passage?

(A) Placebo treatment is a proven tool of modern medicine and its expanded use would
benefit society’s health.
(B) Because modern technology allows for distribution of drugs on a massive scale, the proven
efficacy of the placebo is no longer limited to a privileged few.
(C) The curative power of the placebo is so strong that it should replace proven drugs because the
patients receiving the placebo will then be cured without risking any adverse side effects.
(D) The price of placebo treatment must be kept artificially high because patients have little faith in
inexpensive treatments.
(E) Semi-placebos—drugs that contain only a small amount of the usual dosage—are even more
effective curatives than either the placebo or the full-strength drug.

2. Which one of the following is most analogous to the idea presented in the last
paragraph?

(A) Buying a television at a discount house


(B) Making an additional pledge to charity
(C) Choosing the most expensive dishwasher in a manufacturer’s line
(D) Waiting until a book comes out in paperback
(E) Contributing one dollar to the Presidential Campaign fund on your tax return

The information needed to answer this question is heralded by the pivotal phrase “Even though”.
The implication of that sentence is “you get what you pay for.” This would motivate one to buy
the most expensive item in a manufacturer’s line. Hence the answer is (C).

3. According to the passage, when testing a new drug medical researchers give half of
the subjects the test drug and half a placebo because

(A) proper statistical controls should be observed.


(B) this method reduces the risk of maiming too many subjects if the drug should prove to be
harmful.
(C) all drugs which are tested would prove to be effective otherwise.
(D) most drugs would test positively otherwise.
(E) the cost of dispensing drugs to all the patients is prohibitive.

4. It can be inferred from the passage that the author might

(A) believe that the benefits of a placebo treatment program which leads patients to
believe they were getting a real drug would outweigh the moral issue of lying.
(B) support legislation outlawing the use of placebos.
(C) open up a medical clinic that would treat patients exclusively through placebo methods.
(D) believe that factors other than faith are responsible for the curative power of the placebo.
(E) believe that placebo treatment centers should be tax-exempt because they are nonprofit
businesses.
5. Which one of the following best describes the organization of the material presented
in the passage?

(A) A general proposition is stated; then evidence for its support is given.
(B) Two types of drug treatment—placebo and non-placebo—are compared and contrasted.
(C) A result is stated, its cause is explained, and an application is suggested.
(D) A dilemma is presented and a possible solution is offered.
(E) A series of examples is presented; then a conclusion is drawn from them.

6. Which one of the following most accurately characterizes the author’s attitude
toward placebo treatment?

(A) reserved advocacy


(B) feigned objectivity
(C) summary dismissal
(D) perplexed by its effectiveness
(E) zealous promotion

Placebo (n) : giả dược

Dandruff (n) : gàu

Leprosy (n) : bệnh phong

Faith (n) : đức tin

Efficacy (n) : hiệu quả

Neutralize (v) : trung hòa, vô hiệu

Severe (adj) : nghiêm trọng

Principally (adv) : chủ yếu

Terminal illness (n) : bệnh nan y

Recipent (n) : người nhận

Reserved (adj) : kín đáo

Advocacy (n) : vận động

Feigned (adj) : giả vờ

Perplexed (adj) : bối rối

Zealous (adj) : sốt sắng, hăng hái, nhiệt tâm

Dilemma (n) : tình trạng khó xử

Curative (adj) : chữa bệnh

Pledge (n) : lời hứa, lời thề


Feb 06 – 2

Because the very subject matter of anthropology is so volatile, it’s no surprise that the discipline is
frequently embroiled in controversy. Even when social commentators and outside observers fail to
criticize the latest anthropological theory on human nature, the social science’s own practitioners are
often up in arms over some study or another. Consider Ekman’s landmark study of human emotions
in the 1960s.

At the time, the accepted movement in anthropology was relativism. In an effort to rid the discipline
of accusations of bias, anthropologists attempted to study cultures as isolated systems. The norms,
mores, and practices of each culture were analyzed only in terms of the internal consistency they
possessed and any suggestion of judgment was met by howls of outrage by the anthropological
establishment. Into this arena came Ekman with his startling heresy; emotions, argued Ekman, were
not arbitrary cultural constructs but universal human traits. Ekman had spent years traveling the
world, showing people around the globe photographs of other people expressing six basic emotions:
happiness, sadness, anger, fear, disgust, and surprise. Not one person studied by Ekman failed to
recognize these emotions, whether the person in the photograph was wearing the suit of a Western
businessman or the tribal dress of the Fore foragers of New Guinea.

When Ekman presented his findings at an anthropological conference, he was denounced as a


fascist and a racist. Some of his fellow scientists even took his research to prove not that human
emotions were universal but that the hegemony of Western culture was so complete that even the
most far-flung peoples were socialized into the Western mindset. Ekman was shocked at the
reaction. He thought his findings would be evidence of the brotherhood of man, not of the
subjugation of the world by the West. And yet, Ekman’s conclusions have been replicated again and
again and are now generally accepted in the anthropology community, which apparently is like its
subject matter: quick to anger but perhaps slow to admit mistakes.

1. Which of the following most accurately expresses the main idea of the passage?

A. Despite an initial outcry over his work, Ekman was eventually vindicated by the scientific
community for his research findings.
B. The anthropological community as a whole is characterized by rigidity and an unwillingness to
accept new ideas.
C. Anthropologists take longer to accept controversial ideas about human nature than do scientists
in other fields.
D. The anthropological community’s reaction to certain heretical ideas may reflect a
basic truth about the nature of the discipline.
E. Ekman’s revolutionary study about the universality of human emotions caused a stir in the
anthropological community that reverberates to this day.

First assess the question. This is clearly a main idea question. In order to answer this type of
question, first refer to the notes you took from your analytical reading. The answer to a main idea
question should reflect the major points of the entire passage, not just one part. Focus on the
beginning of the paragraphs and the final few sentences to get a sense of the big picture; unless
you see a word like “however,” “but,” or “although,” don’t look at the details contained in the
body paragraphs.
Earlier, you gathered that the main idea was something like “Ekman’s experience shows how
members of the anthropological community may initially react negatively to a new finding, but may
eventually come to accept the finding.” Now look at the answer choices and see how well they
match your take on the passage. Choice A focuses on Ekman, stating he was vindicated by the
scientific community. But look back at the passage. Ekman and his experience is an example used
by the author to support a statement about the anthropological community. So while choice A may
discuss a part of the passage, it is not the main idea. Choice B makes a claim that is not supported
by the passage. The author claims that the community eventually accepted Ekman’s new idea,
although it caused controversy at first. Choice C compares anthropologists to scientists in other
fields, but the passage makes no explicit comparison to other scientists. Now look at choice D. The
best answer to main idea question encompasses all major parts of the passage. The first paragraph
makes a claim about the nature of the anthropological community, the second and third paragraphs
detail the reaction of the community, and the final sentence reiterates the author’s belief about the
nature of the members of the anthropological community. Choice D is looking pretty good, but you
must check the final answer. Choice E focuses on Ekman’s study, but it doesn’t mention anything
about the nature of the anthropological community, a key idea in the passage. Furthermore, nothing
in the passage supports the idea that the ideas “still reverberate to this day.” So it looks like choice D
is best after all.

2. According to the passage, the results of Ekman’s study were

A. unequivocal in their support of Ekman’s thesis that there are certain emotions that
are universally recognized
B. inconclusive, as they may have simply indicated the hegemony of Western culture rather than the
universality of human emotions
C. invalid because Ekman only used pictures of Western businessmen and Fore tribesmen
D. inflammatory because Ekman’s personal biases influenced the results
E. limited because Ekman did not carry out enough research to properly draw the conclusion he
reached

This is a supporting detail question. It asks you to figure what the passage says about the results of
Ekman’s study. For questions that ask about specific parts of the passage, your goal is to find the
particular lines in which the author discusses the supporting idea asked about in the question. Your
analytical reading should give you an understanding of the structure of the passage, which may help
you figure out where to look for the answer. Alternatively, you can use the words in the question
itself to aid you. The GMAT writers typically use specific words from the passage in the question
stem, so scan around the passage for something like “results of Ekman’s study.” The second
paragraph would probably be the best place to look first because you know from your analysis that
this paragraph details Ekman’s experience. At the end of the paragraph you will find the sentence
“Not one person studied by Ekman failed to recognize these emotions, whether the person in the
photograph was wearing the suit of a Western businessman or the tribal dress of the Fore foragers
of New Guinea.” These lines contain the answer: the results showed that everyone Ekman studied
was able to recognize the emotions pictured.

Now you have to find the answer choice that matches what the passage states. Choice A looks
promising since it says that the results clearly supported Ekman’s thesis. Even if you think choice A
is good, you still must look at the other answer choices; sometimes it’s easy to become enamored
of a certain answer choice but there may be an even better answer. Choice B is incorrect because
the passage doesn’t state that the results indicated Western hegemony. Some scientists argued that
but the passage states that the conclusions have been replicated and are now accepted. There is
nothing in the passage to support choice C. Choice D may look tempting because the results
certainly were inflammatory. However, the passage doesn’t explicitly state that the results were
controversial because of Ekman’s biases. Choice E is also not stated by the author. If anything, the
passage indicates the opposite by saying that Ekman spent years traveling the world for his study.
Thus, choice A turns out to be the best choice.

3. The passage suggests which of the following about anthropology?

A. For much of the discipline’s history it was often charged with making racist assumptions about
the value of other cultures’ practices.
B. The relativist approach is no longer employed by modern anthropologists.
C. At one time, anthropologists evaluated cultures based on a culture’s relationships to
other cultures in the world.
D. Members of the anthropological community are more sensitive to accusations of bias than are the
members of other disciplines.
E. Anthropologists believe that the prevalence of Western cultural norms represents a threat to
indigenous peoples around the world.

Any question that asks what the passage suggests or implies is an inference question. Inference
questions tend to be slightly more difficult than supporting detail questions, even though they both
ask you about the facts in the passage. One difficulty is that inference questions tend to be open
ended; from reading the question, it may not be obvious where exactly in the passage you should
look for the answer. Another difficulty concerns the inference itself. An inference is not merely a
guess or a hunch. Inferences on the GMAT need to be supported by the passage. Any choice that
could be true or might be true just won’t cut it. This question requires you to use the information in
the passage to draw an inference about anthropology. Where should you look? Considering that the
entire passage is about anthropology, it would be inefficient to read practically everything.

When the question doesn’t give you much help figuring out where to look, go to the answer choices.
Take each choice one by one and see if you can prove it to be true based on the passage.
For choice A, look at the parts of the passage that deal with accusations of racism. The last
paragraph states some scientists accused Ekman of racism, but that doesn’t help. The beginning of
the second paragraph said anthropology had been accused of bias, but nothing supports the idea
that these accusations occurred for “much of the discipline’s history.” So while this answer
might be true, it’s not an inference. Choice B is easier to find—relativism appears in the beginning
of the second paragraph. But nowhere does the passage state that relativism is no longer practiced,
so choice B can be eliminated. Now for choice C, look for the part of the passage that talks about
how anthropologists used to evaluate cultures. The second paragraph is a good place to look
because the beginning talks about judging other cultures. The passage states that anthropologists
tried “to study cultures as isolated systems. The norms, mores, and practices of each
culture were analyzed only in terms of the internal consistency they possessed.” Since
this was a change, it means that before, cultures were not studied as isolated systems or in terms of
internal consistency. This makes choice C a good inference. Choice D is easy to eliminate
because the passage doesn’t compare anthropologists to other scientists. The final choice goes too
far for an inference. In the passage, Western hegemony was mentioned by a “few” scientists. But
that doesn’t mean that anthropologists in general feel this way. Choice C is best.
Anthropology (n) : nhân chủng học

Volatile (adj) : bay hơi

Embroil (v) : lôi kéo

Criticize (v) : phê bình, chỉ trích

Relativism (n) : thuyết tương đối

Rid (v) : phóng thích, thoát khỏi

Bias (n) : thiên vị

Norm (n) : quy tắc, định mức

Mores (n) : phong tục, quy ước thiết yếu, đặc trưng của một cộng đồng

Consistency (n) : tính nhất quán

Possess (v) : sở hữu

Arena (n) : phạm vi

Heresy (n) : dị giáo

Denounce (v) : tố cáo, sỉ nhục, phản đối

Arbitrary (adj) : tùy ý, độc đoán, tự tiện

Hegemony (n) : quyền lãnh đạo

Far-flung (adj) : xa xôi

Subjugation (n) : chinh phục

Replicate (v) : nhân rộng

Unequivocal (adj) : rõ ràng, minh bạch

Prevalence (n) : sự đang thịnh hành

Indigenous (adj) : bản địa

Reverberate (v) : vang dội

Stir (n) : sự náo động

Vindicate (v) : minh oan, bào chữa, biện hộ

Outcry (n) : phản đối

Rigidity (n) : sự nghiêm khắc

Heretical (adj) : dị giáo

Discipline (n) : quy luật / for much of : đối với phần lớn của
Feb 07 – 1

According to usage and conventions which are at last being questioned but have by no means been
overcome, the social presence of a woman is different in kind from that of a man. A man’s presence
is dependent upon the promise of power which he embodies. If the promise is large and credible his
presence is striking. If it is small or incredible, he is found to have little presence. The promised
power may be moral, physical, temperamental, economic, social, sexual—but its object is always
exterior to the man. A man’s presence suggests what he is capable of doing to you or for you. His
presence may be fabricated, in the sense that he pretends to be capable of what he is not. But the
pretense is always toward a power which he exercises on others. By contrast, a woman’s presence
ex- presses her own attitude to herself, and defines what can and cannot be done to her. Her
presence is manifest in her gestures, voices, opinions, expressions, clothes, chosen surroundings,
taste—indeed there is nothing she can do which does not contribute to her presence. Presence for a
woman is so intrinsic to her person that men tend to think of it as an almost physical emanation, a
kind of heat or smell or aura.

To be born a woman has been to be born, within an allotted and confined space, into the keeping of
men. The social presence of women has developed as a result of their ingenuity in living under such
tutelage within such a limited space. But this has been at the cost of a woman’s self being split into
two. A woman must continually watch herself. Whilst she is walking across a room or whilst she is
weeping at the death of her father, she can scarcely avoid envisaging herself walking or weeping.
From earliest childhood she has been taught and persuaded to survey herself continually. And so
she comes to consider the surveyor and the surveyed within her as the two constituent yet always
distinct elements of her identity as a woman. She has to survey everything she is and everything
she does because how she appears to others, and ultimately how she appears to men, is of crucial
importance for what is normally thought of as the success of her life. Her own sense of being in
herself is supplanted by a sense of being appreciated as herself by another.

Men survey women before treating them. Consequently how a woman appears to a man can
determine how she will be treated. To acquire some control over this process, women must contain
it and internalize it. That part of a woman’s self which is the surveyor treats the part which is the
surveyed so as to demonstrate to others how her whole self would like to be treated. And this
exemplary treatment of herself by herself constitutes her presence. Every woman’s presence
regulates what is and is not “permissible” within her presence. Every one of her actions—whatever
its direct purpose or motivation—is also read as an indication of how she would like to be treated. If
a woman throws a glass on the floor, this is an example of how she treats her own emotion of anger
and so of how she would wish to be treated by others. If a man does the same, his action is only
read as an expression of his anger. If a woman makes a good joke this is an example of how she
treats the joker in herself and accordingly of how she as joker-woman would like to be treated by
others. Only a man can make a good joke for its own sake.
1. According to “usage and conventions,” appearance is NECESSARILY a part of reality
for

(A) men
(B) women

Compared with the Men in the first paragraph, then discussed in the second paragraph
then discussed with examples in the last paragraph it looks pretty fine, if not perfect.

(C) both men and women


(D) neither men nor women
(E) men always and women occasionally

First thing we have to fix in mind that we are going to answer what we have read in the Passage.
Question #1 purely talks about the whole of the passage, take this question as an inference
question. If you try to find out the exact text to answer the question you will fail. Read whole
passage understand it then pick up the correct choice.

Take a deep look on question statement, the word NECCASSARILY sounds an extreme words so
we have to select an answer according to this.

1st paragraph introduces the topic at hand and compares Women appearance with Men.
2nd paragraph discussed the appearance of Women
3rd paragraph discussed women treatment to the difference scenarios with examples,

2. In analyzing a woman’s customary “social presence,” the author hopes to

(A) justify and reinforce it.


(B) understand and explain it.
(C) expose and discredit it.
(D) demonstrate and criticize it.
(E) sanction and promote it.

3. It can be inferred from the passage that a woman with a Ph.D. in psychology who
gives a lecture to a group of students is probably MOST concerned with

(A) whether her students learn the material.


(B) what the males in the audience think of her. - This talks about the thoughts and not about
women appearance.
(C) how she comes off as a speaker in psychology. - It is related with the women issue of
their appearance and their psyche. This is best analogous example.
(D) finding a husband.
(E) whether a man challenges her.

4. The passage portrays women as


(A) victims
(B) liars
(C) actresses - Definitely this is the answer, Actress are much conscious about their appearance if
you have seen any movie shot, actress mostly try to see their face in the mirror.
(D) politicians
(E) ignorant

5. Which one of the following is NOT implied by the passage?


(A) Women have split personalities.
(B) Men are not image-conscious.
(C) Good looks are more important to women than to men.
(D) A man is defined by what he does, whereas a woman is defined by how she appears.
(E) A man’s presence is extrinsic, whereas a woman’s is intrinsic.

6. The primary purpose of the passage is to


(A) compare and contrast woman’s presence and place in society with that of man’s.
(B) discuss a woman’s presence and place in society and to contrast it with a man’s
presence and place.
(C) illustrate how a woman is oppressed by society.
(D) explain why men are better than women at telling jokes.
(E) illustrate how both men and women are hurt by sexism.

Embody (v) : hiện thân

Credible (adj) : đáng tin cậy

Striking (adj) : nổi bật

Temperamental (adj) : khí chất

Exterior (adj/n) : bên ngoài, vật ở ngoài

Fabricated (adj) : bịa đặt, giả mạo

Instrinsic (adj) : thuộc về bản chất

Emanation (n) : phát ra

Aura (n) : hào quang, hơi thoáng qua

Confined (adj) : hạn chế

Allot (v) : phân bổ, giao cho

Ingenuity (n) : ngây thơ, chất phác, thật thà

Weep (v) : khóc lóc

Envisage (v) : dự tính, nghĩ đến

Supplant (v) : thay thế

Permissible (adj) : có thể chấp nhận

Sake (n) : lợi ích

Reinforce (v) : củng cố

Justify (v) : biện minh, bào chữa


Feb 07 – 2

The collapse of the stock “bubble” of Internet-related companies in 2000–2001 has resulted in more
than its fair share of analysis, hand-wringing, and finger-pointing. A panel discussion at a recent
Technology Today conference in Santa Monica produced a heated debate between two former
luminaries of the dot.com world: investment banker Pat Verhofen and Sue Mickelson, founder and
CEO of Internet retailer Frizbeez.com Verhofen fired the opening shot by placing blame for the
collapse of Internet stocks on the shoulders of Internet entrepreneurs who aggressively promoted
ideas without viable business models. These entrepreneurs were both irresponsible and deceptive,
Verhofen argued, to take investors’ money to fund operations that could not reasonably turn a
profit, such as giving computers away for free or selling bulky objects, such as dog food or furniture,
over the Internet. Many of these companies, he suggested, were little more than arrangements of
smoke and mirrors designed to separate investors from their money.

Mickelson responded that Verhofen was like a fox in a henhouse blaming the rooster for all the dead
chickens. Entrepreneurs cannot be blamed, she argued, for trying to make money for themselves
and other people, because that is what entrepreneurs do. She also stated that you cannot know
what ideas will or will not work until you try them; contemporaries of the Wright brothers said that a
heavier-than-air aircraft could never work, and look at the skies today. Mickelson instead placed the
blame on the unscrupulous bankers and fund managers who hyped Internet stocks in order to cash
in on fees from IPOs and trades. In contrast to entrepreneurs, these financial types actually do have
a responsibility to offer only sound financial advice to their clients. If anyone should bear the blame,
she argued, it should be people like Pat Verhofen.

Indigo Smith, the moderator of the panel, responded that perhaps the true fault lay with the
common investors, who should not have invested in technology stocks in the first place if they
lacked the knowledge to do so properly. While she expressed sympathy for those elderly investors
who lost substantial portions of their retirement savings on flimsy Internet stocks, she observed that
no one forced them to invest in those stocks.

1. Which of the following best describes the structure of the passage?

A. It mentions a puzzling situation, and then describes three approaches people have taken to help
understand that situation.
B. It presents an argument for why something took place, and then offers a refutation of that
argument.
C. It introduces a past phenomenon and then presents three explanations for why the
phenomenon took place.
D. It describes a problem, offers a solution to the problem, and then offers reasons why the solution
could not work.
E. It offers three explanations for a phenomenon and then summarizes what all three have in
common.
2. Which of the following statements presents the strongest conclusion one could draw
based on the information given in the passage?

A. The collapse of the Internet stock “bubble” drove thousands of investors into bankruptcy.
B. People involved with the Internet do not all agree on which party bears the most
responsibility for the collapse of the Internet stock “bubble.”
C. Of all parties involved with the Internet, financial professionals such as investment bankers and
fund managers derived the most profits from the stock “bubble.”
D. The Internet stock “bubble” could not have occurred if entrepreneurs had been honest about the
true financial prospects of their companies.
E. The average investor has no one to blame but himself or herself if he or she invested in an
Internet stock without adequately understanding the true financial prospects of the companies in
question.

The passage provides three opposing viewpoints on this very question, so clearly this is an accurate
statement based on the passage. Answers A and C might be true, but there is inadequate
support for them in the passage. Answers D and E present opinions similar to those of Verhofen
and Smith, respectively, in the passage, but the question does not state that the opinions expressed
in the passage are necessarily true, so these statements cannot be taken as “strong
conclusions.”

3. Which of the following best captures the meaning of the simile attributed to
Mickelson that Verhofen “was like a fox in a henhouse blaming the rooster for all the
dead chickens”?

A. As an entrepreneur, Mickelson understands that similes and other figures of speech can help
convey complex ideas to audiences.
B. Verhofen, as an investment banker, was personally responsible for promoting businesses that he
knew were not viable from a long-term perspective.
C. Foxes, unlike roosters, have no legitimate business in henhouses, and are far more likely than
roosters to kill chickens.
D. As an investment banker, Verhofen was more likely to be the culprit of the crime than
those he identified as responsible.
E. Entrepreneurs cannot be blamed for trying to make money for themselves and other people
because that is what they do.

4. If Mickelson had not used the example of the Wright brothers in her argument, what
other example might have illustrated her point as well?

A. Despite widespread public opinion that the sun revolves around the earth, Galileo Galilei
published findings showing that the earth revolved around the sun; he later retracted this assertion
as a result of pressure from the Church.
B. A tobacco company chose to market cigarettes to children despite widespread public opinion that
such marketing is unethical; over the following decade, the company expanded its share of the
tobacco market.
C. A home electronics company devoted substantial development resources to eight track audio
technology despite widespread industry opinion that cassette tapes were the wave of the future;
eight-tracks were soon replaced by cassette tapes, which in turn were replaced by compact disks.
D. A newspaper chose to publish a story that government lawyers said it could not print; the
newspaper won its case against the government lawyers in a federal court, and the writer of the
story won a Pulitzer Prize.
E. A computer company initiated research into manufacturing a computer for home use
when widespread public opinion held that computers could be useful only for large
corporations or government agencies; personal home computers became a multi billion
dollar market.

5. If Verhofen’s arguments and statements are all correct, which of the following
statements can accurately be inferred?

A. Biotechnology executives who aggressively raise investment capital for


bioengineered products with no conceivable market should be held responsible if
biotechnology stocks crash.
B. Investors should make financial decisions only with the advice of qualified financial advisors, such
as investment bankers or fund managers.
C. If people lose money on investments that they inadequately researched, they have only
themselves to blame.
D. If insurance companies provide home insurance for homes built in a hurricane zone and those
homes are subsequently all destroyed by a major hurricane, the insurance company should be
blamed for any investment losses suffered by its shareholders.
E. The collapse of Internet stocks would not have occurred if companies had not attempted to sell
bulky items, like dog food, over the Internet.

A provides the most similar circumstances to those of Verhofen’s argument, in which Internet
entrepreneurs “who aggressively promoted ideas without viable business models” should
be blamed for the collapse of Internet stocks. B and C have no support in the text, D provides a
different type of example that does not coincide well with Verhofen’s argument, and E overstates
Verhofen’s position, since he identified companies that sell bulky items as one of the causes of
the Internet stock collapse, but not the only cause.

viable (adj) : khả thi

Deceptive (adj) : lừa đảo

Conceivable (adj) : có thể hiểu được

Henhouse (n) : chuồng gà

Culprit (n) : thủ phạm

Comtemporary (n) : người đương thời

Unscrupulous (adj) : vô đạo đức


Feb 08 – 1

Although European decisions during the 16th and 17th centuries to explore, trade with, and colonize
large portions of the world brought tremendous economic wealth and vast geographic influence, the
enormous success of European maritime ventures during the age of exploration also engendered a
litany of unintended consequences for most of the nations with which Europe interacted. Due to
their incredible military force, religious zeal, and uncompromising goal of profit, Europeans often
imposed their traditions, values, and customs on the people with whom they traded. They frequently
acted without regard to the long-term welfare of others as their principal concern was short-term
economic gain. Since many nations that traded with Europe placed high value on their historical
customs, some natives became deeply disconcerted by the changes that occurred as a result of
European power. These factors, coupled with perennial domestic political instability, caused
numerous countries to grow increasingly resistant to European influence.

One potent example of this ideological shift can be seen in the actions of the Tokugawa government
of Japan. In its Seclusion Edict of 1636, the government attempted to extricate cultural interactions
with Europe from the intimate fabric of Japanese society. The Edict attempted to accomplish this by
focusing on three areas. First, it sought to curb cultural exchange by eliminating people bringing
European ideas into Japan. The Edict stated, "Japanese ships shall by no means be sent abroad….All
Japanese residing abroad shall be put to death when they return home." Second, the Edict focused
on limiting trade. Articles 11 through 17 of the Edict imposed stringent regulations on trade and
commerce. Third, the government banned Christianity, which it saw as an import from Europe that
challenged the long-established and well-enshrined religious traditions of Japan. The government
went to considerable lengths to protect its culture. Article eight of the Edict stated, "Even ships shall
not be left untouched in the matter of exterminating Christians."

With the example of Japan and the examples of other countries that chose a different response to
European influence, it is perhaps not too far of a stretch to conclude that Japan made the right
decision in pursuing a path of relative isolationism. As history unfolded during the next 400 years, in
general, countries that embraced European hegemony, whether by choice or by force, tended to
suffer from pernicious wealth inequality, perennial political instability, and protracted
underdevelopment.

1. It can best be inferred from the passage that in 1636, the Japanese government:

A) Saw its citizens living abroad as potential threats


B) Considered all foreign religions a danger
C) Disagreed with the European philosophy that trade brought wealth
D) Foresaw the economic dangers of European trade and imperialism
E) Believed that ideas coming into Japan via foreign interactions provided no positive impact to
Japanese society

The phrase "provided no positive impact" is a strong statement. Although the article is clear that
Japan felt that the European influence damaged its culture, this is not enough evidence to claim that
absolutely nothing positive came from European "ideas".
2. Which of the following best characterizes the most significant motivation for Europe's
behavior with Japan during the 17th century?

A) Religious zeal
B) Long-term political concerns
C) Short-term economic self-interest
D) Cultural imperialism
E) Territorial aggrandizemen

3. The author most likely included the quotation from Article Eight of the Edict at the
end of the second paragraph to:

A) Highlight the venomous anger many Japanese leaders felt toward the importation of foreign
religions
B) Emphasize the determination of the Japanese government to protect itself from
foreign influences it saw as damaging
C) Illustrate how pervasive foreign religious influence had become in Japanese society
D) Emphasize that European economic influence offered no justification for the Edict and the
government relied instead on foreign religious influence to justify the Edict
E) Provide an example of Japan's effort to curb cultural and economic exchange

4. Based upon the passage, the author would likely agree most strongly with which of
the following statements:

A) European decisions made during the 16th and 17th centuries in dealing with Japan represent an
aberration from the typical pattern of European decisions
B) Japanese rulers who responded with ferocity to European influence bear part of the responsibility
for the caustic European-Japanese relationship that ensued
C) With the hindsight of history, Japan likely made the appropriate decision in
extricating itself from European influence
D) European religious and cultural values conflicted with European economic behavior toward Japan
E) The width and breadth of Japan's cultural fabric suffered from its seclusionist policies

5. According to the passage, which of the following constituted the biggest reason for
the Seclusion Edict of 1636?

A) Japanese economic potential would be hampered in the long-term

There is no mention of Japan's self-awareness regarding the long-term economic consequences of


European actions.

B) European trade amounted to a disproportionate transfer of wealth


C) With growing European influence, the potential for European military action against the Japanese
government became too large
D) Traditional Japanese culture and way of life were threatened by European influence
E) Japanese rulers feared the arrival of additional traders and cultural imperialists
6. According to the passage, the Japanese government took all of the following actions
in an attempt to protect Japanese culture and way of life EXCEPT:

A) Prohibit Japanese from visiting other countries, even to see family


B) Execute Japanese citizens who settled in other countries but later decided to return to Japan
C) Heavily regulate foreign economic trade
D) Destroy all remnants of Christianity
E) Prohibit criticism of the feudal shogun system of government

7. The primary purpose of the passage is to:

A) Explain the actions of the Tokugawa government of Japan


B) Compare the results of countries that pursued protectionism with those that pursued globalization
C) Explore the consequences of some European trade and exploration along with
analyzing a country’s response to it
D) Argue for the success of European trade as a means to create wealth and exert influence
E) Elucidate the root of frustration with European imperialism

Engender (v) : gây ra

Litany of (n) : kiện

Unintended (adj) : ngoài ý muốn

Zeal (n) : lòng nhiệt thành

Uncompromising (adj) : không khoan nhượng

Historical custom (n) : phong tục lịch sử

Disconcert (v) : bối rối

Perennial (adj) : lâu năm

Extricate (v) : giải thoát, thoát khỏi

Intimate (adj) : thân mật

Fabric (n) : cách cấu tạo

Curb (v) : ngăn lại

Enshrine (v) : lưu trữ, bảo tồn

Isolationism (n) : cô lập

Unfold (v) : mở ra

Embrace (v) : bao quanh, bao trùm, siết chặt

Hegemony (n) : quyền lãnh đạo

Pernicious (adj) : nguy hiểm

Protract (v) : kéo dài


Elucidate (v) : làm rõ

Frustration (n) : sự thất vọng, chống đối

Imperialism (n) : chủ nghĩa đế quốc

Feudal (adj) : phong kiến

Shogun (n) : tướng quân

Regulate (v) : điều tiết


Feb 08 – 2

Should we really care for the greatest actors of the past could we have them before us? Should we
find them too different from our accent of thought, of feeling, of speech, in a thousand minute
particulars which are of the essence of all three? Dr. Doran's long and interesting records of the
triumphs of Garrick, and other less familiar, but in their day hardly less astonishing, players, do not
relieve one of the doubt. Garrick himself, as sometimes happens with people who have been the
subject of much anecdote and other conversation, here as elsewhere, bears no very distinct
figure. One hardly sees the wood for the trees. On the other hand, the account of Betterton,
"perhaps the greatest of English actors," is delightfully fresh. That intimate friend of Dryden,
Tillotson, Pope, who executed a copy of the actor's portrait by Kneller which is still extant, was
worthy of their friendship; his career brings out the best elements in stage life.

The stage in these volumes presents itself indeed not merely as a mirror of life, but as an illustration
of the utmost intensity of life, in the fortunes and characters of the players. Ups and downs,
generosity, dark fates, the most delicate goodness, have nowhere been more prominent than in the
private existence of those devoted to the public mimicry of men and women. Contact with the stage,
almost throughout its history, presents itself as a kind of touchstone, to bring out the bizarrerie, the
theatrical tricks and contrasts, of the actual world.

1. In the expression One hardly sees the wood for the trees, the author apparently
intends the word trees to be analogous to

A. features of Dorans language style


B. details learned from oral sources
C. personality of a famous actor
D. details of Garricks life
E. stage triumphs of an astonishing player

The wood refers to the bigger picture, the trees to the details. One apparently does not get a
picture of Garrick the man, but one does get along and interesting record of his triumphs. We are
also told that Garrick has been the subject of much conversation and anecdote. Hence the trees
refers to the details of Garricks life learned mainly from oral sources.

2. The "doubt" referred to in the passage concerns whether

A. the stage personalities of the past would appeal on a personal level to people like the
author
B. their contemporaries would have understood famous actors
C. the acting of famous stage personalities would appeal to us today
D. Garrick was as great as he is portrayed
E. historical records can reveal personality

Should we care for the greatest actors means should we like them. The author goes on to ask
whether we would find their ways and ideas too different from our own. These are the doubts that
he raises. The author is not really concerned whether we would like their acting. Hence, A is the
best answer.
3. Information supplied in the passage is sufficient to answer which of the following
questions?
I Who did Doran think was probably the best English actor?
II What did Doran think of Garrick?
III Would the author give a definite answer to the first question posed in the passage?
A. I only
B. II only
C. I and III only
D. II and III only
E. I, II and III

The quotation marks around "perhaps the greatest of English actors," tell us that the author is
quoting from the book he is reviewing, and hence the author of that book, Doran, thinks Betterton
was probably the best.

Doran writes long and interesting records of the triumphs of Garrick but we cannot infer Dorans
opinion of the actor from that. The author would not give a definite answer to the question because
he says the writings of Doran do not relieve one of the doubt.

We can answer question I with the word Betterton and question III with the word no.

Essence (n) : bản chất

Bizarrerie (n) : kỳ quái

Theatrical trick (n) : thủ thuật sân khấu


Feb 09 – 1

One clear advantage of selling goods online is that smaller markets can be served without the seller
needing to invest in heavy inventory costs. Recordings of classical music, for example, are
increasingly hard to find at the larger music chains, where only a handful of recordings sell well
enough to make the inventory costs and use of shelf space worthwhile, but the afcionado can now
locate nearly any classical CD in print on the Internet. In addition, forward-thinking artists without
national reputations have made their music available on personal sites or through services that
provide the musicians a more generous share of profits than that offered by the large record labels.
For some, this has resulted in increased sales volume, greater return on investment, greater control
of product, and a more direct connection with an eager market base. Major orchestras and record
labels have taken note, and have created Web sites where one can purchase individual tracks, full
CDs, archival recordings, and even music exclusively made available for online downloads. Some
symphony orchestras now include, with the price of admission to a concert, the right to download a
recording of the concert afterward. Other services allow the listener unlimited streaming or
downloading for a monthly fee. The question remains as to whether classical music will remain at
the periphery of the online market, just as it is in the big retail chains. The short answer is most
likely yes; classical music is now just one more niche market, albeit one with a particularly long and
distinguished past. Cultural factors—most notably television—have been at work for some time now,
creating a limited audience for music that requires sustained critical listening. However, the digital
economy has ensured two important factors. First, for those with an interest, an impressively wide
range of classical music will be available for some time to come. Second, motivated new artists can
earn enough to continue to produce new recordings.

1. The primary purpose of the passage is to _______.

(A) demonstrate that classical music, though it is part of a culture that is antithetical to its existence,
will exist for some time to come
(B) acknowledge that classical music sales are unlikely to capture a significantly larger part of the
market share of musical recordings, despite the beneficial effect of online sales
(C) discuss the many ways that the Internet has contributed, and will contribute, to sales of smaller
markets
(D) use the effect that online sales has had – and will have – on classical music sales as
an example of how the digital economy can change smaller markets
(E) show how a few motivated artists have revolutionized a feld that was in danger of becoming
extinct
2. It can be inferred from the passage that the author believes that _______.

(A) classical music recordings available exclusively online are equal to those available in stores
(B) the large music chains have not done enough to promote the sales of classical music recordings
(C) the enjoyment of popular music does not necessitate long periods of intense
listening
(D) any classical musician who wishes to increase sales of his or her recordings should set up a
personal Web site
(E) the digital economy has had a mixed effect on classical music sales

The second paragraph states "Cultural Factors--most notably television--have been at work
for some time now, creating a limited audience for music that requires sustained critical
listening " This implies that non-classical (popular) music doesn't require sustained critical
listening. (A) This comparison is never made. (B) the author does not imply that the chains have
not done enough. (D): This is too strong an inference. It might not be true for all classical
musicians. (E) the passage only mensions positive effects.

3. Which of the following is NOT an example of the effect that the digital economy has
had on the selling of classical music, as described in the passage?

(A) Consumers are able to purchase CDs that are unavailable in stores.
(B) Artists find it easier to send promotional information to people who have expressed an interest in
the artists’ music.
(C) A major record label offers its new artists a greater share of profits than in the past.
(D) Listeners download music without having to concern themselves with the cost of each download.
(E) A new artist sells enough copies of a frst CD to pay for the production of a second.

Refer back to the passage for this Specific question. No where in the passage does the author state
that the major labels are offering a greater share of profits. (A): The 1st paragraph states that "the
aficionado can now locate nearly any classical CD in print on the Internet." (B): The 1st
paragraph states that artists can now have "more direct connection with an eager
market base." (D): The last sentence of the 1st paragraph states that "Other services allow
the listener unlimited streaming or downloading for a monthly fee." Since the fee is
monthly, the listener does not have to worry about the cost of individual downloads. (E): The final
sentence of the passage states that "Second, motivated new artists can earn enough to
continue to produce new recordings."
4. It can be inferred from the passage that one factor that has led to the marginalizing
of classical music in retail stores is that _______.

(A) classical music must compete with a wide range of other niche market recordings
(B) physical space is needed for better-selling recordings
(C) major orchestras have had a difficult time selling tickets to live performances
(D) inventory costs of classical music recordings tend to be higher than those associated with
popular music
(E) recordings are displayed in a way that is disadvantageous to the consumer who is interested in a
recording that is not a big seller

This "Inference" question is actually answered directly in the passage. The 1st paragraph states
that "only a handful of recordings sell well enough to make the inventory costs and use
of shelf space worthwhile ..." (A): This niche-against-niche competition is never
mentioned. (C): The passage never states that this is so. (D): This comparison is never
made. (E): How the recordings are displayed is never mentioned.

Necessitate (v) : bắt buộc


Feb 09 – 2

Instead of casting aside traditional values, the Meiji Restoration of 1868 dismantled feudalism and
modernized the country while preserving certain traditions as the foundations for a modern Japan.
The oldest tradition and basis of the entire Japanese value system was respect for and even worship
of the Emperor. During the early centuries of Japanese history, the Shinto cult in which the imperial
family traced its ancestry to the Sun Goddess became the people’s sustaining faith. Although later
subordinated to imported Buddhism and Confucianism, Shintoism was perpetuated in Ise and Izumo
until the Meiji modernizers established it as a quasi state religion.

Another enduring tradition was the hierarchical system of social relations based on feudalism and
reinforced by Neo Confucianism which had been the official ideology of the premodern world.
Confucianism prescribed a pattern of ethical conduct between groups of people within a fixed
hierarchy. Four of the five Confucian relationships were vertical, requiring loyalty and obedience
from the inferior toward the superior. Only the relationship between friend and friend was
horizontal, and even there the emphasis was on reciprocal duties.

1 The author is primarily concerned with

(A) providing a history of the rise of feudalism in Japan


(B) identifying the influences of Confucianism on Japanese society
(C) speculating on the probable development of Japanese society
(D) developing a history of religion in Japan
(E) describing some important features of the Meiji Restoration

2 The passage mentions all of the following as being elements of Japanese society
EXCEPT:

(A) obedience to authority


(B) sense of duty
(C) respect for the Emperor
(D) concern for education
(E) loyalty to one’s superior

3 It can be inferred from the passage that those who led Japan into the modern age
were concerned primarily with

(A) maintaining a stable society


(B) building a new industrial base
(C) expanding the nation’s territory
(D) gaining new adherents of Confucianism
(E) creating a new middle class
Cast aside (v) : bỏ ngoài tai, gạt sang một bên

Restoration (n) : phục hồi

Dismantle (v) : tháo dỡ

Feudalism (n) : chế độ phong kiến

Worship (v) : tôn sùng

Emperor (n) : hoàng đế

Imperial (adj) : hoàng gia

Cult in (n) : sùng bái, cúng tế, thờ

Subordinate (v) : hạ xuống cấp dưới

Buddhism (n) : phật giáo

Confucianism (n) : nho giáo

Shintoism (n) : thần đạo

Perpetuate (v) : kéo dài

Quasi (adj) : gần như

Enduring (adj) : bền vững

Hierarchical (adj) : thứ bậc

Reinforce (v) : gia cố

Hierarchy (n) : hệ thống cấp bậc

Reciprocal (adj) : hỗ tương, đối ứng

Adherent (n) : tín đồ


Feb 10 – 1

Reverse discrimination, minority recruitment, racial quotas, and, more generally, affirmative action
are phrases that carry powerful emotional charges. But why should affirmative action, of all
government policies, be so controversial? In a sense, affirmative action is like other government
programs, e.g., defense, conservation, and public schools. Affirmative action programs are designed
to achieve legitimate government objectives such as improved economic efficiency, reduced social
tension, and general betterment of the public welfare. While it cannot be denied that there is no
guarantee that affirmative action will achieve these results, neither can it be denied that there are
plausible, even powerful, sociological and economic arguments pointing to its likely success.
Government programs, however, entail a cost; i.e., the expenditure of social or economic resources.
Setting aside cases in which the specific user is charged a fee for service (toll roads and tuition at
state institutions), the burdens and benefits of publicly funded or mandated programs are widely
shared. When an individual benefits personally from a government program, it is only because she
or he is one member of a larger beneficiary class, e.g., a farmer; and most government revenue is
obtained through a scheme of general taxation to which all are subject.

Affirmative action programs are exceptions to this general rule, though not, as it might at first seem,
because the beneficiaries of the programs are specific individuals. It is still the case that those who
ultimately benefit from affirmative action do so only by virtue of their status as a member of a larger
group, a particular minority. Rather the difference is the location of the burden. In affirmative
action, the burden of “funding” the program is not shared universally, and that is inherent in the
nature of the case, as can be seen clearly in the case of affirmative action in employment. Often job
promotions are allocated along a single dimension— seniority. When an employer promotes a less
senior worker from a minority group, the person disadvantaged by the move is easily identified: the
worker with greatest seniority on a combined minority-non minority list passed over for promotion.

Now we are confronted with two competing moral sentiments. On the one hand, there is the idea
that those who have been unfairly disadvantaged by past discriminatory practices are entitled to
some kind of assistance. On the other, there is the feeling that no person ought to be deprived of
what is rightfully his, even for the worthwhile service of his fellow humans. In this respect, disability
due to past racial discrimination, at least in so far as there is no connection to the passed-over
worker, is like a natural evil. When a villainous man willfully and without provocation strikes and
injures another, there is not only the feeling that the injured person ought to be compensated but
there is also consensus that the appropriate party to bear the cost is the one who inflicted the
injury. Yet, if the same innocent man stumbled and injured himself, it would be surprising to hear
someone argue that the villainous man ought to be taxed for the injury simply because he might
have tripped the victim had he been given the opportunity. There may very well be agreement that
the victim should be aided in his recovery with money and personal assistance, and many will give
willingly, but there is also agreement that no one individual ought to be singled out and forced to do
what must ultimately be considered an act of charity.
1. The passage is primarily concerned with

(A) comparing affirmative action programs to other government programs


(B) arguing that affirmative action programs are morally justified
(C) analyzing the basis for moral judgments about affirmative action programs
(D) introducing the reader to the importance of affirmative action as a social issue
(E) describing the benefits that can be obtained through affirmative action programs

2. The author mentions toll roads and tuition at state institutions in order to

(A) anticipate a possible objection based on counterexamples


(B) avoid a contradiction between moral sentiments
(C) provide illustrations of common government programs
(D) voice doubts about the social and economic value of affirmative action
(E) offer examples of government programs that are too costly

This is a logical structure question. In the second paragraph the author will describe the general
structure of government programs in order to set up the contrast with affirmative action. The
discussion begins with “Setting aside . . . ,” indicating that the author recognizes such cases and
does not wish to discuss them in detail. Tolls and tuition are exceptions to the general rule, so the
author explicitly sets them aside in order to preempt a possible objection to his analysis based on
claimed counterexamples. (B) is incorrect since the overall point of the passage is to discuss this
dilemma, but the main point of the passage will not answer the question about the logical
substructure of the argument. (C) is incorrect since tolls and tuition are not ordinary government
programs. (D) is incorrect since the author never raises such doubts. Finally, (E) misses the
point of the examples. The point is not that they are costly but that the cost is born by the
specific user.

3. With which of the following statements would the author most likely agree?

(A) Affirmative action programs should be discontinued because they place an unfair burden on non
minority persons who bear the cost of the programs.
(B) Affirmative action programs may be able to achieve legitimate social and economic
goals such as improved efficiency.
(C) Affirmative action programs are justified because they are the only way of correcting injustices
created by past discrimination.
(D) Affirmative action programs must be redesigned so that society as a whole, rather than
particular individuals, bears the cost of the programs.
(E) Affirmative action programs should be abandoned because they serve no useful social function
and place unfair burdens on particular individuals.
4. The author most likely places the word “funding” in quotation marks in order to
remind the reader that

(A) affirmative action programs are costly in terms of government revenues


(B) particular individuals may bear a disproportionate share of the burden of affirmative action
(C) the cost of most government programs is shared by society at large
(D) the beneficiaries of affirmative action are members of larger groups
(E) the cost of affirmative action is not only a monetary expenditure

In paragraph two the author mentions that government programs entail both social and economic
costs. Then, the cost of a specific example, the passed-over worker, is not a government
expenditure in the sense that money is laid out to purchase something. So the author is using the
term “funding” in a nonstandard way, and he wishes to call his readers’ attention to this. (E)
parallels this explanation. (A) is incorrect since it is inconsistent with the reasoning just
provided. (B) is incorrect, for though the author may believe that individuals bear a
disproportionate share of the burden, this is not a response to the question asked. (C) is
incorrect for the same reason: It is a true but non responsive statement. Finally, (D) fails for
the same reason. Though the author notes that affirmative action programs are similar to other
government programs in this respect, this is not an explanation for the author’s placing “funding” in
quotation marks.

5. The “villainous man” introduced at line functions primarily as a(n)

(A) illustration
(B) counterexample
(C) authority
(D) analogy
(E) disclaimer

This is a logical structure question. In the final paragraph, the author analyzes another similar
situation. This technique is called “arguing from analogy.” The strength of the argument depends
on our seeing the similarity and accepting the conclusion of the one argument (the villainous man)
as applicable to the other argument (affirmative action). (A) is perhaps the second-best
response, but the author is not offering an illustration, e.g., an example of affirmative action. To be
sure, the author is attempting to prove a point, but attempting to prove a conclusion is not
equivalent to illustrating a contention. (B) is incorrect since the author adduces the situation to
support his contention. (C) is incorrect, for the author cites no authority. Finally, (E) can be
eliminated since the author uses the case of the villanious man to support, not to weaken, the
case.

6. According to the passage, affirmative action programs are different from most other
government programs in the
(A) legitimacy of the goals the programs are designed to achieve
(B) ways in which costs of the programs are distributed
(C) methods for allocating the benefits of the programs
(D) legal structures that are enacted to achieve the objectives
(E) discretion granted to the executive for implementing the programs
Quota (n) : hạn ngạch – a limited or fixed number or amount of people or things, in particular

Affirmative (adj) : quả quyết, khẳng định

Phrase (n) : cụm từ

Tension (n) : căng thẳng

Betterment (n) : sửa cho tốt hơn

Plausible (adj) : hợp lý

Sociological (adj) : xã hội học

Beneficiary (n) : thụ hưởng, có quyền lợi

Toll (n) : thu phí

Obtain through (v) : thông qua

Scheme (n) : chương trình, kế hoạch

Taxation (n) : sự đánh thuế

Virtue (n) : đức hạnh

Inherent (adj) : vốn có, chỉ định, thuộc về

Burden (n) : gánh nặng

Seniority (n) : thâm niêm

Sentiment (n) : tình cảm, ý kiến

Deprive (v) : tước bỏ

Villainous (adj) : phản diện

Willfully (adv) : cố ý

Provocation (n) : khiêu khích

Consensus (n) : đồng thuận

Stumble (v) : vấp ngã

Trip (v) : quèo chân người nào, đi bước nhẹ

Anticipate (v) : dự đoán

Disclaimer (n) : từ chối trách nhiệm


Feb 10 – 2

In the United States the per capita costs of schooling have risen almost as fast as the cost of
medical treatment. But increased treatment by both doctors and teachers has shown steadily
declining results. Medical expenses concentrated on those above forty-five have doubled several
times over a period of forty years with a resulting 3 percent increase in the life expectancy of men.
The increase in educational expenditures has produced even stranger results; otherwise President
Nixon could not have been moved this spring to promise that every child shall soon have the “Right
to Read” before leaving school.

In the United States it would take eighty billion dollars per year to provide what educators regard as
equal treatment for all in grammar and high school. This is well over twice the $36 billion now being
spent. Independent cost projections prepared at HEW and at the University of Florida indicate that
by 1974 the comparable figures will be $107 billion as against the $45 billion now projected, and
these figures wholly omit the enormous costs of what is called “higher education,” for which demand
is growing even faster. The United States, which spent nearly eighty billion dollars in 1969 for
“defense,” including its deployment in Vietnam, is obviously too poor to provide equal schooling. The
President’s committee for the study of school finance should ask not how to support or how to trim
such increasing costs, but how they can be avoided.

Equal obligatory schooling must be recognized as at least economically unfeasible. In Latin America
the amount of public money spent on each graduate student is between 350 and 1,500 times the
amount spent on the median citizen (that is, the citizen who holds the middle ground between the
poorest and the richest). In the United States the discrepancy is smaller, but the discrimination is
keener. The richest parents, some 10 percent, can afford private education for their children and
help them to benefit from foundation grants. But in addition they obtain ten times the per capita
amount of public funds if this is compared with the per capita expenditure made on the children of
the 10 percent who are poorest. The principal reasons for this are that rich children stay longer in
school, that a year in a university is disproportionately more expensive than a year in high school,
and that most private universities depend—at least indirectly—on tax-derived finances.

Obligatory schooling inevitably polarizes a society; it also grades the nations of the world according
to an international caste system. Countries are rated like castes whose educational dignity is
determined by the average years of schooling of its citizens, a rating which is closely related to per
capita gross national product, and much more painful.

1. Which one of the following best expresses the main idea of the passage?

The answer to a main idea question will summarize the passage, without going beyond it.

(A) The educational shortcomings of the United States, in contrast to those of Latin America, are
merely the result of poor allocation of available resources.
(B) Both education and medical care are severely underfunded.
(C) Defense spending is sapping funds which would be better spent in education.
(D) Obligatory schooling must be scrapped if the goal of educational equality is to be realized.
(E) Obligatory education does not and cannot provide equal education.
2. The author most likely would agree with which one of the following solutions to the
problems presented by obligatory education?

(A) Education should not be obligatory at all.


(B) Education should not be obligatory for those who cannot afford it.
(C) More money should be diverted to education for the poorest.
(D) Countries should cooperate to establish common minimal educational standards.
(E) Future spending should be capped.

3. According to the passage, education is like health care in all of the following ways
EXCEPT:

(A) It has reached a point of diminishing returns, increased spending no longer results in significant
improvement.
(B) It has an inappropriate “more is better” philosophy.
(C) It is unfairly distributed between rich and poor.
(D) The amount of money being spent on older students is increasing.
(E) Its cost has increased nearly as fast.

This is a description question, so we must find the place from which it is drawn. It is the first
paragraph. The sentence “But increased treatment by both doctors and teachers has shown steadily
declining results” shows that both have reached a point of diminishing returns. This eliminates (A)
and (B). Next, the passage states “Medical expenses concentrated on those above forty-five have
doubled several times” and that the demand and costs of higher education are growing faster than
the demand and costs of elementary and high school education. This eliminates (D). Next, the
opening to the passage states that the costs of education “have risen almost as fast as the cost of
medical treatment.” This eliminates (E). Hence, by process of elimination, the answer is (C). We
should, however, verify this. In paragraph three, the author does state that there is a “keen”
discrepancy in the funding of education between rich and poor, but a survey of the passage shows
that at no point does he mention that this is also the case with health care.

5. Which one of the following most accurately characterizes the author’s attitude with
respect to obligatory schooling?

(A) qualified admiration


(B) critical
(C) neutral
(D) ambivalent
(E) resentful
4. Why does the author consider the results from increased educational expenditures to
be “even stranger” than those from increased medical expenditures?
(A) The aging of the population should have had an impact only on medical care, not on education.
(B) The “Right to Read” should be a bare minimum, not a Presidential ideal.

The implication of President Nixon’s promise is that despite increased educational funding many
children cannot even read when they graduate from school.
(C) Educational spending has shown even poorer results than spending on health care, despite
greater increases.

Although at first glance this is a tempting inference, it would be difficult to compare the results of
education and medical care directly (how would we do so?). Regardless, the opening line to the
passage states that educational costs have risen “almost as fast” as medical costs, not faster.
(D) Education has become even more discriminatory than health care.
(E) It inevitably polarizes society.

This is an extension question. We are asked to interpret a statement by the author. The needed
reference is the closing sentence to paragraph one. Remember: extension questions require you to
go beyond the passage, so the answer won’t be explicitly stated in the reference—we will have to
interpret it.

6. By stating “In Latin America the amount of public money spent on each graduate
student is between 350 and 1,500 times the amount spent on the median citizen” and
“In the United States the discrepancy is smaller” the author implies that
(A) equal education is possible in the United States but not in Latin America.

because the import of the passage is that the rich get better schooling and more public funds in the
United States and therefore discrimination is “keener”.
(B) equal education for all at the graduate level is an unrealistic ideal.
(C) educational spending is more efficient in the United States.
(D) higher education is more expensive than lower education both in Latin America and in the
United States, but more so in Latin America.
(E) underfunding of lower education is a world-wide problem.

Infeasible (adj) : không khả thi

Keener (adj) : sâu sắc hơn

Inevitably (adv) : rõ ràng

Polarize (v) : phân cực

Caste (n) : đẳng cấp

Obligatory (adj) : bắt buộc

Neutral (adj) : trung tính

Ambivalent (adj) : lẫn lộn, trái ngược

Resentful (adj) : bực bội, xúc phạm

Cap (v) : giới hạn


Feb 11 – 1

The principle of selection solved the riddle as to how what was purposive could conceivably be
brought about without the intervention of a directing power, the riddle which animate nature
presents to our intelligence at every turn, and in face of which the mind of a Kant could find no way
out, for he regarded a solution of it as not to be hoped for. For, even if we were to assume an
evolutionary force that is continually transforming the most primitive and the simplest forms of life
into ever higher forms, and the homogeneity of primitive times into the infinite variety of the
present, we should still be unable to infer from this alone how each of the numberless forms
adapted to particular conditions of life should have appeared precisely at the right moment in the
history of the earth to which their adaptations were appropriate, and precisely at the proper place in
which all the conditions of life to which they were adapted occurred: the humming-birds at the same
time as the flowers; the trichina at the same time as the pig; the bark-coloured moth at the same
time as the oak, and the wasp-like moth at the same time as the wasp which protects it. Without
processes of selection we should be obliged to assume a "pre-established harmony" after the
famous Leibnizian model, by means of which the clock of the evolution of organisms is so regulated
as to strike in exact synchronism with that of the history of the earth!

All forms of life are strictly adapted to the conditions of their life, and can persist under these
conditions alone. There must therefore be an intrinsic connection between the conditions and the
structural adaptations of the organism, and, since the conditions of life cannot be determined by the
animal itself, the adaptations must be called forth by the conditions. The selection theory teaches us
how this is conceivable, since it enables us to understand that there is a continual production of
what is non-purposive as well as of what is purposive, but the purposive alone survives, while the
non-purposive perishes in the very act of arising. This is the old wisdom taught long ago by
Empedocles.

1. It can be inferred that the author believes that the Leibnitzian model (Highlighted) is

A. ingenious and worthy of serious consideration


B. untenable by all rational people

The word all in B is the best reason for rejecting this answer.
C. an acceptable solution to Kants dilemma
D. unworthy of further consideration
E. an alternative that might still be valid

2. The authors primary purpose in this extract is to

A. suggest that a particular theory explains otherwise puzzling phenomena


B. describe the details of the selection theory for a lay audience
C. justify a particularly controversial model of the origins of life
D. persuade the reader that Empedocles was right
E. prove that selection is the only possible way of looking at evolutionary biology

The author intends to persuade us that selection theory is the answer to the riddle he mentions in
the first sentence (suggesting immediately that A is correct). He does not actually go into
details (eliminate B), or prove that anything is correct (eliminate E). The word particularly
makes C incorrect and D is not even worth considering.
3. The examples in lines 15 - 17 (Highlighted) are intended to
A. reinforce the authors point that is difficult to explain adaptation
B. show that adaptations must take place only at specific times and in specific places

The words must and only in B make that wrong


C. give specific illustration of organisms that are particularly well-adapted to their
conditions

The author mentions specific examples to make his point clear that some forms of life
have arisen with adaptations that fit them very well for one way of life.
D. show organisms that have evolved synchronously in a predestined manner
E. demonstrate that intelligent design is needed for purposive evolution

For, even if we were to assume an evolutionary force………, we should still be unable to


infer … (the author implies that this phenomenon can only being explained by the
selection theory) A must be incorrect

Riddle (n) : câu đố

Purposive (adj) : có mục đích, chủ tâm

Conceivably (adv) : có thể hình dung

Intervention (n) : can thiệp

Animate (v) : to make someone seem more happy or active:

to cause someone or something to be more active or full of life

evolutionary (adj) : tiến hóa

primitive (adj/n) : nguyên thủy / buổi sơ khai, thời kỳ nguyên thủy

homogeneity (n) : đồng nhất

precisely (adv) : đúng

numberless (adj) : vô số

synchronism (n) : tính đồng bộ

persist (v) : kiên trì

harmony (n) : hòa hợp

intrinsic (adj) : nội tại, thuộc về bản chất

perish (v) : diệt vong

the very act of arising (n) : hành vi phát sinh

ingenious (adj) : khéo léo, tinh xảo

untenable (adj) : không thể bảo vệ, không thể sử dụng


Feb 11 – 2

Most Americans are fascinated with their own history, particularly that of the colonial era. Whenever
a crisis affects the current government, pundits and plebeians alike invoke the writings and
teachings of the eighteenth century in order to support or denounce modern viewpoints. Many
citizens wax nostalgic for the glorious times of their nascent union, when some of the most shrewd
and free-thinking minds came together to construct "a more perfect Union.“ A new book by Nathan
Parker, however. suggests that colonial New England was never the egalitarian Eden that modern
Americans make it out to be.

Popular imagination holds that the Puritans were a virtuous group determined to create a new
government through direct democracy. Communities convened town hall meetings, at which policies
were debated and decisions were made by the will of the people. Many renowned international
thinkers, such as France's Alexis de Tocqueville and Hector St. John Crevecoeur, praised this new
American commitment to the voice of the common man. According to Parker, admiration for New
England's first settlers is profoundly miSplaced. In a typical show of their historical revisionism.
Americans have mythologized these town meetings to the point of embarrassment. Parker asserts
that town-hall meetings were open only to a select few male property owners who wielded a strong
financial influence on the community. Thus, the laws that were put into effect as a result of these
meetings hardly an reflected the "consent of the governed". Therefore, there is vast evidence of
voter apathy among the colonists.

Citing the disparity between the roll calls of several meetings and the voter registries of the towns in
which they took place. Parker demonstrates that attendance atm town-hall meetings rarely
exceeded 30 percent of all registered voters. In making these points, Parker hopes to lie to rest the
notion that simple, family-oriented colonial New England was far preferable to the modern America
that many an perceive has outgrown pure democracy. His objective is to lay bare the true nature of
eighteenth-century governance and thus assure Americans that progress can't kill an equality that
never was.

1. The primary purpose of the passage as a whole is to

A. critique a system of logic


B. clarify an ambiguity
C. contrast two diverse notions
D. discredit a commonly held perception
E. question a dubious explanation
2. Which of the following best describes the purpose of the second paragraph?

A. It extols the virtue of the first American settlers who were not daunted by the prospect of
creating an egalitarian society.
B. It first introduces de Tocqueville and Crevecoeur, by whom Parker was first inspired to write.
C. It lists several perceptions about the early American colonies that Parker believes to
be more myth than fact.
D. It serves to emphasize the massive impact that French thinking had on New England's first
settlers.
E. It provides evidence that the Puritans were not nearly as virtuous as they asserted themselves to
be.

3. According to the passage. it is common practice to refer to the colonial era in order to

A. advocate the importance of pure democracy to a fledgling capitalist nation


B. illustrate how American society has always depended upon the family unit that was so highly
esteemed in the eighteenth century
C. establish a historical context for the celebrated writings of de Tocqueville and Crevecoeur
D. praise the perseverance of the Puritans, who never receive the recognition they so richly deserve
E. indicate that the country's rampant growth since its creation has caused it to stray
from its original path toward absolute democracy

4. According to the passage, Parker asserts which of the following about early colonial
town meetings?

A. Those who owned property in the area served as representatives for everyone in the community.
B. Attendance at these meetings was restricted to wealthy landowners.
C. All registered voters were permitted to attend, but fewer than one-third of them actually did.
D. They became the inspiration for what is known today as direct democracy.
E. They were looked upon as models by the framers of the Constitution.

B.Parker asserts that town-hall meetings were open only to a select few male property owners
who wielded a strong financial influence on the community.

C.Parker demonstrates that attendance atm town-hall meetings rarely exceeded 30 percent of all
registered voters.  Incorrect

Fascinate (v) : mê hoặc

Colonial era (adj/n) : kỷ nguyên thuộc địa

Pundit (n) : học giả

Invoke (v) : gọi, cầu nguyện

Plebeian (n) : thường dân

Denounce (v) : tố cáo, phản đối, sỉ nhục

Wax (v) : đánh bóng

Nostalgic (adj) : hoài niệm


Nascent (adj) : mời thành lập, non trẻ

Shrewd (adj) : thông minh

Egalitarian (adj) : bình đẳng

Puritan (n) : thanh giao – a member of a group of English Protestants of the late 16th and 17th
centuries who regarded the Reformation of the Church of England under Elizabeth as incomplete
and sought to simplify and regulate forms of wordship

Reformation (n) : cải cách

Protestant (n) : người phản đối

Regulate (v) : điều tiết, chỉnh đốn, sắp đặt

Virtuous (adj) : đức hạnh

Convent (v) : triệu tập

Renowned (adj) : nổi tiếng

Praise (v) : ca ngợi

Profoundly (adv) : sâu sắc

Revisionism (n) : chủ nghĩa xét lại

Mythologize (v) : thần thoại hóa

Embarrassment (n) : sự khó chịu, sự lúng túng

Wield (v) : mang lại

Consent (n) : sự bằng lòng, sự tán thành

The governed (n) : chính phủ

Apathy (n) : thờ ơ, lãnh đạm

Roll call (n) : điểm danh

Governance (n) : sự cai trị

Lay (v) : làm rạp xuống, trình ra

Assure (v) : cam đoan

Progress (n) : sự tiến bộ

Common practice (n) : thực tế phổ biến

Advocate (v) : biện hộ, bênh vực

Extol (v) : trích xuất, khen ngợi

Rampant (adj) : hung hăng, cuồng bạo, phóng túng, không bị kiềm chế

Stray from (v) : đi lạc


Feb 12 – 1

The origins of tea as a beverage can be traced back more than 5,000 years. Chinese mythology first
addresses the drink in 3000 BC, when the emperor Nin Song was said to have discovered it. Nin
Song was something of a visionary. Among other innovations, he believed that water should be
boiled before drinking as a health precaution. As the story goes, he was traveling with some
members of the court when they stopped to rest. Some leaves from a bush fell into the water being
boiled for the weary travelers, and thus was tea born. In AD 800, a man named Lu Yu wrote the
first known book on tea cultivation and preparation. The work, called the Ch’a Ching, melded Zen
Buddhist teachings with the art and craft of tea, forever linking the drink to spirituality.

In AD 1191, the cultivation and brewing of the leaves spread to Japan when a monk named Yeisei
returned from pilgrimage, bringing seeds back with him. Yeisei had observed tea being used in and
enhancing meditation and spiritual awareness in China. He shared this discovery with his peers and
the tradition quickly caught on—all the way to the highest levels of society, including the imperial
court. Tea was so well-received in Japan that it was elevated to an art form, culminating in the
creation of the well-known Japanese Tea Ceremony. The ceremony evolved and grew both more
intricate and more exclusive, with students of the art receiving years of practice and training before
they were allowed to perform it.

The once-lowly leaf had been raised to the pinnacle of spiritual and social grace. In the words of
Lafcadio Hearn, an historian and writer of Irish origin who emigrated to Japan in the late 19th
Century, “The Tea ceremony requires years of training and practice to graduate in art … yet the
whole of this art, as to its detail, signifies no more than the making and serving of a cup of tea. The
supremely important matter is that the act be performed in the most perfect, most polite, most
graceful, most charming manner possible.”

1. The main purpose of this passage is to

A. Trace the historical progression of tea from its origins to the present day.
B. Give brief highlights from the history of the cultivation of tea.
C. Provide an anecdotal account how tea became a drink.

Answer choice C is similarly mentioned, but it’s nowhere near the bulk of the passage.
D. Highlight some important elements of the history of preparing and drinking tea.
E. Argue against the notion of tea drinking as a valid social art.

2. The last sentence of the first paragraph serves to illustrate which of the following
about tea?

A. The mistake that led to tea drinking’s ultimate elevation as a social grace
B. The accidental and fortunate nature of how tea was discovered
C. The spreading seeds of the habit of drinking tea
D. The link between tea and Zen Buddhist practice of pilgrimage
E. The unusually rapid way that tea was developed into a beverage

3. Which of the following inferences may be drawn from the discussion of Lu Yu’s work?
A. Before 800 AD, it was largely unknown how to cultivate tea.
B. Some people even today drink tea for reasons other than its physical benefits.
C. Drinking tea was primarily a Zen Buddhist practice until the late 700s.
D. The Ch’a Ching is one of the earliest works of Chinese origin that is concerned with agriculture.
E. Lu Yu was interested in popularizing tea in countries other than China.

4. Based on the passage, Lafcadio Hearn would have agreed with which of the following
statements about Japanese Tea Ceremony?
A. It is needlessly complex and intricate.
B. It is important that students of the art spend many years mastering it.
C. It is the pinnacle of Japanese taste and culture.
D. It is both a simple act and one that is rich with cultural significance.
E. It is an inextricable part of Japanese history and spirituality.

This is an applying information question that asks you to make a decision based on the information
given in the passage. After reading the quote by Lafcadio Hearn, we can see that he sees the
Japanese Tea Ceremony as something of a contrast, particularly by the use of the sentence “… yet
the whole of this art … signifies nothing more than the making and serving of tea
…” Answer choice A, however, is too extreme o be supported by the quote; Hearn is not strongly
negative about the practice of tea ceremony. Answer choice B is perhaps something that is believed
by members of Japanese society, but we have no support that Hearn believes it
is “important.” Answer choice C is also extreme. It is a statement from elsewhere in the passage,
but not an opinion expressed by Hearn. Answer choice E is somewhat supported by the passage,
though the word “inextricably” is a little extreme; however, it is not necessarily Hearn’s opinion.
Only answer choice D expresses the contradiction that Hearn saw in the practice of Japanese Tea
Ceremony, and expresses it in a non-extreme, non-offensive way.

Mythology (n) : thần thoại

Visionary (n) : người có tầm nhìn

Precaution (n) : đề phòng

Court (n) : triều đình

Spirituality (n) : tâm linh

Monk (n) : nhà sư

Pilgrimage (n) : cuộc đi hành hương

Culminate (v) : lên đến đỉnh cao

Intricate (adj) : phức tạp

Exclusive (adj) : chọn lọc

Pinnacle (n) : đỉnh cao/ ngọn/ đỉnh núi / chỏm

Signify (v) : biểu thị

Supremely (adv) : cực kỳ, tuyệt vời

Anecdotal (adj) : giai thoại

Inextricable (adj) : không thể thiếu được


Feb 12 – 2

On the surface, the conquest of the Aztec empire by Herman Cortes is one of the most amazing
military accomplishments in history. With a small fighting force numbering in the hundreds, Cortes
led the Spanish explorers into victory against an Aztec population that many believe topped 21
million. In light of such a seemingly impossible victory, the obvious question is: how did a small
group of foreign fighters manage to topple one of the world's strongest, wealthiest, and most
successful military empires?

Several factors led to Cortes' success. First, the Spanish exploited animosity toward the Aztecs
among rival groups and convinced thousands of locals to fight. In one account of a battle, it is
recorded that at least 200,000 natives fought with Cortes. Next, the Spanish possessed superior
military equipment in the form of European cannons, guns, and crossbows, leading to effective and
efficient disposal of Aztec defenses. For example, Spanish cannons quickly defeated large Aztec
walls that had protected the empire against big and less technically advanced armies.

Despite the Spanish advantages, the Aztecs probably could have succeeded in defending their
capital city of Tenochtitlan had they leveraged their incredible population base to increase their
army's size and ensured that no rogue cities would ally with Cortes. In order to accomplish this later
goal, Aztec leader Motecuhzoma needed to send envoys to neighboring cities telling their inhabitants
about the horrors of Spanish conquest and the inevitability of Spanish betrayal.

In addition, the Aztecs should have exploited the fact that the battle was taking place on their
territory. No reason existed for the Aztecs to consent to a conventional battle, which heavily favored
the Spanish. Motecuhzoma's forces should have thought outside the box and allowed Cortes into the
city, only to subsequently use hundreds of thousands of fighters to prevent escape and proceed in
surprise "door-to-door" combat. With this type of battle, the Aztecs would have largely thwarted
Spanish technological supremacy. However, in the end, the superior weaponry of the Spanish, the
pent-up resentment of Aztec rivals, the failure of Aztec diplomacy, and the lack of an unconventional
Aztec war plan led to one of the most surprising military outcomes in the past one thousand years.

1. Which of the following best characterizes the main point the author is trying to
convey in the passage?
A) Aztec failure to fight an unconventional war led to an unnecessary defeat
B) Spanish victory was neither as impressive nor as surprising as it may first appear
C) Resentment toward the Aztecs led to their demise
D) Herman Cortes masterminded an amazing military accomplishment
E) The myopic vision of the Aztecs led to their unnecessary downfall

In order to ascertain the main point that the author is trying to make, it is important to
examine logical flow of the passage.
1st Paragraph: Explain a seemingly amazing accomplishment and ask whether it really is as
impressive as it first appears.
2nd Paragraph: Explain factors that made the impressive accomplishment not as impressive.
3rd Paragraph: Explain how the seemingly amazing accomplishment didn't have to turn out the
way it did.
4th Paragraph: Explain how the seemingly amazing accomplishment didn't have to turn out the
way it did.
2. The passage is sequentially organized in which of the following ways?
A) Introduce an enigma; explain the reasons for the enigma; discuss the inevitability of
the enigma
B) Define a problem; explain the sources of the problem; offer a solution to the problem

The third and fourth paragraphs are not offering a solution to a problem but rather discussing the
inevitability of an outcome.
C) Introduce a mystery; offer an explanation for the mystery; provide an alternative explanation for
the mystery
D) Pose a question; offer an answer to the question; offer an alternative answer to the question
E) Define a problem; explain the likelihood of the problem; discuss the consequences of the problem

In order to see the sequential ordering, break down the logical flow of the passage.

1st Paragraph: Introduce something that looks very impressive on the surface and ask how it
happened.
2nd Paragraph: Offer several factors that help explain what seemed so impressive and
unbelievable.
3rd Paragraph: Provide several ways that the seemingly unbelievable was not inevitable.
4th Paragraph: Continue with paragraph 3. Conclude by noting that the seemingly unbelievable
and unexplainable was both explainable and not inevitable.

To summarize the sequential organization:

Introduce an enigma ("how did a small group of foreign fighters manage to topple one of the
world's strongest, wealthiest, and most successful military empires?") Explain reasons for the
enigma (2nd paragraph) Discuss the inevitability of the enigma (3rd and 4th paragraphs)

3. The author implies which of the following about the Aztec view toward an
unconventional military confrontation of the Spanish?

A) The Aztecs did not consider it


B) The Aztecs considered it, but rejected it out of beliefs about how battles ought to be fought
C) The Aztecs considered this, but it was too late
D) The Aztecs were certain a victory could be achieved via traditional combat
E) The Aztecs felt the geography of Tenochtitlan did not favor this strategy
4. According to the passage, all of the following led to Cortes' success EXCEPT:

A) Advanced crossbows
B) Nimble military force

Although the passage mentions that Cortes' army was small, it implies that this as a weakness. The
passage never states that the military was nimble nor does it mention this as a source of success.
C) Local Spanish allies

The passage mentions this as a source of success: "In one account of a battle, it is recorded that at
least 200,000 natives fought with Cortes."
D) Local tribal friction
E) Quick destruction of Aztec walls

Conquest (n) : sự chinh phục, sự xâm chiếm

Accomplishment (n) : thành tựu

Animosity (n) : thù oán, sự ác cảm

Leverage (v) : tận dụng

Envoy (n) : đặc sứ ngoại giao, phái viên

Inhabitant (n) : cư dân

Consent (v) : tán thành

Conventional (adj) : thông thường

Thwart (v) : cản trở

Supermacy (n) : quyền tối cao

Resentment (n) : oán giận

Diplomacy (n) : ngoại giao

Myopic (adj) : cận thị

Enigma (n) : bí ẩn, câu đố

Nimble (adj) : nhanh nhẹn, lanh lợi, trí óc minh mẫn


Feb 13 – 1

The human species came into being at the time of the greatest biological diversity in the history of
the Earth. Today, as human populations expand and alter the natural environment, they are
reducing biological diversity to its lowest level since the end of the Mesozoic era, 65 million years
ago. The ultimate consequences of this biological collision are beyond calculation, but they are
certain to be harmful. That, in essence, is the biodiversity crisis.

The history of global diversity can be summarized as follows: after the initial flowering of
multicellular animals, there was a swift rise in the number of species in early Paleozoic times
(between 600 and 430 million years ago), then plateaulike stagnation for the remaining 200 million
years of the Paleozoic era, and finally a slow but steady climb through the Mesozoic and Cenozoic
eras to diversity’s all-time high. This history suggests that biological diversity was hard won and a
long time in coming. Furthermore, this pattern of increase was set back by five massive extinction
episodes. The most recent of these, during the Cretaceous period, is by far the most famous,
because it ended the age of the dinosaurs, conferred hegemony on the mammals, and ultimately
made possible the ascendancy of the human species. But the cretaceous crisis was minor compared
with the Permian extinctions 240 million years ago, during which between 77 and 96 percent of
marine animal species perished. It took 5 million years, well into Mesozoic times, for species
diversity to begin a significant recovery.

Within the past 10,000 years biological diversity has entered a wholly new era. Human activity has
had a devastating effect on species diversity, and the rate of human-induced extinctions is
accelerating. Half of the bird species of Polynesia have been eliminated through hunting and the
destruction of native forests. Hundreds of fish species endemic to Lake Victoria are now threatened
with extinction following the careless introduction of one species of fish, the Nile perch. The list of
such biogeographic disasters is extensive.

Because every species is unique and irreplaceable, the loss of biodiversity is the most profound
process of environmental change. Its consequences are also the least predictable because the value
of Earth’s biota (the fauna and flora collectively) remains largely unstudied and unappreciated;
unlike material and cultural wealth, which we understand because they are the substance of our
everyday lives, biological wealth is usually taken for granted. This is a serious strategic error, one
that will be increasingly regretted as time passes. The biota is not only part of a country’s heritage,
the product of millions of years of evolution centered on that place; it is also a potential source for
immense untapped material wealth in the form of food, medicine, and other commercially important
substance.
1. Which one of the following best expresses the main idea of the passage?

(A) The reduction in biodiversity is an irreversible process that represents a setback both for science
and for society as a whole.
(B) The material and cultural wealth of a nation are insignificant when compared with the country’s
biological wealth.
(C) The enormous diversity of life on Earth could not have come about without periodic extinctions
that have conferred preeminence on one species at the expense of another.
(D) The human species is in the process of initiating a massive extinction episode that may make
past episodes look minor by comparison.
(E) The current decline in species diversity is human-induced tragedy of incalculable
proportions that has potentially grave consequences for the human species.

2. Which one of the following situations is most analogous to the history of global
diversity summarized in Highlighted part of the passage?

(A) The number of fish in a lake declines abruptly as a result of water pollution, then makes a slow
comeback after cleanup efforts and the passage of ordinances against dumping.
(B) The concentration of chlorine in the water supply of large city fluctuates widely before stabilizing
at a constant and safe level.
(C) An old-fashioned article of clothing goes in and out of style periodically as a result of features in
fashion magazines and the popularity of certain period films.
(D) After valuable mineral deposits are discovered, the population of a geographic region booms
then levels off and begins to decrease at a slow and steady pace.
(E) The variety of styles stocked by a shoe store increases rapidly after the store opens,
holds constant for many months, and then gradually creeps upward.

3. The author suggests which one of the following about the Cretaceous crisis?

(A) It was the second most devastating extinction episode in history.


(B) It was the most devastating extinction episode up until that time.
(C) It was less devastating to species diversity than is the current biodiversity crisis.
(D) The rate of extinction among marine animal species as a result of the crisis did not
approach 77 percent.
(E) The dinosaurs comprised the great majority of species that perished during the crisis.

4. The author mentions the Nile perch in order to provide an example of

(A) a species that has become extinct through human activity


(B) the typical lack of foresight that has led to biogeographic disaster
(C) a marine animal species that survived the Permian extinctions
(D) a species that is a potential source of material wealth
(E) the kind of action that is necessary to reverse the decline in species diversity

5. All of the following are explicitly mentioned in the passage as contributing to the
extinction of species EXCEPT
(A) hunting
(B) pollution
(C) deforestation
(D) the growth of human populations
(E) human-engineered changes in the environment

6. The passage suggests which one of the following about material and cultural wealth?
(A) Because we can readily assess the value of material and cultural wealth, we tend not
to take them for granted.
(B) Just as the biota is a source of potential material wealth, it is an untapped source of cultural
wealth as well.
(C) Some degree of material and cultural wealth may have to be sacrificed if we are to protect our
biological heritage.
(D) Material and cultural wealth are of less value than biological wealth because they have evolved
over a shorter period of time.
(E) Material wealth and biological wealth are interdependent in a way that material wealth and
cultural wealth are not.

7. The author would be most likely to agree with which one of the following statements
about the consequences of the biodiversity crisis?
(A) The loss of species diversity will have as immediate an impact on the material of nations as on
their biological wealth.
(B) The crisis will likely end the hegemony of the human race and bring about the ascendancy of
another species.
(C) The effects of the loss of species diversity will be dire, but we cannot yet tell how
dire.
(D) It is more fruitful to discuss the consequences of the crisis in terms of the potential loss to
humanity than in strictly biological loss to humanity than in strictly biological terms.
(E) The consequences of the crisis can be minimized, but the pace of extinctions can not be
reversed.

Stagnation (n) : sự đọng lại, đình trệ

Devastating (adj) : tàn phá

Human-induced (n) : do con người gây ra

Collectively (adv) : tập thể

Fauna (n) : hệ động vật

Flora (n) : hệ thực vật

Taken smt for granted (v) : được xem là đương nhiên

Heritage (n) : gia tài, di sản

Dire (adj) : khốc liệt, tàn khốc, tai hại

Foresight (n) : tầm nhìn xa, sự lo trước


Feb 13 – 2

Forget hostile aliens. According to a forthcoming book by noted astrophysicist Egbert Larson, the
intrepid humans who first attempt interstellar space travel will face far more daunting challenges
before they ever meet the Little Green Men. Larson begins with the problem of relativistic time
dilation. If you travel all the way to Alpha Centauri, you’d like to come back and tell your friends
about it, right? It’s not too likely to happen, though. If Mr. Einstein was right about relativity—and
we’re not going to say he wasn’t—then time slows down when you approach the speed of light. A
person traveling at any velocity near the speed of light will age only days for every week, month, or
even year that passes on earth. Relativity does not present a problem for interstellar space travel,
per se, but it does mean that interstellar civilizations or even just interstellar communications will
require a mind-boggling amount of calendar juggling.

Did we mention that you’d have to travel at near the speed of light? That’s because the distance
between stars is so vast that even if you could travel at the speed of light— which, Larson reminds
us, you can’t—it would take more than four years to reach our closest star neighbors, Alpha Proxima
and Alpha Centauri, and decades or centuries to reach the other stars in our “immediate
neighborhood.” And if you tried to accelerate directly to the speed of light like they do in the movies,
you’d be instantly splattered on the back of your theoretical spacecraft. Achieving anything close to
light speed will require sustained acceleration at a level that human bodies can withstand—say, a
crushing two gravities—for over a year. Better hope somebody brings some chips.

Speaking of chips, food is going to be a problem. Since it is economically, if not physically,


impossible to accelerate 200 years’ worth of food to nearly the speed of light, and since you’re not
likely to find any grocery stores along the way, someone will have to figure out how to make food in
space. Keeping a crew alive on the way turns out to be the trickiest part of all. Once you’ve got the
nearly impossible physics of space travel worked out, you still have to figure out the chemistry and
biology of keeping your air and water clean and keeping your crew fed and safe from radiation and
infection, and—did we mention the 200 years?—you’ll probably need several generations of crew
members to complete the trip. Ever been on a bus for more than 24 hours? It’s not a pretty picture.

We applaud Larson for his insightful writing and his scrupulous attention to scientific detail. For
those of you seeking a cold, hard look at the reality of interstellar space travel, this is a stellar read.
But be warned: Larson doesn’t let you down gently. For those of you sincerely hoping to beam up
with Scotty—and you know who you are—you might want to give this one a pass.

1. According to the information given in the passage, if two 20-year-old twins lived on
earth, and one of them left on a journey for Alpha Centauri at very close to the speed of
light, then managed to survive the journey and return to earth having aged 40 years
during the journey, what could she expect to find upon her return to earth?

A. Her great-grandmother
B. Her twin at the age of 20
C. Her twin at the age of 40
D. Her twin at the age of 60
E. Her twin’s great-grandchildren
2. Which of the following inventions, if it could be perfected and manufactured at a
viable cost, would address the most challenges to human interstellar space travel, as
presented in the passage?

A. A ram-scoop drive that can accelerate a spacecraft of any size to four-fifths of the speed of light
within 24 hours
B. A cold-sleep capsule that essentially halts the passage of time for human inhabitants
while protecting them from all physical harm
C. A sustainable biosphere that reliably generates healthy food and automatically cleans air and
water
D. A neutrino-based communications system that permits instantaneous communication across any
distance without any relativistic time dilation
E. An impervious force field that protects the ship and its inhabitants from radiation, meteor strikes,
or hostile alien attacks

3. The author of the passage most likely mentions “Little Green Men” in the first
paragraph for what purpose?

A. To poke fun at the ignorance of most science fiction readers


B. To introduce a daunting challenge that will have to be addressed before human interstellar space
travel can become possible
C. To draw a comparison between the attempts of humans to voyage in space and the more
successful attempts of other civilizations
D. To draw an amusing distinction between a supposed danger of space travel, as
presented in the popular media, and the actual challenges posed by interstellar space
travel, as perceived by scientists
E. To suggest that the concept of human interstellar space travel is as much of a myth as the “Little
Green Men” that appear in science fiction movies and television programs

Hostile (adj) : thù địch

Astrophysicist (n) : nhà vật lý thiên văn

Intrepid (adj) : gan dạ

Interstellar (adj) liên sao, ở khoảng giữa các sao

Daunting (adj) : nản chí

Dilation (n) : sự giãn nở

Age (v) : già đi

Mind-boggling (adj) : dị thường, không thể tin được

Splatter (v) : văng tung tóe, bắn tung tóe

Relativity (n) : thuyết tương đối

Applaud (v) : hoan nghênh

Scrupulous (adj) : chu đáo


Feb 14 – 1

James Joyce revolutionized the novel, the short story, and modern literature as we know it. He was
born in Dublin, the first of 10 children in a Catholic family. His father was a civil servant whose poor
financial judgment left the family impoverished for much of Joyce’s youth. Young James attended
Dublin’s fine Jesuit schools, which gave him a firm grounding in theology and classical languages—
subjects that appeared repeatedly in his later work. The story of his early life and his intellectual
rebellion against Catholicism and Irish nationalism are told in the largely autobiographical novel A
Portrait of the Artist as a Young Man.

In 1902, at the age of 20, Joyce left Dublin to spend the rest of his life in Paris, Trieste, Rome, and
Zurich, with only occasional visits back home. Despite this self-imposed exile, Dublin was the setting
for most of his writings. Dubliners (1914), Joyce’s most accessible work, is a collection of short
stories describing the paralyzing social mores of middle-class Catholic life. “The Dead,” the final
story in the collection, is frequently listed as one of the finest short stories ever written.

Joyce’s next book, Ulysses, took seven years to write; once he finished writing it, he almost couldn’t
find anyone to publish it. Upon the novel’s publication, both Ireland and the United States
immediately banned it as obscene. Despite these obstacles, Ulysses has come to be generally
recognized as the greatest twentieth-century novel written in English. The novel was revolutionary in
many ways. The structure was unique: Joyce recreated one full day in the life of his protagonist,
Leopold Bloom, and modeled the actions of the story on those of Ulysses in the Odyssey. In
recounting Bloom’s day, Joyce mentions everything that happens to Bloom—including thoughts,
bodily functions, and sexual acts—providing a level of physical actuality that had never before been
achieved in literature. To provide a psychological insight comparable to the physical detail, Joyce
employed a then-revolutionary technique called stream of consciousness,in which the protagonist’s
thoughts are laid bare to the reader.

From 1922 until 1939, Joyce worked on a vast, experimental novel that eventually became known as
Finnegan’s Wake. The novel, which recounts “the history of the world” through a family’s dreams,
employs its own “night language” of puns, foreign words, and literary allusions. It has no clear
chronology or plot, and it begins and ends on incomplete sentences that flow into each other. Many
of Joyce’s supporters thought he was wasting his time on the project, although the playwright
Samuel Beckett, who later won the Nobel Prize for Literature, helped Joyce compile the final text
when his eyesight was failing. Today, Finnegan’s Wake is viewed as Joyce’s most obscure and
possibly most brilliant work.

1. Which of the following would make the most appropriate title for this passage?

A. The Long Way Home: Ulysses and Finnegan’s Wake


B. James Joyce, Ulysses, and the Battle against Censorship
C. The Works of James Joyce, Ireland’s Literary Genius
D. The Hidden Value of James Joyce’s Great Novels
E. A Portrait of James Joyce as a Young Man
2. Based on the information in the passage, which of the following would be the most
accurate statement about Dubliners and Finnegan’s Wake?

A. Dubliners contains one of the greatest short stories in the English language, and Finnegan’s Wake
is the greatest story in the English language.
B. Many of the chief characters in Finnegan’s Wake were earlier introduced in Dubliners.
C. The linguistic experimentation of Dubliners paved the way for the “night language” of Finnegan’s
Wake.
D. Dubliners is a longer book than Finnegan’s Wake.
E. Dubliners is a more accessible book than Finnegan’s Wake.

3. Joyce’s works helped introduce all except which of the following literary elements
into modern English literature?

A. Narration through second-person address


B. Novel structure based on real-time chronology
C. Linguistic experimentation
D. Literary realism concerning physical reality
E. Stream of consciousness

The passage never mentions second-person address. There is support for all of the other
answers in the passage: B—“The structure was unique: Joyce recreated one full day in
the life of his protagonist”; C—“The novel … employs its own ‘night language’ of puns, foreign
words, and literary allusions”; D—“Joyce mentions everything that happens to Bloom—including
thoughts, bodily functions, and sexual acts—providing a level of physical actuality that had never
before been achieved in literature”; E—“Joyce employed a then-revolutionary technique called
stream of consciousness.”

4. According to the passage, in what year was Joyce born?

A. 1880
B. 1882
C. 1885
D. 1902
E. 1914

Revolutionize (v) : cách mạng hóa

Civil servant (n) : công chức

Impoverished (adj) : nghèo khó

Theology (n) : thần học

Rebellion (n) : nổi loạn, chống lại nhà cầm quyền

Catholicism (n) : công giáo

Irish nationalism (n) : chủ nghĩa dân tộc ailen

Exile (n) : lưu vong, tha hương, viễn xứ


Self-imposed (adj) : tự áp đặt

Paralyzing (adj) : làm tê liệt

Finest (adj) : tốt nhất

Obscene (adj) : tục tĩu, ghê tởm

Protagonist (n) : nhân vật chính

Psychological (adj) : tâm lý

Insight (n) : cái nhìn sâu sắc

Pun (n) : chơi chữ, lời nói cợt

Allusion (n) : ám chỉ

Chronology (n) : niên đại

Plot (n) : tình tiết

Compile (v) : biên dịch, biên soạn

Eyesight (n) : thị lực

Obcure (adj) : bí mật, khó hiểu

Narration (n) : tường thuật


Feb 14 – 2

Although websites such as Facebook and MySpace experienced exponential growth during the
middle of the first decade of the 21st century, some users remain oblivious to the fact that the
information they post online can come back to haunt them. First, employers can monitor employees
who maintain a blog, photo diary, or website. Employers can look for controversial employee
opinions, sensitive information disclosures, or wildly inappropriate conduct. For example, a North
Carolina newspaper fired one of its features writers after she created a blog on which she
anonymously wrote about the idiosyncrasies of her job and coworkers.

The second unintended use of information from social networking websites is employers who check
on prospective employees. A June 11, 2006 New York Times article reported that many companies
recruiting on college campuses use search engines and social networking websites such as MySpace,
Xanga, and Facebook to conduct background checks. Although the use of MySpace or Google to
scrutinize a student’s background is somewhat unsettling to many undergraduates, the Times noted
that the utilization of Facebook is especially shocking to students who believe that Facebook is
limited to current students and recent alumni.

Corporate recruiters and prospective employers are not the only people interested in college
students’ lives. The third unintended use of social networking websites is college administrators who
monitor the Internet—especially Facebook—for student misconduct. For example, a college in
Boston’s Back Bay expelled its student Government Association President for joining a Facebook
group highly critical of a campus police sergeant. In addition, fifteen students at a state university in
North Carolina faced charges in court for underage drinking because of photos that appeared on
Facebook.

Although more users of websites such as Facebook are becoming aware of the potential pitfalls of
online identities, many regular users still fail to take three basic security precautions. First, only
make your information available to a specific list of individuals whom you approve. Second, regularly
search for potentially harmful information about yourself that may have been posted by mistake or
by a disgruntled former associate. Third, never post blatantly offensive material under your name or
on your page as, despite the best precautions, this material will likely make its way to the wider
world. By taking these simple steps, members of the digital world can realize the many benefits of e-
community without experiencing some of the damaging unintended consequences.

1 Based upon the passage, the author implies which of the following:

A) Information obtained unwillingly from the Internet is permissible in court


B) It is impossible to protect yourself from unintended uses of information online
C) Making information available only to people whom you trust compromises your online community
D) Even if you restrict who can view your data, the government may still access it
E) Done properly, posting prurient information about oneself poses no substantial risk
2 Which of the following best describes the author's logical flow in the passage?
A) Define a problem, provide examples of it, offer means of remedying it, and offer a
brief evaluation of the issue at hand
B) Provide examples of a problem, offer a counter point, provide a resolution of the conflicting
views, and offer a brief evaluation of the issue at hand
C) Provide examples of a problem, provide means of remedying it, offer a brief evaluation of the
issue at hand, and provide a contrasting evaluation of the issue at hand
D) Define a problem, provide examples of it, offer a brief evaluation of the issue at hand, and offer
suggestions to support that evaluation
E) Define a problem, offer a contrasting view of the issue at hand, offer a brief evaluation of the
issue at hand, and offer a solution to the conflicting views

Define a problem: "some users remain oblivious to the fact that the information they post online
can come back to haunt them."
Provide three examples of the problem: "First, employers can monitor employees who maintain
a blog...The second unintended use of information...The third unintended use of social networking
websites is..."
Provide Ways to Remedy the Problem: "many regular users still fail to take three basic security
precautions. First, only make your information available...Second, regularly search for...Third, never
post blatantly offensive"
Evaluate the Issue at Hand: "By taking these simple steps, members of the digital world can
realize the many benefits of e-community without experiencing some of the damaging unintended
consequences."

3 The author implies that users should take all of the following actions to protect their
online privacy EXCEPT:

A) Know to whom you make your online information available


B) Actively hunt for misinformation or damaging information posted about you or under your name
C) Speak with recruiters to inform them of any misinformation published about you
D) Carefully select and limit who can view your electronic profile
E) Avoid uploading information that would be exceedingly damaging if it were discovered

4 The tone of the passage suggests that the author's view toward e-community and the
digital world can best be described as:

A) Largely Pessimistic
B) Frustrated
C) Guardedly Optimistic
D) Distressed
E) Strongly Optimistic

5 According to the passage, all of the following represent a possible threat to privacy or
an unintended use of data EXCEPT:
A) Disgruntled past associates posting damaging information online
B) Colleges or universities disciplining students for expressing politically incorrect or institutionally
disowned opinions
C) Government officials using online information, obtained against one’s will, to bring legal
proceedings
D) Potential employers conducting unauthorized and potentially invasive background checks via
Google or Facebook
E) Malicious users impersonating one's identity to commit identity fraud

6 The primary purpose of the passage is to:


A) Explain the growth of the digital world through the lens of privacy
B) Discuss the risks of the digital world and suggest ways to protect yourself
C) Evaluate the pros and cons of active participation in the digital world
D) Propose steps Facebook, MySpace, and Google can take to protect user privacy
E) Illustrate potential unintended uses of private information

7 Which of the following best describes the relationship of the fourth paragraph to the
remainder of the passage?
A) It offers detailed examples to support previous assertions
B) It provides suggestions to ameliorate the previously mentioned problems
C) It summarizes the points of the preceding paragraphs
D) It offers counter evidence and an alternative point of view to the claims made earlier in the
passage
E) It reconciles conflicting claims

Ameliorate (v) : cải thiện

Idiosyncrasy (n) : đặc điểm riêng

Unintended (adj) : ngoài ý muốn

Scrutinize (v) : xem xét kỹ lưỡng

Disgruntled (adj) : bất mãn

Former associate (n) : cựu cộng sự

Blatantly (adv) : ngang nhiên, trắng trợn

Sergeant (n) : trung sĩ

Prurient (adj) : dâm ô, dâm dục

Unwillingly (adv) : bất đắc dĩ

Malicious (adj) : độc hại, có ác tánh, thù oán

Impersonate (v) : mạo danh

Remedy (v) : khắc phục

Distressed (adj) : đau khổ, buồn rầu


Feb 15 – 1

In Roman times, defeated enemies were generally put to death as criminals for having offended the
emperor of Rome. In the Middle Ages, however, the practice of ransoming, or returning prisoners in
exchange for money, became common. Though some saw this custom as a step towards a more
humane society, the primary reasons behind it were economic rather than humanitarian.

In those times, rulers had only a limited ability to raise taxes. They could neither force their subjects
to fight nor pay them to do so. The promise of material compensation in the form of goods and
ransom was therefore the only way of inducing combatants to participate in a war. In the Middle
Ages, the predominant incentive for the individual soldier to participate in a war was the expectation
of spoils. Although collecting ransom clearly brought financial gain, keeping a prisoner and arranging
for his exchange had its costs. Consequently, several procedures were devised to reduce transaction
costs. One such device was a rule asserting that the prisoner had to assess his own value. This
compelled the prisoner to establish a value without much distortion; indicating too low a value would
increase the captive’s chances of being killed, while indicating too high a value would either ruin him
financially or create a prohibitively expensive ransom that would also result in death.

1. The primary purpose of the passage is to

(A) discuss the economic basis of the medieval practice of exchanging prisoners for
ransom
(B) examine the history of the treatment of prisoners of war
(C) emphasize the importance of a warrior’s “word of honor” during the Middle Ages
(D) explore three ways of reducing the costs of ransom
(E) demonstrate why warriors of the Middle Ages looked forward to battles

2. It can be inferred from the passage that a medieval soldier

(A) was less likely to kill captured members of opposing armies than was a soldier of the
Roman Empire
(B) was similar to a 20th-century terrorist in that he operated on a basically independent level and
was motivated solely by economic incentives
(C) had few economic options and chose to fight because it was the only way to earn an adequate
living
(D) was motivated to spare prisoners’ lives by humanitarian rather than economic ideals
(E) had no respect for his captured enemies since captives were typically regarded as weak

3. Which of the following best describes the change in policy from executing prisoners in
Roman times to ransoming prisoners in the Middle Ages?
(A) The emperors of Rome demanded more respect than did medieval rulers and thus Roman
subjects went to greater lengths to defend their nation.
(B) It was a reflection of the lesser degree of direct control medieval rulers had over
their subjects.
(C) It became a show of strength and honor for warriors of the Middle Ages to be able to capture
and return their enemies.
(D) Medieval soldiers were not as humanitarian as their ransoming practices might have indicated.
(E) Medieval soldiers demonstrated more concern about economic policy than did their Roman
counterparts.
4. The author uses the phrase “without much distortion”(Highlighted) in order

(A) to indicate that prisoners would fairly assess their worth


(B) to emphasize the important role medieval prisoners played in determining whether they should
be ransomed
(C) to explain how prisoners often paid more than an appropriate ransom in order to increase their
chances for survival
(D) suggest that captors and captives often had understanding relationships
(E) to show that when in prison a soldier’s view could become distorted

Ransom (v) : chuộc lại, trả tiền chuộc

Humanitarian (n) : nhân đạo

Combatant (n) : chiến binh

Predominant (adj) : chiếm ưu thế, mạnh hơn

Spoil (n) : chiến lợi phẩm

Devise (v) : nghĩ ra

Compell (v) : bắt buộc

Dirtortion (n) : méo mó, biến thể, làm sai lạc

Captive (adj/n) : bị cầm tù / người bị bắt , tù nhân

Prohibitively (adv) : nghiêm cấm

Medieval (adj) : trung cổ

Counterpart (n) : đối tác


Feb 15 – 2

As a business model, the world of publishing has always been a somewhat sleepy enclave, but now
all that seems poised to change. Several Line companies have moved aggressively into a
new (5) business endeavor whose genesis comes from the question: Who owns the great works of
literature? Text-on-demand is not a completely new idea, of course. In the 1990s, the Gutenberg
project sought volunteers to type literary classics that had (10) expired copyrights into word
processing files so that scholars would have searchable databases for their research. Most of the
works of Shakespeare, Cervantes, Proust, and Moliere were to be found free online by as early as
1995.

(15) However, now large-scale companies have moved into the market, with scanners and
business plans, and are looking for bargain basement content. These companies are striking deals
with libraries, and some publishers, to (20) be able to provide their content, for a price, to
individual buyers over the Internet.

At stake are the rights to an estimated store of 30 million books, most of which are now out of
print. Many of these books are now also in the (25) public domain, giving any company the right
to sell them online. Still, a good portion of the books a general audience might actually want to
buy is still under copyright. The urgent question: Who owns those copyrights? In the case of all too
(30) many books put out more than 20 years ago by now-defunct publishing companies,the answer
is unclear—a situation the new text-on-demand companies are eager to exploit. An association
of publishers has sued, claiming massive copyright (35) infringement. The case is several
years away from trial.

1. The primary purpose of the passage is to

A. present the results of a statistical analysis and propose further study


B. explain a recent development and explore its consequences
C. identify the reasons for a trend and recommend measures to address it
D. outline several theories about a phenomenon and advocate one of them
E. describe the potential consequences of implementing a new policy and argue in favor of that
policy

2. It can be inferred from the passage that the works of Shakespeare, Cervantes, and
Molière

A. are some of the most popular works of literature


B. are no longer copyrighted
C. are among the works for which the association of publishers is suing text-on-demand companies
D. do not currently exist as searchable databases
E. were owned by now-defunct publishing companies
3. Which of the following is an example of a book that a text-on-demand company
would not have to acquire the rights to?

A. A book still under copyright


B. A book more than 20 years old
C. A book in the public domain
D. A book a general audience might want to buy
E. A book not already owned by publishers the company has a deal with

The passage states in the last paragraph that these companies want to exploit the unclear situation
of who owns the copyrights to certain books. They would want to print something that is not under
copyright, so it would be in the public domain, as the passage indicates in lines 25–26. For (B), the
passage does not say that all books over 20 years old may be freely printed.

4. It can be inferred from the passage that a common practice of text-on-demand


companies is to

A. use scanners to find books they want to acquire


B. create business plans well before they have any actual business
C. buy content at premium prices
D. acquire the rights to books for as little as possible
E. attempt to supplant the role of traditional publishers

Advocate (v) : biện hộ

Supplant (v) : thay thế, dành chỗ

Enclave (n) : bao vây

Poise (v) : sẵn sàng

Endeavor (n) : nỗ lực

Genesis (n) : nguồn gốc, khởi điểm

At stake (adj) : bị đe đọa

Domain (n) : tài sản

Now-defunct (adj) : bây giờ không còn tồn tại

Eager (adj) : háo hức

Infringement (n) : sự vi phạm


Feb 16 – 1

The most casual observer of Nature recognizes in almost every instance that comes under his notice
in everyday life, without the aid of logical definition, the broad distinctions between an animal, a
plant, and a stone or mineral. To him, the old definition that an animal is possessed of life and
locomotion, a plant of life without locomotion, and a mineral deficient in both, seems to be
sufficient, until some day he travels beyond the circuit of diurnal routine, and encounters a sponge
or a zoophyte, which possesses only one of his supposed attributes of animal life, but which he is
assured is nevertheless a member of the animal kingdom.

Such an encounter usually perplexes the neophyte at first, but rather than confess his
generalizations to have been too gross, he will tenaciously contend that the sponge must be a plant,
until the evidence produced is so strong that he is compelled to desert his position and seek refuge
in the declaration that one kingdom runs into the other so imperceptibly that no line of demarcation
can be drawn between them. Between these two extremes of broad distinction, and no distinction,
lies the ground occupied by the scientific student, who, whilst admitting that logical definition fails ¡n
assigning briefly and tersely the bounds of the three kingdoms, contends that such limits exist so
positively, that the universal scientific mind accepts the recognized limit without controversy or
contradiction.

1. What is the primary purpose of the first paragraph of the passage?

(A) To explain the differences between animals, plants and minerals


(B) To introduce an exception to acknowledged characteristics of animals, plants and
minerals
(C) To conclude how neophytes can, at times, be wrong in their perceptions
(D) To clarify to the common man the characteristics of a sponge and a zoophyte
(E) To redefine the old broad distinctions between animals, plants and stones

2. The author would most probably agree with each of the following about
neophytes EXCEPT

(A) They are generally unaware that exceptions exist


(B) When presented with proof, they are open to acceptance
(C) They generally do not travel outside their daily routine
(D) They justify their position vehemently
(E) They do not agree with the scientific student

3. According to the passage, which of the following is most probably true of a zoophyte?
(A) It is capable of movement
(B) It is a living being
(C) It is capable of movement and is also a living being
(D) It does not contain chlorophyll as plants do
(E) It is not an animal

The passage states that animals possess two attributes—life and locomotion. It further states that a
sponge or a zoophyte possesses only one of these attributes. Then, this attribute must be life
because it would be a little absurd to assume that zoophytes are capable of movement but do not
possess life. Hence, (B) is the correct answer.
Instance (n) : ví dụ, trường hợp

Locomotion (n) : sự vận động

Deficient in (adj) : thiếu, không đủ

Diurnal (adj) : trong một ngày

Encounter (v) : gặp, đụng độ

Assure (v) : cam đoan

Nevertheless (adv) : tuy nhiên

Perplex (v) : bối rối

Neophyte (n) : tân sinh ( a person who is new to a subject, skill or belief )

Generalization (n) : khái quát hóa, kiến thức phổ thông

Gross (adj) : hiển nhiên, rõ ràng

Tenaciously (adv) : ngoan cường

Compell (v) : bắt buộc

Refuge (n) : nơi ẩn náu

Desert (v) : bỏ, trốn đi

Declaration (n) : lời tuyên bố

Imperceptibly (adv) : không thể nhận ra

Demarcation (n) : phân giới, vạch biên giới

Assign (v) : thừa nhận, chỉ định

Briefly (adv) : ngắn gọn

Tersely (adv) : chặt chẽ

Bound (n) : giới hạn

Contradiction (n) : mâu thuẫn

Acknowledged (adj) : thừa nhận


Feb 16 – 2

Until the 1970s it was assumed that, despite the very large number of species that appeared during
the Cambrian explosion, nearly all fit into the same rather small number of phyla that exist today.
Each phylum—a group of organisms with the same basic pattern of organization, such as the radial
symmetry of jellyfish and other coelenterates or the segmented structure of worms and other
annelids—was seen as evolutionarily stable.

Innumerable individual species have arisen and died out, but development and extinction were
assumed to take place within existing phyla; the elimination of entire phyla was thought to be
extremely rare. A diverse group of marine fossils, known collectively as the Problematica, present
difficulties for this interpretation. They show patterns of organization so bizarre that it is hard to fit
any of them into present-day phyla. They include the banana-shaped Tullimonstrum and the spiked,
spiny Hallucigenia, creatures whose very names reflect the classifier‘s discomfort.

The ―Ediacaran fauna,‖ which respired, absorbed nutrients, and eliminated wastes directly through
their external surfaces, are also included among the Problematica. Theirs was an approach taken by
only a few modern multicelled creatures (such as tapeworms) that are otherwise totally unlike them.
Several theorists have argued that the Problematica are not just hard to classify—they are evidence
that the conventional view of the Cambrian explosion is wrong. They contend that the Cambrian
explosion represented the simultaneous appearance of a much larger number of animal phyla than
exists today. Each was a separate ―experiment‖ in basic body design, and the Cambrian seas
teemed with many different phyla, or basic body plans, each represented by only a few species.

Today, the number of phyla has fallen drastically, but each surviving phylum contains a much larger
number of species. The Problematica, then, were not unsuccessful variants within present-day
phyla; each represented a distinct phylum in its own right. Revisionists contend that the selection
process eliminated not only particular unfavourable traits, but entire body plans and approaches to
survival. The Ediacaran fauna, for example, represented a particular structural solution to the basic
problems of gas and fluid exchange with the environment.

This approach to body engineering was discarded at the same time as the Ediacaran fauna
themselves were wiped out; given the improbability of duplicating an entire body plan through
chance mutation, it was unlikely that this particular approach would ever be tried again.

Revisionists and conventional theorists agree that modern marine species are products of natural
selection. Up until 30 years ago, the pattern of early marine animal evolution seemed to be well
established. Most present-day marine animal phyla had appeared during the ―Cambrian explosion,‖
an extraordinary burgeoning of multicellular life in the warm seas of the Cambrian period, between
570 and 500 million years ago.
1. The description by the author in the third paragraph of how the Ediacaran fauna
carried out respiration, absorption, and excretion tends to support the view that the
Ediacaran fauna:

A. were probably not members of any present-day phylum.


B. had physiological processes different from those of any other known organisms.
C. could not absorb or excrete fluids.
D. were members of the same phylum as Tullimonstrum.
E. were not much dissimilar from other existing fauna

2. The passage implies that conventional and revisionist theorists disagree about all of
the following EXCEPT:

A. the accuracy of the conventional view of early marine evolution.


B. the probable number of marine animal phyla during the Cambrian period.
C. the likelihood of entire phyla becoming extinct.
D. the applicability of the theory of natural selection to the Cambrian period.

They agree with this statement in the first sentence of the six paragraph
E. the number of species within individual phyla

Evaluate the wording carefully. The two sides disagree on all the answer choices except the
correct one, which means that they agree on the correct choice. Predict a point of agreement
between the two sides. Para 6th opening line gives a big hint: The two sides agree that ―modern
marine species are products of natural selection.‖ (D) jumps out quickly when the prediction is made
beforehand.

3. According to the passage, the Problematica are difficult to classify because:

I. some had unusual shapes.


II. some of them functioned physiologically differently from modern organisms.
III. they became extinct at the end of the Cambrian period.

A. I only
B. II only
C. I and II only
D. I and III only
E. I, II and III

Mapping the Passage

1st and 2nd Para discuss a traditional theory of phyla evolution.


2nd and 3rd present fossil evidence, the Problematica, that challenges the traditional view of phyla.
3rd presents theorists who argue that the Problematica disprove the traditional view.
4th and 5th present the new view‘s main tenet: natural selection involved not only experiments
with individual traits within a phyla, but also with whole phyla.
6th reviews the traditional theory of phyla evolution.
Pattern of organization (n) : mô hình tổ chức

Symmetry (n) : đối xứng

Radial (adj) : xuyên tâm

Coelenterate (n) : đồng nghiệp

Evolutionarily (adv) : tiến hóa

Bizarre (adj) : kỳ quái

Respire (v) : hô hấp

Simultaneous (adj) : đồng thời

Teem (v) : có nhiều

Variant (n) : biến thể

Fluid (n/adj) : chất lỏng

Wipe out (v) : bị xóa sổ

Improbability (n) : không có khả năng

Mutation (n) : đột biến

Burgeon (v) : phát triển

Applicability (n) : khả năng ứng dụng


Feb 17 – 1

Many readers, I suspect, will take the title of this article [Women, Fire, and Dangerous Things] as
suggesting that women, fire, and dangerous things have something in common—say, that women
are fiery and dangerous. Most feminists I’ve mentioned it to have loved the title for that reason,
though some have hated it for the same reason. But the chain of inference—from conjunction to
categorization to commonality—is the norm. The inference is based on the common idea of what it
means to be in the same category: things are categorized together on the basis of what they have
in common. The idea that categories are defined by common properties is not only our everyday folk
theory of what a category is, it is also the principle technical theory—one that has been with us for
more than two thousand years.

The classical view that categories are based on shared properties is not entirely wrong. We often do
categorize things on that basis. But that is only a small part of the story. In recent years it has
become clear that categorization is far more complex than that. A new theory of categorization,
called prototype theory, has emerged. It shows that human categorization is based on principles
that extend far beyond those envisioned in the classical theory. One of our goals is to survey the
complexities of the way people really categorize. For example, the title of this book was inspired by
the Australian aboriginal language Dyirbal, which has a category, balan, that actually includes
women, fire, and dangerous things. It also includes birds that are not dangerous, as well as
exceptional animals, such as the platypus, bandicoot, and echidna. This is not simply a matter of
categorization by common properties.

Categorization is not a matter to be taken lightly. There is nothing more basic than categorization to
our thought, perception, action and speech. Every time we see something as a kind of thing, for
example, a tree, we are categorizing. Whenever we reason about kinds of things—chairs, nations,
illnesses, emotions, any kind of thing at all—we are employing categories. Whenever we
intentionally perform any kind of action, say something as mundane as writing with a pencil,
hammering with a hammer, or ironing clothes, we are using categories. The particular action we
perform on that occasion is a kind of motor activity, that is, it is in a particular category of motor
actions. They are never done in exactly the same way, yet despite the differences in particular
movements, they are all movements of a kind, and we know how to make movements of that kind.
And any time we either produce or understand any utterance of any reasonable length, we are
employing dozens if not hundreds of categories: categories of speech sounds, of words, of phrases
and clauses, as well as conceptual categories. Without the ability to categorize, we could not
function at all, either in the physical world or in our social and intellectual lives.
1. The author probably chose Women, Fire, and Dangerous Things as the title of the
article because

I. he thought that since the Dyirbal placed all three items in the same category, women, fire, and
dangerous things necessarily had something in common.

II. he was hoping to draw attention to the fact that because items have been placed in the same
category doesn’t mean that they necessarily have anything in common

III. he wanted to use the Dyirbal classification system as an example of how primitive classifications
are not as functional as contemporary Western classification systems.

(A) I only
(B) II only
(C) III only
(D) II and III only
(E) I, II, and III

2. According to the author,

I. categorizing is a fundamental activity of people.


II. whenever a word refers to a kind of thing, it signifies a category.
III. one has to be able to categorize in order to function in our culture.

(A) I only
(B) II only
(C) I and II only
(D) II and III only
(E) I, II, and III

3. Which one of the following facts would most weaken the significance of the author’s
title?

(A) The discovery that all the birds and animals classified as balan in Dyirbal are female
(B) The discovery that the male Dyirbal culture considers females to be both fiery and dangerous
(C) The discovery that all items in the balan category are considered female
(D) The discovery that neither fire nor women are considered dangerous
(E) The discovery that other cultures have categories similar to the balan category

4. If linguistic experts cannot perceive how women, fire, and dangerous things in the
category balan have at least one thing in common, it follows that

(A) there probably is something other than shared properties that led to all items in
balan being placed in that category.
(B) the anthropologists simply weren’t able to perceive what the items had in common.
(C) the anthropologists might not have been able to see what the items had in common.
(D) the items do not have anything in common.
(E) the Australian aboriginal culture is rather mystic.
5. Which one of the following sentences would best complete the last paragraph of the
passage?

(A) An understanding of how we categorize is central to any understanding of how we


think and how we function, and therefore central to an understanding of what makes us
human.

(B) The prototype theory is only the latest in a series of new and improved theories of
categorization; undoubtedly even better theories will replace it.

(C) The prototype theory of categories has not only unified a major branch of linguistics, but it has
applications to mathematics and physics as well.

(D) An understanding of how the prototype theory of categorization evolved from the classical
theory is essential to any understanding of how we think and how we function in society.

(E) To fully understand how modern Australian society functions, we must study how it is influenced
by aboriginal culture—most specifically how aborigines organize and classify their surroundings.

Anthropologist (n) : nhà nhân chủng học

Fiery (adj) : bốc lửa, nóng gắt

Inference (n) : suy luận

Conjunction (n) : sự kết hợp

Categorization (n) : phân loại

Commonality (n) : điểm chung

Norm (n) : bình thường/tiêu chuẩn/định mức

Property (n) : tính chất, đặc tính

Prototype (n) : nguyên mẫu/ vật mẫu đầu tiên

Emerge (v) : nổi lên

Envision (v) : hình dung

Aboriginal (adj) : thổ dân/ người bản xứ

Platypus (n) : thú mỏ vịt

Bandicoot (n) : giống chuột lớn

Mundane (adj) : trần tục

Utterance (n) : cách nói, sự phát âm

Linguistic (adj) : ngôn ngữ học

Primitive (adj) : nguyên thủy


Feb 17 – 2

In many underdeveloped countries, the state plays an important and increasingly varied role in
economic development today. There are four general arguments, all of them related, for state
participation in economic development. First, the entrance requirements in terms of financial capital
and capital equipment are very large in certain industries, and the size of these obstacles will serve
as barriers to entry on the part of private investors. One can imagine that these obstacles are
imposing in industries such as steel production, automobiles, electronics, and parts of the textile
industry. In addition, there are what Myint calls "technical indivisibilities in social overhead capital."
Public utilities, transport, and communications facilities must be in place before industrial
development can occur, and they do not lend themselves to small-scale improvements.

A related argument centres of the demand side of the economy. This economy is seen as
fragmented, disconnected, and incapable of using inputs from other parts of the economy.
Consequently, economic activity in one part of the economy does not generate the dynamism in
other sectors that is expected in more cohesive economies. Industrialization necessarily involves
many different sectors; economic enterprises will thrive best in an environment in which they draw
on inputs from related economic sectors and, in turn, release their own goods for industrial
utilization within their own economies.

A third argument concerns the low-leve equilibrium trap in which less developed countries find
themselves. At subsistence levels, societies consume exactly what they produce. There is no
remaining surplus for reinvestment. As per-capita income rises, however, the additional income will
not be used for savings and investment. Instead, it will have the effect of increasing the population
that will eat up the surplus and force the society to its former subsistence position. Fortunately, after
a certain point, the rate of population growth will decrease; economic growth will intersect with and
eventually outstrip population growth. The private sector, however, will not be able to provide the
one-shot large dose of capital to push economic growth beyond those levels where population
increase eat up the incremental advances.

The final argument concerns the relationship between delayed development and the state. Countries
wishing to industrialize today have more competitors, and these competitors occupy a more
differentiated industrial terrain than previously. This means that the available niches in the
international system are more limited. For today's industrializers, therefore, the process of
industrialization cannot be a haphazard affair, nor can the pace, content, and direction be left solely
to market forces. Part of the reason for a strong state presence, then, relates specifically to the
competitive international environment in which modern countries and firms must operate.

1. According to the passage, all of the following are arguments for state economics
intervention EXCEPT
A.the start-up costs of initial investments are beyond the capacities of many private investors.
B. the state must mediate relations betweent he demand and supply sides of the economy.
C. the pace and processes of industrialization are too important to be left solely to market
trends.
D.the livelihoods and security of workers should not be subject to the variability of
industrial trends.
E. public amenities are rquired to facilitate a favourable business environment .
2. Which of the following best states the central point of the passage?
A. Without state intervention, many less developed countries will not be able to carry
out the interrelated tasks necessary to achieve industrialization.
B. Underdeveloped countries face a crisis of overpopulation and lack of effective demand that
cannot be overcome without outside assistance.
C. State participation plays a secondary role as compared to private capital investment in the
industrialization of underdeveloped countries.
D. Less developed countries are trapped in an inescapable cycle of low production and demand.
E. State economic planning can ensure the rapid development of nonindustrialized countries' natural
resources.

3. The author suggests all of the following as appropriate roles for the state in economic
development EXCEPT

A. safeguarding against the domination of local markets by a single source of capital


B. financing industries with large capital requirements
C. helping to co-ordinate demand among different economic sectors
D. providing capital inputs sufficient for growth to surpass increases in per capita consumption
E. developing communication and transportation facilities to service industry

4. The author suggests which of the following about the "technical indivisibilities in the
social overhead capital" (end of the first paragraph) and the "low-level equilibrium trap"
(start of the third paragraph)?

A. The first leads to rapid technological progress; the second creates the demand for technologically
sophisticated products.
B. Both enhance the developmental effects of private sector investment.
C. Neither is relevant to formulating a strategy for economic growth.
D. The first is a barrier to private investment; the second can attract it.
E. The first can prevent development from occurring; the second can negate its effects.

5.Which of the following, if true, would cast doubt on the author's argument that state
participation is important in launching large-scale industries?

I. Co-ordination of demand among different economic sectors requires a state planning agency.
II. Associations of private-sector investors can raise large amounts of capital by pooling their
resources.
III. Transportation and communications facilities can be built up through a series of small-scale
improvements.

A.I only
B.II only
C. I and II only
D.II and III only
E.I, II and III
6. According to the passage, the "low-level equilibrium trap" in underdeveloped
countries results from

A.the tendency for societies to produce more than they can use
B.intervention of the state in economic development
C.the inability of market forces to overcome the effects of population growth
D.the fragmented and disconnected nature of the demand side of the economy
E.one-shot, large doses of capital intended to spur economic growth

Livelihood (n) : đời sống, kế sinh nhai

Intervention (n) : sự can thiệp

Tendency (n) : xu hướng

Pool (v) : chung vốn

Negate (v) : phủ nhận, làm cho kém hiệu quả


Feb 18 – 1

Between 1965 and 1970, welfare caseloads more than doubled and costs tripled. The Nixon
administration was unable to secure a legislative majority for comprehensive welfare reform.
Legislative welfare reform raised contentious issues of who is entitled to support, how much, and on
what terms—precisely the types of issues that have defied political resolution throughout welfare‘s
history.

As a mechanism of policy change, the Nixon administration turned to a common managerial tool—
performance monitoring. Middle-level officials at the Office of Management and Budget (OMB) and
the Department of Health, Education and Welfare (HEW) crafted quality control—a system for
monitoring the accuracy of state welfare payments—into an instrument for indirectly influencing
states to become more restrictive in the provision of welfare. Quality control‘s manifest purpose was
to achieve fiscal accountability.

Through this instrument HEW could monitor state welfare payments and withhold federal
reimbursement from those that it deemed to be improper. However, quality control
also served a latent, political function, partly reflected in its design. It penalized states only for
overpayments and payments made to ineligible individuals.

Quality control‘s effectiveness depended on the uncoordinated responses of street-level bureaucrats


in hundreds of local welfare offices to new demands that administrative reform imposed at the
workplace. For example, welfare workers translated administrative concern for procedural uniformity
into demands that welfare applicants routinely produce scores of documents of dubious relevance to
their eligibility.

Applicants who could not meet these procedural demands, whether reasonable or not, were denied
welfare. Administrative reform traded errors of liberality for errors of stringency. Behaviours directed
toward the helping aspects of welfare policy were virtually displaced as workers responded to
incentives to maximize measured attributes of performance, namely procedural uniformity and
productivity. At the same time, worker discretion to make unreasonable procedural demands was
virtually unchecked.

Quality control did not overtly breach the integrity of theoretical entitlement to welfare promised by
statute and supported by legal precedent. Rather, it seemed designed to protect this promise. But in
practice, quality control appears to have initiated a process of effective disentitlement. Its adverse
effects were unmeasured and unobserved, leaving quality control‘s manifest legitimacy unimpaired.
Government institutions and officials were thus insulated from the effects of their actions. In this
sense, quality control ironically eroded the government accountability that it was ostensibly intended
to guarantee. Furthermore, through quality control, federal authorities could indirectly influence
state administrative practices without directly encroaching on areas of nominal state authority.
Performance measurement backed by fiscal sanctions proved to be a relatively potent, if imperfectly
cast, instrument for penetrating a decentralized bureaucracy.
1. All of the following are mentioned in the passage by the author as adverse effects of
quality control EXCEPT:

A. undue emphasis on administrative paperwork and procedures.


B. arbitrary and inconsistent penalties for state welfare agencies.
C. a decrease in the number of people who were eligible for welfare benefits.
D. lack of accountability for certain systematic infringements of the welfare system.
E. initiation of a process of effective dis-entitlement

2. In paragraph 4, the phrase ―uncoordinated responses of street-level bureaucrats‖ is


used in order to:

A. support the author‘s claim that unreasonable administrative procedures caused many applicants
to be denied welfare benefits.
B. refute the theory that quality control was used to hold states to a higher standard of
accountability in their fiscal administration.
C. prove that quality control policies were implemented to serve a political rather than a social
agenda.
D. provide a potential reason for the ineffectiveness of performance monitoring on
general welfare reform.
E. criticise bureaucrats for the state of affairs with regards to quality control

3. What does the author of the passage suggest about the use of common managerial
tools to effect policy changes in the welfare system?

A. Procedural changes in welfare agencies should be established in ways that assure adherence to
regulations for both workers and applicants.
B. Administrative reform methods like performance monitoring may cause welfare
organizations to become overly restrictive in their policies.
C. State payments and federal reimbursement funding can be effectively monitored through changes
in welfare administration at the national level.
D. Implementation of quality control methods helped to hold the federal government accountable
for its actions.
E. Such tools have completely failed to effect policy changes in the past

Secure (v) : đảm bảo

Legislative (adj) : lập pháp

Reform (n/v) : cải cách

Comprehensive (adj) : toàn diện

Contentious (adj) : gây tranh cãi

Entitled (adj) : được phép

Defy (v) : bất chấp

Resolution (n) : nghị quyết


Provision (n) : điều khoản

Fiscal (adj) : tài chính

Accountability (n) : trách nhiệm giải trình

Reimbursement (n) : hoàn trả, bồi thường

Deem (v) : coi là

Improper (adj) : không đúng

Latent (adj) : ngầm, kín đáo

Penalize (v) : phạt

Ineligible (adj) : không đủ điều kiện

Bureaucrat (n) : quan chức

Impose (v) : áp đặt, bắt buộc

Uniformity (n) : tính đồng nhất

Procedural (adj) : thủ tục

Liberality (n) : tự do

Stringency (n) : nghiêm ngặt, minh bạch

Virtually (adv) : hầu như

Discretion (n) : tự do hành động, tùy ý

Overly (adv) : công khai

Breach (v) : vi phạm

Integrity (n) : chính trực, thanh liêm

Precedent (n) : tiền lệ

Initiate (v) : khởi xướng

Adverse (adj) : bất lợi, trái ngược

Legitimacy (n) : tính hợp pháp

Unimpaired (adj) : không bị suy yếu

Insulate (v) : tách riêng ra

Ironically (adv) : trớ trêu thay

Erode (v) : xói mòn

Encroach (v) : lạm dụng, vượt quyền


Sanction (n) : chế tài

Potent (n) : tiềm năng

Penetrate (v) : thâm nhập, hiểu rõ

Bureaucracy (n) : quan liêu

Cast (v/n/adj) : đúc/ bỏ phiếu/ sự đổ vào khuôn

Adherence (n) : tuân thủ

Infringement (n) : xâm phạn, vi phạm

Arbitrary (adj) : tùy ý, độc đoán

Inconsistent (adj) : không nhất quán

Emphasis (n) : nhấn mạnh


Feb 18 – 2

In all industrial countries, the seventies were a time of slowdown in real economic growth. In some
countries, people resisted the decline in real income growth and tried to maintain their accustomed
growth in living standards by demanding higher wages and salaries. In other countries, where there
was a higher degree of ‘social consensus’, it was recognized that such a wage-price spiral would be
damaging to the society as a whole, and people accepted the lower real income growth without
demanding a pushing up of wages and prices. In testing this hypothesis, researchers examined the
experience of eighteen industrial countries in the seventies. It was seen that the differences in the
inflation performances of these countries were well-explained by the indicator representing the
degrees of social consciousness.

Two rationales are advanced for the role of social consensus. The first is based on conflict over the
distribution of a known loss in aggregate real income, while the second is based on imperfect
information regarding the size of the loss. The two rationales are not mutually exclusive, but the first
represents a greater departure from economic orthodoxy than the second. Let us first suppose that
there is a big increase in the price of imported oil and that the size of the resulting loss of aggregate
real income is known to all. In countries having a high degree of consensus, each group of
individuals may be willing to scale down its claim on output in the same proportion as the aggregate
reduction. On the other hand, in countries lacking such consensus, there may be an unwillingness to
scale down demands. In the latter case, inflationary pressures will emerge even though the process
is ultimately likely to be counterproductive or irrational from the point of view of the society as a
whole.

According to the second rationale, workers demand no more than what is warranted by productivity,
and if they know that productivity has fallen or oil prices have increased, they are willing to accept a
lower wage. However, workers will not generally have enough information to know the value of their
contribution to production, and they will not necessarily believe statements by the firm that
productivity growth has declined. They will not be certain whether such statements are true or
whether these statements represent attempts by the firm to cheat them. Statements by the firms
will have greater credibility where the firms have established a reputation for fairness than where
there is a high degree of mistrust between workers and employers. The greater the degree of trust,
the less will be the workers’ resistance to reduction in the level or growth rate of real wage.

Both rationales suggest that real wages may be relatively inflexible, at least in downward direction,
when there is a lack of social consensus.

1. Which of the following can be concluded from the information given in the passage?

A. Economic slowdown causes inflation.


B. Unwillingness to scale down demands in the event of a known loss in aggregate real income
prevents social consensus.
C. People's acceptance of lower real income growth can prevent inflation.
D. Reliable indicators of social consciousness exist.
E. Workers are generally resistant to reduction in the level or growth rate of real wage.
2. Which one of the following will most strengthen the author's viewpoint that lack of
social consensus makes wages inflexible?

A. An agreement among workers that wage-price spiral is damaging to society as a whole.


B. A report that concludes that the high inflation in a given country is a result of its workers'
unwillingness to accept lower wages.
C. Empirical evidence that the greater the mistrust between workers and employers, the more the
inflation in the country.
D. A news report that a workers' union refused a wage-reducing proposal by its
management.
E. A research indicating that inflationary pressures are likely to be counterproductive for a nation's
economy.

3. The authors believe which of the following regarding the two rationales advanced for
the role of social consensus?

I. Both the rationales challenge traditional economics.


II. Both the rationales explain the role of social consensus in inflation performance.
III. The rationales can co-exist.

A. I only
B. II only
C. II and III only
D. I and II only
E. I, II, and III

From the line mentioned below it is clear that both rationales challenged the traditional one while
the magnitude of difference was more in first one.
"The two rationales are not mutually exclusive, but the first represents a greater departure
from economic orthodoxy than the second"

Resist (v) : chống lại

Accustomed (adj) : thói quen

Consensus (n) : sự đồng thuận

Spiral (n) : đường xoắn ốc, vòng xoắn

Rationale (n) : lý do

Not mutually exclusive (adj) : không loại trừ lẫn nhau

Orthodoxy (n) : chính thống

Irrational (adj) : phi lý


Feb 19 – 1

VO2 max is more than merely a trendy buzzword for workout aficionados to bandy about. As
athletics grows ever more scientific in orientation, measurable indicators of performance grow in
importance. For the endurance athlete, the ability to keep muscles supplied with oxygen is currently
viewed as the single most effective indicator of ability. This measurement, VO2 max, can range from
40 ml/kg/min for an average fit young adult to in excess of 90 ml/kg/min for a cardiovascular
superstar like Lance Armstrong.

For sprinters, on the other hand, oxygen replenishment is a peripheral concern since the short
duration of the activity ensures that almost all energy production relies on the phosphagen or lactic
acid processes. While the exact method of energy production shifts after approximately thirty
seconds of activity, this general rule holds true for any athlete where the period of exertion is less
than approximately two minutes. Short distance runners and swimmers, weight lifters, and those
participating in team sports where individual effort varies over time are all examples of athletes for
whom oxygen uptake primarily affects the length of the recovery period following exertion. In these
cases a training program centered on VO2 max may not provide optimum results.

Once this initial period of exertion is over, however, energy production shifts to the aerobic process
of glycogen metabolism and, eventually, fat metabolism. It is at this point that the ability of the
body to supply muscle tissue with oxygen, as measured by VO2 max, becomes a key component.
Training efforts in this area are highly variable, with studies showing negligible improvement in
some participants but a doubling of VO2 max in others. The average VO2-max-centered training
program sees approximately 17% improvement in measured capacity. The detailed mechanism that
is at work in the transfer of oxygen to muscle tissue remains poorly understood. Lung capacity,
circulatory capacity, blood chemistry, and as yet unknown characteristics of individual muscle cells
may all play a role.

(1) Assuming that the information contained within this article is accurate, which of the
following claims about VO2 max can be most reasonably made?

(A) Some individuals have more than twice the VO2 max of an average fit young adult.
(B) It depends primarily on unexplained properties of individual muscle cells.
(C) It contributes to the metabolism of lactic acid and phosphagen.
(D) The mechanism of VO2 max shifts after thirty seconds of activity.
(E) It can be increased by up to 17% through a focused training program.

(E) But the passage does not state that.

The passage says:"The average VO2-max-centered training program sees APPROXIMATELY 17%
improvement in measured capacity."

So nowhere in the passage says that it can be increased UP TO 17%. It can be 17,000001 or 17,3.
The limit is not 17,0000000%.

(2) In the passage, the author's opinion regarding VO2 max could best be characterized
by which of the following statements?
(A) It is a useful indicator of performance for endurance athletes but should be ignored by those
with events lasting over thirty seconds. The idea after "but" is out of scope
(B) It is merely a trendy buzzword in the athletic community.
(C) At least some of its popularity may be due to the increasingly scientific orientation
of sports training.

"VO2 max is more than merely a trendy buzzword for workout aficionados to bandy
about. As athletics grows ever more scientific in orientation, measurable indicators of
performance grow in importance"

(D) It is a good indicator of the performance of distance runners, cyclists, and weightlifters.
(E) Eventually its popularity as a performance indicator will be eclipsed by other measures.

(3) A top-level sprinter can run 400 meters in approximately 45 seconds. Based on this
information and that contained within the article, assuming that it is correct, which of
the following claims can be most reasonably made?
(A) During a 400 meter sprint, a top level athlete's primary energy source is some
combination of the phosphagen and lactic acid processes.
(B) VO2 max only plays a role in the final stages of a 400 meter race when run by a top-level
sprinter.
(C) Fat metabolism plays a significant role in the performance of a top-level sprinter during a 400
meter race.
(D) VO2 max plays no role in the recovery period at the conclusion of a top-level sprinter's 400
meter race.
(E) No top-level sprinter should consider VO2 max in his or her training.

The question focuses on 400 meter top sprinter, not VO2 max

(4) According to the information in this article, which of the following can be inferred
about the role of VO2 max in exercise?
(A) It is entirely unimportant to sprinters.
(B) It plays a greater role early in exertion.
(C) It should not be a part of the training regimen of a professional football player.
(D) It is the only area of fitness that is scientifically measured.
(E) It is a strong indicator of performance for endurance athletes.

Buzzword (n) : từ thông dụng

Aficionado (n) : người hâm mộ - a person who is very knowledgeable and enthusiastic about an activity, subject or
pastime

Knowledgeable (adj) : có kiến thức

Bandy (v) : nói đùa, bàn luận

Endurance (n) : sức bền

Cardiovascular (adj) : tim mạch

Sprinter (n) : chạy nước rút

Replenishment (n) : sự bổ sung, cung cấp

Exertion (n) : gắng sức

Negligible (adj) : không đáng kể


Feb 19 – 2

A conventional view of nineteenth-century Britain holds that iron manufacturers and textile
manufacturers from the north of England became the wealthiest and most powerful people in
society after about 1832. According to Marxist historians, these industrialists were the target of the
working class in its struggle for power. A new study by Rubinstein, however, suggests that the real
wealth lay with the bankers and merchants of London. Rubinstein does not deny that a northern
industrial elite existed but argues that it was consistently outnumbered and outdone by a London-
based commercial elite. His claims are provocative and deserve consideration.

Rubinstein’s claim about the location of wealth comes from his investigation of probate records.
These indicate the value of personal property, excluding real property (buildings and land), left by
individuals at death. It does seem as if large fortunes were more frequently made in commerce than
in industry and, within industry, more frequently from alcohol or tobacco than from textiles or metal.
However, such records do not unequivocally make Rubinstein’s case. Uncertainties abound about
how the probate rules for valuing assets were actually applied. Mills and factories, being real
property, were clearly excluded: machinery may also have been, for the same reason. What the
valuation conventions were for stock-in-trade (goods for sale) is also uncertain. It is possible that
their probate values were much lower than their actual market value: cash or near-cash, such as
bank balances or stocks, were, on the other hand, invariably considered at full face value. A further
complication is that probate valuations probably took no notice of a business’s goodwill (favor with
the public) which, since it represents expectations about future profit-making, would today very
often be a large fraction of market value. Whether factors like these introduced systematic biases
into the probate valuations of individuals with different types of businesses would be worth
investigating.

The orthodox view that the wealthiest individuals were the most powerful is also questioned by
Rubinstein’s study. The problem for this orthodox view is that Rubinstein finds many millionaires
who are totally unknown to nineteenth-century historians: the reason for their obscurity could be
that they were not powerful. Indeed, Rubinstein dismisses any notion that great wealth had
anything to do with entry into the governing elite, as represented by bishops, higher civil servants,
and chairmen of manufacturing companies. The only requirements were university attendance and a
father with a middle-class income.

Rubinstein, in another study, has begun to buttress his findings about the location of wealth by
analyzing income tax returns, which reveal a geographical distribution of middle-class incomes
similar to that of wealthy incomes revealed by probate records. But until further confirmatory
investigation is done, his claims can only be considered partially convincing.
1. The main idea of the passage is that

(A) the Marxist interpretation of the relationship between class and power in nineteenth-century
Britain is no longer viable
(B) a simple equation between wealth and power is unlikely to be supported by new data from
nineteenth-century British archives
(C) a recent historical investigation has challenged but not disproved the orthodox view
of the distribution of wealth and the relationship of wealth to power in nineteenth-
century Britain
(D) probate records provide the historian with a revealing but incomplete glimpse of the extent and
location of wealth in nineteenth-century Britain
(E) an attempt has been made to confirm the findings of a new historical study of nineteenth-
century Britain, but complete confirmation is likely to remain elusive

2. The author of the passage implies that probate records as a source of information
about wealth in nineteenth-century Britain are

(A) self-contradictory and misleading


(B) ambiguous and outdated
(C) controversial but readily available
(D) revealing but difficult to interpret
(E) widely used by historians but fully understandable only by specialists

3. The author suggests that the total probate valuations of the personal property of
individuals holding goods for sale in nineteenth-century Britain may have been

(A) affected by the valuation conventions for such goods


(B) less accurate than the valuations for such goods provided by income tax returns
(C) less, on average, if such goods were tobacco-related than if they were alcohol-related
(D) greater, on average, than the total probate valuations of those individuals who held bank
balances
(E) dependent on whether such goods were held by industrialists or by merchants or bankers

4. According to the passage, Rubinstein has provided evidence that challenges which
one of the following claims about nineteenth-century Britain?

(A) The distribution of great wealth between commerce and industry was not equal.
(B) Large incomes were typically made in alcohol and tobacco rather than in textiles and metal.
(C) A London-based commercial elite can be identified.
(D) An official governing elite can be identified.
(E) There was a necessary relationship between great wealth and power.

5. The author mentions that goodwill was probably excluded from the probate valuation
of a business in nineteenth-century Britain most likely in order to

(A) give an example of a business asset about which little was known in the nineteenth century
(B) suggest that the probate valuations of certain businesses may have been significant
underestimations of their true market value
(C) make the point that this exclusion probably had an equal impact on the probate valuations of all
nineteenth-century British businesses
(D) indicate that expectations about future profit-making is the single most important factor in
determining the market value of certain businesses
(E) argue that the twentieth-century method of determining probate valuations of a business may
be consistently superior to the nineteenth-century method

6. Which one of the following studies would provide support for Rubinstein’s claims?

(A) a study that indicated that many members of the commercial elite in nineteenth-century London
had insignificant holdings of real property
(B) a study that indicated that in the nineteenth century, industrialists from the north of England
were in fact a target for working-class people
(C) a study that indicated that, in nineteenth-century Britain, probate values of goods for sale were
not as high as probate values of cash assets
(D) a study that indicated that the wealth of nineteenth-century British industrialists did
not appear to be significantly greater when the full value of their real property holdings
was actually considered
(E) a study that indicated that at least some members of the official governing elite in nineteenth-
century Britain owned more real property than had previously been thought to be the case

This question basically asks what can we use to strengthen the claims? so for that we need to find
the weakness which is given in the q3! that is the uncertainity in calculating probate values!

Choice d suggests thats 2 variables of studying probate value does not give a lot of difference so it
can be trusted for the difference is minimal and hence the claims can be accepted?

7. Which one of the following, if true, would cast the most doubt on Rubinstein’s
argument concerning wealth and the official governing elite in nineteenth-century
Britain?

(A) Entry into this elite was more dependent on university attendance than on religious background.
(B) Attendance at a prestigious university was probably more crucial than a certain minimum family
income in gaining entry into this elite.
(C) Bishops as a group were somewhat wealthier, at the point of entry into this elite, than were
higher civil servants or chairmen of manufacturing companies.
(D) The families of many members of this elite owned few, if any, shares in iron industries and
textile industries in the north of England.
(E) The composition of this elite included vice-chancellors, many of whom held office
because of their wealth.

Composition (n) : thành phần

Vice-chancellor (n) : phó hiệu trưởng

Struggle (n) : đấu tranh

Outnumber (adj) : đông hơn

Outdone (v) : thua kém

Industrialist (n) : nhà công nghiệp


Provocative (adj) : trêu chọc, khiêu khích

Probate record (n) : hồ sơ chứng thực

Unequivocally (adv) : dứt khoát

Probate (adj) : quản chế

Mill (n) : xưởng chế tạo

Convention (n) : quy ước, hiệp ước

Valuation (n) : định giá

Invariably (adv) : luôn luôn

Goodwill (n) : thiện chí

Fraction (n) : miếng nhỏ, phần, phân số

Bias (n) : thiên vị

Orthodox (adj) : chính thống

Obscurity (n) : sự không có tên tuổi, tình trạng ít người biết đến

Dismiss (v) : bỏ qua

Governing elite (n) : giới cầm quyền

Bishop (n) : giám mục

Civil servant (n) : quan chức

Buttress (v) : làm cho lý lẽ thêm vững chắc

Insignificant (adj) : tầm thường


Feb 20 – 1

Like our political society, the university is under severe attack today and perhaps for the same
reason; namely, that we have accomplished much of what we have set out to do in this generation,
that we have done so imperfectly, and while we have been doing so, we have said a lot of things
that simply are not true. For example, we have earnestly declared that full equality of opportunity in
universities exists for everyone, regardless of economic circumstance, race, or religion. This has
never been true. When it was least true, the assertion was not attacked. Now that it is nearly true,
not only the assertion but the university itself is locked in mortal combat with the seekers of
perfection. In another sense the university has failed. It has stored great quantities of knowledge; it
teaches more people; and despite its failures, it teaches them better. It is in the application of this
knowledge that the failure has come. Of the great branches of knowledge—the sciences, the social
sciences, and the humanities—the sciences are applied, sometimes almost as soon as they are
learned. Strenuous and occasionally successful efforts are made to apply the social sciences, but
almost never are the humanities well applied. We do not use philosophy in defining our conduct.

We do not use literature as a source of real and vicarious experience to save us the trouble of living
every life again on our own.The great tasks of the university in the next generation are to search
the past to form the future, to begin an earnest search for a new and relevant set of values, and to
learn to use the knowledge we have for the questions that come before us. The university should
use one-fourth of a student’s time in his undergraduate years and organize it into courses which
might be called history, and literature and philosophy, and anything else appropriate and organize
these around primary problems. The difference between a primary problem and a secondary or even
tertiary problem is that primary problems tend to be around for a long time, whereas the less
important ones get solved. One primary problem is that of interfering with what some call human
destiny and others call biological development, which is partly the result of genetic circumstance and
partly the result of accidental environmental conditions. It is anticipated that the next generation,
and perhaps this one, will be able to interfere chemically with the actual development of an
individual and perhaps biologically by interfering with his genes. Obviously, there are benefits both
to individuals and to society from eliminating, or at least improving, mentally and physically
deformed persons.

On the other hand, there could be very serious consequences if this knowledge were used with
premeditation to produce superior and subordinate classes, each genetically prepared to carry out a
predetermined mission. This can be done, but what happens to free will and the rights of the
individual? Here we have a primary problem that will still exist when we are all dead. Of course, the
traditional faculty members would say, “But the students won’t learn enough to go to graduate
school.” And certainly they would not learn everything we are in the habit of making them learn, but
they would learn some other things. Surely, in the other three-quarters of their time, they would
learn what they usually do, and they might even learn to think about it by carrying new habits into
their more conventional courses. The advantages would be overwhelmingly greater than the
disadvantages. After all, the purpose of education is not only to impart knowledge but to teach
students to use the knowledge that they either have or will find, to teach them to ask and seek
answers for important questions.
1. The author suggests that the university’s greatest shortcoming is its failure to

(A) attempt to provide equal opportunity for all


(B) offer courses in philosophy and the humanities
(C) prepare students adequately for professional studies
(D) help students see the relevance of the humanities to real problems
(E) require students to include in their curricula liberal arts courses

2. It can be inferred that the author presupposes that the reader will regard a course in
literature as a course

(A) with little or no practical value


(B) of interest only to academic scholars
(C) required by most universities for graduation
(D) uniquely relevant to today’s primary problems
(E) used to teach students good writing skills

3. Which of the following questions does the author answer in the passage?

(A) What are some of the secondary problems faced by the past generation?
(B) How can we improve the performance of our political society?
(C) Has any particular educational institution tried the proposal introduced by the author?
(D) What is a possible objection to the proposal offered in the passage?
(E) Why is the university of today a better imparter of knowledge than the university of the past?

4. Which of the following questions would the author most likely consider a primary
question?

(A) Should Congress increase the level of Social Security benefits?


(B) Is it appropriate for the state to use capital punishment?
(C) Who is the best candidate for president in the next presidential election?
(D) At what month can the fetus be considered medically viable outside the mother’s womb?
(E) What measures should be taken to solve the problem of world hunger?

The difference between a primary problem and a secondary or even tertiary problem is that primary
problems tend to be around for a long time, whereas the less important ones get solved.

Here we have a primary problem that will still exist when we are all dead.

This is an application question. The author uses the term “primary problems” to refer to questions of
grave importance that are not susceptible to an easy answer. Each of the incorrect answers poses a
question that can be answered with a short answer. (A) can be answered with a yes or no. (C) can
be answered with a name. (D) can be answered with a date. (E) can be answered with a series
of proposals. And even if the answers are not absolutely indisputable, the questions will
soon become dead issues. The only problem that is likely to still be around after “we are
all dead” is the one of capital punishment.
Severe (adj) : nghiêm trọng

Namely (adv) : cụ thể là

Earnestly (adv) : tha thiết

Humanity (n) : nhân văn

Strenuous (adj) : nỗ lực

Vicarious (adj) : gián tiếp

Earnest (adj) : đúng đắn

Tertiary (adj) : hạng thứ ba

Interfere (v) : can thiệp

Anticipate (v) : dự đoán

Deform (v) : biến dạng

Hunger (n) : nạn đói

Fetus (n) : thai nhi

Womb (n) : tử cung

Capital punishment (n) : hình phạt tử hình


Feb 21 – 1

A sanctuary may be defined as a place where Man is passive and the rest of Nature active. Till quite
recently Nature had her own sanctuaries, where man either did not go at all or only as a tool-using
animal in comparatively small numbers. But now, in this machinery age, there is no place left where
man cannot go with overwhelming forces at his command. He can strangle to death all the nobler
wild life in the world to-day. To-morrow he certainly will have done so, unless he exercises due
foresight and self-control in the mean time.

There is not the slightest doubt that birds and mammals are now being killed off much faster than
they can breed. And it is always the largest and noblest forms of life that suffer most. The whales
and elephants, lions and eagles, go.The rats and flies, and all mean parasites, remain. This is
inevitable in certain cases. But it is wanton killing off that I am speaking of to-night. Civilized man
begins by destroying the very forms of wild life he learns to appreciate most when he becomes still
more civilized. The obvious remedy is to begin conservation at an earlier stage, when it is easier and
better in every way, by enforcing laws for close seasons, game preserves, the selective protection of
certain species, and sanctuaries.

I have just defined a sanctuary as a place where man is passive and the rest of Nature active. But
this general definition is too absolute for any special case. The mere fact that man has to protect a
sanctuary does away with his purely passive attitude. Then, he can be beneficially active by
destroying pests and parasites, like bot-flies or mosquitoes, and by finding antidotes for diseases like
the epidemic which periodically kills off the rabbits and thus starves many of the carnivora to death.
But, except in cases where experiment has proved his intervention to be beneficial, the less he
upsets the balance of Nature the better, even when he tries to be an earthly Providence.

1. The author implies that his first definition of a sanctuary is

A. totally wrong
B. somewhat idealistic
C. unhelpful
D. indefensible
E. immutable

2. The authors argument that destroying bot-flies and mosquitoes would be a beneficial
action is most weakened by all of the following except

A. parasites have an important role to play in the regulation of populations


B. the elimination of any species can have unpredictable effects on the balance of nature
C. the pests themselves are part of the food chain
D. these insects have been introduced to the area by human activities
E. elimination of these insects would require the use of insecticides that kill a wide range of insects
3. It can be inferred that the passage is

A. part of an article in a scientific journal


B. extracted from the minutes of a nature club
C. part of a speech delivered to an educated audience
D. a speech delivered in a court of law
E. from a polemical article published in a magazine

4. The purpose of the final paragraph is

A. to sum up the main points of the authors argument


B. to urge a solution to an increasingly pressing problem
C. to qualify the authors definition of an important term
D. to propose a program
E. to suggest that man should not intervene in natural environments

Sanctuary (n) : khu bảo tồn, nơi thiêng liêng, nơi trú ẩn

Strangle (v) : bóp nghẹt

Nobler (adj) : vĩ đại, quý tộc

Foresight (n) : tầm nhìn xa

Inevitable (adj) : chắc chắn xảy ra

Wanton (adj) : bừa bãi

Absolute (adj) : tuyệt đối

Purely passive attitude (adv/adj/n) : thái độ hoàn toàn thụ động

Beneficially (adv) : có lợi

Antidote (n) : thuốc giải độc

Epidemic (n) : bệnh truyền nhiễm

Periodically (adv) : định kỳ

Carnivora (n) : thú ăn thịt

Earthly (adj) : trần gian

Providence (n) : tạo hóa

Immutable (adj) : bất biến


Feb 21 - 2

The ‘trophic contamination hypothesis’ posits that shorebirds accumulate industrial and urban
pollution at stopover sites, toxins that are subsequently released in sudden high doses as fat is
burned during migratory flights, disrupting the bird’s ability to make migratory decisions. For
example, large contaminant doses might hamper refueling by altering the satiation signal in
shorebirds so that they do not accumulate sufficient fat for migration. A recent study found that, out
of those shorebirds that were unable to migrate, some weighed as much as 20% less than the
average migrating bird of their species. Whether such findings are a result of shorebirds suffering
from trophic contamination, or whether such birds simply cut their migrations short by landing in a
foreign ecosystem, is unlikely to be resolved until further studies are conducted.

One promising line of research involves organochlorines, toxins deposited on mudflats in the 1970s
and 1980s, now buried by sediments but finally close enough to the surface to be of issue to
foraging shorebirds. Organochlorines should be more accessible to long-billed shorebirds that probe
deeply for prey than to short-billed species that forage at or near the surface. We predict that an
increased number of long-billed shorebirds will either be unable to migrate or will be found along an
aberrant flight path

1. The most immediate effect on birds that have accumulated toxins in their fat deposits
is

A a tendency to navigate along a divergent flight path


B an inability to realize when they have eaten a sufficient amount of food
C a diminished capacity to retrace their migratory route
D an increased likelihood to exhibit aggression towards other species
E an increased likelihood to exhibit aggression towards other species

2. According to the passage, the long-billed shorebird is expected to be more likely than
the short-billed shorebird to have trouble migrating because

A it is more vulnerable to the effects of organochlorines


B it typically is unable to differentiate between a foreign ecosystem and a familiar one
C it stops feeding before it is fully satiated
D it grazes in parts of the mudflat in which the surface is known to have a higher concentration of
organochlorines
E it digs deeper into the earth and is therefore more likely to encounter toxins

3. the author implies that foreign ecosystems have which potential effect on shorebirds?

A They can make a bird more vulnerable to predators.


B They can expose shorebirds to foreign toxins.
C They can diminish a bird’s ability to navigate.
D They can lead to a reduction in the bird’s weight.
E They can cause a bird to become separated from its flock.
Trophic (adj) : dinh dưỡng

Posit (v) : đặt ra

Accumulate (v) : tích trữ

Stop-over site (adj/n) : điểm dừng chân

Migratory (adj) : di cư

Hamper (v) : cản trở

Satiation (n) : sự thỏa mãn, sự bão hòa

Organochlonrine (n) : clo hữu cơ

Mudflat (n) : bãi bồi

Sediment (n) : trầm tích

Forage (v) : thức ăn gia xúc

Probe (v) : thăm dò

Aberrant (adj) : khác thường, lạc đường

Long-billed (adj) : mỏ dài

Flock (n) : bầy đàn

Graze (v) : ăn cỏ, chăn thả


Feb 22 – 1

In 1978, as public reaction to the Love Canal landfill leak snowballed, the Environmental Protection
Agency counted 425,000 industrial plants across the country jointly producing 45 million tons of
toxic waste per year. Well over 50 percent of these wastes were not being disposed of according to
existing public health standards. In November 1981, a ban was placed on burial of toxic wastes. This
ban provided at best a stopgap to permit debate on long-term approaches to landfill regulation. At
the same time, it exacerbated the problem of illicit abandonment of wastes. Unmarked barrels of
toxic products, which could not be opened to determine their contents, were abandoned in deserted
factories by bankrupt manufacturing firms or dropped off in empty fields by fly-by-night disposal
companies, which multiplied as an indirect result of the ban.

The underlying reason for industry irresponsibility with regard to waste is economic. The Chemical
Manufacturers’ Association denounced the landfill ban as unworkable and costly, while contending
that even the currently permissible landfill techniques are unrealistically capital-intensive.
Specifically, industry representatives claim that the EPA requires unnecessarily frequent and
extensive site monitoring.

The Hazardous Waste Treatment Council, which represents companies involved in recovering,
recycling, incinerating, biodegrading, evaporating, and chemically treating wastes, supports
permanent landfill restrictions. The processes utilized by these companies face relatively little public
opposition. Their sites tend to look from the outside like normal industrial plants, in contrast to the
fenced-off "fallout zone” appearance of a landfill. Less appealing activities, such as landfarming
(whereby wastes are handled in the traditional manner of horse manure—mixed with soil, aerated,
and biodegraded by microorganisms) are generally located well away from cities. Most important,
these processes leave no toxic end product which must be containerized and stored underground.

These procedures span a wide range of availability and cost-effectiveness. The electroplating
industry, for example, produces some acidic wastes that can be neutralized by combination with
everyday lime. At the opposite end of the cost-effectiveness spectrum, ion exchange
chromatography provides an extremely costly, though safe, solution for other plating effluvia.
Cooperative recycling, whereby a waste treatment company utilizes the wastes of one
manufacturing process to neutralize those produced by another industry, has proved a great success
in Germany. Its development in the United States has, unfortunately, been retarded by secrecy
about product components.
Even if all known waste alternatives were operating at maximum potential, they still would not
obviate the need for landfill. They would, however, provide a partial solution for the petrochemical
and chemical manufacturing industries in the face of regulatory stringency.

1. It can be inferred that the petrochemical and chemical manufacturing industries.

A. have offered compelling arguments against EPA regulations


B. have largely relied until now on containerization and burial to dispose of toxic wastes
C. are responsible for more public health violations than other industries
D. are opposed on principle to government regulation of waste disposal
E. are more competitive and secretive about their processes than most U.S. industries
Even if all known waste alternatives were operating at maximum potential, they still would not
obviate the need for landfill. They would, however, provide a partial solution for the petrochemical
and chemical manufacturing industries in the face of regulatory stringency.

 Petrochemical and chemical manufacturing industries still use landfill to dispose their waste
( burial of toxic wastes )

2. According to the passage, on which issue are the positions of the Chemical
Manufacturers' Association and the Hazardous Waste Treatment Council opposed?

A. The viability of the EPA ban on burial of containerized waste


B. The necessity of containerization and underground storage
C. The necessity of stringent EPA landfill guidelines
D. The desirability of developing alternatives to current landfill technology
E. The feasibility of using recovery/recycling processes in the U.S.

It is a Detail Question, In the second paragraph, the passage states that the CMA is opposed to the
landfill ban for economic reasons. In the third paragraph, the passage reveals that
HWTC "supports permanent landfill restrictions." Since landfill disposal requires burial, (A)
matches the prediction and is the correct choice.

(B) is incorrect because neither side actually disputes the need for some containerization and burial.
The CMA is all for it, while the HWTC uses processes that do not require it.

(C) is tricky. The CMA does dispute the specific guidelines, but the author never says that it
disputes the necessity for some such guidelines.

Both (D) and (E) focus on alternatives to current landfill and recycling techniques, which the CMA
never discusses in the passage.

3.The alternatives to landfill mentioned in the third and fourth paragraphs are
characterized by all of the following EXCEPT that.

A. they are not a major focus of conservationist protests

B. some are not widely available in this country

C. they require secrecy about product components

D. their end products are nontoxic


E. some are unrealistically expensive

4 According to the passage, all of the following statements are true of landfills EXCEPT
that.

A. they were temporarily banned in late 1981


B. the Chemical Manufacturers' Association opposes the present laws regulating them
C. they tend to present an unsightly appearance
D. their use is impeded by industrial secretiveness
E. they cannot be completely eliminated at this time
Landfill (n) : bãi rác

Dispose (v) : xử lý

Burial (n) : chôn cất

Exacerbate (v) : trầm trọng hơn

Illicit (adj) : bất hợp pháp, trái phép

Abondonment (n) : vứt bỏ

Underlying (adj) : cơ bản

Unworkable (adj) : không khả thi

Permissible (adj) : cho phép, có thể chấp nhận

Unrealistically (adv) : phi thực tế

Incinerate (v) : thiêu hủy

Biodegrade (v): phân hủy sinh học

Evaporate (v) : bốc hơi

Fallout zone (n) : khu vực bụi phóng xạ

Manure (n) : phân để bón cây

Containerize (v) : đóng gói

Electroplate (v) : mạ điện

Acidic (adj) : có tính axit

Neutralize (v) : trung hòa

Lime (n) : vôi

Effluvia (n) : nước thải

Retard (v) : trì hoãn

Secrecy (n) : bí mật

Obviate (v) : cản trở

Feasibility (n) : tính khả thi

Viability (n) : khả năng tồn tại

Impede (v) : cản trở


Feb 22 – 2

There is an intriguing note to the current call upon civil rights law to help remedy the undervaluation
of women’s work. Until fairly recently, government was not expected to solve workers’ economic
grievances, however valid they might be. Many assumed that the responsibility lay with workers
themselves. Collective bargaining was the preferred instrument for pursuing pay equity for women.
Rather than call upon the law to regulate the market from the outside, one could try to reshape or
otherwise influence the market so that women themselves would be better able to address the
problem. This could be done by raising absolute wage levels in low paying, predominantly
female industries (such as retail clothing) or by changing the pay relationship between largely
female and largely male occupations within a single industry, such as auto manufacturing. Through
union representation, employees in traditionally female jobs in an industry could identify the actual
degree of underpayment of their work and then, as a group, pressure their employer to remedy it.
In addition, this process would encourage those affected-men and women alike-to be sensitive to
the limits of available resources, to be pragmatic about the pace at which the wage structure could
be revised.

I do not mean to suggest that collective bargaining is a foolproof means for closing the gender gap
in wages. To the extent that the problem involves the undervaluation of nonunion female
occupations in an otherwise unionized industry, political hurdles will discourage unionized employees
from supporting revisions in the wage structure. And to the extent that the problem is the
concentration of women in low-paying industries- textiles, for example-the product market imposes
serious economic constraints on a substantial closing of the wage gap.

Despite the imperfections of tools like collective bargaining for redressing wage disparities between
men and women, a reliance on law or government is favorable for neither individual firms nor our
economy as a whole. Nonetheless, although opponents of mandatory public remedies may correctly
fear those remedies as being a cure worse than the disease, they are wrong when they imply that
the current system of wage determination by business management is perfectly healthy.

1. In line 10 (Bolded), this most likely refers to

(A) increasing the wages of women and men in a single industry


(B) bringing about changes in market conditions
(C) changing the dynamic of collective bargaining
(D) relying on civil rights law to remedy economic grievances
(E) applying group pressure on an employer

2. According to the author, the process of unionization and collective bargaining could
do all of the following EXCEPT

(A) overcome market pressures that keep wages in some industries lower than in others
(B) encourage worker flexibility in adjusting a new pay scale to economic conditions
(C) help workers to apply group pressure on employers
(D) aid in determining the degree to which women are being underpaid
(E) sensitize workers to the limits of their industry’s ability to institute change
3. Which of the following best summarizes the author’s main point?

(A) Pay inequity for women exists because of the lack of unionization in traditionally female
occupations.
(B) Government regulation of industry to achieve pay equity for women is unnecessary because
management has the power to effectively determine wages.
(C) Unionization would solve all industry problems relating to the valuation of women’s work.
(D) Government regulation of women’s wages is necessary only in those industries where collective
bargaining is ineffective.
(E) Collective bargaining is preferable to government actions in redressing the
undervaluation of women’s work.

4. The author mentions textiles (Highlighted) in order to

(A) demonstrate the potential harm of government regulation of industry


(B) outline a strategy for achieving pay equity for women
(C) indicate how quickly employees can reasonably expect to achieve pay equity
(D) give an example of a situation in which collective bargaining may be ineffective
(E) show why civil rights laws are the most important tool for increasing women’s wages

5. It can be inferred that the author’s attitude toward opponents of government


regulation of wage determination mentioned in the last paragraph is characterized by
which of the following?

I. Distrust of their motives


II. Sympathy with some of their concerns
III. Disagreement with some of their assumptions
IV. Opposition to their political principles

(A) I only
(B) III only
(C) I and II only
(D) II and III only
(E) I, II, and IV

Nonetheless, although opponents of mandatory public remedies may correctly fear those remedies
as being a cure worse than the disease,…

The answer to this question is in the last paragraph, where the author refers to the “opponents” of
government regulation. The author says that they aren’t right about everything, although they are
right about the evils of government intervention. This confirms options II and III—the author is
sympathetic, but disagrees with part of their argument. Since only (D) includes both II and III, it
must be the correct answer. Statements I and IV suggest that the author’s hostile to the opponents.
The author generally agrees with them! The author takes issue with one point only.
6. In the final paragraph, the author addresses “opponents of mandatory public
remedies” (Highlighted) by

(A) arguing that those remedies would benefit the economy


(B) implying that alternative methods of correcting wage disparities would be worse
(C) asserting that the present approach to setting wages is flawed
(D) defending civil rights legislation as a solution to social problems
(E) insisting that those remedies are a viable means of correcting wage disparities

7. The passage refers to which of the following as reasons for preferring collective
bargaining to legislation as a method of ending the undervaluation of women’s work?

I. The greater responsiveness of collective bargaining to existing conditions that affect wage levels
II. The general desirability of using private rather than public remedies
III. The potential of collective bargaining for achieving a uniform national solution to the problem of
gender wage disparities

(A) I only
(B) III only
(C) I and II only
(D) II and III only
(E) I, II, and III

The answer to this question will appear in the first paragraph, which lists all the reasons collective
bargaining is good. Option I is implied at the end of the paragraph with “sensitive to the limits.”
Option II occurs at the beginning of the paragraph, which endorses self-help over civil rights law.
Option III, however, is not found here. In fact, option III appears in the discussion of the
weaknesses of collective bargaining in paragraph 2.

Intriguing (adj) : hấp dẫn

Grievance (n) ; bất bình

Collective (adj) : tập thể

Bargain (v) : thương lượng

Predominantly (adv) : chủ yếu

Degree (n) : mức độ

Pragmatic (adj) : thực dụng, căn cứ vào sự thật

Revise (v) : sửa đổi, xem lại

Foolproof (adj) : điên rồ, hoàn hảo

Hurdle (n) : rào cản

Impose (v) : áp đặt

Constaint (n) ; hạn chế


Redress (v) : khắc phục

Disparity (n) : sự chênh lệch

Reliance (n) : sự phụ thuộc

Mandatory (n) : bắt buộc

Opponent (n) : đối thủ, người phản đối

Responsiveness (n) : sự đáp ứng

Flaw (v) : thiếu sót

Desirability (n) : sự mong muốn


Feb 23 – 1

Prior to the fall of the Union of Soviet Socialist Republics (USSR), Mikhail Gorbachev, seeing a
country falling behind its Western rival and a people increasingly clamoring for change, addressed
the growing internal unrest in the summer of 1987 by introducing a series of reforms known as
perestroika (literally, restructuring). In Perestroika: New Thinking for Our Country and the World,
Mikhail Gorbachev discussed his analysis of the problems facing the USSR and his plans to solve
them. Perhaps the most pressing and visible problem facing the USSR in the last 1980s came in the
form of the country’s consistently mediocre economic performance, despite its vast natural resource
wealth and large labor force. Gorbachev flatly admitted that economic failures were increasing and
current policies were failing to offer a sustainable remedy. Failing to take advantage of the
numerous scientific and technological advancements available, the USSR relied on inefficient and
outdated business models. As a result, Gorbachev said, "in the last fifteen years the national income
growth rates had declined by more than a half and by the beginning of the eighties had fallen to a
level close to economic stagnation." With business executives focused on using more resources (in
order to employ more people) instead of becoming more efficient, the country produced poor quality
products unable to compete in a global economy. Further, this inefficiency led to shortages: "the
Soviet Union, the world’s biggest producer of steel, raw materials, fuel and energy, has shortfalls in
them due to wasteful or inefficient use."

The decrepit economy engendered social unrest and woe that only compounded economic
difficulties and societal misery. Gorbachev wrote of "a gradual erosion of the ideological and moral
values of our people" and noted the considerable growth in "alcoholism, drug addiction and crime."
Accentuating these difficulties, the Communist government often ignored the needs of the average
citizen, causing distrust and resentment. Perhaps the most destructive element of the social
unraveling and inadequate government response was the mediocre education system. Gorbachev
said, "Creative thinking was driven out from the social sciences, and superfluous and voluntarist
assessments and judgments were declared indisputable truths." Although Gorbachev also opined
about the growing public disbelief in the content of the immense government propaganda
campaigns, the extent to which economic underdevelopment and social deviance gripped Soviet
culture made the collapse of the USSR virtually inevitable in the minds of many observers. When
combined with glasnost (literally, openness), Gorbachev’s plan that allowed greater transparency,
perestroika actually served to hasten the collapse of the USSR. Contrary to its purpose, perestroika
ensured that the fall of the USSR would occur sooner rather than later. Only a few years after
Gorbachev implemented changes that would have been unthinkable and antithetical to the
philosophy of previous leaders like Lenin, Stalin, and Khrushchev, the USSR fell.

1 Which of the following best describes the primary objective of the passage?

A) Argue that the implementation of perestroika caused the fall of the Soviet Union
B) Explain perestroika along with its roots and consequences
C) Analyze the pros and cons of Mikhail Gorbachev's decision to implement perestroika
D) Explain the short-falls of a communist system and offer remedies
E) Discuss the role of Mikhail Gorbachev in propelling the USSR towards ceasing to exist
2 The passage implies that which of the following was most true of the Soviet economy
prior to perestroika:

A) Suffered from underperformance due to excessive government regulation and micro-


management
B) Failed to meet its potential as a result of corruption and bureaucratic overhead
C) Lacked adequate natural resources to grow efficiently, regardless of business management
D) Focused on achieving high-employment rather than export-capable products
E) With declining growth and stagnation, stood in the worst shape ever in USSR history

3 Based upon the passage, the author would likely agree most with which of the
following characterizations of the impact of the USSR's troubled economy during the
days leading up to perestroika?

A) Cause for renewed determination in communist philosophy


B) Reason that natives looked increasingly to the West and capitalism
C) Source of frustration and discomfort among citizens that fueled social friction
D) Justification for the USSR's neglect of the needs of many citizens
E) Primary cause of the USSR's poor educational system

4 According to the passage, which of the following best describes the relationship
between perestroika and the fall of the USSR?

A) Perestroika mildly delayed the fall of the USSR, although the decline of the Soviet Republic was
inevitable
B) Perestroika hastened the decline of the USSR
C) Perestroika enabled the USSR to pursue much needed restructuring
D) Perestroika softened the impact from the collapse of the USSR
E) Perestroika had little relationship to the decline of the USSR, which was inevitable anyway

5 In the context of the passage, the author most likely uses the word "unthinkable" (in
the last sentence) to help convey which of the following points about the changes
Gorbachev implemented in perestroika?

A) They would have never crossed the mind of Lenin as being conceptually possible, let alone
desirable or feasible
B) They would have been difficult for the mind of Lenin to comprehend intelligibly
C) They would have been seen by Lenin as undesirable and poor choices
D) They would have been seen by Lenin as incomprehensible yet appealing
E) They would have been considered highly desirable

Only a few years after Gorbachev implemented changes that would have been unthinkable and
antithetical to the philosophy of previous leaders like Lenin, Stalin, and Khrushchev, the USSR fell.
6 Gorbachev offers all of the following as evidence of the need for perestroika EXCEPT:

A) Shortages in natural resources due to inefficiency


B) Declines in economic output and growth
C) Slides in moral values of citizens
D) Erosion of new and ingenious thinking
E) Frustrations with the results of past reform efforts

7 Which of the following words best describes the passage's tone?

A) Primarily Analytical
B) Highly Critical
C) Frustrated
D) Not Objective
E) Deeply Introspective

Throughout the passage, the author is explaining perestroika, discussing why Gorbachev felt it was
necessary, and arguing about its impact on the fall of the USSR. The passage takes the tone of an
analytical essay. Further, the author appears to be rather objective, avoiding ad hominem attacks,
slurs, etc. and instead relying of quotations from Gorbachev himself.

Frustrated (adj) : bực bội

Introspective (adj) : hướng nội

Clamor (v) : kêu la

Internal unrest (adj/n) : bất ổn nội bộ

Mediocre (adj) : tầm thường

Sustainable (adj) : bền vững

Stagnation (n) : sự đình trệ

Shortfall (n) : thiếu sót

Decrepit (adj) : suy yếu

Engender (v) : gây ra

Woe (n) : buồn rầu

Accentuale (v) : nêu ra, nhấn mạnh

Assessment (n) : đánh giá

Superfluous (adj) : vô dụng, dư thừa

Indisputable (adj) : không thể chối cãi

Propaganda (n) : tuyên truyền, truyền giáo

Disbelief (n) : hoài nghi


Immense (adj) : nhiều

Deviance (n) : lệch lạc

Hasten (v) : hối thúc

Antithetical (adj) : phản diện, phản đối

Friction (n) : xích mích, bất hòa, ma sát

Comprehend (v) : hiểu

Intelligibly (adv) : thông minh

Incomprehensible yet appealing (adj/adv/adj) : không thể hiểu được / nhưng / hấp dẫn

Desirable (adj) : kỳ vọng, mong muốn, say mê


Feb 23 – 2

Unlike the United States with its generalissimo politicians - Washington, Jackson, Grant, and
Eisenhower- the ‘martial arts’ have been conspicuously absent from Canadian politics. The exception
to the rule is former Prime Minister Pierre Elliot Trudeau, who became the first Canadian leader to
bring a gunslinger ethos to Canadian politics. Trudeau introduced Canada to the refined art of single
combat; it was the politics of “doing it my way”. Single-combat confrontation implied much more
than the renegade in power did, and far less than the tricks of William Lyon Mackenzie King, prime
minister intermittently between 1921 and 1948.

Trudeau’s unique background prepared him for the role of authoritarian leader he would assume
later in life. Born on October 18, 1919, Trudeau lived in French-speaking Montreal, but heard English
at home from his mother, making it easy for the young politician to appeal to all sectors of Canada,
a bilingual country. As a young man, he walked and cycled through Europe, finding himself on
occasion on the wrong side of the bars in foreign jails. By 1940, Trudeau entered the law faculty at
the University of Montreal. As a student he enlisted in the Canadian Officers Training Corps, where
he was given a commission as a lieutenant, a rank he held until his retirement in 1947. Trudeau, a
renowned sportsman, held a brown belt in karate, knew how to skin dive and could descend 150
feet off a cliff with ease. He continued performing flamboyant physical feats even in later life as
Canada’s fifteenth prime minister, astounding Canadians with his prowess. The public’s adoration
made it possible for him to practice his personal brand of ‘do it my way’ politics, initiating profound
and long-lasting changes to his country.

Other leaders would never have undertaken to deal with such taboo issues as divorce, abortion and
homosexuality– matters likely to infuriate conservative Canada from coast to coast. Even the
powerful Mackenzie King dared not touch any of the three, though Trudeau tackled them together in
an omnibus bill as Minister of Justice under Lester B. Pearson. His reason for loosening legislation on
these issues was, as he put poetically put it, "The state has no business in the bedrooms of the
nation." The myths makers have it that this was Trudeau's first deliberate ‘gun slinging’ move,
performed with the ultimate goal of attaining national leadership. Contrary to popular belief,
Trudeau had no leadership aspirations at the time; all he had was a passion for combat that eclipsed
other religious considerations. Trudeau instigated far-reaching changes in legislation governing
divorce, abortion and homosexuality that have had a major impact on Canada, shaping the country
into what it is today.

1. The primary focus of the passage is on which of the following?

A. Comparing two Canadian prime ministers and contrasting their personal style of leadership
B. Describing the leadership style of one of Canada’s prime ministers
C. Evaluating the success of the leadership style of one of Canada’s prime ministers
D. Summarizing the contribution of one of Canada’s prime ministers
E. Tracing the long-term impact of legislation put forward by one of Canada’s prime ministers
2. It can be inferred from the passage that former Prime Minister William Lyon
Mackenzie King

A. Was opposed to abortion and put forward legislation making it illegal


B. Was opposed to abortion but did not put forward legislation making it illegal
C. Was in favor of abortion and put forward legislation making it legal
D. Was in favor of abortion but did not put forward legislation making it legal
E. Did not put forward legislation making abortion legal

3. The author of the passage mentions Trudeau’s accomplishments in sports primarily in


order to

A. Explain the source of Trudeau’s physical stamina


B. Illustrate that he had earned the adoration of the Canadian public
C. Contrast it to his personal brand of ‘do it my way’ politics
D. Provide one reason why he was able to single-handedly push through legislation that
should normally have shocked conservative Canada
E. Provide one reason why he was able to single-handedly thwart legislation that should normally
have shocked conservative Canada

4. The quotation "The state has no business in the bedrooms of the nation,"
(Highlighted)is most probably used to

A. Present the opinion that the state should have less of a say in issues that are
essentially not public matters
B. Present the opinion that the state should have less of a say in issues that are essentially public
matters
C. Present the opinion that the state should have more of a say in issues that are essentially not
personal matters
D. Provide an example of Pierre Elliot Trudeau’s flamboyant style that he used to captivate the
Canadian public
E. Contrast Pierre Elliot Trudeau’s flamboyant style with that of Lester B. Pearson

Generalissimo (n) : tổng thống lĩnh/ đại nguyên soái

Martial art (n) : võ thuật

Conspicously (adv) : dễ thấy

Ethos (n) : đạo đức, phong tục, tập quán

Gunslinger (n) : xạ thủ

Refine (adj) : có học thức, lỗi lạc, tinh chế

Confrontation (n) : đối đầu

Renegade (v/n) : bội ước/ người bội giáo, người phản đảng

Authoritarian (adj/n) : độc đoán, hống hách

Bilingual (adj) : song ngữ


Enlist (v) : nhập ngũ

Commission (n) : sự ủy quyền, giấy ủy quyền

Lieutenant (n) : trung úy

Cliff (n) : vách đá

Descend (v) : hạ xuống

Flamboyant (adj) : lòe loẹt, chói chang

Astound (v) : kinh ngạc

Prowess (n) : năng lực

Initate (v) : khởi xướng

Profound (adj) : thâm thúy, uyên thâm

Undertaken (v) : thực hiện

Taboo (adj) : cấm kỵ

Infuriate (v) : làm tức giận

Conservative (adj) : đảng viên bảo thủ, người theo đảng bảo thủ

Tackle (v) : giải quyết

Loosen (v) : nới lỏng

Deliberate (adj) : cố ý, chủ tâm

Eclipse (v) : lu mờ

Instigate (v) : khởi xướng

Far-reaching (adj) : sâu rộng, vươn xa

Thwrat (v) : cản trở


Feb 24 – 1

It would be enormously convenient to have a single, generally accepted index of the economic and
social welfare of the people of the United States. A glance at it would tell us how much better or
worse off we had become each year, and we would judge the desirability of any proposed action by
asking whether it would raise or lower this index. Some recent put must satisfy. One aspect,
population change, is now handled by converting output to a per capita basis on the assumption
that, other things equal, twice as many people need twice as many goods and services to be equally
well off. But an index of needs would also account for differences in the requirements for living as
the population becomes more urbanized and suburbanized; for the changes in national defense
requirements; and for changes in the effect of weather on our needs. The index would have to tell
us the cost of meeting our needs in a base year compared with the cost of meeting them equally
well under the circumstances prevailing in every other year. Measures of “needs” shade into
measures of the human and physical environment in which we live. We all are enormously affected
by the people around us. Can we go where we like without fear of attack? We are also affected by
the physical environment—purity of water and air, accessibility of parkland and other conditions. To
measure this requires accurate data, but such data are generally deficient. Moreover, weighting is
required: to combine robberies and murders in a crime index; to combine pollution of the Potomac
and pollution of Lake Erie into a water pollution index; and then to combine crime and water
pollution into some general index. But there is no basis for weighting these beyond individual
preference. There are further problems. To measure welfare we would need an index of the
“goodness” of the distribution of income. There is surely consensus that given the same total income
and output, a distribution with fewer families in poverty would be better, but what is the ideal
distribution? Even if we could construct indexes of output, real costs, needs, state of the
environment, we could not compute a welfare index because we have no system of weights to
combine them.

1. The author’s primary concern is to

(A) refute arguments for a position


(B) make a proposal and defend it
(C) attack the sincerity of an opponent
(D) show defects in a proposal
(E) review literature relevant to a problem

2. The author implies that use of man-hours is not an appropriate measure of real cost
because it

(A) ignores the conditions under which the output is generated


(B) fails to take into consideration the environmental costs of production
(C) overemphasizes the output of real goods as opposed to services
(D) is not an effective method for reducing unemployment
(E) was never intended to be a general measure of welfare
3. It can be inferred from the passage that the most important reason a single index of
welfare cannot be designed is

(A) the cost associated with producing the index would be prohibitive
(B) considerable empirical research would have to be done regarding output and needs
(C) any weighting of various measures into a general index would be inherently
subjective and arbitrary
(D) production of the relevant data would require time, thus the index would be only a reflection of
past welfare
(E) accurate statistics on crime and pollution are not yet available

4. The author regards the idea of a general index of welfare as a(n)

(A) unrealistic dream


(B) scientific reality
(C) important contribution
(D) future necessity
(E) desirable change

5. According to the passage, the GNP is a(n)

(A) fairly accurate measure of output


(B) reliable estimate of needs
(C) accurate forecaster of welfare
(D) precise measure of welfare
(E) potential measure of general welfare

6. According to the passage, an adequate measure of need must take into account all of
the following EXCEPT

(A) changing size of the population


(B) changing effects on people of the weather
(C) differences in needs of urban and suburban populations
(D) changing requirements for governmental programs such as defense
(E) accessibility of parkland and other amenities

7. The passage is most likely

(A) an address to a symposium on public policy decisions


(B) a chapter in a general introduction to statistics
(C) a pamphlet on government programs to aid the poor
(D) the introduction to a treatise on the foundations of government
(E) a speech by a university president to a graduating class

Enormously (adv) : rất nhiều

Index (n) : mục lục, chỉ báo

Deficient (adj) : không đủ


Goodness (n) : nhân đạo

Consensus (n) : đồng thuận

Distribution (n) : phân chia theo hạng

Compute (v) : tính toán

Pamphlet (n) : cuốn sách nhỏ, bài luận ngắn

Treatise (n) : khái luận, bài luận, chuyên luận

Symposium (n) : hội nghị chuyên đề

Amenity (n) : tiện nghi

Arbitrary (adj) : chuyên quyền, độc đoán, tùy ý

Refute (v) : bác bỏ

Defect (n) : nhược điểm, khuyết điểm


Feb 24 – 2

One reason we are able to recognize speech, despite all the acoustic variation in the signal, and
even in very difficult listening conditions, is that the speech situation contains a great deal of
redundancy—more information than is strictly necessary to decode the message. There is, firstly,
our general ability to make predictions about the nature of speech, based on our previous linguistic
experience—our knowledge of the speakers, subject matter, language, and so on. But in addition,
the wide range of frequencies found in every signal presents us with far more information than we
need in order to recognize what is being said. As a result, we are able to focus our auditory
attention on just the relevant distinguishing features of the signal—features that have come to be
known as acoustic cues.

What are these cues, and how can we prove their role in the perception of speech? It is not possible
to obtain this information simply by carrying out an acoustic analysis of natural speech: this would
tell us what acoustic information is present but not what features of the signal are actually used by
listeners in order to identify speech sounds. The best an acoustic description can do is give us a
rough idea as to what a cue might be. But to learn about listeners' perception, we need a different
approach.

1. The first paragraph of the passage implies which of the following concerning sound
and communication?

A. A lack of redundancy in certain signals does not preclude communication.


B. The excess of sound encoded in speech serves as an encumbrance to the exchange of
information.
C. Many features of a sound can be lost without necessarily compromising
communication between two speakers.
D. The human processing of acoustic sound does not depend on contextual factors.
E. Some languages depend on social cues in order for speakers to be mutually intelligible.

2. The primary purpose of the passage is to

A. recommend a particular means of learning about a complex phenomenon


B. point out the shortcomings in a line of research
C. explain a process and discuss why one approach will not yield desirable results
D. prescribe an approach to understand with greater specificity a physiological process
E. warn researchers against relying on a specific methodology

The author is not warning, prescribing or pointing out any shortcomings. He is only explaining how
we are able to understand speech even if the listening conditions are difficult.
3. The main reason that the author of the passage discounts using a purely acoustic
analysis to understand the way in which humans are able to recognize sounds is that
A. the physical properties of speech and the perception of those physical properties are
qualitatively different
B. redundancy, or the excess of information, typically ensures that a signal is faithfully transmitted
between two speakers
C. no two people process the same cue in exactly the same way
D. the relevant distinguishing features of a sound tend to merge different cues into a generalized
sound so as to render any analysis ineffectual
E. humans ignore certain features of the acoustic sound that may figure prominently into the
perception of sound

Qualitatively (adv) : phẩm tính

Acoustic (adj) : âm hưởng học

Cue (n) : tín hiệu

Decode (v) : giải mã

Linguistic (adj) : ngôn ngữ học

Auditory (adj) : thính giác

Present (v) : bày tỏ

Discount (v) : không kể đến, không tính vào, khuyên không nên

Perception of speech (n) : nhận thức về lời nói

Property (n) : tính chất


Feb 25 – 01

The bond between the domestic dog and humans is such that the dog exists in every part of the
world inhabited by people. The relationship between the two species stretches back tens of
thousands of years, to the first domestication of the wolf. Every modern breed of domestic dog, of
which there are more than 400 today, is descended from this wild ancestor. Prehistoric humans had
contact with other wild canids, such as jackals, foxes, dholes, and African hunting dogs, but only the
wolf possessed the characteristics that allowed for integration into human life, which implies social
assimilation, as opposed to mere domestication, which requires only a taming of wild instincts in the
animal. So while dogs have been integrated, animals such as cows, sheep, and goats
have not.

The traits in the wolf that allowed for integration are threefold. First, the wolf is a highly social
animal, living in packs, akin to the social networks of humans. In the absence of this behavior, it
would have been difficult for the first captive wolf pups to remain in a human settlement, constantly
surrounded by other creatures. Wild canids such as the jackal and fox are solitary animals and
would not have adapted easily, if at all, to social living. Second, wolves possess a system of social
stratification remarkably similar to that of humans: each member of the group is aware of its rank in
the chain of dominance and is loyal to higher-ranking members. Humans exploited this innate
sensitivity to hierarchy by raising wolf cubs to be submissive. Third, wolves are highly intelligent
creatures, able to learn tasks quickly. Without this attribute, the wolf would have been of little use
to early man, since it does not provide meat, milk, or wool. The more intelligent the captive wolf,
the more likely humans would have sought to breed it, resulting in perpetuation of this inborn
intelligence and culminating in the remarkable cognitive abilities of modern dogs.

The development of different breeds from the wolf appears to have begun as early as 2000 B.C. in
ancient Egypt and parts of western Asia. The first recorded instances of physically distinct breeds
come from these areas. In Egypt, dogs resembling modern greyhounds were prevalent, while in
western Asia dogs resembling modern mastiffs were common. The distinct body types of these
breeds perhaps reflect the different purposes for which they were bred: greyhounds for chasing
swift prey, such as hares, and mastiffs for grappling with larger prey, such as boars or antelope. But
this is only speculation; the historical record from this period is too sparse to allow certainty on the
matter.

1. Which of the following most accurately states the main idea of the passage?

(A)Humans value dogs more than they value any other domestic animal.
(B) The domestication of the wolf has no parallel in any other animal.
(C) Certain desirable traits not present in other animals allowed the wolf to become the
modern dog.
(D)Domestication of the wolf has been unquestionably successful.
(E) Wolves are similar to humans in several important aspects.

Main idea : So while dogs have been integrated, animals such as cows, sheep, and goats have not.
2. The passage suggests which of the following about the modern dog?

(A) It is the best social companion of all domesticated animals.


(B) Its intelligence is unsurpassed among household animals.
(C) It cannot survive outside of a social environment.
(D) Its body type always reflects the purpose for which it was bred.
(E) It perceives humans as superior to itself.

3. All of the following statements are supported by the passage EXCEPT:

(A) Sheep are not part of the social fabric of human life.
(B) It is not possible to domesticate jackals or foxes.
(C) Submissiveness is a desirable trait in domesticated animals.
(D) Humans tended to domesticate those animals that provided some benefit to them.
(E) The purpose of early breeds of dog is not known.

Bond (n) : liên kết, rằng buộc

Inhabit (v) : trú ngụ

Stretch (v) : trải dài

Domestication (n) : thuần hóa

Prehistoric (adj) : tiền sử

Assimilation (n) : đồng hoá, xem giống như

Tame (v) : thuần hóa

Instinct (n) : bản năng

Akin (adj) : giống như

Captive (adj) : nuôi nhốt

Settlement (n) : thuộc địa

Solitary (adj) : đơn độc

Stratification (n) : sự phân tầng

Innate (adj) : bẩm sinh, thiên phú

Hierachy (n) : hệ thống cấp bậc

Cub (n) : thú con

Submissive (adj) : dễ bảo, phục tùng

Wool (n) : len

Perpetuation (n) : sự tồn tại

Cognitive (adj) : nhận thức


Inborn (adj) : bẩm sinh

Culminate (v) : lên đến đỉnh điểm

Resemble (v) : giống với vật gì

Swift (adj) : nhanh nhẹn

Grapple (v) : vật lộn

Boar (n) : heo rừng

Antelope (n) : linh dương

Sparse (adj) : thưa thớt


Feb 25 – 2

Although hard statistics are difficult to come by, there is substantial anecdotal evidence that use of
performance-enhancing drugs, or doping, is rampant in professional sports. Of perhaps greater
significance to society are the estimated 1.5 million amateur athletes who use steroids, either to
improve their appearance or to emulate the performance of their favorite professional athletes. This
chemical epidemic is a pernicious threat to both the nation’s health and our collective sense of “fair
play.”

Nonprescription anabolic steroids have been illegal in the United States since 1991, and most
professional sports leagues have banned them since the 1980s. These bans are partly a matter of
fairness—a talented athlete trained to the peak of her ability simply cannot compete with an
equivalent athlete using steroids—but also based on issues of health. Anabolic androgenic steroids
(“anabolic” means that they build tissues; “androgenic” means that they increase masculine traits)
have been linked to liver damage, kidney tumors, high blood pressure, balding, and acne. They
function by increasing the body’s level of testosterone, the primary male sex hormone. In men, this
dramatic increase in testosterone can lead to the shrinking of testicles, infertility, and the
development of breasts; in women, it can lead to the growth of facial hair and permanent damage
to the reproductive system. Steroids have also been linked to a range of psychological problems,
including depression and psychotic rage.

The punishments for getting caught using steroids are severe, and the serious health consequences
are well documented. Despite this, millions of professional and amateur athletes continue to use
performance-enhancing drugs. Why is this? One clear pattern is that many athletes will do whatever
it takes to get an edge on the competition. Since the 1950s, Olympic athletes have played a catand-
mouse game with Olympic Committee officials to get away with doping, because the drugs really do
work. Athletes who dope are simply stronger and faster than their competitors who play fair.
Professional athletes in football and baseball have found that steroids and human growth hormone
can give them the edge to score that extra touchdown or home run, and in the modern sports
market, those results can translate into millions of dollars in salary. For the millions of less talented
athletes in gyms and playing fields across the country, drugs seem like the only way to approach the
abilities of their heroes in professional sports.

The other clear pattern, unfortunately, is that it has been all too easy for abusers to get away with
it. Steroid abuse is often regarded as a “victimless crime.” One of the favored ways to trick the
testers is to use “designer” steroids. There are thousands of permutations of testosterone, such as
THG, that can be produced in a lab. Chemists have discovered that they can create new drugs that
produce androgenic effects but do not set off the standard doping tests. Other methods have been
to use the steroids but stop a few weeks before testing, to use other chemicals to mask the traces
of steroids, or to switch in a “clean” sample of urine at the testing site. Other athletes use steroid
precursors, such as androstenedione, that have androgenic effects similar to those of steroids but
are not illegal because they are not technically steroids. The sad fact is that unless the government
and professional sports organizations are willing to get tough on the steroid problem, the use of
performance-enhancing drugs in sports is not going to end.
1. What appears to be the primary purpose of this passage?

A. To educate readers about the health threats involved in the use of performanceenhancing drugs
B. To analyze the ways in which professional athletes have eluded attempts to screen for
performance-enhancing drugs
C. To discuss the reasons why performance-enhancing drugs are a dangerous and
persistent problem for society
D. To complain about the inadequate efforts by government and professional sports organizations to
eliminate the problem of performance-enhancing drugs
E. To argue that athletes, both professional and amateur, should not use performance enhancing
drugs on the grounds that they are both dangerous and unfair

2. According to the passage, all of the following are known potential consequences of
steroid use except for which of the following?

A. Damage to reproductive organs


B. Decreased blood pressure
C. Increases in the user’s strength and speed
D. Kidney tumors
E. Increased risk of depression

3. The author’s attitude toward the problem of steroid abuse is best described as which
of the following?

A. Cautious but optimistic


B. Judgmental but supportive
C. Ambivalent but resigned
D. Curious but subjective
E. Concerned but pessimistic

4. Which of the following can be inferred about a long-distance race in which both
athletes who use performance-enhancing drugs and those who do not use these drugs
compete?

A. The athletes using the drugs will be caught by the proper authorities and ejected from the race.
B. The athletes using the drugs will have a better chance of winning the race.
C. The athletes using the drugs will use steroid precursors that produce effects similar to those of
androgenic drugs but are not technically steroids.
D. The athletes using the drugs are more likely to be professionals in their sport than the athletes
who do not use such drugs.
E. The athletes using the drugs will be more likely to use any means possible to win the race,
including intentional sabotage of the other racers’ equipment.
5. The relationship of an athlete who does not use performance-enhancing drugs to an
athlete who does use such drugs is most similar to which of the following?

A. The relationship of a farmer selling milk from cows that have been given bovine growth hormone,
a legal drug that promotes greater than normal milk production, to a farmer selling milk from cows
that have not been given bovine growth hormone
B. The relationship of a chess player to a competitor who uses psychological tricks in order to gain
an advantage
C. The relationship of a boxer in the lightweight class to a boxer in the heavyweight class
D. The relationship of a person taking a standardized test according to the rules to a
person taking the same test while using an illegal hidden calculator
E. The relationship of a person entering a pig in an agricultural contest to a person entering a
guinea pig in the same contest

6. According to the passage, which of the following can be inferred about the “designer”
steroid THG?

A. It can increase masculine traits in users without setting off standard doping tests.
B. It does not cause the health problems associated with traditional anabolic steroids.
C. Even if professional sports organizations could detect THG, they would take no action against
those who use it.
D. It is a chemical permutation of progesterone, a hormone that has powerful effects on the human
body.
E. Because it is a “designer” steroid, it is more expensive than generic steroids.

7. Which of the following best expresses the role of the third paragraph in the overall
structure of the passage?

A. It redirects the theme of the passage from presenting a problem to explaining the
reasons for the problem’s severity.
B. It introduces a new concept that defines the rest of the passage.
C. It provides an answer to a question posed in the first two paragraphs.
D. It refutes the central hypothesis of the second paragraph and poses a question that is answered
in the following paragraphs.
E. It narrows the focus of the passage from the general themes of the first two paragraphs to the
more specific themes of the last two paragraphs.

Redirect (v) : chuyển hướng

Theme (n) : chủ đề

Severity (n) : mức độ nghiêm trọng

Persistent (adj) : dai dẳng


Feb 26 – 1

Founded at the dawn of the modern industrial era, the nearly forgotten Women's Trade Union
League (WTUL) played an instrumental role Line in advancing the cause of working women
throughout the early part of the twentieth century. In the face of considerable adversity, the WTUL
made a contribution far greater than did most historical footnotes.

The organization's successes did not come easily; conflict beset the WTUL in many forms. During
those early days of American unions, organized labor was aggressively opposed by both industry
and government. The WTUL, which represented a largely unskilled labor force, had little leverage
against these powerful opponents. Also, because of the skill level of its workers as well as inherent
societal gender bias, the WTUL had great difficulty finding allies among other unions. Even the large
and powerful American Federation of Labor (AFL), which nominally took the WTUL under its wing,
kept it at a distance. Because the AFI2s power stemmed from its highly skilled labor force, the
organization saw little economic benefit in working with the WTUL. The affiliation provided the AFL
with political cover, allowing it to claim support for women workers; in return, the WTUL gained a
potent but largely absent ally.

The WTUL also had to overcome internal discord. While the majority of the group's members were
working women, a sizeable and powerful minority consisted of middle- and upper-class social
reformers whose goals extended beyond labor reform. While workers argued that the WTUL should
focus its efforts on collective bargaining and working conditions, the reformers looked beyond the
workplace, seeking state and national legislation aimed at education reform and urban poverty relief
as well as workplace issues. Despite these obstacles, the WTUL accomplished a great deal. The
organization was instrumental in the passage of state laws mandating an eight-hour workday, a
minimum wage for women, and a ban on child labor. It provided seed money to women who
organized workers in specific plants and industries, and it also established strike funds and soup
kitchens to support striking unionists. After the tragic Triangle Shirtwaist Company fire of 1911, the
WTUL launched a four-year investigation whose conclusions formed the basis of much subsequent
workplace safety legislation. The organization also offered a political base for all reform-minded
women, and thus helped develop the next generation of American leaders. Eleanor Roosevelt was
one of many prominent figures to emerge from the WTUL.

The organization began a slow death in the late 1920s, when the Great Depression choked (60) off
its funding. The organization limped through the 1940s; the death knell eventually rang in 1950, at
the onset of the McCarthy era. A turn-of-the-century labor organization dedicated to social reform,
one that during its heyday was regarded by many as "radical," stood little chance of weathering that
storm. This humble ending, however, does nothing to diminish the accomplishments of an
organization that is yet to receive its historical due.

1. The primary purpose of this passage is to

A. describe the barriers confronting women in the contemporary workplace


B. compare and contrast the methods of two labor unions of the early industrial era
C. critique the methods employed by an important labor union
D. rebuke historians for failing to cover the women’s labor movement adequately
E. call readers’ attention to an overlooked contributor to American history
2. Which of the following best characterizes the American Federation of Labor’s view of
the Women’s Trade Union League, as it is presented in the passage?

A. The WTUL was an important component of the AFL’s multifront assault on industry and its
treatment of workers.
B. Because of Eleanor Roosevelt’s affiliation with the organization, the WTUL was a vehicle through
which the AFL could gain access to the White House.
C. The WTUL was to be avoided because the radical element within it attracted unwanted
government scrutiny.
D. The WTUL offered the AFL some political capital but little that would assist it in labor
negotiations.
E. The WTUL was weakened by its hesitance in pursuing widespread social reform beyond the
workplace.

3. Each of the following is cited in the passage as an accomplishment of the Women’s


Trade Union League EXCEPT

A. it organized a highly skilled workforce to increase its bargaining power


B. it contributed to the development of a group of leaders in America
C. it provided essential support to striking women
D. it helped fund start-up unions for women
E. it contributed to the passage of important social and labor reform legislation

4. The passage suggests which of the following about the “middle- and upper-class
social reformers” (Highlighted) ?

A. They did not understand, nor were they sympathetic to, the plight of poor women workers.
B. Their naive interest in Communism was ultimately detrimental to the Women’s Trade Union
League.
C. It was because of their social and political power that the Women’s Trade Union League was able
to form an alliance with the American Federation of Labor.
D. They represented only an insignificant fraction of the leadership of Women’s Trade Union League.
E. They sought to advance a broad political agenda of societal improvement.

Adversity (n) : nghịch cảnh

Beset (v) : bao vây

Bias (n) : thành kiến, thiên vị

Affiliation (n) : liên kết

Potent (adj) : thế lực

Discord (n) : mối bất hòa

Instrumental (adj) : góp phần vào

Mandate (v) : bắt buộc

Seed money (n) : tiền vốn


Emerge (v) : nổi lên

Prominent (adj) : nổi bật

Limp (v) : đi khập khiễng

Heyday (n) : thời kỳ hoàng kim

A turn-of-the-century labor organization (n) : một tổ chức thế kỷ

Radical (adj) : căn bản, cấp tiến

Rebuke (v) : quở trách


Feb 26 – 2

Although I am familiar with Rembrandt's work, through photographs and black and white
reproductions, I invariably experience a shock from the colour standpoint whenever I come in touch
with one of his pictures. I was especially struck with that masterpiece of his at the Hermitage, called
the ‘Slav Prince’, which, by the way, I am convinced is a portrait of himself; anyone who has had the
idea suggested cannot doubt it for a moment; it is Rembrandt's own face without question. The
reproductions I have seen of this picture, and, in fact, of all Rembrandt's works, are so poor and so
unsatisfactory that I was determined, after my visit to St. Petersburg, to devise a means by which
facsimile reproductions in colour of Rembrandt's pictures could be set before the public. The black
and white reproductions and the photographs I put on one side at once, because of the impossibility
of suggesting colour thereby.

Rembrandt has been reproduced in photograph and photogravure, and by every mechanical process
imaginable, but all such reproductions are not only disappointing, but wrong. The light and shade
have never been given their true value, and as for colour, it has scarcely been attempted.

After many years of careful thought and consideration as to the best, or the only possible, manner
of giving to those who love the master a work which should really be a genuine reproduction of his
pictures, I have adapted and developed the modern process of color printing, so as to bring it into
sympathy with the subject.

1. Which of the following would the author most probably agree with?

(A) Rembrandt’s paintings have been reproduced effectively in black and white
(B) The works of Rembrandt have been misrepresented
(C) Facsimile reproductions of Rembrandt’s paintings are available in St. Petersburg
(D) The interplay of light and shade in Rembrandt’s paintings has never completely been
replicated
(E) Rembrandt’s paintings are a disappointment to the public

The light and shade have never been given their true value, and as for colour, it has scarcely been
attempted.
2. Why does the author mention the ‘Slav Prince’ in the passage?

(A) To provide an example of Rembrandt’s work that does not measure up to his other works
(B) To prove that the Slav Prince is a portrait of Rembrandt himself
(C) To provide an example of a problem he mentions earlier in the passage
(D) To assert that color printing was the only way to reproduce great works of art such as the Slav
Prince
(E) To highlight the importance of using the right colors in a painting
3. Which of the following can be concluded about the Slav Prince from the information
in the passage?

(A) It is a self portrait of Rembrandt


(B) It is Rembrandt’s best work ever
(C) No other work of Rembrandt has been reproduced as many times as the Slav Prince
(D) The colors of this painting can be improved through color printing
(E) It is housed at The Hermitage

Invariably (adv) : luôn luôn

Standpoint (n) : quan điểm

Devise (v) : nghĩ ra

Facsimile (v) : mô phỏng

Photogravure (n) : thuật khắc hình

Replicate (v) : nhân rộng


Feb 27 – 1

Feminist literary criticism is literary criticism informed by feminist theory, or more broadly by the
politics of feminism. Its history has been widespread and varied, from classic works of nineteenth-
century women authors such as George Eliot and Margaret Fuller, to cutting-edge theoretical work in
women's studies and gender studies by "third-wave" authors. In the most general and simple terms,
feminist literary criticism before the 1970s—in the first and second waves of feminism—was
concerned with the politics of women's authorship and the representation of women's condition
within literature, including the depiction of fictional female characters. In addition, feminist criticism
was further concerned with the exclusion of women from the western literary canon – an exclusion
that most feminist critics suggest is due to the views of women authors not being considered
universal.

Since the development of more complex conceptions of gender and subjectivity and third-wave
feminism, modern feminist literary criticism has taken a variety of new routes, namely in the
tradition of the Frankfurt School's critical theory. It has considered gender in the terms of Freudian
and Lacanian psychoanalysis, as part of the deconstruction of existing relations of power, and as a
concrete political investment. It has also been closely associated with the birth and growth of gay
studies. The more traditionally central feminist concern with the representation and politics of
women's lives has continued to play an active role in criticism. More specifically, modern feminist
criticism deals with those issues related to the patriarchal programming within key aspects of society
including education, politics, and the work force.

Recently, Lisa Tuttle has defined feminist theory as asking "new questions of old texts."
Consequently she cites the following as the primary goals of feminist criticism: to uncover a female
tradition of writing; to interpret symbolism of women's writing so that it will not be lost or ignored
by the male point of view; to analyze women writers and their writings from a female perspective;
to examine sexism in literature; and to increase awareness of the sexual politics of language and
style. Only through such analysis, she argues, can a proper view of feminist criticism be framed
moving forward.

1. According to the passage, which of the following would be a likely reason that a
George Eliot novel was not considered among the western literary canon?

(A) George Eliot’s political and social views were considered unorthodox.
(B) The two primary characters of the novel were women.
(C) The women characters in the novel held positions of power in society.
(D) George Eliot believed in voting equality for women.
(E) Male critics did not properly understand her work.

2. Which of the following can be inferred from the passage about modern feminist
criticism?

(A) It was primarily formed by scholars in the Frankfurt School of thought.

‘Primarily’ is incorrect, modern feminist criticism also be formed in terms of freudian and lacanian
(B) It shares many components with the modern gay movement.
(C) It contradicts much of the previous era’s feminist criticism.
(D) It considers gender differently than does earlier feminist criticism.
(E) Women played a dominant role in education.

Modern feminist literary criticism has taken a variety of new routes, namely in the tradition of the
Frankfurt School's critical theory. It has considered gender in the terms of Freudian and Lacanian
psychoanalysis

3. According to the passage, all of the following are considered new routes of modern
feminist literary criticism EXCEPT:
(A) viewing gender in terms of Freudian and Lacanian psychoanalysis.
(B) deconstructing existing relations of power in relation to gender.
(C) regarding gender as a tangible political contribution.
(D) focusing on the representation and politics of women’s lives.
(E) scrutinizing gender in the tradition of the Frankfurt School’s critical theory.

D The more traditionally central feminist concern with the representation and politics of
women's lives has continued to play an active role in criticism.

A-B-C-E Modern feminist criticism deals with those issues related to the patriarchal
programming within key aspects of society including education, politics, and the work force.

Patriarchal (adj) : gia trưởng

Key aspect (n) : khía cạnh quan trọng

Concrete (v) : cụ thể hóa

Contradict (v) : mâu thuẫn, nói ngược lại

Unorthodox (adj) : không chính thống

Exclusion (n) : loại trừ

Cutting-edge (adj) : tiên tiến

Theoretical (adj) : lý thuyết

Authorship (n) : quyền tác giả

Depiction (n) : miêu tả

Fictional (adj) : hư cấu

Canon (n) : kinh điển

Interpret (v) : giải nghĩa

Symbolism (n) : biểu tượng

Sexism (n) : phân biệt giới tính

Feminist criticism (n) : phê bình của nữ quyền

Psycho-analysis (n) : nhà phân tích tâm lý học

Scrutinize (v) : xem xét kỹ lưỡng


Feb 27 – 2

The classical realist theory of international relations has long dominated both academic institutions
and the American government. Even at the birth of the nation, early political thinkers, such as
Alexander Hamilton, promoted a realist view of international relations and sought to influence the
actions of the government based on this perspective. While the classical realist school of
international relations is not entirely homogeneous in nature, there are certain premises that all
classical realists share.

The primary principle underlying classical realism is a concern with issues of war and peace.
Specifically, classical realists ask, what are the causes of war and what are the conditions of peace?
The members of the classical realist school mainly attribute war and conflict to what is termed the
security dilemma. In the absence of any prevailing global authority, each nation is required to
address its own security needs. However, each nation’s quest for security—through military
buildups, alliances, or territorial defenses—necessarily unsettles other nations. These nations react
to feelings of insecurity by engaging in their own aggressive actions, which leads other nations to
react similarly, perpetuating the cycle.

It is important to note that for realists, unlike idealists or liberal internationalists, international
conflict is a necessary consequence of the structural anarchy that nations find themselves in.
Whereas other schools may see international conflict as the result of evil dictators, historical chance,
flawed sociopolitical systems, or ignorance of world affairs, classical realists see war as the logical
result of a system that by its nature lacks a true central authority.

Hand in hand with this view of conflict as an inevitable condition of the global power structure is the
realists’ view of the nation as a unitary actor. Because classical realists see international relations as
a continuing struggle for dominance, the nation can not be viewed as a collection of individuals with
disparate wants, goals, and ideologies. The realist view requires the formulation of a national
interest, which in its simplest terms refers to the nation’s ability to survive, maintain its security, and
achieve some level of power relative to its competitors. Realism is not without its critics, many of
whom challenge the premise that war is the natural condition of international relations or that there
can be a truly national interest. However, the realist school of international relations continues to
shape foreign policy because of the successes it has had in describing real world interactions
between nations.

1. It can be inferred from the passage that members of the classical realist school would
be LEAST likely to support

A. an international policy based on building a strong military force to deter threats


B. an international policy that seeks to reduce threats of war by providing humanitarian
aid to potential aggressor countries
C. a domestic policy that attempts to unify the nation’s citizens behind a common cause
D. a domestic policy that allocates a majority of the country’s budget for defense spending
E. an international policy based on joining a common defense contract with other nations
2. Which of the following, if true, would best support the classical realist theory of
international conflict as it is described in the passage?

A. Some countries ruled by dictators maintain peaceful relations with their neighbors.
B. Despite the presence of a world superpower, many countries continue to fight wars with their
neighbors.
C. War has existed from the beginning of recorded history.
D. After the nations of the world form an authoritative world court, wars decrease
dramatically.
E. Some countries are able to capture territories from other countries without fear of international
consequences.

This is an apply information question. In order to answer it, you must first go to the passage to
understand the theory of conflict. This information is found in the second paragraph: “In the
absence of any prevailing global authority, each nation is required to address its own
security needs.” Next, you have to figure out which choice best supports this view. Choice
A doesn’t do much.The classical realist school isn’t concerned with the actions of dictators. Choice
B seems to hurt the theory. It states that there is a prevailing global authority and yet conflict
continues. Choice C just states that war is a persistent problem, but it doesn’t address the causes
of it, so this choice doesn’t necessarily support the theory. Choice D does support the theory
because it shows that the presence of a global authority reduces war. Remember, the realist view
argued that the lack of a global authority led to war, so the presence of a global authority should
reduce war. That is what choice D states. Choice E doesn’t address the causes of war or the
presence of a global authority.

Whereas other schools may see international conflict as the result of evil dictators, historical chance,
flawed sociopolitical systems, or ignorance of world affairs, classical realists see war as the logical
result of a system that by its nature lacks a true central authority.

3. According to the passage, the formation of a national interest serves what function in
the classical realist theory of war and peace?

A. It is a convenience used by theorists to describe national interests where none exist.


B. It provides the necessary justification for the classical realist view of a continuous
global power struggle.
C. It is less important to the theory than is the idea of the nation as a unitary actor.
D. It is a description of the policies used by world leaders to convince their citizens that war is
necessary.
E. It is the part of the theory that receives the most criticism from opponents.
4. The author most likely regards the classical realist theory of international relations
with

A. general apathy
B. skeptical dismissal
C. veiled disapproval
D. glowing approval
E. qualified acceptance

This is a tone question. Some tone questions require you to consider the passage as a whole, while
some ask about a specific part. This is a more general tone question. The passage describes the
classical realist view and the author ends by stating that the theory has had “successes” in
describing relations in the world.Thus, the tone must be somewhat positive. That leaves only
choices D and E as possibilities. Between the two, choice D is too strong. Passages on the GMAT
rarely if ever will contain “glowing approval” for their topics.

Classical realist (n) : hiện thực cổ điển

Dominate (v) : thống trị

Homogeneous (adj) : đồng nhất

Security dilemma (n) : vấn đề an ninh

Dilemma (n) : tình trạng khó xử

Prevailing (adj) : thịnh hành

Unsettle (v) : làm ảnh hưởng

Perpetuate (v) : làm cho lâu dài

Idealist (n) : thuyết quan niệm, người duy tâm

Liberal internationalist (n) : người theo chủ nghĩa quốc tế tự do

Anarchy (n) : vô chính phủ, tình trạng hỗn loạn

World affair (n) : các vấn đề thế giới

Inevitable (adj) : chắc chắn xảy ra

Unitary (adj) : đơn nhất

Disparate (adj) : khác biệt

Formulation (n) : xây dựng, bày tỏ, ý kiến

Glowing (adj) : bảy tỏ sự khen ngợi tuyệt vời


Feb 28 – 1

It has long been a tenet of business theory that the best decisions are made after careful review
and consideration. Only after weighing all the options and studying projections, say
most (5) professors of business, can a practical decision be made.

Now, that model is being questioned by some business thinkers in the light of the theories of
Malcolm Gladwell, who states that human beings (10) often make better decisions in the blink of an
eye. It is, at first glance, a theory so counter- intuitive as to seem almost ludicrous. Behind any
decision, Gladwell posits, there is a behind-the-scenes subconscious process in which the brain
(15) analyzes; ranks in order of importance; compares and contrasts vast amounts of information;
and dismisses extraneous factors, seemingly almost instantaneously, often arriving at a conclusion
in less than two seconds. Citing a multitude of (20) studies and examples from life, Gladwell shows
how that split-second decision is often better informed than a drawn-out examination.

Evanston and Cramer were the first to apply this theory to the business world. Evanston
(25) videotaped the job interviews of 400 applicants at different firms. He then played only 10
seconds of each videotape to independent human resources specialists. The specialists were able to
pick out the applicants who were hired with an accuracy of (30) over 90%.

Cramer took the experiment even further, using only five seconds of videotape, without sound. To
his astonishment, the rate of accuracy with which the HR specialists were able to predict the
(35) successful applicants fell only to 82%.

Critics argue that these results illustrate a problem with stereotyping that impedes human resources
specialists from hiring the best candidates even when they have the time to (40) get below the
surface: going for the candidate who “looks the part.” Gladwell argues that, on the contrary, the
human mind is able to make complicated decisions quickly and that intuition often trumps an
extended decision-making (45) process.

1. The primary purpose of the passage is to

A. discuss reasons an accepted business theory is being reexamined


B. present evidence that resolves a contradiction in business theory
C. describe a tenet of business practices and how that tenet can be tested in today’s economic
environment
D. argue that a counter-intuitive new business idea is, in the final analysis, incorrect
E. present evidence that invalidates a new business model

2. According to the passage, all of the following are examples of the subconscious
processes by which the brain makes a decision EXCEPT

A. analysis of information
B. ranking of information
C. comparison and contrast of information
D. rejecting information that is not pertinent
E. consulting a multitude of studies and examples
3. The author’s attitude toward the long-held view that decisions should be made
carefully over time expressed in lines 1–5 can best be described as

A. dismissive and scornful


B. respectful but questioning
C. admiring and deferential
D. uncertain but optimistic
E. condescending and impatient

4. The author most likely mentions the results of Cramer’s extension of Evanston’s
experiment in order to

A. show that Cramer’s hypothesis was correct, while Evanston’s hypothesis turned out to be
incorrect
B. show that Evanston’s hypothesis was correct, while Cramer’s hypothesis turned out to be
incorrect
C. demonstrate that while both experiments were scientifically rigorous, neither ended up being
scientifically valid
D. illustrate that the principle of subconscious decisions continues to work even when
less information is available
E. demonstrate that Cramer’s experiment was 8% more accurate than Evanston’s, even though his
subjects had less information to work with

5. It can be inferred that the critics referred to in line 36 believed the results of the two
experiments had less to do with the innate decision-making of the subjects than with

A. the excellent decision-making of Evanston and Cramer


B. the expertise of Malcolm Gladwell, who originated the theory
C. not choosing candidates who “looked the part”
D. the use of videotape as a method of choosing candidates
E. their unconscious use of visual stereotypes in making their selections

Tenet (n) : nguyên lý, giáo lý

Counter (adj) : trái ngược

Intuitive (adj) : trực quan

Ludicrous (adj) : lố bịch, khôi hài, buồn cười

Posit (v) : đặt ra

Subconscious (adj) : thuộc về tiềm thức

Dismiss (v) : bỏ qua

Extraneous (adj) : không liên quan

Multitude (adj) : nhiều

Astonishment (n) : sự ngạc nhiên


Stereotype (v/n) : khuôn mẫu, rập khuôn

Impede (v) : cản trở

Intuition (n) : trực giác

Condescend (v) : hạ mình, nhường nhịn

Scornful (adj) : khinh bỉ

Deferential (adj) : bảo vệ, cung kính

Look the part (v) : trông có vẻ giống như vậy


Feb 28 – 2

The poverty line, also known as the poverty threshold, is defined as the minimum level of income
that is necessary for the people of a specific nation.The poverty line of a country is calculated by
identifying the total amount of necessary expenses that an average adult requires over the course of
one year, with the main such expense usually being house rent.Additionally, this calculation also
typically includes the basic expenses of food and clothing.This measure of minimum income required
is purely quantitative and does not estimate either the specific needs of people from different low-
income groups or the different ways in which low-income groups experience poverty in different
countries.The solely quantitative basis of calculating the minimum required income, therefore, may
in fact lead to either the overestimation or the underestimation of the number of people who
actually live under the poverty line in a given country.Another reason that renders the poverty line
short of being a very reliable indicator of actual poverty levels is that certain nations, usually
developed ones, with strong welfare systems fail to account for ‘in-kind’ transfers when calculating
the poverty threshold.Such calculations do not regard the benefits received from antipoverty
programs such as food stamps, housing assistance, etc. as income, thereby presenting a distorted
picture of the level of poverty in the nation. Another reason that the poverty line may not be the
most reliable indicator of actual poverty levels is that certain nations, usually developed ones,
include systems of social welfare that remain unaccounted for while calculating the proportion of
population that does not earn the minimum required income.

1. The primary purpose of the passage is to


A. suggest that the poverty line should not be used as an indicator of actual poverty levels in
developed nations, since its use could lead to a miscalculation of the number of people actually
living in poverty
B. provide a basis for the conclusion that the economic prosperity of certain countries may lead to a
miscalculation of their respective poverty lines
C. identify the reasons that the poverty line does not account for certain sources of income such as
social welfare systems
D. argue that, because of the way it is calculated, poverty line, as a measure of the
number of people living in poverty, may give inaccurate results
E. propose that the poverty line be replaced by an improved system of calculation for the minimum
income required

2. Which of the following can be inferred from the last two sentences of the passage?
A. By treating the benefits as income, the nations would not be correctly estimating the extent of
poverty in their countries.
B. Although the antipoverty programs give benefits to the people, their effects are not very
significant and hence the nations fail to account for them.
C. A calculation done with considering the benefits as income would present a more realistic picture
of the poverty in the nation, just as it probably does in developing nations.
D. These nations run a potential risk of overestimating the extent of poverty in their countries by not
treating the benefits as income.
E. Housing assistance and food stamps are indeed income for the people who receive them, and
hence such people should not get benefits from any other antipoverty program.
3. Which of the following statements about the poverty line would the author most
likely agree with?
A. It is usually used in developed nations to falsify data about the number of people actually living in
poverty.
B. Quantitative measurements are likely to be the only accurate measure of the number of people
living in poverty.
C. It is not easy to calculate because of the difficulty of gathering data about people from low-
income groups.
D. More than one form of measurement may need to be utilized to develop more
accurate estimates of poverty levels.
E. Income from social welfare systems should not be taken into account while calculating the
minimum required income for an average adult.

4. Which of the following is mentioned in the passage?


A. A qualitative basis for calculating the minimum income required would not result in
overestimating the number of people living below the poverty line.
B. Quantitative parameters are poor indicators of poverty.
C. The cost of the basic requirements of housing, food and clothing is not the same in any two
economies.
D. The calculation of poverty in developed nations is affected by their misnaming of antipoverty
programs.
E. The quantitative basis for calculating the poverty line excludes estimation of the
specific needs of people from various low-income groups.

Exclude (v) : không bao gồm

Falsity (v) : làm sai lệch

Poverty line (n) : chuẩn nghèo


Mar 01 – 1

Research shows that when people work with a positive mind-set, performance on nearly every
level—productivity, creativity, engagement—improves. Yet the correlation between happiness and
performance is perhaps not so easily understood by many. For one, most people believe that
success precedes happiness, but, because success is a moving target, the happiness that results
from success is fleeting. Instead people who cultivate a positive mind-set perform better in the face
of a challenge. In a meta-analysis of 225 academic studies, researchers found strong evidence of
directional causality between life satisfaction and successful business outcomes.

What is even more encouraging is that even though our genetics and our environment have an
impact on how happy we are, our general sense of well-being is surprisingly malleable. Our habits,
our interactions with coworkers, our attitude towards stress—all these aspects can be managed to
increase our happiness and our chances of success. Habits such as taking short breaks and
constantly having a positive engagement with coworkers not only reduce stress substantially but
also enhance our productivity and thus better our prospects of professional success. Accordingly,
encouraging employees to accept some level of stress as an inevitability and perceive it as a
motivator and a fuel to growth, is vital for boosting the performance of the workforce.

1. Which of the following statements would the author NOT agree with?

A. Two people with the same genetic make-up and environment can have different happiness
quotients.
B. For happiness to result from success, the latter needs to be a stationary target.
C. Increment in remuneration is not the only important factor responsible for improving the
performance of employees.
D. Some factors beyond that we have little control over impact how happy we are.
E. The order in which success and happiness occur is not necessarily fixed.

2. The author is primarily concerned with

A. showing how happiness can be cultivated rather than passively experienced


B. guiding employers with respect to increasing the overall happiness and success quotient of the
employees
C. clarifying why happiness is not dependent on factors other than a positive outlook
D.proposing that happiness and the resultant success at work are rather dependent on
our attitude
E. outlining the ways in which one can achieve professional success

3. According to the passage, which of the following is true?

A. Most people use success to measure their happiness.


B. Even when the situation is tough, a positive outlook can trick the brain in to believing otherwise.
C. One of the most important ways of boosting employee performance is encouraging them to
accept some level of stress as a part of the job.
D. Developing common habits with coworkers can help reduce stress significantly.
E. When happiness is triggered by success, it is anything but everlasting.

Everlasting (adj) : mãi mãi, bất diệt, vĩnh viễn


Order (n) : thứ tự

Increment (n) : sự lớn lên, sự tăng lên

Remuneration (n) : thù lao, sự khen thưởng

Stationary (adj) : đứng im, không thay đổi

Precede (v) : đứng trước, tới trước

Fleeting (adj) : không lâu dài, thoáng qua

Cultivate (v) : trau dồi

Causality (n) : quan hệ nhân quả

Correlation (n) : sự tương quan, liên quan

Well-being (n) : hạnh phúc

Malleable (adj) : dễ uốn

‘it is anything but everlasting = nó là mọi thứ trừ mãi mãi ‘


Mar 01 – 2

Given his luminous treatment of light, sky, and water, J.M.W. Turner (1775-1851) is often viewed in
hindsight as a precursor of Impressionism. Yet as Turner authority Andrew Wilton has argued, his
roots lie in a specifically eighteenth century tradition, that of the "sublime." Before landscape
painting was accepted in England as the rendition of everyday reality, it was seen as the expression
of a state of spiritual exaltation.

The roots of the notion of the sublime, Wilton notes, go back to antiquity: Longinus observed
(according to an eighteenth century paraphrase) that "the effect of the sublime is to lift up the
soul...so that participating, as it were, of the splendors of the divinity, it becomes filled with joy and
exultation." The sublime, therefore, was understood to produce an effect of elevation toward unity
with divine. In its origins, the sublime was associated with literary rather than visual art, as its
connotations of power and mystery could most easily be conveyed in words; and its subject matter
was epic, historical, or religious. To eighteenth century commentators, Homer, the Bible, and Milton
were quintessentially sublime. When the concept was applied to painting, this narrative emphasis
was maintained, leading almost by necessity to a focus on the human figure; for Joshua Reynolds,
Michelangelo's Sistine Chapel frescoes exemplified the sublime in art. Because it did not show
figures (except incidentally) landscape was necessarily seen as inferior.

The transition to the conception that produced Turner's landscapes had several sources. One was
the eighteenth century's quasi-religious excitement in the scientific investigation of nature, shown
for example when Addison exclaimed upon the astronomer's "pleasing astonishment, to see so many
worlds, hanging one above another, and sliding round their axles in such an amazing pomp and
solemnity." A second was the rise of a middle class with the leisure to travel, which led to an
interest in the Rigged vistas of Wales and Scotland. Finally, James Thomson's immensely popular
nature epic "The Seasons" (1726-30) applied blank verse, with its connotations of loftiness, to
portrayal of nature's immensities. By the latter part of the century, there was a well-defined notion
of the sublime in literature and painting, which included nature while by no means excluding earlier
referents. According to Edmund Burke's definitive essay of 1757, the sublime in nature was closely
tied up with vastness, lack of habitation and cultivation, and danger— which, as in the reaction to
high mountain passes or storms at sea, was conducive to awe. These qualities, as evoked in the
painting of landscapes (and urban vistas, an important though subordinate field), produced a series
of genres that, Wilton stresses, form the key to Turner's work: the "picturesque sublime," the
"terrific" (wild crags, cataracts, etc.), the sublime of the sea, mountains, and darkness, and finally
the "architectural sublime" and the urban sublime.

(1) According to the passage, landscapes were not originally seen as embodying the
sublime because

(A) the narrative connotations of the sublime implied an emphasis on the human figure
X
(B) only religious subjects were seen as embodying the sublime
(C) Michelangelo did not paint landscape
(D) landscape was viewed purely as the visual representation of everyday nature scenes
(E) nature was not conceived as a source of awe and wonder
In its origins, the sublime was associated with literary rather than visual art, as its connotations of
power and mystery could most easily be conveyed in words; and its subject matter was epic,
historical, or religious. To eighteenth century commentators, Homer, the Bible, and Milton were
quintessentially sublime. When the concept was applied to painting, this narrative emphasis was
maintained, leading almost by necessity to a focus on the human figure; for Joshua Reynolds,
Michelangelo's Sistine Chapel frescoes exemplified the sublime in art. Because it did not show
figures (except incidentally) landscape was necessarily seen as inferior.

(2) The author gives specific examples of all of the following EXCEPT
(A) the conception of the sublime held in antiquity
(B) the subject matter which might be considered as representing the "terrific"
(C) a work of visual art considered as embodying the sublime by an eighteenth century authority
(D) a historical figure exemplifying the sublime
(E) a conception similar to that of the sublime in a non artistic context

The third passage mentioned about the sublime in scientific investigation of nature such as :
astronomer’s works …

One was the eighteenth century's quasi-religious excitement in the scientific investigation of nature,
shown for example when Addison exclaimed upon the astronomer's "pleasing astonishment, to see
so many worlds, hanging one above another, and sliding round their axles in such an amazing pomp
and solemnity." A second was the rise of a middle class with the leisure to travel, which led to an
interest in the Rigged vistas of Wales and Scotland. Finally, James Thomson's immensely popular
nature epic "The Seasons" (1726-30) applied blank verse, with its connotations of loftiness, to
portrayal of nature's immensities.

(3) According to the author, Burke contributed to the development of the concept of the
sublime by
(A) classifying the genres of the sublime in art
(B) broadening the conception of the sublime to include nature
(C) giving a more clear cut definition of the sublime than earlier writers
(D) defining some of the qualities in nature that could be considered sublime
(E) rejecting Longinus's identification of the sublime with religious experience

Luminous (adj) : sáng chói, minh bạch, sáng suốt

Hindsight (n) : nhận thức muộn màng

Precursor (n) : tiền thân

Impressionism (n) : chủ nghĩa ấn tượng

Sublime (adj) : cao siêu

Landscape (n) : phong cảnh

Rendition (n) : sự biểu hiện, sự thông dịch

Spiritual (adj) : tâm linh, tinh thần

Exaltation (n) : sự tôn vinh

Antiquity (n) : đời xưa, cổ xưa


Splendor (n) : sự huy hoàng, sự lộng lẫy, sự sáng lạng

Divinity (n) : người hay vật được tôn thờ

Elevation (n) : phẩm giá, phẩm hạnh

Connotation (n) : ý nghĩa, sự hàm súc

Convey (v) : chuyển tải

Epic (n) : anh hùng ca

Quintessentially (adv) : tinh túy

Narrative (adj) : tường thuật, kể chuyện

Human figure (n) : nhân vật con người

Exemplify (v) : nêu gương, làm thí dụ, mẫu mực

Incidentally (adv) : tình cờ

Inferior (adj) : kém hơn

Source (n) : nguyên nhân

Quasi (adj) : gần như, hầu như

Astronomer (n) : nhà thiên văn học

Astonishment (n) : sự kinh ngạc, ngạc nhiên

Axle (n) : trục

Pomp (n) : sự lộng lẫy, rực rỡ, hào hoa

Solemnity (n) : trang trọng

Blank verse (n) : thơ không vần

Loftiness (n) : lòng cao thượng

Immensity (n) : sự mênh mông, bao la

Vastness (n) : tính bao la, to lớn

Awe (n) : sự kinh hãi, mối sợ hãi

Evoke (v) : gợi lên

Genre (n) : thể loại

Picturesque (adj) : đẹp như tranh vẽ

Cataract (n) : thác nước lớn

Wild crag (n) : hoang dã

Embody (v) : hiện thân


Mar 02 – 1

After the end of World War II, a pervasive, but unfortunately fallacious, economic perspective took
hold. Based on the United States’ successful emergence from the Depression, the idea that war was
good for an economy became fashionable. However, linking the United States economic recovery
with its entry into World War II is a prime example of flawed economic thinking.

Supporters of the war benefits economy theory hold that a country at war is a country with a
booming economy. Industry must produce weapons, supplies, food, and clothing for the troops. The
increased production necessitates the hiring of more people, reducing unemployment. More
employment means more money in the pockets of citizens, who are then likely to go out and spend
that money, helping the retail sector of the economy. Retail shops experience an increase in
business and may need to hire more workers, further reducing unemployment and adding to the
economic momentum. While this scenario sounds good in theory, it does not accurately represent
what truly happens in a war time economy.

In reality, the government can fund a war in a combination of three ways. It can raise taxes, cut
spending on other areas, or increase the national debt. Each of these strategies has a negative
impact on the economy. An increase in taxes takes money out of an individual’s hands, leading to a
reduction in consumer spending. Clearly, there is no net benefit to the economy in that case. Cutting
spending in other areas has its costs as well, even if they are not as obvious. Any reduction in
government spending means the imposition of a greater burden on the benefactors of that
government spending. Cutbacks in a particular program mean that the people who normally depend
on that program now must spend more of their money to make up for the government cuts. This
also takes money out of consumers’ hands and leaves the economy depressed. Of course, a
government could go into debt during the war, but such a strategy simply means that at some point
in the future, taxes must be increased or spending decreased. Plus, the interest on the debt must be
paid as well.

1. The “pervasive…economic perspective” mentioned in line 1 took hold because

A. observers took the appearance of one phenomenon with another to indicate that one
caused the other
B. the U.S. would not have emerged from the Depression had it not entered World War II
C. the booming economy during wartime created thousands of jobs in the U.S.
D. most people are not trained to think in economic terms
E. economists confused an event that was necessary for an outcome to occur with one that is
merely sufficient to bring about that outcome

2. Which of the following situations best mirrors the effect that cutting spending in
government programs has, as detailed in the passage?
A. Government cutbacks on public works maintenance lead to a deterioration of roads, which
creates more work for private construction firms.
B. A decrease in the federal education budget causes certain schools to close, which forces families
to send their children to schools that are farther away.
C. A federal decrease in unemployment payments causes some individuals who would otherwise
remain on unemployment to seek jobs.
D. Government cuts in housing subsidies results in fewer houses being built.
E. A reduction in the federal spending on food safety inspections leads to a rash of
illnesses and an increase in the amount of money spent on medicine.

3. The passage implies which of the following about a government that funds a war by
increasing the national debt?
A. It is no worse off than it would be funding a war by cutting spending or increasing taxes.
B. The initial costs it incurs are less than with the other two methods, but the future
costs are greater.
C. It must increase taxes in order to pay off the interest on the debt.
D. If the government does not increase taxes or decrease spending, its economy will not recover.
E. It receives a net benefit to the economy greater than it would achieve with either of the other
two methods.

This is an inference question. You can find the answer in the final paragraph, where the author says
“Of course, a government could go into debt during the war, but such a strategy simply means that
at some point in the future, taxes must be increased or spending decreased. Plus, the interest on
the debt must be paid as well.” Choice A is the opposite of what the passage states—going into debt
means not only will the government have to deal with the problems associated with increasing taxes
or cutting spending, but it must also pay the interest on the debt. Choice B is a better answer. In
the short term, the government doesn’t have the problems associated with the other two solutions,
but must face those problems, plus interest payments, in the future. Choice C goes too far because
it is not clear that a government “must” increase taxes.The passage says spending could be
decreased. Choice D makes an extreme and unsupported claim because the author doesn’t say the
economy “will not recover” unless certain actions are taken. Choice E is similar to A and is incorrect
based on the information in the passage.

4. The second paragraph of the passage performs which of the following functions?
A. It describes the common economic benefits of a wartime economy.
B. It provides the background information necessary to understand the information in the third
paragraph.
C. It explains what happened to the United States’ economy during World War II.
D. It presents a possible objection to the author’s main thesis.
E. It helps explain why individuals might hold the viewpoint presented in the first
paragraph.

Pervasive (adj) : phổ biến

Fallacious (adj) : ngụy biện, niềm tin sai lầm

Imposition (n) : áp đặt, đánh thuế, bắt buộc

Benefactor (n) : người làm việc từ thiện, nhà hảo tâm

Incur (v) : phát sinh

Rash (n) : phát ban


Mar 02 – 2

James Porter (1905-1970) was the first scholar to identify the African influence on visual art in the
Americans, and much of what is known about the cultural legacy that African-American artists
inherited from their African forebears has come to us by way of his work. Porter, a painter and art
historian, began by studying African-American crafts of the eighteenth and nineteenth centuries.
This research revealed that many of the household items created by African-American men and
women—walking sticks, jugs, and textiles—displayed characteristics that linked them
iconographically to artifacts of West Africa. Porter then went on to establish clearly the range of the
cultural territory inherited by later African-American artists.

An example of this aspect of Porter’s research occurs in his essay “Robert S. Duncanson, Midwestern
Romantic-Realist.” The work of Duncanson, a nineteenth-century painter of the Hudson River
school, like that of his predecessor in the movement, Joshua Johnston, was commonly thought to
have been created by a Euro-American artist. Porter proved definitively that both Duncanson and
Johnston were of African ancestry. Porter published this finding and thousands of others in a
comprehensive volume tracing the history of African-American art. At the time of its first printing in
1943, only two other books devoted exclusively to the accomplishments of African-American artists
existed. Both of these books were written by Alain LeRoy Locke, a professor at the university where
Porter also taught. While these earlier studies by Locke are interesting for being the first to survey
the field, neither addressed the critical issue of African precursors; Porter’s book addressed this
issue, painstakingly integrating the history of African-American art into the larger history of art in the
Americas without separating it from those qualities that gave it its unique ties to African artisanship.
Porter may have been especially attuned to these ties because of his conscious effort to maintain
them in his own paintings, many of which combine the style of the genre portrait with evidence of
an extensive knowledge of the cultural history of various African peoples.

In his later years, Porter wrote additional chapters for later editions of his book, constantly revising
and correcting his findings, some of which had been based of necessity on fragmentary evidence.
Among his later achievements were his definitive reckoning of the birth year of the painter Patrick
Reason, long a point of scholarly uncertainty, and his identification of an unmarked grave in San
Francisco as that of the sculptor Edmonia Lewis. At his death, Porter left extensive notes for
unfinished project aimed at exploring the influence of African art on the art of the Western world
generally, a body of research whose riches scholars still have not exhausted.
1. Which one of the following most accurately states the main idea of the passage?

(A) Because the connections between African-American art and other art in the Americas had been
established by earlier scholars, Porter’s work focused on showing African-American art’s connections
to African artisanship.
(B) In addition to showing the connections between African-American art and African artisanship,
Porter’s most important achievement was illustrating the links between African-American art and
other art in Americas.
(C) Despite the fact that his last book remains unfinished, Porter’s work was the first to devote its
attention exclusively to the accomplishments of African-American artists.
(D) Although showing the connections between African-American art and African artisanship,
Porter’s work concentrated primarily on placing African-American art in the context of Western art in
general.
(E) While not the first body of scholarship to treat the subject of African-American art,
Porter’s work was the first to show the connections between African-American art and
African artisanship.

2. The discussion of Locke’s books is intended primarily to

(A) argue that Porte’s book depended upon Locke’s pioneering scholarship
(B) highlight an important way in which Porter’s work differed from previous work in his
field
(C) suggest an explanation for why Porter’s book was little known outside academic circles
(D) support the claim that Porter was not the first to notice African influences in African-American
art
(E) argue that Locke’s example was a major influence o Porter’s decision to publish his findings

3. The passage states which one of the following about the 1943 edition of Porter’s book
on African-American art?

(A) It received little scholarly attention at first.


(B) It was revised and improved upon in later editions.
(C) It took issue with several of Locke’s conclusions.
(D) It is considered the definitive version of Porter’s work.
(E) It explored the influence of African art on western art in general.

4. Given the information in the passage, Porter’s identification of the ancestry of


Duncanson and Johnston provides conclusive evidence for which one of the following
statements?

(A) Some of the characteristics defining the Hudson River school are iconographically linked to
Weston African artisanship.
(B) Some of the works of Duncanson and Johnston are not in the style of the Hudson River school.
(C) Some of the work of Euro-American painters displays similarities to African-American crafts of
the eighteenth and nineteenth centuries.
(D) Some of the works of the Hudson River school were done by African-American
painters.
(E) Some of the works of Duncanson and Johnston were influenced by West African artifacts.
5. Which one of the following can most reasonably be inferred from the passage about
the study that Porter left unfinished at his death?
(A) If completed, it would have contradicted some of the conclusions contained in his earlier book.
(B) If completed, it would have amended some of the conclusions contained in his earlier book.
(C) If completed, it would have brought up to date the comprehensive history of African-American
art begun in his earlier book.
(D) If completed, it would have expanded upon the project of his earlier book by
broadening the scope of inquiry found in the earlier book.
(E) If completed, it would have supported some of the theories put forth by Porter’s contemporaries
since the publication of his earlier book.

6. Which of the following hypothetical observations is most closely analogous to the


discoveries Porter made about African-American crafts of the eighteenth and nineteenth
centuries?
(A) Contemporary Haitian social customs have a unique character dependent on but different from
both their African and French origins.
(B) Popular music in the United States, some of which is based on African musical traditions, often
influences music being composed on the African continent.
(C) Many novels written in Canada by Chinese immigrants exhibit narrative themes very
similar to those found in Chinese folktales.
(D) Extensive Indian immigration to England has made traditional Indian foods nearly as popular
there as the traditional English foods that had been popular there before Indian immigration.
(E) Some Mexican muralists of the early twentieth century consciously imitated the art of native
peoples as a response to the Spanish influences that had predominated in Mexican art.

7. The passage most strongly supports which one of the following inferences about
Porter’s own paintings?
(A) They often contained figures or images derived from the work of African artisans.
(B) They fueled his interest in pursuing a career in art history.
(C) They were used in Porter’s book to show the extent of African influence on African-American art.
(D) They were a deliberate attempt to prove his theories about art history.
(E) They were done after all of his academic work had been completed.

Porter’s book addressed this issue, painstakingly integrating the history of African-American art into
the larger history of art in the Americas without separating it from those qualities that gave it its
unique ties to African artisanship. Porter may have been especially attuned to these ties
because of his conscious effort to maintain them in his own paintings, many of which combine
the style of the genre portrait with evidence of an extensive knowledge of the cultural history of
various African peoples.
8. Based on the passage, which one of the following, if true, would have been most
relevant to the project Porter was working on at the time of his death?

(A) African-American crafts of the eighteenth and nineteenth centuries have certain resemblances to
European folk crafts of earlier periods.
(B) The paintings of some twentieth-century European artists prefigured certain stylistic
developments in North African graphic art.
(C) The designs of many of the quilts made by African-American women in the nineteenth century
reflect designs of European trade goods.
(D) After the movement of large numbers of African-Americans to cities, the African influences in the
work of many African-American painters increased.
(E) Several portraits by certain twentieth-century European painters were modeled
after examples of Central African ceremonial masks.

Inherit (v) : thừa kế, thừa hưởng

Forebear (n) : tổ tiên

Craft (n) : đồ thủ công

Jug (n) : cái bình

Iconographically (adv) : mang tính biểu tượng

Artifact (n) : hiện vật

Predecessor (n) : tiền nhân

Definitively (adv) : dứt khoát

Precursor (n) : người đi trước

Painstakingly (adv) : siêng năng

Artisanship (n) : nghệ thuật

Attune (v) : làm cho hài hòa

Tie (n) : mối rằng buộc

Extensive (adj) : sâu rộng

Revise (v) : xem lại

Fragmentary (adj) : rời rạc, mảnh vỡ

Reckoning (n) : sự ước lượng

Sculptor (n) : nhà điêu khắc

Inquiry (n) : sự điều tra, sự tìm tòi

Contemporary (adj/n) : đồng thời/ người đương thời

Model (v) : bắt chước theo kiểu mẫu

Prefigure (v) : tiền định, đặt nền móng

Derive (v) : nguồn gốc, phát hiện, trích dẫn

Deliberate (adj) : cố ý, chủ tâm, cố tình


Mar 03 – 1

Henry David Thoreau was an American author, poet, philosopher, abolitionist, naturalist, and leading
transcendentalist. He is best known for his book Walden, a reflection upon simple living in natural
surroundings, and his essay Resistance to Civil Government (also known as Civil Disobedience), an
argument for disobedience in an unjust state.

Thoreau’s Civil Disobedience is considered by many to be the most important political document of
the 19th century. Thoreau's philosophy of civil disobedience later influenced the political thoughts
and actions of such notable figures as Leo Tolstoy, Mohandas Gandhi, and Martin Luther King, Jr.
Thoreau is even sometimes cited as an anarchist. Though Civil Disobedience seems to call for
improving rather than abolishing government —"I ask for, not at once no government, but at
once a better government"— the direction of this improvement points toward anarchism: "That
government is best which governs not at all; and when men are prepared for it, that will be the kind
of government which they will have."

In one analysis of Civil Disobedience, Richard Drinnon partly blames Thoreau for the ambiguity,
noting that Thoreau's "sly satire, his liking for wide margins for his writing, and his fondness for
paradox provided ammunition for widely divergent interpretations of Civil Disobedience.” Regardless
of whether Thoreau intended to create this ambiguity, it cannot be denied that this work has
influenced the political thoughts of both anarchists and those committed to improving and changing
government through civil disobedience.

1. The author uses the bolded quotation in order to:

A. contradict the notion that he is an anarchist.


B. highlight one interpretation of Thoreau's essay.
C. support the quotation that follows.
D. suggest that Thoreau was purposefully vague.
E. suggest that Henry David was an anarchist. (author used word ‘though’ to imply that he didn’t
directly consider Thoreau as an anarchist but rather something beyond that thought)

Thoreau is even sometimes cited as an anarchist.Though Civil Disobedience seems to call for
improving rather than abolishing government —"I ask for, not at once no government, but at
once a better government"— the direction of this improvement points toward anarchism: "That
government is best which governs not at all; and when men are prepared for it, that will be the kind
of government which they will have."
The blue sentence is not a quotation because it belongs to the author

2. The view that Thoreau may have indeed been an anarchist depends on which of the
following assumptions?

A. No other texts of Thoreau directly contradict this view.


B. Not all of Thoreau’s texts support this view
C. Other analyses of Civil Disobedience support this view.
D. All of Thoreau’s texts support this view.
E. Thoreau was not involved in some local politics.
3. The primary purpose of this passage is to:

A. suggest that Henry David Thoreau was an anarchist.


B. discuss two important works of Henry David Thoreau.
C. highlight the importance of a particular essay.
D. question the current analysis of Thoreau’s work.
E. discuss Thoreau’s importance as a political philosopher.

Abolitionist (n) : người theo chủ nghĩa bãi bỏ, ví dụ một tập quán hoặc tổ chức, đặc biệt là hình
phạt tử hình hoặc nô lệ

Naturalist (n) : nhà tự nhiên học

Transcendentalist (n) : người theo thuyết tiên nghiệm – một loại tri thức có thể có được mà không
cần hoặc trước kinh nghiệm. Ví dụ toán học và logic… là tiên nghiệm vì chúng là các tư tưởng xuất
phát chỉ từ tuy duy mà thôi ( khoa học về tự nhiên và xã hội thường được coi là hậu nghiệm – loại
tri thức đòi hỏi kinh nghiệm )

Resistance (n) : kháng chiến, sức chống cự

Disobedience (n) : sự bất tuân

Civil (adj) : dân sự

Unjust (adj) : bất công

Anarchist (n) : người theo chủ nghĩa vô chính phủ

Abolish (v) : bãi bỏ

Toward (giới từ) : về phía

Ambiguity (n) : sự mơ hồ

Sly (adj) : ranh mãnh, xảo quyệt

Satire (n) : châm biếm, chế nhạo

Fondness (n) : thật thà

Paradox (n) : nghịch lý

Ammunition (n) : đạn dược, quân nhu, dụng cụ quân đội

Divergent (adj) : khác nhau

Vague (adj) : mơ hồ, không rõ ràng, mập mờ

Quotation (n) : một trích dẫn câu nói hoặc bài phát biểu nhưng không phải của tác giả hoặc người
nói – a group of words taken from a text or speech and reported by someone other than the original
author or speaker
Mar 03 – 2

Since the 1994 introduction to supermarket shelves of Calgene’s Flavr Savr tomato, the first
genetically modified food item to be offered to consumers, transgenic foods have made a quiet
transformation of the food production system in America. The amount of farmland planted with
transgenic crops exploded from barely over 10,000 acres in 1994 to approximately 200 million acres
in 2004. The Grocery Manufacturers of America estimate that transgenic crops—primarily corn and
soybeans—now appear in approximately 75 percent of all processed foods found in grocery stores.
While many in the industry applaud this transformation as a beneficial use of technology that will
improve products and profits, many observers are concerned that the long-term effects of these
transgenic foods are still poorly understood.

Humans have been modifying food since the dawn of agriculture. The difference between
conventional selective breeding and biotechnology is that instead of relying on natural but targeted
reproductive processes, biotechnology relies on the tools of recombinant DNA technology, primarily
restriction endonucleases and ligase enzymes, to alter the genetic codes of organisms in ways that
could not occur in nature. One transgenic food organism, the “Bt potato,” combines the genetic
code of a regular potato with that of the Bacillus thuringiensis bacterium, resulting in a potato that is
poisonous to many types of insect pests, but still edible by humans

Critics of bioengineering warn that we have no long-term studies on the effects of such manipulation
of food organisms. Modified foods might have unintended consequences for the health of people
who eat them. The modifications introduced into domesticated crops might spread into wild plant
species, fundamentally altering the ecosystem. Many critics have argued that the introduction of
bioengineered foods gives biotechnology companies undue power over the processes of food
production, and thereby leaves small farmers and farmers in the developing world at the mercy of
large corporations. Perhaps the most common criticism leveled at transgenic foods is that they are
“not natural.

Despite these criticisms, the agricultural industry in America has continued to move ahead with
further research into and planting of transgenic crops. Government regulatory agencies, such as the
EPA, the FDA, and the Department of Agriculture, have essentially given biotechnology companies
free rein as long as they can establish that transgenic crops are “substantially similar” to
conventional crops, and biotechnology companies have so far successfully fought off all efforts to
have transgenic foods labeled as such. To date, no confirmed case of harmful health effects from
transgenic foods has been documented. American consumers, although they consistently voice
doubts about transgenic foods when asked about them on surveys, are apparently unconcerned that
they eat, on average, several genetically modified foods each week.

1. Which of the following statements best summarizes the main idea of the passage?

A. The tools of recombinant DNA technology have spurred a fundamental transformation in the way
in which food is engineered and grown.
B. A majority of Americans now eat transgenic foods because of the benefits in nutrition and cost
offered by these new products.
C. Despite the fact that transgenic foods have been proven unsafe for human consumption,
biotechnology companies have quietly achieved widespread distribution of these foods.
D. Genetically modified foods, although they face criticism from those who doubt their
safety, have become a significant part of the American food production system.
E. Biotechnology offers a solution to the growing levels of hunger and malnutrition in the developing
world, although critics worry that transgenic foods may have unintended health consequences

This is a standard main idea question. The first paragraph contains the main ideas of the passage,
primarily in its last sentence: “While many in the industry applaud this transformation as a
beneficial use of technology that will improve products and profits, many observers are
concerned that the long-term effects of these transgenic foods are still poorly
understood.”

Answer choice D, the correct answer, addresses the main themes of the passage in a balanced
tone that is characteristic of the passage itself.

Answer choice E focuses on a topic—the developing world—that is not a major focus of the
passage.

2. The passage mentions each of the following as concerns raised by critics of transgenic
foods EXCEPT:

A. Transgenic foods might affect the health of consumers in unforeseen ways.


B. Transgenic foods may be more resistant to insect pests than conventional crops.
C. An agricultural distribution system based on genetically modified food will leave small farmers
dependent on biotechnology conglomerates.
D. There is no data on what the health effects may be for humans who eat transgenic foods over a
period of several decades.
E. Plant species in the wild might become contaminated by genetically modified species

3. Which of the following inferences drawn from the statements in the final paragraph,
if accurate, might best explain the apparently contradictory finding that Americans
voice concerns over transgenic foods in polls but consume these foods on a regular
basis?

A. Government regulatory agencies have approved the sale of transgenic foods to consumers as
long as the producers can establish that the transgenic foods are “substantially similar” to
conventional foods.
B. American farmers planted over 200 million acres with transgenic crops in 2004, a substantial
increase from the acreage planted with transgenic crops a decade earlier.
C. American consumers may be unaware that they are eating transgenic foods because
of the biotechnology industry’s success in preventing the labeling of these foods.
D. Critics of genetic engineering have been unsuccessful in their attempts to impose bans on
transgenic food through the EPA and FDA.
E. As of the writing of the passage, no deaths or other adverse health effects had been documented
in connection with the consumption of transgenic foods.

4. The relationship between a conventional potato and a “Bt potato” is most similar to
which of the following?

A. The relationship between a conventional automobile and one that has been equipped
with a revolutionary new engine technology that allows it to fly.
B. The relationship between a regular hamburger and one that has been made poisonous through
the addition of bleach.
C. The relationship between a laptop computer and a similar computer that is twice as fast as the
first computer because it has a more powerful central processing unit.
D. A wild chicken and a domesticated chicken that has been bred to gain weight three times faster
than a wild chicken.
E. A boiled potato and a baked potato with all the fixings.

This is an applying information question. You have to determine what the relationship is between a
conventional potato and a “Bt potato,” and then see which of the answer choices presents a
corresponding relationship. From the information in the second paragraph, we know that a Bt potato
is a potato that has been modified through a new kind of technology so that it has a potentially
useful attribute—it is poisonous to insect pests but not to humans—that is not normally found in
potatoes.

Answer choice A presents a parallel situation: a car has been modified by technology so that it
possesses a potentially useful attribute—the ability to fly—that is not normally encountered in
automobiles. This looks like a strong answer, but you need to eliminate the other answer choices.

Answer choice C is partially parallel in that the second item may be more useful than the first, but
there is no mention that the first was modified to produce the second, and the distinguishing
attribute of the second computer, greater speed, is a difference in degree, and does not represent
an entirely new attribute.

Answer choice D is incorrect on similar grounds; a fatter chicken is just a variant on the old kind
of chicken, rather than a chicken with an entirely new attribute. Also, it could be argued that
selective breeding, since it is an old technology, is not a good match for the new technology that
produced the Bt potato.

5. The second paragraph plays what role in the passage?

A. It presents the first of two arguments discussed in the passage.


B. It suggests that the hypothesis advanced in the first paragraph is incomplete.
C. It refutes an argument and suggests an area for further study, which is discussed in the rest of
the passage.
D. It suggests a new way of looking at a topic that differs from the conventional view presented in
the first paragraph.
E. It provides background information relevant to the topic discussed in the following
paragraphs.

6. The author’s attitude toward transgenic food can best be described as which of the
following?

A. Angry and concerned.


B. Accepting and optimistic.
C. Enthusiastically ambivalent.
D. Interested but uncommitted.
E. Hopeful but suspicious.
This is a tone and style question. You can gauge an author’s attitude by the use of tone words with
strong positive or negative connotations. This passage, however, has very little in the way of tone
words, so you have to gauge the author’s attitude by the choice of information presented.

Answer choice D, “Interested but uncommitted,” does appear to be an accurate description,


because the author is clearly interested enough in the subject to have written the passage, but he or
she does not explicitly commit to either side.

The terms in answer choice E are too strong in both directions; the author seems somewhat
accepting of the developments in agriculture, but does not make any statements that could be
characterized as “hopeful.” The author’s decision to enumerate the concerns of critics without fully
refuting them suggests that the author could be suspicious, but there is no clear evidence that the
author is necessarily suspicious.

Answer choice D is the best answer. Note that D is the most moderate of the answer choices;
this is characteristic of tone questions. ACT rarely permits the authors of passages to express strong
emotions. When you see emotional terms like angry or enthusiastic in answer choices for this type
of question, those choices are usually wrong.

Poll (n) : cuộc thăm dò ý kiến

Recombinant (adj) : tái tổ hợp

Edible (adj) : ăn được

Manipulation (n) : sự điều khiển, thao tác

Fight off (v) : đẩy lui

Uncommitted (adj) : không chắc chắn

Conglomerate (n) : tập đoàn


Mar 04 – 2

Should taxpayers pay to read the results of public funded research or do they have the right to
those. This issue is the center of hotly contested debate in the House of Representatives as a part of
the Research Works Act. The Research Works Act would forbid the National Institute of Health to
require, as it now does, that its grantees provide copies of the papers they publish in peer-reviewed
journals to the library. If the bill passes, to read the results of federally funded research, most
Americans would have to buy access to individual articles at a cost of $15 or $30 apiece, making
citizens pay for research already funded by them.

Publishers of journals such as Cell, Science, and The New England Journal of Medicine, who are
backing the bill, argue that they add value to the finished product and that requiring them to
provide free access to journal articles within a year of publication denies them their fair
compensation and makes it difficult for them to generate profits comparable to the profits in the
industry. Furthermore, they claim that while the research may be publically funded, the journals are
not, claiming that they add significant value in the peer review process that makes the published
articles worthwhile.

But in fact, these journals receive billions of dollars in subscription payments, a good portion of their
revenue today that is derived largely from public funds. Moreover, even the peer review process,
which the journals claim is their primary value add, is funded by public funds. The researchers who
volunteer their time to review their peers’ work come primarily from universities and research
organizations that are funded by taxpayers’ dollars.

Rather than rolling back public access, Congress should move to enshrine a simple principle in
United States law: if taxpayers paid for it, they own it. This is already the case for scientific papers
published by researchers at the N.I.H. campus in Bethesda, Md., whose work has been explicitly
excluded from copyright protection since 1976 because it was funded by the public. It would be
easy to extend this coverage to all works funded by the federal government.

1. The primary purpose of the passage is to

A. Demonstrate how the publishing industry is misusing the time of Congress for their own financial
benefit.
B. Illustrate a moral dilemma facing the House of Representatives.
C. Evaluate the pros and cons of passing the Research Works Act.
D. Discuss and disprove the claims put forth by the backers of Research Works Act.
E. Advocate all publically funded research to be made open source.

‘The Research Works Act would forbid the National Institute of Health to require, as it
now does, that its grantees provide copies of the papers they publish in peer-reviewed
journals to the library.’ If the bill passes, to read the results of federally funded research, most
Americans would have to buy access to individual articles at a cost of $15 or $30 apiece, making
citizens pay for research already funded by them.

Blue sentence is the claims of researcher works act


2. Which of the following if true would undermine the claims of publishers of Cell,
Science, and The New England Journal of Medicine?

A. Publishers such as Cell and Science that are backing the Research Works Act already have higher
revenues than their European counterparts.
B. Publishers such as Cell and Science that are backing the Research Works Act are
much more profitable than publishers who publish independent research works and do
not provide free access to public.
C. The government agencies are able to negotiate serious discounts because they argue that they
are paying only for priority access for research that would be made public anyway.
D. While the overall revenue of the publishing industry in general has gone down due to the rise of
internet, the overall revenue of the publishers backing the Research Works Act has been remained
largely unchanged.
E. There is little risk to the overall profitability of the Cell, Science, etc. even if the Research Works
Act is not passed.

Publishers of journals such as Cell, Science, and The New England Journal of Medicine, who are
backing the bill, argue that they add value to the finished product and that requiring them to
provide free access to journal articles within a year of publication denies them their fair
compensation and makes it difficult for them to generate profits comparable to the profits in the
industry

3. The third paragraph performs which of the following functions

A. It summarizes the discussion thus far and suggests additional reasons why the publishers may
have been unfairly compensated.
B. It refutes the claims made by the publishers of Science, Cell, and The New England
Journal of Medicine.
C. It discusses a scenario in which the arguments of publishers of Science, Cell, and The New
England Journal of Medicine may not hold ground.
D. It puts forward new evidence to distract the focus of the House of Representatives.
E. It cites a specific case to illustrate the inconsistency in the claims made by the publishers of
Science, Cell, and The New England Journal of Medicine in the previous paragraph.

Put forth (v) : đưa ra

Inconsistency (n) : mâu thuẫn

Forbid (v) : ngăn cấm

Back (v) : ủng hộ

Explicity (adv) : rõ ràng


Mar 05 – 1

What is it that keeps the developing world in an apparent state of perpetual poverty? Poor
education, lack of basic medical care, and the Line absence of democratic structures all certainly
(5) contribute to these nations’ plight. However, according to Peruvian economist Hernando de
Soto, the overriding cause is the overwhelming prevalence of black market activity, well outside the
formal economy, in these countries. The losses (10) incurred from this condition are twofold. First,
they deny the government tax revenues which could be used to improve education, medical
treatment, and government efficiency. More important, however, they deny earners the chance to
accumulate assets (15) recognized by law and thus prevent them from leveraging those assets to
borrow. Reforming these nations’ legal systems in order to confer ownership through titling, De Soto
argues, would help the poor there access the assets their work (20) should be generating. These
assets could then be used to buy homes and construct businesses, thus building a more stable and
prosperous economy. De Soto estimates the value of these assets, which he terms “dead capital,” at
nearly $10 trillion (25) worldwide.

De Soto is not the first to locate the developing world’s problems in the domain of property rights.
Others have tried property rights reform and failed. According to de Soto, this is because (30) his
predecessors attempted to model their plans on existing, successful property rights systems. In
other words, they tried to transplant American and British property law to an inhospitable host. De
Soto argues that within many of the extralegal (35) markets of the developing world, mutually
agreed upon rules for distributing assets and recognizing property rights already exist. Rather than
force these markets to adjust to a new, foreign system of property titling, reformers should focus on
(40) codifying the existing systems wherever it is practical to do so. This would facilitate a quicker,
more natural transition to an economy that builds wealth rather than squanders it.

1. The author’s primary goal in the passage is to

A. compare several failed attempts to address a problem


B. respond to criticism of a new theory
C. identify the problems inherent in a new economic theory
D. describe a novel approach to an old problem
E. compare different property rights systems in the industrial world

Use POE. The author briefly mentions failed attempts, but it’s not the main point of the passage,
so (A) is incorrect. Choices (B) and (C) are incorrect because the author doesn’t mention any
criticisms or problems with the new theory. Choice (D) is supported by the passage because
the author says that poverty is a perpetual problem and that de Soto’s approach is different, or
novel. Choice (E) is incorrect because while it is briefly mentioned, it’s not the main point of the
passage.

2. According to the passage, de Soto believes that the quickest way to address poverty
in the developing world is to

A. increase funding for education


B. build the infrastructure to support lending
C. ensure medical care for all citizens
D. aggressively root out corruption in government
E. increase tax rates on all citizens in developing countries
3. The author’s assertion that “reformers should focus on codifying the existing systems
wherever it is practical to do so” (lines 39–41) suggests that

A. in some instances, current systems are inadequate to meet the needs of a market
economy
B. these systems are already written down and need only be enacted as law
C. where it is impractical to codify existing systems, countries should adopt American property law
D. the existing systems are superior to those currently in use in modern industrialized countries
E. improving education and medical care in these countries should take priority over reforming
property laws

4. The term “dead capital” (line 24) refers to

A. loans that are never repaid


B. failed investments in new businesses
C. cities ruined by over-industrialization
D. the proceeds of extralegal commerce
E. property passed from generation to generation

Perpetual (adj) : vĩnh viễn

Plight (n) : tình thế

Overriding (adj) : tàn phá

Overwhelming (adj) : áp đảo

Prevalence (n) : tỷ lệ lưu hành, sự đang thịnh hành

Accumulate (v) : tích trữ

Leverage (v) : tận dụng

Confer (v) : trao, thương thuyết, đàm phán

Prosperous (adj) : thịnh vượng

Inhospitable (adj) : khắc nghiệt

Extralegal (adj) : ngoại đạo, vượt quá thẩm quyền của pháp luật, không được pháp luật quy định

Codify (v) : mã hóa, lập điều lệ

Facilitate (v) : tạo điều kiện

Squander (n) : sự phung phí

Inherent (adj) : vốn có, thuộc về

Proceed (n) : tiền thu được


Mar 05 – 2

Nitrogen, which comprises 78 percent of Earth's atmosphere, is necessary for plant nutrition. But
agriculture and industry have doubled the rate at which nitrogen is "fixed" and in so doing may have
raised a serious environmental threat. Despite the natural abundance of nitrogen, most of it is in the
form of an inert gas, N2, which cannot be used by living things. To nourish plants, it must be fixed—
bonded with hydrogen or oxygen. In nature, some of this fixing is done by lightning, but most of it
is the work of certain algae and bacteria. Human activity has greatly increased the supply of usable
nitrogen. The largest artificial source of fixed nitrogen is chemical fertilizer. The cultivation of plants
such as legumes that carry nitrogen-fixing bacteria on root nodules is another major source.
Automobiles, factories, and power plants release significant amounts of fixed nitrogen by bu ming
fossil fuels such as oil and coal. Other sources include forest fires and sewage treatment.

It is uncertain what effect these changes have had so far, but the potential for harm is considerable.
Burning forests and fossil fuel creates fixed-nitrogen gases that increase smog and acid rain,
damage the ozone layer, and add to global warming by trapping heat in the atmosphere. Synthetic
fertilizers, along with this airborne fixed nitrogen, cause plants that thrive on nitrogen to drive out
other species, which may become extinct along with the animals that depend on them. Also, since
many bacteria and fungi feed on nitrogen, these nitrogen-rich plants decompose faster than other
plants. This prevents the plant community from storing larger amounts of carbon. The failure of
plant communities to absorb more carbon dioxide and store it as carbon may become another
contributor to global warming.

Finally, fixed nitrogen in the air settles and adds to the fixed nitrogen already in the soil from
chemical fertilizers. This fixed nitrogen runs off, or seeps into rivers and streams, where, with fixed
nitrogen from sewage treatment plants, it flows into lakes and oceans. Besides making drinking
water unsafe, this can lead to algae blooms that kill fish. It can also spur the growth of small aquatic
plants that cloud the water and deprive larger plants of sunlight. When these larger plants die, they
are consumed by bacteria that multiply and deplete the water of oxygen, rendering it incapable of
sustaining life.

1.According to the passage, which of the following may be an effect of artificial


fertilizers?

I. Increased acid rain


II. Extinction of some plant species
III. Algae blooms

A) I only
B) II only
C) I and II only
D) II and III only
E) I, II, and III
2. It can be inferred that the author would be most likely to agree with which of the
following statements?

A) Most of the fixed nitrogen in the environment today is of human origin.

Other sources include forest fires and sewage treatment.


B) Nitrogen in the form of N2 is relatively harmless.
C) Excess fixed hydrogen has done irreversible harm to the environment.
D) The plants that are grown for food are not the ones that do best in a high-nitrogen environment.
E) Algae blooms are caused by a loss of oxygen in the water.

The passage concerns about the harmless of one variant form of normal nitrogen – Fixed nitrogen

3. Which of the following areas would probably be hardest hit by the effects of fixed
nitrogen?

A) a region where thunderstorms are frequent


B) the part of a river upstream from a sewage treatment facility
C) an area where legumes are grown
D) a bay at the mouth of a major river that flows past farms that use chemical fertilizers
E) a region where fungi are grown commercially

4. Which of the following statements, if true, would most weaken the author's claim?

A) Forest fires generate more fixed oxygen than do automobiles.


B) Fixed-nitrogen gasses are not the only gasses that trap heat in the atmosphere
C) The demand for food grown without chemical fertilizers has been increasing.
D) Earth's forests have enormous untapped potential to absorb and store carbon
dioxide.
E) Most of the bacteria and fungi that consume nitrogen are harmless to humans.

‘When these larger plants die, they are consumed by bacteria that multiply and deplete the water of
oxygen, rendering it incapable of sustaining life.’

This statement is not relevant with E because even macteria isn’t harmless to humans,
it still can be harmless to other living lifes

As per para 2-'Synthetic fertilizers, along with this airborne fixed nitrogen, cause plants that thrive
on nitrogen to drive out other species, which may become extinct along with the animals that
depend on them. Also, since many bacteria and fungi feed on nitrogen, these nitrogen-rich plants
decompose faster than other plants. This prevents the plant community from storing larger
amounts of carbon. The failure of plant communities to absorb more carbon dioxide and
store it as carbon may become another contributor to global warming. '
If Earth's forests have enormous untapped potential to absorb and store carbon dioxide, then it
won't contribute to global warming as is claimed by the author.

Comprise (v) : bao gồm, chứa đựng

Inert (adj) : trơ

Nourish (adj) : nuôi, cấp dưỡng


Bond (v) : liên kết

Legume (n) : cây họ đậu

Sewag (n) : nước thải

Fungi (n) : nấm

Spur (v) : thúc đẩy

Deprive (v) : cướp lấy, chiếm lấy

Deplete (v) : làm cạn kiệt

Render (v) : làm trở nên

Untapped (adj) : chưa được khai thác

Hardest hit (n) : bị ảnh hưởng mạnh nhất


Mar 06 – 1

As the American workforce gets grayer, age discrimination will likely become a more prominent issue
in the courts. It is, of course, illegal to discriminate against an employee because of his or (5) her
age, and yet it is not illegal to dismiss a worker because he has a high salary and expensive health
care.

This apparent contradiction is at the heart of a raft of cases now making their way through the
(10) courts. The outcome of these cases will have broad implications for the workplace in the
coming years. By 2020, the Bureau of Labor Statistics has projected that more than half of all
workers will be over 40—many of whom, by dint of seniority and (15) promotions, will be earning
higher than median salaries, eligible for more stock options, and carrying higher health care costs as
a result of a larger number of dependents and the increased cost of health care for older workers.

(20) Is it any wonder that a bottom-line oriented business might want to shed these workers,
whose productivity is likely to plummet in the next few years, even as they become more expensive
employees?

(25) Still, the legal challenges of implementing this policy are daunting. Businesses have the right to
rate workers on their productivity and to rank them against their peers. But they are not allowed to
prejudge individuals based on their sex, race, or (30) age. Each worker must be treated on his or
her own merits, rather than by how they fit into a larger profile of the group they belong to.

For companies looking to lay off these workers, the cost of making a mistake is high; while only
(35) one in three age discrimination suits are won by the plaintiff, the awards tend to be steep and
the political fall-out harsh.

1. The primary purpose of the passage is to


A. advocate on behalf of the older American worker who could soon face dismissal
B. describe the origin of two theories of labor law and their effects on the workplace
C. present an overview of the legal ramifications of a practice some call discriminatory
D. describe the process by which America’s workforce is getting older
E. describe the methods by which a company could reduce its bottom line

2. Which of the following best describes the organization of the second paragraph of the
passage?
A. An assertion is made and then briefly contradicted.
B. A contradiction is stated and then quickly resolved.
C. A new theory is described and then qualified.
D. An apparent inconsistency is stated and its consequences outlined.
E. A conventional model is described and an alternative is introduced.

3. Which of the following, if true, would most effectively weaken the author’s assertion
that a “bottom-line oriented business” might want to lay off older workers?
A. A new study shows that, on average, younger workers earn less and have lower associated
medical costs than older workers.
B. Older workers have a higher rate of absenteeism than younger workers.
C. A new study shows that older workers are in fact more productive and have fewer
medical expenses compared to younger workers.
D. A forecasted downturn in the economy will erode profits in may American businesses.
E. A new bill scheduled to become law will make it easier for employers to employ illegal aliens.

4. It can be inferred from the passage regarding the relationship between American
companies and older Americans that
A. what is good for American companies is not necessarily good for older Americans
B. American companies are prohibited by law from practices that discriminate based on age
C. large monetary judgments from age discrimination suits might prove more expensive than paying
older employees’ salaries
D. by the year 2030, the percentage of older employees will be even higher than in the year 2020
E. some older employees may well be more productive than some younger employees

It is, of course, illegal to discriminate against an employee because of his or her age, and
yet it is not illegal to dismiss a worker because he has a high salary and expensive health care.
5. The author mentions all of the following as driving up the cost to employers for
employing workers over the age of 40 EXCEPT
A. the cost of out-placement services
B. a larger number of dependents
C. increased cost of health care
D. higher median salaries
E. the cost of employee stock options

Grayer (adj) : rầu rĩ, xám xịt, buồn rầu

Prominent (adj) : nổi bật

Dismiss (v) : cách chức, đuổi

Raft (n) : số nhiều

Seniority (n) : thâm niên

Orient (v) : định hướng

Bottom-line in business : this term is often used and refers to the profitability of a
business after all expenses are deducted from revenues. Bottom line profits are net
profits after all the costs of the business have been accounted.

Bottom-line oriented business : Công việc kinh doanh mà mục chính là lợi nhuận sau cùng, bỏ qua
các rào cản về đạo đức nếu cần thiết

Shed (v) : rũ bỏ

Plummet (v) : lao dốc, xuống cấp nhanh chóng

Plaintiff (n) : nguyên đơn, người thưa kiện

Defendant (n) : bị đơn, người bị kiện

Steep (adj) : dốc ( ám chỉ lên nhanh hay xuống nhanh, rủi ro cao )

Fall-out (n) : sự thất bại

Harsh (adj) : khắc nghiệt


Mar 06 – 2

Monogamy is not natural — hardly any species practice it, except for birds. Social monogamy
wherein two creatures mate and work together to meet their needs is especially uncommon among
nonavian warm-bloods; only about 5% of the 4,000 mammal species are monogamous. Since
mating with one female at a time lowers a male’s chances of producing as many offspring as
possible, what good, evolutionarily speaking, can come of monogamy? Why would mammals be
monogamous?

One reason, according to a report in the Proceedings of the National Academy of Sciences (PNAS),
suggests that males stayed with one female to ensure their young were not killed by another male,
but survived to reproduce to carry on their genetic lineage. Based on breeding and parenting
behaviors of 230 primate species over several generations, the researchers determined that males
balanced the need to spread their gene pool against the need to protect their young from being
killed. The attacking males needed to kill the young so they could breed with its mother, who would
delay conception of another offspring if she were nursing. So the father hung around to protect his
genetic line and help raise the young so the mother could reproduce again sooner. “This is the first
time that theories for evolution of monogamy have been systematically tested, conclusively showing
that infanticide is the driver of monogamy,” trumpeted Christopher Opie, a researcher in
anthropology. “This brings to a close the long-running debate about the origin of monogamy in
primates.”

Well, not so fast! Another study, published in the journal Science, used a similar analysis, but across
a wider sample — about 2,500 mammal species. Those authors, Dieter Lukas and Tim Clutton-
Brock, found no correlation between infanticide and monogamy. They suggested that monogamy is
about location and supply. “Monogamy develops where females live at low density,” says Lukas.
Males cannot fend off rival suitors from more than one female at a time because they’re too spread
out. To ensure their young are the ones the female is carrying, they stick with one female. “It’s a
consequence of resource defense.” This study notes that in monogamous mammalian species, the
females tend to be solitary and intolerant of other females. Unlike those of ungulates, who are rarely
monogamous, these mammals’ nutritional needs are greater, and they therefore shoo off
competitors for food resources.

As for human monogamy, the PNAS study is more comfortable extrapolating its results. Because
humans have big brains, their infants take longer to nurture and are vulnerable for longer.
Therefore, human males needed to protect their child-rearing female until breeding was done. So
how important were kids in man’s move towards monogamy? It’s a fascinating fight, but ultimately,
whether monogamy is natural is less relevant than whether it’s desirable. Human monogamy seems
to be both an acquired taste and a social necessity. The question remains whether it’s worth the
cost of learning it.

1. Which one of the following can be supported by the information given in the passage?

A. Humans are warm-blooded.


B. Safety of genetic line is essential for evolution.
C. Infanticide is the driver of monogamy in primates.
D. Ungulate females are intolerant of other females.
E. Producing as many offspring as possible will ensure genetic progression.
2. Which one of the following best describes the content of the passage?

A. A chronology of the development of different theories of evolution of human monogamy


B. An effort to explore origins of monogamy and questioning human adoption of the
practice
C. An argument concerning whether or not monogamy is natural and desirable
D. An attempt to determine which theory best explains the monogamous behavior of humans
E. A discussion of how the human monogamy differs from that of other primates

3. All of the following are key aspects of the PNAS study EXCEPT:

A. Nursing mothers delay conception of another offspring.


B. Protecting young from being killed is a necessity for evolution.
C. Male primates balance between spreading their gene pool and protecting their young from being
killed.
D. Kids are of utmost importance in making men move towards monogamy.
E. Male parental help allows females to reproduce again sooner.

4. It can be inferred from the passage that further research on which one of the
following topics will explain origin of human monogamy with greater clarity?

A. formation of social structure and customs


B. whether monogamy is desired
C. effect of monogamy on human evolution
D. how human monogamy differs from monogamy in other species
E. monogamy in areas with low density female population

Monogamy (n) : chế độ một vợ một chồng (adj : monogamous)

Nonavian (adj) : không thuộc gia cầm (relating to birds)

Lineage (n) : dòng dõi, huyết thống, nòi giống

Primate (n) : linh trưởng

Conception (n) : hành động thụ thai

Infanticide (n) : tội giết trẻ nhỏ dưới 1 năm tuổi, kẻ giết trẻ sơ sinh

Fend off (v) : chống đỡ

Suitor (n) : người cầu hôn

Ungulate (n) : động vật móng guốc

Extrapolate (v) : ngoại suy – mở rộng các kết luận có được tự sự quan sát một hiện tượng sang
những hiện tượng ngoài phạm vi đã được xem xét

Extrapolation (n) : sự ngoại suy – quá trình ước tính vượt ra ngoài phạm vi quan sát ban đầu

Nurture (v) : dưỡng dục

Formation (n) : sự hình thành, sự cấu thành, hệ thống


Mar 07 – 1

The Japanese system of kanban (just-in-time inventory control), credited for much of the Japanese
automobile industry's success in the 1980s, may be falling out of favor. The large-scale
(5) demands of international trade reward more planned schedules of production and have led to a
reintroduction of an old friend: the specialized worker.

The conflict between most marketing and production (10) departments is well documented.
Marketing departments need to respond quickly to changing customer wants and needs; production
departments need to plan their production on a two- week schedule with each department
informing (15) other departments of its backlogs, inventories and outputs. To make
the kanban system work however, the manufacturers had to train production employees to shift on
a moment's notice to any job in the factory, as needed, thus diminishing (20) the role of the
specialized worker.

But kanban may have caused its own demise. By being so efficient and responsive, Japanese auto
mobile manufacturers find themselves confronted with a large and predictable global demand.
(25) Requests for automobiles are increasing at such a steady rate that managers can now
determine production schedules up to ten months in advance and, as a result, workers no longer
need to jump from position to position. Since long-term (30) scheduling has become
possible, productivity has risen 27 percent.

1. It can be inferred from the passage that the conflict between marketing departments
and production departments

A. resulted from both departments’ failure to communicate concerning backlogs, inventories, and
outputs.
B. led to the training of marketing employees in the kanban system, enabling them to shift rapidly to
any job in the factory.
C. stemmed from the production department’s need to schedule and the marketing
department’s need to respond quickly.
D. rested on the shortened two-week timeframe of the kanban system.
E. was the primary reason for the lack of success of the Japanese automobile industry in the 1980s.

2. The author cites the productivity increase of 27 percent in order to

A. prove the argument that kanban workers are more efficient than specialized workers.
B. provide an example of how rapid response to global demands can improve productivity.
C. argue against the reintroduction of the specialized worker.
D. support the thesis that predictable demand may encourage an otherwise less efficient
practice.
E. challenge the theory that kanban workers are less efficient in a global market.

This question asks about what the author does, so you need to be conscious of just how short a
GMAT passage is. The author has only about 250 words—he can illustrate, illuminate, and elucidate;
display, discuss, or divulge. The author cannot prove, refute, resolve, or define—not in 250 words.
So keep that in mind as you work through your Four-Step Approach, and especially as you use POE.
3. It can be inferred from the passage that specialized workers

A. require more training than employees under the kanban system.


B. play a more limited role in the kanban system.
C. are useful only when long-term planning becomes possible.
D. respond to changes in demand quickly and efficiently.
E. handle backlog and inventory problems more efficiently than do kanban-trained workers.

4. The passage is primarily concerned with

A. pointing out the variety of management styles at work in the automobile industry.

This answer choice is too broad. The passage is about kanban, not the automobile industry as a
whole.
B. establishing kanban as a viable option in the automobile industry.
C. illustrating the effects of kanban on employee training.
D. showing that kanban can work only in the Japanese automobile industry.
E. describing how a business practice can render itself obsolete.

Render itself (v) : khiến bản thân nó trở nên

Obsolete (adj) : lỗi thời

Business practice (n) : thực tiễn kinh doanh

Demise (n) : sự chết

Backlog (n) : tồn đọng


Mar 08 - 1

Music and literature, rivals among the arts, have not coexisted without intruding on each other's
terrain. Ever since what we think of as "literature" developed out of the sounds of spoken, sung, and
chanted art, (5) writing has aspired to the condition of music, in which form contributes significantly
to content. Nowhere is this truer than in the African American tradition, whose music is often
considered its greatest artistic achievement and one of the greatest contributions to (10) North
American art. But while many African American writers have used musicians and music as theme
and metaphor in their writing, none had attempted to draw upon a musical genre as the structuring
principle for an entire novel until Toni Morrison did so in her 1992 (15) novel Jazz, a novel set in the
Harlem section of New York City in 1926.

In Jazz, the connection to music is found not only in the novel's plot but, more strikingly, in the way
in which the story is told. The narration slips easily from (20) the third-person omniscience of the
narrator's disembodied voice-which, though sensitive and sympathetic, claims no particular identity,
gender, or immersion in specific social circumstances-to the first-person lyricism of key characters.
But throughout (25) these shifts, the narrator is both generous with the characters' voices and
protective of his or her mastery over the narrative as a whole. On the one hand, the central
characters are given the responsibility of relating their parts of the overarching story, but on
(30) the other hand, their sections are set off by quotation marks, reminders that the narrator is
allowing them to speak. In this way, the narrative is analogous in structure to the playing of a jazz
band which intertwines its ensemble sound with the individuality (35) of embedded solo
performances.

In jazz, composer and conductor Duke Ellington was the first to construct his compositions with his
individual musicians and their unique "voices" in mind. Yet no matter how lengthy his musicians'
(40) improvisations, no matter how bold or inventive their solos might be, they always performed
within the undeniable logic of the composer's frame-they always, in other words, performed as if
with quotation marks around their improvisations and solos. It is this (45) same effect that Toni
Morrison has achieved in Jazz, a literary rendering of an art of composition that Duke Ellington
perfected around the time in which Jazz is set.

In this novel, Morrison has found a way, (50) paradoxically, to create the sense of an ensemble of
characters improvising within the fixed scope of a carefully constructed collective narration. By
simulating the style of a genius of music while exhibiting Morrison's own linguistic virtuosity,
(55) Jazz serves to redefine the very possibilities of narrative point of view.
1. Which one of the following most accurately states the main point of the passage?

(A) In Jazz, Morrison has realized a significant artistic achievement in creating the first African
American work of fiction whose plot, themes, and setting are all drawn from the world of jazz.
(B) Morrison's striking description of a musical ensemble performance containing solo improvisations
constitutes an important artistic innovation and makes Jazz an important model for other writers.
(C) Although many African American writers have used music as a central metaphor in their works,
Morrison's 1992 novel is unique and innovative for using jazz as its central metaphor.
(D) Building on the works of many African American writers and musical composers, Morrison has
over the years developed an innovative jazzlike style of narration, which she used especially
effectively in the novel Jazz.
(E) In Jazz, Morrison has succeeded in creating an original and effective narrative
strategy that is a literary analogue of Duke Ellington's style of musical composition.

2. The author's discussion in the first paragraph proceeds in which one of the following
ways?

(A) from a common claim about the arts, to a denial of this claim as applied to a particular artistic
tradition, to a hypothesis about a particular individual
(B) from a general remark about two art forms, to a similar observation about a
particular artistic tradition, to a specific comment about a particular work that
exemplifies the prior remarks
(C) from a description of a common claim about two art forms, to some specific evidence that
supports that claim, to an inference regarding a particular individual to whom that claim applies
(D) from an observation about a specific art form, to a more general claim about the applicability of
that observation to other art forms, to a particular counterexample to the first observation
(E) from general comments about the arts, to a purported counterexample to the general comments
as applied to a particular artistic tradition, to a description of a particular work that bears out the
original comments

3. The author's assertion in lines 10-16 would be most called into question if which one
of the following were true?

(A) Even a casual reading of Jazz makes it evident that the author has intentionally tried to simulate
a style of jazz performance in the narration of the story.
(B) A small number of African American novelists writing earlier in the twentieth
century sought to base the form of their work on the typical structure of blues music.
(C) All novels about nonliterary arts and artists appear as if their authors have tried to make their
narrative styles reminiscent of the arts in question.
(D) Depending partly on whether or not it is read aloud, any novel can be found to be somewhat
musical in nature.
(E) A smaller number of African American writers than of non-African American writers in North
America have written novels whose plots and characters have to do with music.
4. The information in the passage most supports which one of the following statements
regarding Ellington?

(A) Morrison has explicitly credited him with inspiring the style of narration that she developed in
Jazz.
(B) He prevented his musicians from performing lengthy solos in order to preserve the unity of his
compositions.
(C) He is a minor character in Morrison's Jazz.
(D) He composed music that was originally intended to be performed by the specific
musicians he conducted.
(E) Though he composed and conducted primarily jazz, he also composed some music of other
genres.

5. The author's primary purpose in the passage is to

(A) analyze and commend the variety of contributions to the art of the novel made by a particular
writer
(B) contrast a particular African American writer's work with the work of African American
practitioners of another art
(C) describe a particular aspect of one work by a particular writer
(D) demonstrate the ways in which two apparently dissimilar arts are, on a deeper analysis, actually
quite similar
(E) detail the thematic concerns in the work of a particular writer and identify the sources of those
concerns

6. Each of the following excerpts from the passage exhibits the author's attitude toward
the novel Jazz EXCEPT:

(A) " ... whose music is often considered its greatest artistic achievement and one of the
greatest contributions to North American art" (lines 8-10)
(B) "In Jazz, the connection to music is found not only in the novel's plot but, more strikingly, in the
way in which the story is told" (lines 17-19)
(C) "The narration slips easily from the third-person omniscience of the narrator's disembodied
voice ... " (lines 19-21)
(D) " ... Morrison has found a way, paradoxically, to create the sense of an ensemble of characters
improvising within the fixed scope ... " (lines 49-51)
(E) "By simulating the style of a genius of music while exhibiting Morrison's own linguistic
virtuosity ... " (lines 52-54)
7. It can be inferred from the passage that the author would be most likely to believe
which one of the following?

(A) In Jazz, Morrison has perfected a style of narration that had been attempted with little success
by other North American writers in the twentieth century.
(B) Because of its use of narrative techniques inspired by jazz, Morrison's novel represents the most
successful representation to date of the milieu in which jazz musicians live and work.
(C) In Jazz, Morrison develops her narrative in such a way that the voices of individual characters
are sometimes difficult to distinguish, in much the same way that individual musicians' voices merge
in ensemble jazz playing.
(D) The structural analogy between Jazz and Duke Ellington's compositional style
involves more than simply the technique of shifting between first-person and third-
person narrators.
(E) Morrison disguises the important structural connections between her narrative and Duke
Ellington's jazz compositions by making the transitions between first- and third-person narrators
appear easy.

Eliminate A and C because Morrison used Jazz as the structuring principle for an entire novel but
Morrison had no objects finished in the Jazz field.

8. The passage contains information that most helps to answer which one of the
following questions?

(A) Do any African American visual artists also attempt to emulate African American music in their
work?
(B) In what way is Jazz stylistically similar to other literary works by Morrison?

The passage mentions about only one work done by Morrison


(C) After the publication of Jazz, did critics quickly acknowledge the innovative nature of the
narrative style that Morrison uses in that novel?
(D) How many works by African American writers have been inspired by the music of Duke
Ellington?
(E) What characteristic of Jazz is also present in the work of some other African
American writers?

Intrude (v) : xâm nhập

Terrain (n) : địa hình, địa thế, vị trí

Metaphor (n) : ẩn dụ

Plot (n) : cốt truyện

Strikingly (adv) : nổi bật

Narration (n) : sự tường thuật

Narrator (n) : người dẫn chuyện

Omniscience (n) : toàn tri – khả năng biết tất cả mọi thứ

Lyricism (n) : thơ trữ tình


Mastery (n) : quyền lực, sự hiểu biết rõ ràng

Overarching (adj) : bao trùm

Intertwine (v) : đan xen

Ensemble (n) : sự hòa hợp, toàn thể

Composer (n) : nhà soạn nhạc

Conductor (n) : nhạc trưởng

Composition (n) : sáng tác

Improvisation (n) : ngẫu hứng

Bold (adj) : liều lĩnh, quả quyết

Inventive (adj) : sáng tạo

Simulate (v) : mô phỏng

Virtuosity (n) : tài năng

Linguistic (adj) : ngôn ngữ học

Possibility (n) : khả năng

Distinguish (v) : phân biệt

Disguise (v) : che đậy, giả dạng


Mar 07 – 2

In response to rapidly rising crime rates, legislators in Georgian England initiated a policy of
imposing mandatory capital punishment for what to modem eyes is an astonishing range of crimes.
Over 200 crimes were punishable by hanging: not only murder and kidnapping, but forgery, petty
theft, and "posing as a gypsy." However, although the number of crimes punishable by death
increased, and more and more criminals were brought to trial, the numbers of people who were
actually hanged fell.

Simple decency alone accounts for many of the instances in which the English chose not to use their
lethal laws. Judges could commute the death sentence on suitably penitent felons. Juries could
undervalue stolen goods so as to bilk the prosecution. (Since the law demanded that anyone who
stole 40 shillings or more must hang, hundreds of theft of goods valued by the jury at 39 shillings
were handed down every year for the convictions.) However, so many remissions seem to indicate
that the exercising of mercy was also part of an active strategy. If a convicted felon could obtain
references and sub-petitions from the clergy or other respectable persons, he could seek the Royal
Mercy by petitioning the Home Secretary. It was within the power of the king to override courts and
sentences and he often did, proving to his subjects that he was concerned with their cares.

Simply repealing the hanging laws would have had quite a different effect than maintaining but not
always using them. If each reprieve were a special case where the ruler chose mercy over death,
then that decision would be repaid in devotion and gratitude. Without the Royal Mercy and judicial
commission, so many Englishmen would have met their end at the gallows that it would have
provoked general riots.

1. The author mentions the crimes of forgery, petty theft, and "posing as a gypsy"
(Highlighted) in order to

A) give examples of crimes which were typically punishable by death in Georgian England

B) illustrate the types of crime prevalent in Georgian England

C) provide examples of crimes that were formerly considered to be much more serious than they are
now

D) highlight how the death penalty in Georgian England was even applied to what would
now be considered minor crimes

E) contrast the seventy of punishment in Georgian England with the 4. magnitude of the crime
finance and other political issues.
2. The primary purpose of the passage is to

A) outline judicial measures brought in response to increasing crime rates in Georgian England

B) demonstrate how the Royal Mercy and judicial commission were used to secure the king’s
position in Georgian England

C) show why the number of people hanged in Georgian England fell, despite rise in
number of capital offenses and criminal trials

D) review and evaluate the effectiveness of capital punishment in Georgian England

E) define the limits of judicial and royal power in Georgian England

3. According to the passage, which of the following statements is NOT true of the legal
system of Georgian England?

A) The law provided for mandatory penalties for some crimes.

B) Judges were compelled to adhere to the letter of the law.

C) Juries had latitude to determine the precise charges on which they convicted prisoners.

D) There was a system of appeals in place.

E) The king had ultimate authority over the administration of justice.

Legislator (n) : nhà lập pháp

Initiate (v) : khởi xướng

Astonishing (adj) : kinh ngạc

Forgery (n) : giả mạo

Pose (v) : tạo dáng

Decency (n) : sự quyết định

Lethal (adj) : gây chết người

Commute (v) : giao hoán

Penitent (adj) : sám hối, hối hận

Felon (n) : kẻ phạm tội

Jury (n) : bồi thẩm đoàn

Prosecution (n) : truy tố

Remission (n) : sự giảm bớt, sự xá tội


Petition (n) : kiến nghị

Clergy (n) : giáo sĩ

Home secretary (n) : bộ trưởng nội vụ

Repeal (v) : bãi bỏ

Reprieve (n) : sự xá trạng

Devotion (n) : sự tận tâm, lòng mộ đạo

Judical commission (n) : ủy ban tư pháp

Riot (n) : bạo loạn

Compel (v) : bắt buộc

Adhere (v) : tuân thủ

Appeal (n) : kháng cáo


Mar 08 – 2

Pretty much everybody agrees that clean air is a good thing, right? Evidently not so. Since the
1960s, when people started talking about clean air in the first place, the American energy industry,
which includes coal companies, oil companies, and utility companies, has dragged its heels on every
initiative to improve the quality of the air we breathe. Even after the Clean Air Act of 1970 and its
amendments in 1977 and 1990 made it clear that controlling air pollution is a national priority, these
companies have found tricks and loopholes to avoid compliance.

Perhaps the most egregious loophole is the one that allows older power plants to disregard limits on
sulfur dioxide emissions until they undergo a major renovation, at which point they have to comply.
Sulfur dioxide from coal-burning power plants is the primary cause of acid rain in North America.
The Clean Air Act states that when coal burning power plants upgrade their equipment, they must
then comply with sulfur dioxide limitations by either installing scrubbing equipment that cleans the
emissions or using fuel with lower sulfur content. The law tied the timing of compliance to major
renovations in order to give power plants a grace period in which to comply. Many power plants,
however, have exploited a loophole in this law by instituting a series of “minor” renovations that, in
effect, upgrade their equipment without requiring them to comply with the Clean Air Act. Some
plants have cheated the system by undergoing “minor” renovations for decades.

The power companies claim that they have to resort to these underhanded measures because the
cost of compliance with the Clean Air Act is too high. And if everyone else is cheating the system,
why should they have to install costly sulfur dioxide scrubbers? This cost argument falls apart upon
scrutiny. Since 1977, more than 400 power plants across the country have managed to comply with
the restrictions and are still making money. The sulfur dioxide scrubbing equipment has turned out
to be far less expensive than the power industry naysayers claimed it would be. Many power plants
have even complied with the emissions limits and reduced their operating costs by switching from
high-sulfur Appalachian coal to the low-sulfur coal produced in western states such as Wyoming and
Idaho. Western coal is not only cleaner than eastern coal, but also, because it is generally closer to
the surface, as much as 30 percent less expensive to extract.

Clearly, the costs of compliance with the Clean Air Act can be justified, but if these companies were
honest, such justifications would not have to be made. If they were honest, they would
acknowledge the costs of not complying: the health costs of increased rates of asthma and lung
cancer in high-emissions areas; the environmental costs of acid-scarred forests and lakes; the
aesthetic costs of a haze of sulfur dioxide cutting visibility across the eastern United States to only
half of what it was in preindustrial times. When you look at the true costs you have to ask, is any
cost too high for clean air?

1. According to the information given in the passage, sulfur dioxide emissions are linked
to all except which of the following phenomena?

A. Reduced visibility in the eastern United States


B. Damage to the ozone hole
C. Increased rates of asthma
D. Acid rain
E. Damaged forests
2. In the fourth paragraph, the passage mentions the “400 power plants” for what
purpose?

A. To provide concrete evidence that many power plants have complied with the Clean
Air Act provisions without undergoing ruinous financial hardship
B. To demonstrate the size and influence of the energy industry in the United States
C. To demonstrate that only a fraction of the power plants in the country have complied with the
Clean Air Act, while hundreds of others have avoided compliance through tricks and loopholes
D. To demonstrate that companies can both comply with the Clean Air Act and achieve reductions in
their operating costs by employing new, more efficient technologies
E. To suggest that those companies that have not complied are in the minority

3. Which of the following statements, if true, would provide the strongest argument for
a utility company spokesman wishing to refute the arguments expressed in the
passage?

A. Over the last decade, the energy industry has funded an environmental initiative that has planted
more than 200,000 new trees.
B. The dangers of acid rain to human health have been wildly exaggerated by environmental
extremists who seek to scare the general public.
C. The specifications of the Clean Air Act, although well intentioned, in practice require
power plants to adopt less efficient technologies that increase emissions of atmospheric
pollutants other than sulfur dioxide that have been linked to equally serious problems.
D. A substantial upgrade to a coal-burning power plant that includes the installation of sulfur dioxide
scrubbing equipment can cost hundreds of millions of dollars, although companies can often recoup
most of these costs over the following years as a result of efficiency benefits from the upgrade.
E. The scientific data upon which the Clean Air Act was based have not been corroborated by the
scientists at the Center for Atmospheric Truth, a research group funded by a consortium of energy
companies.
Mar 09 – 1

The two construction periods at Aztec, New Mexico as indicated by the modern tree-ring dating are
corroborated nicely by other evidence found by archaeologist Earl H. Morris in the 1920’s that Aztec
actually was built by one group of people, abandoned, and then reoccupied at a later date by a
slightly different group of people.Throughout all the rooms he dug during his early 20th century
excavations, Morris found sterile layers of windblown sand and ruined debris from falling walls and
ceilings.In this debris and under the sand, he found Chaco-like pottery and artifacts.In addition
there were surprisingly few burial sites.Even today, archeologists have located few Chaco-type
burials in Chaco Canyon itself.Whatever the burial customs of the Chaco people may have been,
they have eluded archeologists for many decades.The absence of burials of this period at Aztec is a
clue that probably a group of Chaco-like people, bearing the distinctive Chaco culture, may actually
have moved into the Aztec area.

Granting that the local sandstone was not quite as easily worked as that at Chaco, the large-size
rooms, the high ceilings, the banded-veneer masonry walls, the large doorways, and other
techniques used were very similar to the architectural techniques of the Chaco area.Overlying the
Chaco debris and sterile sand layers, Morris found pottery, household utensils, and burials
characteristic of the classic Mesa Verde Period—a period which occurred later than the great Chaco
Period.In addition, there were obvious architectural signs of rebuilding and remodeling within the
pueblo.Large Chaco-type rooms had been made smaller by wattle-and-daub partition walls, while
doorways had been shortened and narrowed more like the ones at Mesa Verde.Thus there were two
definite periods of occupation at Aztec, one by a Chaco-like people and one by a Mesa Verde-type
people.

1. The author of the passage likely mentions both Morris and the tree-ring dates in order
to
A. Explain how both natural and man-made evidence contributes to debunking a myth about a
particular site.
B. Describe how the later remodeling of Aztec was in the Mesa Verde-style.
C. Challenge the idea that the Mesa Verde people preceded the Chaco people at Aztec.
D. Bolster his/her own argument through both recently discovered and historical
evidence.
E. Prove that only the Chaco and Mesa Verde people lived at Aztec.

2. The author is primarily concerned with


A. Refuting a misconception with specific support
B. Describing the findings of an archaeologist with whom the author partially agrees
C. Stating a theory and providing support for it
D. Reframing the timeline for the habitation of a historical site
E. Arguing for the existence of different cultures

3. Which of the following is implied about the Aztec site?


A. It was not extensively excavated until the 1920’s.
B. Its population was a mix of Chaco and Mesa Verde peoples.
C. Aztec was uninhabited by the end of the Mesa Verde period.
D. The Mesa Verde people did not take most of their possessions with them when they abandoned
the site.
E. The Aztec site excavated under Morris was expected to have more burial sites than
found.

4. Which of the following can be inferred from the passage?


A. The pottery belonging to the Chaco period was distinctively different from that
belonging to the Mesa Verde period.
B. The Chaco people died out before the Mesa Verde people did.
C. The dwellings of the Chaco Canyon were more intimate in size than those of the Mesa Verde
people.
D. The burial customs of the Mesa Verde people are also unknown.
E. The Aztec people never actually lived in Aztec, New Mexico.

Corroborate (v) : chứng thực

Sterile (adj) : vô trùng – free from bacteria or other living microorganism

Debris (n) : mảnh vụn

Pottery (n) : đồ gốm

Burial site (n) : bãi chôn lấp

Elude (v) : trốn tránh, thoát khỏi

Bear (v) : mang theo

Distinctive (adj) : đặc biệt, đặc sắc, riêng biệt

Masonry (n) : vật xây cất bằng gạch

Veneer (n) : tấm gỗ ép

Overlie (v) : nằm ở bên trên

Utensil (n) : nông cụ, đồ dùng

Peublo (n) : cộng đồng người mỹ bản địa

Partition (n) : vách ngăn

Dwelling (n) : nơi trú ngụ

Bolster (v) : tăng cường

Uninhabited (adj) : không có người ở


Mar 09 – 2

Kazuko Nakane’s history of the early Japanese immigrants to central California’s Pajaro Valley
focuses on the development of farming communities there from 1890 to 1940. The Issei (first-
generation immigrants) were brought into the Pajaro Valley to raise sugar beets. Like Issei laborers
in American cities, Japanese men in rural areas sought employment via the “boss” system. The
system comprised three elements: immigrant wage laborers; Issei boardinghouses where laborers
stayed; and labor contractors, who gathered workers for a particular job and then negotiated a
contract between workers and employer. This same system was originally utilized by the Chinese
laborers who had preceded the Japanese. A related institution was the “labor club,” which provided
job information and negotiated employment contracts and other legal matters, such as the rental of
land, for Issei who chose to belong and paid an annual fee to the cooperative for membership.

When the local sugar beet industry collapsed in 1902, the Issei began to lease land from the valley’s
strawberry farmers. The Japanese provided the labor and the crop was divided between laborers
and landowners. The Issei thus moved quickly from wage-labor employment to sharecropping
agreements. A limited amount of economic progress was made as some Issei were able to rent or
buy farmland directly, while others joined together to form farming corporations. As the Issei began
to operate farms, they began to marry and start families, forming an established Japanese American
community. Unfortunately, the Issei’s efforts to attain agricultural independence were hampered by
government restrictions, such as the Alien Land Law of 1913. But immigrants could circumvent such
exclusionary laws by leasing or purchasing land in their American-born children’s names.

Nakane’s case study of one rural Japanese American community provides valuable information about
the lives and experiences of the Issei. It is, however, too particularistic. This limitation derives from
Nakane’s methodology—that of oral history—which cannot substitute for a broader theoretical or
comparative perspective. Future research might well consider two issues raised by her study: were
the Issei of the Pajaro Valley similar to or different from Issei in urban settings, and what variations
existed between rural Japanese American communities?

1. The primary purpose of the passage is to

(A) defend a controversial hypothesis presented in a history of early Japanese immigrants to


California
(B) dismiss a history of an early Japanese settlement in California as narrow and ill constructed
(C) summarize and critique a history of an early Japanese settlement in California
(D) compare a history of one Japanese American community with studies of Japanese settlements
throughout California
(E) examine the differences between Japanese and Chinese immigrants to central California in the
1890’s
2. Which of the following best describes a “labor club,” as defined in the passage?

(A) An organization to which Issei were compelled to belong if they sought employment in the
Pajaro Valley
(B) An association whose members included labor contractors and landowning “bosses”
(C) A type of farming corporation set up by Issei who had resided in the Pajaro Valley for some time
(D) A cooperative association whose members were dues-paying Japanese laborers
(E) A social organization to which Japanese laborers and their families belonged

3. Based on information in the passage, which of the following statements concerning


the Alien Land Law of 1913 is most accurate?

(A) It excluded American-born citizens of Japanese ancestry from landownership.


(B) It sought to restrict the number of foreign immigrants to California.
(C) It successfully prevented Issei from ever purchasing farmland.
(D) It was applicable to first-generation immigrants but not to their American-born
children.
(E) It was passed under pressure from the Pajaro Valley’s strawberry farmers.

4. Several Issei families join together to purchase a strawberry field and the necessary
farming equipment. Such a situation best exemplifies which of the following, as it is
described in the passage?

(A) A typical sharecropping agreement


(B) A farming corporation
(C) A “labor club”
(D) The “boss” system
(E) Circumvention of the Alien Land Law

5. The passage suggests that which of the following was an indirect consequence of the
collapse of the sugar beet industry in the Pajaro Valley?

(A) The Issei formed a permanent, family-based community.


(B) Boardinghouses were built to accommodate the Issei.
(C) The Issei began to lease land in their children’s names.
(D) The Issei adopted a labor contract system similar to that used by Chinese immigrants.
(E) The Issei suffered a massive dislocation caused by unemployment.

6. The author of the passage would most likely agree that which of the following, if it
had been included in Nakane’s study, would best remedy the particularistic nature of
that study?
(A) A statistical table comparing per capita income of Issei wage laborers and sharecroppers in the
Pajaro Valley
(B) A statistical table showing per capita income of Issei in the Pajaro Valley from 1890 to 1940
(C) A statistical table showing rates of farm ownership by Japanese Americans in four
central California counties from 1890 to 1940
(D) A discussion of original company documents dealing with the Pajaro Valley sugar beet industry
at the turn of the century
(E) Transcripts of interviews conducted with members of the Pajaro Valley Japanese American
community who were born in the 1920’s and 1930’s

7. It can be inferred from the passage that, when the Issei began to lease land from the
Valley’s strawberry farmers, the Issei most probably did which of the following?

(A) They used profits made from selling the strawberry crop to hire other Issei.
(B) They negotiated such agricultural contracts using the “boss” system.
(C) They paid for the use of the land with a share of the strawberry crop.
(D) They earned higher wages than when they raised sugar beets.
(E) They violated the Alien Land Law.

Sugar beet (n) : củ cải đường

Laborer (n) : người lao động

Boardinghouse (n) : nhà trọ

Precede (v) : tới trước

Lease (v) : cho thuê

Hamper (v) : cản trở

Circumvent (v) : phá vỡ

Particularistic (adj) : đặc thù, tỉ mỉ

Methodology (n) : phương pháp luận

Substitute (n) : thay thế

Comparative (adj) : so sánh

Dues-pay (v) : đóng phí

Reside (v) : cư trú


Mar 10 – 1

Morris is familiar as the Oscar-winning director of haunting, enigmatic documentary feature films.
Yet running through all of Morris’ movies as an obsessive leitmotif, are questions, about the
misleading nature of visual representation, the ethical shadows lurking in the margins of the picture
frame or the movie still, the myriad ways in which we interpret images selectively to confirm our
pre-existing biases.

Virtually never seen on-camera, Morris as an interviewer typically adopts an attitude that suggests a
film noir detective or forensic scientist with a PhD in philosophy. He’s an erudite gumshoe, weighing
the facts , now and then interjecting a wisecracking observation, always more intent on exposing his
subjects’ (and the audience’s) blind spots than on arriving at comforting conclusions. Moral
ambiguity is his métier, enlightened doubtfulness is his default mode.

In “Believing Is Seeing”, Morris again assumes his sphinx-like stance, posing endless riddles and
rhetorical questions throughout the six thematically concentric essays that make up the book.

His opening chapter recounts the fascinating, long-running debate over whether the esteemed
British photographer Roger Fenton artfully rearranged clusters of Russian cannon balls on a desolate
road to create his famous, iconic shot of a Crimean War landscape. A Susan Sontag essay, asserting
that Fenton did in fact stage this chilling vision of arbitrary death in April 1855, sets Morris’ restless
mind in motion.

In the concluding essay, “Whose Father Is He?” Morris similarly revisits a long-ago visual crime
scene, so to speak. This time it’s the Gettysburg battlefield of July 1863, where among the
thousands of dead was an otherwise unidentifiable Union solider clutching in his hand an ambrotype
of three young children. The story of the ensuing inquiry into the fallen patriot’s identity is a shaggy-
dog tale with a gothic-horror twist.

So in a way are Morris’ probings of the stomach-churning acts committed by U. S. troops at Abu
Ghraib. Although much of the material presented here was previously laid out in “Standard
Operating Procedure,” Morris’ assertion, that the repellent photos of soldiers leering over an Iraqi
corpse helped camouflage the crimes’ higher-ranking perpetrators, is an argument that warrants
retelling – and national soul-searching.

At its core, though, “Believing is Seeing” is an elegantly conceived and ingeniously constructed work
of cultural psycho-anthropology wrapped around a warning about the dangers of drawing inferences
about the motives of photographers based on the split-second snapshots of life that they present to
us. It’s also a cautionary lesson for navigating apparitions dancing before our eyes.
1. The primary purpose of the passage is to

A) Summarize Morris’ exposition on the vindicative aspects of photographs.


B) make a case that photographs can draw at least as much from reality as fantasy.
C) consider a compelling challenge to the constructivist view of photography.

D) untangle the mysteries behind some of the world’s most iconic documentary photographs.
E) present an expose regarding manipulation of photographic evidence.

2. The author mentions "Susan Sontag" (Highlighted) primarily in order to

A) support Fenton’s Crimean photo-thesis.


B) corroborate Morris ‘argument on Fenton’s culpability.
C) investigate the controversy behind Fenton’s iconic photo.
D) provide another example of photo-journalism.
E) Suggest that she is a Fenton scholar.

3. Which of the following most accurately describes Morris’ opinion of the Abu Gharib
crimes?

A) The photograph could serve as both an expose, and as a cover-up.


B) The photograph exposes wrong doing up the entire chain of command.
C) The photograph reveals prisoner-abuse far beyond the scope of the “Standard Operating
Procedure”.
D) The photograph functioned as a clinical autopsy of criminal misconduct.
E) The photographs in question imply that different protocols of war apply for problematic nations.

Compelling (adj) : thuyết phục

Constructivist (n) : thuyết kiến tạo

Corroborate (v) : chứng thực

Culpability (n) : sự có tội

Enigmatic (adj) : bí ẩn

Obsessive (adj) : ám ảnh

Leitmotif (n) : nét chủ đạo

Misleading (adj) : gây hiểu nhầm

Lurk (v) : ẩn nấp

Margin (n) : lề trang sách

Myriad (adj) : vô số

Interpret (v) : giải nghĩa

Pre-existing (adj) : tồn tại từ trước giờ

Bias (n) : thành kiến


Virtually (adv) : hầu như

Noir detective (n) : thể loại phim tội phạm, tiểu thuyết điều tra

Forensic (adj) : pháp y

Erudite (adj) : uyên bác, thông thái

Gumshoe (n) : thám tử

Interject (v) : xen vào

Wisecracking (adj) : thông thái

Moral ambiguity (n) : sự mơ hồ về đạo đức

Métier (n) : nghề nghiệp

Doubtfulness (n) : sự nghi ngờ

Enlighten (v) : giác ngộ, làm sáng tỏ

Sphinx-like (adj) : giống nhân sư

Stance (n) : lập trường

Riddle (n) : câu đố

Rhetorical (adj) : hùng biện

Thematically (adv) : theo chủ đề

Concentric (adj) : đồng tâm

Recount (v) : kể lại

Esteem (v) : quý trọng, yêu mến

Artfully (adv) : một cách nghệ thuật

Rearrange (v) : sắp xếp lại

Cluster (n) : cụm, đoàn, thành bầy

Desolate (adj) : hoang vắng

Iconic shot (n) : ảnh chụp mang tính biểu tượng

Arbitrary (adj) : tùy ý, tự tiện, độc đoán

Restless (adj) : bồn chồn

Clutch (v) : nắm chặt, bám lấy

Ensuing (adj) : tiếp theo, theo sau

Inquiry (n) : điều tra, do thăm tin tức

Patriot (n) : nhà ái quốc


Corpse (n) : tử thi

Camouflage (n) : ngụy trang

Perpetrator (n) : thủ phạm

Elegantly (adv) : thanh lịch

Conceived (adj) : quan niệm, nhận thức

Ingeniously (adv) : khéo léo

Inference (n) : kết luận, suy luận

Cautionary (adv) : thận trọng

Navigate (v) : điều hướng

Apparition (n) : ma quỷ, sự xuất hiện, tầm nhìn rõ, sự hiện rõ

Vindicative (v) : minh oan


Mar 10 – 2

English language scholars generally agree that the modern English language developed from several
sources: the Anglo-Saxon language, or Old English, spoken by the Germanic peoples who migrated
to the island of Britain in the fifth century; the Old Norse influences of the Vikings and the Danish
kings of England in the ninth and tenth centuries; the French influence of the Norman invaders in
the eleventh century; and the Latin influences of the earlier Roman inhabitants and the Catholic
Church. However, one mystery remains. When the Anglo-Saxons arrived in Britain, there were
numerous Celtic inhabitants dwelling alongside what remained of the Roman population. Why, then,
did the Anglo-Saxons, and thus the English, not absorb more of the Celtic languages? The English
language ultimately adopted very few Celtic words, so few in fact that scholars are at a loss to
explain the reason with any certainty. One thing is certain: the Celtic languages are in no way
related to Anglo-Saxon, indeed developing from an entirely different family of languages, so there is
no question that the Anglo-Saxons did not adopt Celtic words simply because they already had very
similar words of their own. So, what happened? Some scholars have suggested that the Anglo-
Saxons already had enough words of their own and thus did not need to borrow from the Celts,
even upon arriving in a new place. For instance, if the day-to-day elements of life in Britain were
similar enough to those in the Anglo-Saxon homeland, the Anglo-Saxons would not feel the need to
make use of foreign words to describe their new life. This theory, however, is inconsistent with
evidence that the Anglo-Saxons borrowed everyday words from other languages such as Old Norse
and French. Other scholars have suggested the theory that the Anglo-Saxons chose to avoid the
Celtic words because the Celts were essentially a conquered people - an explanation that is strongly
supported by the rapid disappearance of Celts from south and central England and their subsequent
movement north and west into what would become Cornwall, Wales, and Scotland.

Leading linguistic scholar David Crystal disagrees with this latter hypothesis, however. He points out
that among the Anglo-Saxons it was not uncommon to find children with Welsh names. The great
Christian poet Cdmon and Cdwalla, the king of Wessex in the seventh century, were both
noteworthy and highly respected Anglo-Saxons who bore Welsh names. From a purely practical
perspective, it is unlikely that Anglo-Saxon parents would bestow Celtic names on their children if
those names were closely associated with a despised language or a group of people deemed
inferior. As a modern example, during World War I people in England began changing their names
to avoid sounding too Germanic. Even the royal family, up to that point bearing the name Saxe-
Coburg-Gotha, changed the family name to Windsor due to the long connection of that name with a
specifically English history. Additionally, the respected Battenburg family in England, closely
connected to the monarchy, felt the need to change their name to Mountbatten, as it had a less
decidedly German connotation.

Perhaps more significantly, David Crystal raises the possibility that the word cross, steeped in
important religious meaning for many English speakers, came from a Celtic background. In Latin,
the word is crux, and the Scandinavians rendered it kross. But there is, on the whole, very little
linguistic influence on early English religious terminology from the Germanic languages or the
Germanic peoples, who were decidedly pagan upon their arrival to England. On the other hand, the
Irish Celts were enthusiastic and thorough in their missionary efforts to England and other parts of
Europe, and they rendered the Latin crux as cros in Old Irish and as croes in Welsh. It is highly
possible that the English word cross and the Old Norse word kross were influenced by the Irish
missionary work. It is unlikely that the mystery of the missing Celtic words will ever be solved
satisfactorily, but what little evidence remains suggests that the mystery can no longer be written
off as a case of a conquered people becoming linguistically obsolete

1. Which of the following best states the main idea of the passage?

A.Although linguistic scholars do not know why the English language has so few Celtic
words, it can no longer be assumed that the Anglo-Saxons avoided Celtic words in the
belief that the Celts were inferior.
B.The possible Celtic derivation of the word cross suggests that the Anglo-Saxons interacted more
closely with the Celts than was previously thought.

One example of evidence can’t be seen to be a best state the main idea of the passage
C.New evidence suggests that the traditional belief about the Anglo-Saxon, Old Norse, French, and
Latin influences on the English language is erroneous and misleading.
D.The actions taken by the English during World War I indicate strongly that their forebears
eradicated Celtic words for similar reasons.
E.The appearance of Welsh names among significant Anglo-Saxon figures indicates that of all the
Celtic peoples, the Welsh had the greatest linguistic impact on Anglo-Saxon daily life.

2. The use of the word "connotation" (Highlighted) most closely suggests which of the
following?

A. Clear relationship
B. Linguistic origin
C. Theoretical definition
D. Potential association
E. Emotional correlation

3. The discussion of the word cross in the passage is intended to show which of the
following?

A. Although they were previously ignored by scholars, it is clear that many important Celtic words
were indeed absorbed into the English language.
B. Scholars now realize that many Celtic words influenced Old Norse words and not the other way
around.
C. It is incorrect to assume that there was a very great influence on the English language from Celtic
words.
D. Linguistic scholar David Crystal believes that Celtic words make up an important part of the
English language.
E. The significance of the few Celtic words within the English language suggests a more
important influence than was previously thought.

4. The author provides examples of English behavior toward German last names during
World War I in order to do which of the following?
A. Prove definitely that human nature does not change
B. Undermine the theory of the Welsh influence on English names
C. Use a fairly recent event to provide context for a hypothesis
D. Show that the English changed names because they considered Germans inferior
E. Suggest that many of the so-called "English" names are really German
5. Which of the following best describes the author's attitude toward the theory that
there are few Celtic words in the English language because the Celts migrated and had
no contact with the Anglo-Saxons?
A. Self-righteous insistence
B. Scholarly disagreement
C. Patronizing disapproval
D. Justifiable concern
E. Vitriolic dissent

6. The primary purpose of the passage is to do which of the following?


A. Caution against making an historical judgment without considering further linguistic
evidence
B. Introduce a new theory and support it with linguistic evidence
C. Defend a scholarly position by citing leading authorities in the field
D. Dispute a long-held scholarly position by disproving the linguistic evidence in support of it
E. Compare several theories and argue in support of one of them

7. The passage suggests that the author would probably agree with which one of the
following?
A. There is less Latin and Old Norse influence on the English language that there is Celtic influence.
B. Although there seem to be few Celtic words within the English language, these words
suggest a significant linguistic role.
C. The possible Celtic derivation of the English word cross alone suggests that the English viewed
the Celts favorably.
D. The Anglo-Saxons did not adopt many Celtic words because they had enough everyday words in
their own language.
E. Because some of the Anglo-Saxons gave their children Welsh names, the Anglo-Saxon people
unquestionably had a high opinion of the Celts.

Despise (v) : coi thường

Missionary (adj) : nhà truyền giáo

Thorough (adj) : kỹ lưỡng, hết lòng

Caution (n) : thận trọng

Self-righteous (adj) : tự cho là đúng đắn

Insistence (n) : nhấn mạnh

Patronize (v) : bảo trợ, cư xử

Justifiable (adj) : chính đáng

Vitriolic (adj) : chỉ trích cay đắng, ác ý

Dissent (n) : bất đồng quan điểm

Connotation (n) : ý nghĩa, sự hàm súc


Mar 11 – 2

The debate over the environment crisis is not new: anxiety about industry’s impact on the
environment has existed for over a century. What is new is the extreme polarization of views.
Mounting evidence of humanity’s capacity to damage the environment irreversibly coupled with
suspicions that government, industry, and even science might be impotent to prevent environmental
destruction have provoked accusatory polemics on the part of environmentalists. In turn, these
polemics have elicited a corresponding backlash (a strong adverse reaction (as to a recent political
or social development)) from industry. The sad effect of this polarization is that it is now even more
difficult for industry than it was a hundred years ago to respond appropriately to impact analyses
that demand action.

Unlike today’s adversaries, earlier ecological reformers shared with advocates of industrial growth a
confidence in timely corrective action. George P. Marsh’s pioneering conservation tract Man and
Nature (1864) elicited wide acclaim without embittered denials. Man and Nature castigated Earth’s
despoilers for heedless greed, declaring that humanity “has brought the face of the Earth to a
desolation almost as complete as that of the Moon.” But no entrepreneur of industrialists sought to
refute Marsh’s accusations, to defend the gutting (to destroy the essential power or effectiveness of
“inflation gutting the economy” of forests or the slaughter of wildlife as economically essential, or to
dismiss his ecological warnings as hysterical. To the contrary, they generally agreed with him.

Why? Marsh and his followers took environmental improvement and economic progress as givens:
they disputed not the desirability of conquering nature but the bungling way in which the conquest
was carried out. Blame was not personalized (to make personal or individual; specifically: to mark as
the property of a particular person “personalized stationery”), Marsh denounced general greed
rather than particular entrepreneurs, and the media did not hound (underwrite: to pursue with or as
if with hounds) malefactors. Further, corrective measures seemed to entail no sacrifice, to demand
no draconian remedies. Self-interest underwrote (to guarantee financial support of) most prescribed
reforms. Marsh’s emphasis on future stewardship (the conducting, supervising, or managing of
something; especially: the careful and responsible management of something entrusted to one's
care “stewardship of our natural resources”) was then a widely accepted ideal (if not practice). His
ecological admonitions were in keeping with the Enlightenment premise that humanity’s mission was
to subdue and transform nature.

Not until the 1960s did a gloomier perspective gain popular ground. Fredric Clements’ equilibrium
model of ecology, developed in the 1930s, seemed consistent with mounting environmental
disasters. In this view, nature was most fruitful when least altered. Left undisturbed, flora and fauna
gradually attained maximum diversity and stability. Despoliation thwarted the culmination or
shortened the duration of this beneficent climax: technology did not improve nature but destroyed
it.
The equilibrium model became an ecological mystique: environmental interference was now taboo,
wilderness adored. Nature as unfinished fabric perfected by human ingenuity gave way to the image
nature debased and endangered by technology. In contrast to the Enlightenment vision of nature,
according to which rational managers construct an ever more improved environment, twentieth-
century reformers’ vision of nature calls for a reduction of human interference in order to restore
environmental stability.
1. Which one of the following most accurately states the main idea of the passage?

(A) Mounting evidence of humanity’s capacity to damage the environment should motivate action to
prevent further damage.
(B) The ecological mystique identified with Frederic Clements has become a religious conviction
among ecological reformers.
(C) George P. Marsh’s ideas about conservation and stewardship have heavily influenced the present
debate over the environment.
(D) The views of ecologists and industrial growth advocates concerning the
environment have only recently become polarized.
(E) General greed, rather than particular individuals or industries, should be blamed for the
environmental crisis.

2. The author refers to the equilibrium model of ecology as an “ecological mystique”


(Highlighted) most likely in order to do which one of the following?

(A) underscore the fervor with which twentieth-century reformers adhere to the
equilibrium model
(B) point out that the equilibrium model of ecology has recently been supported by empirical
scientific research
(C) express appreciation for how plants and animals attain maximum diversity and stability when left
alone
(D) indicate that the idea of twentieth-century ecological reformers are often so theoretical as to be
difficult to understand
(E) indicate how widespread support is for the equilibrium model of ecology in the scientific
community

3. Which one of the following practices is most clearly an application of Frederic


Clements’ equilibrium model of ecology?

(A) introducing a species into an environment to which it is not help control the spread of another
species that no longer has any natural predators
(B) developing incentives for industries to take corrective measures to protect the environment
(C) using scientific methods to increase the stability of plants and animals in areas where species
are in danger of becoming extinct
(D) using technology to develop plant and animal resources but balancing that development with
stringent restrictions on technology
(E) setting areas of land aside to be maintained as wilderness from which the use of
extraction of natural resources is prohibited

environmental interference was now taboo, wilderness adored

4. The passage suggests that George P. Marsh and today’s ecological reformers would
be most likely to agree with which one of the following statements?

(A) Regulating industries in order to protect the environment does not conflict with the self interest
of those industries.
(B) Solving the environmental crisis does not require drastic and costly remedies.
(C) Human despoliation of the Earth has caused widespread environmental damage.
(D) Environmental improvement and economic progress are equally important goals.
(E) Rather than blaming specific industries, general greed should be denounced as the cause of
environmental destruction.

5. The passage is primarily concerned with which one of the following?

(A) providing examples of possible solutions to a current crisis


(B) explaining how conflicting viewpoints in a current debate are equally valid
(C) determining which of two conflicting viewpoints in a current debate is more persuasive
(D) outlining the background and development of conflicting viewpoints in a current
debate
(E) demonstrating weaknesses in the arguments made by one side in a current debate

Extreme polarization (n) : phân cực cực đoan

Mount (v) : gắn kết

Impotent (adj) : bất lực

Provoke (v) : kích động

Accusatory (adj) : buộc tội

Polemic (n) : chính trị, sự tranh luận

Elicit (v) : gợi ra

Backlash (n) : phản ứng dữ dội

Adversary (n) : đối thủ

Ecological (adj) : sinh thái

Corrective (adj) : mong muốn sửa chữa

Embittered (adj) : nóng nảy

Denial (n) : cự tuyệt

Desolation (n) : sự tàn phá

Bungling (adj) : không khôn khéo

Malefactor (n) : người bất lương

Draconian (adj) : hà khắc

Stewardship (n) : sự quản lý

Admonition (n) : khuyên bảo, khuyên răn

Premise (n) : tiền đề, cơ sở

Enlightment (n) : khai sáng, giác ngộ

Subdue (v) : khuất phục


Fruitful (adj) : hiệu quả

Alter (v) : thay đổi

Despoliation (n) : sự chiếm đoạt

Culmination (n) : đỉnh cao

Mystique (n) : bí ẩn

Taboo (adj) : cấm kỵ

Extraction (n) : sự khai thác

Underscore (v) : nhấn mạnh, gạch dưới

Fervor (n) : sự nhiệt thành, hăng hái

Adhere (v) : tuân thủ


Mar 12 – 1

Flatfish, such as the flounder, are among the few vertebrates that lack approximate bilateral
symmetry (symmetry in which structures to the left and right of the body's midline are mirror
images). Most striking among the many asymmetries evident in an adult flatfish is eye' placement:
before maturity one eye migrates, so that in an adult flatfish both eyes are on the same side of the
head. While in most species with asymmetries virtually all adults share the same asymmetry,
members of the starry flounder species can be either left-eyed (both eyes on the left side of head)
or right-eyed. In the waters between the United States and Japan, the starry flounder populations
vary from about 50 percent left-eyed off the United States West Coast, through about 70 percent
left eyed halfway between the United States and Japan, to nearly 100 percent left-eyed off the
Japanese coast.

Biologists call this kind of gradual variation over·a certain geographic range a "cline" and interpret
dines as strong indications that the variation is adaptive, a response to environmental differences.
For the starry flounder this interpretation implies that a geometric difference (between fish that are
mirror images of one another) is adaptive, that left-eyedness in the Japanese starry flounder has
been selected for, which provokes a perplexing question: what is the selective advantage in having
both eyes on one side rather than on the other?

The ease with which a fish can reverse the effect of the sidedness of its eye asymmetry simply by
turning around has caused biologists to study internal anatomy, especially the optic nerves, for the
answer. In all flatfish, the optic nerves cross, so that the right optic nerve is joined to the brain's left
side and vice versa. This crossing introduces an asymmetry, as one optic nerve must cross above or
below the other. G. H. Parker reasoned that if, for example, a flatfish's left eye migrated when the
right optic nerve was on top, there would be a twisting of nerves, which might be mechanically
disadvantageous. For starry flounders, then, the left-eyed variety would be selected against, since in
a starry flounder the left optic nerve is uppermost.

The problem with the above explanation is that the Japanese starry flounder population is almost
exclusively left-eyed, and natural selection never promotes a purely less advantageous variation. As
other explanations proved equally untenable, biologists concluded that there is no important
adaptive difference between left-eyedness and right-eyedness and that the two characteristics are
genetically associated with some other adaptively significant characteristic. This situation is one
commonly encountered by evolutionary biologists, who must often decide whether a characteristic is
adaptive or selectively neutral. As for the left-eyed and right-eyed flatfish, their difference, however
striking, appears to be an evolutionary red herring.

1. According to the passage, starry flounder differ from most other species of flatfish in
that starry flounder

(A) are not basically bilaterally symmetric


(B) do not become asymmetric until adulthood
(C) do not all share the same asymmetry
(D) have both eyes on the same side of the head
(E) tend to cluster in only certain geographic regions
2. The author would be most likely to agree with which of the following statements
about left-eyedness and right-eyedness in the starry flounder?

I. They are adaptive variations by the starry flounder to environmental differences.


II. They do not seem to give obvious selective advantages to the starry flounder.
III. They occur in different proportions in different locations.

(A) I only
(B) II only
(C) I and III only
(D) II and III only
(E) I, II, and III

For the starry flounder this interpretation implies that a geometric difference (between
fish that are mirror images of one another) is adaptive

3. According to the passage, a possible disadvantage associated with eye migration in


flatfish is that the optic nerves can

(A) adhere to one another


(B) detach from the eyes
(C) cross
(D) stretch
(E) twist

4. Which of the following best describes the organization of the passage as a whole?

(A) A phenomenon is described and an interpretation presented and rejected.


(B) A generalization is made and supporting evidence is supplied and weighed.
(C) A contradiction is noted and a resolution is suggested and then modified.
(D) A series of observations is presented and explained in terms of the dominant theory.
(E) A hypothesis is introduced and corroborated in the light of new evidence.

5. The passage supplies information for answering which of the following questions?

(A) Why are Japanese starry flounder mostly left-eyed?


(B) Why should the eye-sidedness in starry flounder be considered selectively neutral?
(C) Why have biologists recently become interested in whether a characteristic is adaptive or
selectively neutral?
(D) How do the eyes in flatfish migrate?
(E) How did Parker make his discoveries about the anatomy of optic nerves in flatfish?

6. Which of the following is most clearly similar to a cline as it is described in the second
paragraph of the passage?

(A) A vegetable market in which the various items are grouped according to place of origin
(B) A wheat field in which different varieties of wheat are planted to yield a crop that will bring the
maximum profit
(C) A flower stall in which the various species of flowers are arranged according to their price
(D) A housing development in which the length of the front struts supporting the porch
of each house increases as houses are built up the hill
(E) A national park in which the ranger stations are placed so as to be inconspicuous, and yet as
easily accessible as possible

7. Which of the following phrases from the passage best expresses the author's
conclusion about the meaning of the difference between left -eyed and right-eyed
flatfish?

(A) "Most striking"


(B) "variation is adaptive"
(C) "mechanically disadvantageous"
(D) "adaptively significant"
(E) "evolutionary red herring"

Flatfish (n) : cá dẹt

Flounder (n) : cá bơn

Vertebrate (adj) : động vật có xương sống

Bilateral (adj) : song phương

Symmetry (n) : đối xứng

Mirror image (n) : ảnh phản chiếu

Most striking (adj) : nổi bật nhất

Asymmetry (n) : không đối xứng

Gradual (adj) : dần dần

Adaptive (adj) : thích nghi

Perplexing (adj) : bối rối

Optic (adj) : thị giác

Vice versa (adv) : ngược lại

Uppermost (adj) : trên cùng

Porch (n) : hiên nhà

Strut (n) : thanh chống


Mar 12 – 2

Economists use the phrase trade deficit to describe a condition when the dollar value of an
economy's imports exceed the dollar value of an economy's exports. Some political analysts believe
that trade deficits are a sign of weakness in an economy. They opine that trade deficits cause an
economy to lose jobs and drain the wealth of a nation. A closer analysis, however, reveals that trade
deficits might not be a cause of economic weakness.

One misconception is that trade deficits cause an increase in unemployment. The problem with this
analysis is that it confuses the loss of certain jobs with a decrease in the total number of jobs. For
example, when two nations drop their barriers to trade, certain industries in one country might incur
losses in the new more competitive market, resulting in a job loss. But each nation will move more
efficiently towards production in areas of comparative advantage, while the consumers of each
nation will be in a position to purchase goods at the most competitive prices. While it is beneficial
for a nation or region to have comparative advantages in profitable sectors of the economy, efforts
to offset the lack of comparative advantage by protecting an economy will only lead to inefficiently
high subsidies, whether from a government or in the form of higher prices paid by consumers.

Another misconception surrounding trade deficits is that they drain the wealth of a nation. However,
just because money flows from an economy doesn't mean that the economy becomes weaker.
Trade deficits are offset by capital account surpluses. The current account measures the various
flows of goods across countries while the capital account refers to a country's assets. Countries that
have current account surpluses will have capital account deficits, and vice versa. This capital account
surplus means that more assets in the form of cash, property, and other assets are flowing into the
home country. Having greater assets means that the country has more money to invest in business,
which in turn raises productivity.

1. The author's main point is that

a) although trade deficits entail current account deficits, the countervailing capital account surplus
creates substantial economic benefits
b) every nation should aim to incur large trade deficits because, contrary to common opinion, these
deficits actually improve the economy
c) the strength of a nation's economy depends on the size of its trade deficits with other nations
whose high subsidies create inflated prices for their consumers
d) the claim that a trade deficit creates an unhealthy economy relies on two common
misconceptions about a trade deficit's effects on jobs and a nation's wealth
e) because global economic systems are so complex, misconceptions about employment rate, a
nation's wealth, and especially trade deficits are common

2. Which of the following, if true, would strengthen the author's conclusion that greater
assets mean higher productivity?

a) The assets that a country builds up when the economy is healthy are likely to be invested abroad.
b) The more assets a country has, the more its citizens can save to purchase homes.
c) Each current account surplus is offset by an equal capital account deficit, and vice versa.
d) When assets flow into a capital account, they are owned by private enterprise.
e) A country's assets are typically invested in domestic businesses.
3. The misunderstanding addressed in the second sentence of paragraph 2 is most
similar to which one of the following situations?

a) Investors in a company sell its stock when sales of a certain product dip, even though
the company's quarterly profits rose.
b) A babysitter decides to work for a family with three children instead of a family with two children,
even though the pay is same.
c) A movie-goer avoids seeing a film that received only one poor review, although it was from a
prominent critic, and instead sees a film that got many bad reviews.
d) To ensure the kitchen will be able to handle orders, a restaurant decreases the prices of its
appetizers after it began serving food to guests sitting at the bar.
e) In spite of a construction company's new policy of shorter breaks for its workers, productivity
drops.

4. According to the passage, which of the following is true of trade deficits?

a) Trade deficits are exclusive to the United States and other developed countries.
b) Trade deficits occur when a country has a capital account surplus and a current
account deficit.
c) Trade deficits can lead to budget deficits.
d) Countries should work to prevent trade deficits.
e) Political analysts and economists are in agreement about the consequences of a trade deficit.

Confuse (v) : nhầm lẫn

Offset (v) : bù lại

Subsidy (n) : tiền trợ cấp

Drain (v) : chảy ra

Capital account (n) : tài khoản vốn – chỉ sự ghi lại các giao dịch liên quan tới sự vận động đi vào và đi ra
của luồng vốn quốc gia – theo đó, vốn đổ vào trong nước thông qua khoản đi vay, đầu tư từ nước ngoài
vào trong nước sẽ được ghi trong khoản mục Có (+) của bằng cân đối thanh toán, ngược lại, vốn rời khỏi
quốc gia khi cho vay, mua tài sản nước ngoài sẽ được hạch toán vào khoản mục Nợ (-)

Current account (n) : tài khoản vãng lai – là khoản mục trong cán cân thanh toán ghi chép lại các giao
dịch về hàng hóa và dịch vụ xuất khẩu nhập khẩu, thu nhập tiền lương lãi suất và lợi nhuận chuyển về
nước và chuyển giao, viện trợ đóng góp quà biếu
Mar 13 – 1

One of the most studied senses is vision. Scientists have carefully unraveled the connections of brain
cells in the visual system and have studied how they respond to light. so we have many clues about
how the brain takes visual (5) images apart. What is particularly elusive, however, is how the brain
puts the pieces back together, turning two-dimensional patterns of light on the retinas into our
perception of the visual world. In one case, however, the perception of color, we are beginning to
get a good idea of (10) how the brain operates.

Most people think that the balance of red, green, and blue light reflected from an object into the eye
determines the object's color. It is easy to demonstrate that this notion is not true, however, simply
by noting that objects remain the (15) same color in daylight, fluorescent light. and incandescent
light, each of which contains a mix of wavelengths of light very different from the others. Edwin
Land, inventor of the instant camera, has provided an explanation of this phe- nomenon in what he
calls the retinex theory, a term that (20) combines “retina” and “cortex” to suggest that both parts
of the visual system are involved in perceiving color.

Retinex theory proposes that the retina and the cortex cooperate to perform some complex
computations on the basis of light received from all areas within the visual (25) landscape. A
separate computation is carried out for each of three wavelengths of light that correspond to what
we normally think of as red, green, and blue; the wavelengths to which the three types of receptors
in the retina are most sensitive. According to the theory, the color we perceive at (30) a particular
location is determined by three numbers, com- puted by dividing the amount of light received from
that location at each wavelength by a weighted average of the amount of light at that wavelength
received from all parts of the field of vision. The weighted average gives more (35) weight to light
coming from close to the location in ques- tion than to that coming from far away. The three
numbers, coordinates in a color space of three dimensions, uniquely determine the color we see,
just as the three dimensions of physical space uniquely define the location of an object. (40) Land
has conducted a number of experiments showing that the numbers computed in this way correctly
predict what color an observer will see under a number of unusual light-ing conditions.

This remarkable theory suggests that our visual systems (45) evolved so that we see the colors of
objects as the same, regardless of the mix of wavelengths of light falling on our retinas.
Furthermore, this complex computation is carried out virtually instantaneously without our even
being aware of it.
1. According to the passage, the proportions of red, green, and blue light reflected by an
object cannot be the sole determinants of the object’s color because

A. color information about three wavelengths is not sufficient to produce the full spectrum of
possible colors.
B. the perceived color of an object changes with the ambient lighting of the object’s environment.
C. the image of an object is formed not by light coming from the object itself, but from other parts
of the field of vision.
D. variations in the mix of wavelengths illuminating an object do not affect its color.
E. this information varies according to the object’s proximity to the viewer.

Most people think that the balance of red, green, and blue light reflected from an object into the eye
determines the object's color. It is easy to demonstrate that this notion is not true,
however, simply by noting that objects remain the (15) same color in daylight,
fluorescent light. and incandescent light, each of which contains a mix of wavelengths
of light very different from the others.
We know that even the objects’re under different lights, they still remain the same color, but with
different mix of wavelengths ( tricky ). The question ask us about object color, not wavelengths

2. The passage suggests that Edwin Land created the name retinex (line 19) for his
optical theory in order to

A. distinguish his theory from rival theories of the retina’s operation.


B. indicate that both the retina and the cortex are involved in color perception.
C. differentiate between the actions of the retina and the actions of the cortex.
D. imply that properties ascribed to the retina actually belong to the cortex.
E. indicate that the cortex and the retina work together in perceiving location.

3. It can be inferred from the passage that if the balance of red, green, and blue light
entering the eye determined color, the apparent color of an object could be expected to
change if the object were moved

A. from a blue background to a bright yellow background.


B. from a sunlit room to a room with fluorescent lights.
C. to a different set of coordinates in physical space.
D. close enough to take up the viewer’s entire field of vision.
E. to a new area in the viewer’s visual landscape.

The information that daylight, fluorescent light, and incandescent light each contain a mix of
wavelengths of light very different from the others.

Land has conducted a number of experiments showing that the numbers computed in this way
correctly predict what color an observer will see under a number of unusual light-ing conditions.

Unravel (v) : làm sáng tỏ

Visual (adj) : thị giác


Apart (adv) : riêng lẻ

Elusive (adj) : khó nắm bắt, không thể tránh khỏi

Retina (n) : võng mạc

Fluorescent (adj) : huỳnh quang

Incandescent (adj) : sợi đốt

Wavelength (n) : bước sóng

Cortex (n) : vỏ não

Computation (n) : tính toán

Receptor (n) : thụ thể

Uniquely (adv) : độc đáo

Instantaneously (adv) : tức thời

Iluminate (v) : chiếu sáng

Sole (adj) : duy nhất

Determinant (n) : yếu tố quyết định

Proximity (n) : trạng thái ở gần


Mar 13 – 2

In the early 1970s, a new system of organizing the growing acquisitions of corporations was
introduced. Called the growth/share matrix, this tool seemed to operate on the most logical of
assumptions: Corporations should sell off (5) their losing divisions as determined by the divisions’
positions on the matrix, and retain and increase those divisions that the matrix considered
successful.

According to the Harvard Business Review, the Boston Consulting Group (BCG) introduced the
matrix in response (10) to corporations that had entered the heyday of acquisition and
diversification of the 1960s and early 1970s, and subsequently faltered with the energy crisis of
1973. The matrix worked by ordering each division according to its position within its industry
overall. Thus, managers had a tool for (15) understanding the relative success of those businesses
with whose fields they were unfamiliar. Enthusiasm over the matrix and its simplicity and apparent
logic obscured one of the problems inherent in the initial situation: the wide range of acquisitions
these corporations had purchased.

(20) The matrix evaluated the performance of the divisions in terms of their competitiveness within
their fields and their cash value, but failed to analyze the relationships among di- visions that made
up a corporation's holdings. For instance, a corporation that owned a newspaper chain and a paper
(25) mill would be advised to consider more than just the relation of the paper mill's performance
to that of other mills. Beyond this, the matrix underestimated the amount of debt a corporation
could safely assume. And finally, the matrix was unable to provide information regarding the
corporation's (30) ability to manage even those successes identified by the matrix.

Simply having a number of separately competitively successful companies does not ensure that
companies will be able to support their owners without proper management (35) and
understanding. Despite the clarity and effectiveness of the growth/share matrix as a tool for
determining divisions’ performance, it could not long compensate for the difficulties present in the
initial situation it sought to alleviate: that of corporations believing that their particular management
(40) styles would function effectively for any type of smaller business they might acquire.

1. Which of the following best describes the main idea of the passage?

A. The growth/share matrix was a failure as an acquisition research tool, and hurt many
corporations.
B. The growth/share matrix, though eagerly embraced at first, could not completely
solve the problems it sought to address.
C. Corporations that acquire holdings that are both overly diversified and unrelated will not succeed
in the business world.
D. Management style should be of primary concern when a corporation is deciding which divisions to
retain and which to divest.
E. No one corporate tool can ever compensate for a lack of management skills and well-thought-out
acquisition planning.
2. According to the passage, all of the following were problems associated with
corporations’ reliance on the growth/share matrix EXCEPT

A. the overestimation by the matrix of the negative effect that debt might have on a corporation.
B. not considering divisions’ relation to one another within each corporation’s holdings.
C. the failure of the matrix to compensate for the lack of knowledge the corporations had about
their own holdings.
D. the matrix’s inability to correctly order divisions within their overall industries.
E. the matrix’s lack of focus on a corporation’s ability to manage its acquisitions.

Beyond this, the matrix underestimated the amount of debt a corporation could safely assume

 Ma trận đánh giá quá thấp khả năng chịu đựng gánh nặng nợ của tập đoàn

3. It can be inferred from the passage that the author suggests which of the following
concerning some corporations during the energy crisis of 1973?

A. The troubles of these corporations were related to problems of conforming their


management styles to their new holdings.
B. Lack of fuel led many companies to have trouble powering their acquisitions.
C. Corporations’ reliance on the growth/share matrix led them to mismanage their holdings.
D. Overenthusiastic buying of smaller companies left many corporations unwieldy and difficult to
manage.
E. Too little diversification forced companies to find a tool to estimate the relative success of
companies with whose fields they were unfamiliar.

Subsequently (adv) : sau đó

Falter (v) : lưỡng lự

Obscure (v) : bị che khuất

Inherent (adj) : vốn có

mill (n) : xưởng chế tạo

alleviate (v) : giảm bớt

comform (v) : phù hợp


Mar 14 – 1

Congressman Hastings has proposed that Congress should abolish the Electoral College system for
electing the president and replace it with a system of direct popular election. The Electoral College
system is flawed, he argues, because it runs directly counter to the democratic principle that every
citizen’s vote should count equally.

Because of the winner-take-all system in which the candidate who receives the most popular votes
in a state receives all of that state’s electoral votes, the citizens who voted for the losing candidate
are effectively disenfranchised from the national election, even if their candidate lost the state by
only a handful of votes. Moreover, because each state’s number of electors is the same as its
number of members of Congress, the citizens of small states get a disproportionately larger vote
than citizens of more populous states. In the 1988 election, for example, the combined voting-age
population of the six least populous states— Alaska, Delaware, North Dakota, South Dakota,
Vermont, and Wyoming—was 3,119,000. These six states held 21 electoral votes among them.
Florida, with a voting-age population of 9,614,000, also had 21 electoral votes. Because of inequities
of this nature, there have been four presidential elections in which the candidate who won the
Electoral College actually lost the popular vote: 1824, 1876, 1888, and 2000.

Congressman Markham has argued that Hastings’s proposed changes are unnecessary and even
dangerous. First of all, he argues, the Electoral College system, whatever its flaws, has resulted in a
stable democratic government for more than 200 years, which shows that it is doing something
right. Second, the winner-take-all system helps create decisive majorities in the Electoral College,
thereby reducing the problem of disputed elections that we might see in the event of direct popular
elections. Third, the current system of allocating electors helps protect the interests of small states,
which would be largely neglected in favor of large states if the Electoral College were based entirely
on population. Protecting these states’ rights is essential to upholding the principle of federalism (in
which the states and the federal government maintain distinct powers).

When the Electoral College system was first formalized by the Twelfth Amendment in 1804, a direct
popular vote would have been impossible to implement, and the Electoral College was probably the
best way to approximate the will of the people. Advances in technology and communication,
however, now mean that a direct popular vote would be as simple, if not simpler, to administer than
the current Electoral College system. Alternative ways to reform the system would be to do away
with the winner take-all system of state electors, to base the numbers of electors strictly on state
populations, or to have a direct popular election but to weight the votes from different states
differently in order to preserve the influence of small states.

1. The passage is primarily concerned with which of the following?

A. Evaluating the merits of the Electoral College system as a means of protecting federalism
B. Examining the impact of the Twelfth Amendment on the history of the American presidency
C. Disputing the validity of the American democratic process
D. Presenting arguments regarding the best way to elect the president of the United
States
E. Comparing arguments regarding the nature of democratic processes
2. Which of the following best describes the structure of the passage?

A. It presents a critique of an institution, then provides a defense of that institution, and


then offers possible compromises between the two positions.
B. It presents an argument, lists problems with that argument, and then ultimately refutes the
argument.
C. It discusses both sides of a controversial topic and then chooses the side with the stronger
arguments.
D. It analyzes flaws in a traditional institution and then provides a series of steps to remedy the
flaws.
E. It presents two competing viewpoints and then shows the logical errors in both positions.

3. From the information provided in the paragraph, what can be inferred about the
election of 1892?

A. Its result was disputed on the grounds that smaller states had a disproportionate influence on the
outcome.
B. The winner of the election received more popular votes than his opponent.
C. The winner-take-all system of state electoral votes led to a decisive majority for the winner in the
Electoral College.
D. Reforming the Electoral College system was an issue of contention in this election.
E. This election led to the formalization of the Electoral College by the Twelfth Amendment.

It’s a trick question , the passage never mention about the result of the election of 1892 so we can’t
infer anything but there is one sure thing that to win the election, one has to have more number of
votes than his opponent

4. Hastings’s response to Markham’s argument that the history of the American


government “shows that it [the Electoral College] is doing something right” (Highlighted)
would most likely be which of the following?

A. Under the current system, each voter in Alaska has proportionately three times as much voting
power as each candidate in Florida.
B. We do not know whether or not the American government would have been equally stable had
the president been elected by a direct popular election since the beginning.
C. If the candidate who lost the popular election won the presidency four times in 200
years, there is something wrong with the system.
D. Maintaining a strong federal system is less important than upholding the principle that each vote
should count equally.
E. A process that maintained the Electoral College but removed the winner-take-all system would
substantially reduce the disenfranchisement that occurs under the current system.
5. According to the passage’s presentation of Markham’s sentiments regarding
federalism, which of the following systems for electing the president would be most
objectionable to him?

A. An Electoral College system identical to the one currently in use


B. An Electoral College in which each state’s number of electors was based strictly on population,
but the winner-take-all system was maintained
C. An Electoral College in which each state’s number of electors was based strictly on that state’s
number of members of Congress, but in which the winner-take-all system was replaced by a divided
electoral vote proportional to the state’s popular vote
D. A direct popular election in which the votes from citizens of smaller states were given more
weight than citizens from larger states
E. A direct popular election

6. Which of the following examples from international politics, if true, would give
Markham the most support in his argument against Hastings?

A. A nation in Africa that modeled its government on the American governmental system after
achieving independence from a European colonial power recently entered into a civil war that has
effectively ended any true democratic processes.
B. The ancient city-state of Athens had a form of direct popular election in the fifth century B.C.E.,
but this government fell as a result of the Peloponnesian War that Athens fought against Sparta.
C. A South American nation that deposed its long-standing military dictatorship and instituted in its
place a democratic government with a president elected through a direct popular election has
experienced both economic growth and improved relations with the international community.
D. A nation in Central Europe that recently changed its government from a longstanding
parliamentary monarchy to a government led by a popularly elected premier was
recently thrown into chaos when the popularly elected premier declared a dictatorship.
E. The system of proportional power sharing by the members of the European Union has resulted in
a number of thorny disputes between member states that will probably grow more severe as new
nations from Eastern Europe enter the EU.

7. According to the information given in the passage, which of the following statements
about Florida and South Dakota is most accurate?

A. Florida is a larger state in area than South Dakota.


B. South Dakota has a larger population than Florida.
C. The ratio of members of Congress to electors in the Electoral College is lower for the state of
Florida than it is for South Dakota.

We can’t assume that because we don’t have specific number of elector in south dakota

D. South Dakota has more members of Congress per voting-age citizen than Florida
does.
E. A higher percentage of the voting-age population in South Dakota exercises its constitutional right
to vote than is observed among the voting-age population of Florida.

Member of congress per voting age citizen

Congressman (n) : nghị sĩ quốc hội

Abolish (v) : bãi bỏ

Electoral college (n) : cử tri đoàn

Disenfranchise (v) : tước quyền

Elector (n) : cử tri, người đi bầu

Populous (adj) : đông dân

Voting-age population (n) : dân số trong độ tuổi bỏ phiếu

Inequity (n) : bất bình đẳng

Decisive (adj) : quyết đoán

Majority (n) : đa số

Dispute (v) : tranh chấp

Neglect (v) : bỏ mặc

Uphold (v) : duy trì

Federalism (n) : liên bang

Amendment (n) : sửa đổi

Administer (v) : quản trị, điều hành

Compromise (v) : thỏa hiệp


Mar 14 – 2

To understand John Rawls's theory of justice, one first needs to grasp what he was reacting against.
The dominant approach in pre-Rawls political philosophy was utilitarianism, which emphasized
maximizing the fulfillment of people's preferences. At first sight, utilitarianism seems plausible-what
else should we do but try to achieve the most satisfaction possible for the greatest number of
people?-but the theory has some odd consequences. Suppose executing an innocent person will
appease a mob, and that doing so will therefore increase total satisfaction. Incredibly, a utilitarian
would have to endorse the execution. Rawls accordingly complains that, in the utilitarian view, there
is no reason "why the violation of the liberty of a few might not be made right by the greater good
shared by many."

If we reject utilitarianism and its view about the aim of the good life, how can we know what justice
requires? Rawls offers an ingenious answer. He asserts that even if people do not agree on the aim
of the good life, they can accept a fair procedure for settling what the principles of justice should be.
This is key to Rawls's theory: Whatever arises from a fair procedure is just.

But what is a fair procedure? Rawls again has a clever approach, beginning with his famous veil of
ignorance. Suppose five children have to divide a cake among themselves. One child cuts the cake
but does not know who will get which shares. The child is likely to divide the cake into equal shares
to avoid the possibility of receiving the smallest share, an arrangement that the others will also
admit to be fair. By denying the child information that would bias the result, a fair outcome can be
achieved.

Rawls generalizes the point of this example of the veil of ignorance. His thought experiment features
a situation, which he calls the original position, in which people are self-interested but do not know
their own station in life, abilities, tastes, or even gender. Under the limits of this ignorance,
individuals motivated by self-interest endeavor to arrive at a solution in which they will not lose,
because nobody loses. The result will be a just arrangement.

Rawls thinks that people, regardless of their plan of life, want certain "primary goods." These
include rights and liberties, powers and opportunities, and income and wealth. Without these
primary goods, people cannot accomplish their goals, whatever they may be. Hence, any individual
in the original position will agree that everyone should get at least a minimum amount of these
primary goods. Unfortunately, this is an inherently redistributionist idea, since the primary goods are
not natural properties of human beings. If someone lacks a primary good, it must be provided, at
the expense of others if necessary.

1) The author's primary purpose in the passage is to

(A) show why a once-dominant theory was abandoned


(B) describe the novel way in which a theory addresses a problem
(C) sketch the historical development of a celebrated theory
(D) debate the pros and cons of a complex theory
(E) argue for the truth of a controversial theory.
2) According to the passage, Rawls uses which one of the following devices to explain
his theory?

(A) a thought experiment


(B) a process of elimination
(C) an empirical study of social institutions
(D) a deduction from a few basic principles
(E) a consideration of the meaning of words

3) With which one of the following statements would both Rawls and the author of the
passage be most likely to agree?

(A) There are situations in which it is permissible to treat the fulfillment of one person's
preferences as more important than the fulfillment of the majority's preferences.
(B) Unless individuals set aside their own self-interest, they cannot make fair judgments about the
distribution of goods.
(C) If an individual lacks a good, society must sometimes provide that good, even if this means
taking it from others.
(D) Most people agree about which of the primary goods is the most valuable.
(E) It is fair to sacrifice the individual's interests if doing so will maximize the satisfaction of the
majority.

Option C feels like a good choice because of the last paragraph, but the author subtly indicates that
they do not agree with this statement. Their use of the word "unfortunately" in the below quote
from the last paragraph indicates that the author would prefer not to make society pay for the
goods of other individuals, while Rawls indicates that he agrees with that concept.

4) The author's stance toward Rawls's theory is most accurately described as one of

(A) scholarly neutrality with respect both to its objectives and its development
(B) disdain for its pretensions camouflaged by declarations of respect for its author
(C) sympathy with its recommendations tempered with skepticism about its cogency
(D) enthusiasm for its aims mingled with doubts about its practicality
(E) admiration for its ingenuity coupled with misgivings about some of its implications

"Rawls offers an ingenious answer."

"Rawls again has a clever approach, beginning with his famous veil of ignorance."

"Rawls thinks that people, regardless of their plan of life, want certain "primary
goods." .... Unfortunately, this is an inherently redistributionist idea, since the primary goods are
not natural properties of human beings."

The first two are praising his intelligence, but the use of the word "Unfortunately" in the last
sentence indicates that the author is unhappy about at least one implication of Rawl's theory

Theory of justice (n) : lý thuyết công bằng

Grasp (v) : hiểu biết


Utilitarianism (n) : chủ nghĩa thực dụng

Preference (n) : quyền ưu tiên, sở thích

Fulfillment (n) : làm trọn vẹn, thực hiện

Plausible (adj) : có lý

Appease (v) : xoa dịu

Mob (n) : đám đông

Incredibly (adv) : đáng kinh ngạc

Endorse (v) : thừa nhận

Ingenious (adj) : khéo léo

Veil of ignorance (n) : bức màn vô minh

Ignorance (n) : thiếu hiểu biết

Endeavor (v) : nỗ lực

Ingenuity (n) : thật thà, chất phác

Misgiving (n) : sự đa nghi, nghi ngờ

Mingle (v) : trộn lẫn

Cogency (n) : sức thuyết phục

Stance (n) : lập trường


Mar 15 – 1

Justice is the first virtue of social institutions, as truth is of systems of thought. A theory, however
elegant and economical, must be rejected or revised if it is untrue. Likewise, laws and institutions,
no matter how efficient (5) and well arranged, must be reformed or abolished if they are unjust.
Each person possesses inviolability, founded on justice, which even the welfare of society as a whole
cannot override. For this reason, justice denies that the loss of freedom for some is made right by a
greater good shared (10) by others. It does not allow that the sacrifices imposed on a few are
outweighed by the larger sum of advantages enjoyed by many.

Therefore, in a just society the liberties of equal citizenship are taken as settled; the rights secured
by justice are not (15) subject to political bargaining or to the calculus of social interests. The only
thing that permits us to acquiesce to an erroneous theory is the lack of a better one; analogously,
an injustice is tolerable only when it is necessary to avoid an even greater injustice. As primary
virtues of human activities (20) , truth and justice are uncompromising.

These propositions seem to express our intuitive conviction of the primacy of justice. One might
inquire whether these contentions or others similar to them are sound, and if so, how they can be
accounted for. To this end, it is necessary (25) to work out a theory of justice in light of which
these assertions can be interpreted and assessed.

Begin by considering the role of principles of justice. Assume that a society is a more or less self-
sufficient association of persons who, in their relations to one another, (30) recognize certain rules
of conduct as binding and who, for the most part, act in accordance with them. Suppose further that
these rules specify a system of cooperation designed to advance the good of those taking part in it.
Then, although a society is a cooperative venture for mutual (35) advancement, it is typically
marked by a conflict as well as by an identity of interests. There is an identity of interests since
social cooperation makes possible a better life for all than any would have if each were to live solely
by his own efforts. There is a conflict of interests since persons are not (40) indifferent to how the
greater benefits produced by their collaborations are distributed, for in order to pursue their ends
they each prefer a larger to a lesser share.

A set of principles is required for choosing among the various social arrangements that determine
this division of (45) advantages and for underwriting an agreement on the proper distributive
shares. These principles are the principles of social justice: they provide a way for assigning rights
and duties in the basic institutions of society and they define the appropriate distribution of the
benefits and burdens of (50) social cooperation.

1. It can be inferred from the passage that the author believes that one can permit the
use of an imperfect theory of justice

A. under no circumstances, for to do so would violate the idea of justice.


B. only if it would allow for the greater good of the society.
C. only if a more just theory cannot be found.
D. whenever it is agreed upon by a majority of the society.
E. whenever it would effectively prevent certain injustices.
2. According to the passage, a society is characterized by all of the following EXCEPT

A. self-sufficient individuals.

A society may be self-sufficient, but the individuals within it are not.


B. a collaborative effort for common progress.
C. an adherence to certain rules of behavior.

D. a conflict of interest among individuals.


E. a common interest in social cooperation.

Begin by considering the role of principles of justice. Assume that a society is a more or less self-
sufficient association of persons who, in their relations to one another, (30) recognize certain
rules of conduct as binding and who, for the most part, act in accordance with them. Suppose
further that these rules specify a system of cooperation designed to advance the good of
those taking part in it. Then, although a society is a cooperative venture for mutual
(35) advancement, it is typically marked by a conflict as well as by an identity of interests. There
is an identity of interests since social cooperation makes possible a better life for all than any
would have if each were to live solely by his own efforts. There is a conflict of interests since
persons are not (40) indifferent to how the greater benefits produced by their collaborations are
distributed, for in order to pursue their ends they each prefer a larger to a lesser share.

3. The author implies that, in order to ascertain whether convictions concerning the
inherent nature of justice are valid, one must

A. determine the context in which they can be assessed.


B. shed light on the role of the principles of justice.
C. meet all of the requirements of a just society.
D. account for the principle of justice in society.
E. establish a theory of justice with which to evaluate them.

To this end, it is necessary (25) to work out a theory of justice in light of which these assertions
can be interpreted and assessed.

Social institution (n) : tổ chức xã hội

Elegant (adj) : thanh lịch

Arrange (v) : sắp xếp, bố trí, đồng ý

Unjust (adj) : bất công

Acquiesce (v) : thông qua

Erroneous (adj) : sai lầm

Analogously (adv) : tương tự

Tolerable (adj) : có thể chịu được

Proposition (n) : mệnh đề


Intuitive (adj) : trực quan

Contention (n) : sự thảo luận, ganh đua, cạnh tranh, chủ trương

Sound (adj) : đúng đắn, có cơ sở, có căn cứ, logic

Assess (v) : đánh giá

Assertion (n) : khẳng định

Venture (n) : liên doanh

Underwrite (v) : bảo hiểm, bảo lãnh

Ascertain (v) : chắc chắn

Inherent nature of justice (n) : bản chất vốn có của công lý


Mar 15 – 2

Currently the focus of research into artificial intelligence (AI) is on relatively limited applications,
including guiding airplanes and missiles, understanding language, detecting credit card fraud, and
diagnosing medical conditions from electrocardiograms. While these avenues certainly contain
promise for commercial and industrial interests, the populace at large typically is more interested in
a slightly less consequential application of AI. In 1997, Deep Blue, a chess playing computer
program, defeated grand master Gary Kasparov in a tournament, marking the first time a computer
was able to best a human world champion.

But these more “frivolous” applications of AI are actually valuable indicators of the state of
technology. Computer engineers are constantly reviving and reinventing the programming
algorithms in an effort to make computers think more like people do. And the exponential increases
in processing and storage technology are allowing artificial intelligence researchers to greatly
increase the power of AI programs while simultaneously reducing the size and computing needs of
the machines. For example, the Deep Blue program required 256 specialized processors to analyze
the millions and millions of combinations of moves. Each of these processors was about one
hundred times faster than a standard home computer, which means Deep Blue was about 25,600
times faster than a 1997 personal computer.

Only five years later, in 2002, AI engineers unleashed a new chess playing program, Deep Fritz.
Deep Fritz played Vladimir Kramnik, the highest ranked chess player in the world, to a draw, but this
result was anything but a failure to the AI community. Whereas Deep Blue required 256 processors
to achieve its victory, Deep Fritz had a mere eight. Deep Blue could analyze 200 million moves per
second, but Deep Fritz could deal with only 2.5 million. Yet Deep Fritz’s chess playing abilities more
closely resemble those of a person. Deep Blue was able to defeat Kasparov by brute strength alone,
calculating millions of possible moves and counter moves. Deep Fritz played Kramnik to a draw by
using advanced pattern recognition skills, which allowed it to be competitive despite considering
fewer move combinations.

1. The main idea of the passage is that

A. computer chess programs will soon be able to defeat any human challenger
B. computer chess programs are now being designed to be more efficient and perform fewer
calculations
C. certain applications of AI technology provide insight into advances in the field
D. the public pays too much attention to frivolous applications of AI technology
E. the Deep Fritz chess program is more successful than the Deep Blue program
2. According to the passage, the AI community did not view Deep Fritz’s draw as a
failure because

A. Kramnik was a higher ranked chess player than Kasporov and was thus harder to defeat
B. researchers did not expect Deep Fritz to win because it only had eight processors

The passage doesn't state or imply that engineers didn't expect deep fritz to win as it has only 8
processors. This is a comparision drawn by author and as per the passage it is used only for
comparison between deep Blue and deep Fritz.
C. Deep Fritz’s performance suggested that programmers were approaching one of their
goals
D. Deep Blue’s victory had already proven that a computer could beat a grand champion in chess
E. human players had five years to adapt to competing against chess programs

3. The author implies that human chess players

A. will not lose to Deep Fritz


B. will soon be unable to defeat most computer chess programs
C. are not able to analyze possible move combinations when playingchess
D. rely on pattern recognition skills when playing chess
E. could potentially beat Deep Fritz but not Deep Blue

Yet Deep Fritz’s chess playing abilities more closely resemble those of a person. Deep
Blue was able to defeat Kasparov by brute strength alone, calculating millions of possible moves and
counter moves. Deep Fritz played Kramnik to a draw by using advanced pattern
recognition skills, which allowed it to be competitive despite considering fewer move
combinations.

4. The primary purpose of the passage is to

A. contrast the abilities of two computer programs


B. encourage researchers to pursue more serious inquiries
C. detail the state of research in a particular field
D. compare human chess players to computer chess programs
E. challenge a long-held assumption about AI

This results was anything but a failure (sentence) : kết quả này là bất cứ điều gì ngoại trừ sự
thất bại
Mar 16 – 1

Dear Sirs,

Given all the coverage that the emergence of hybrid cars has received in your pages in recent
months, your readers may be interested to learn that gasoline-electric hybrids are not a new
phenomenon at all, but rather the latest incarnation of an idea that has been kicking around for over
a century. Indeed, the hybrid car has been around almost as long as the automobile itself.

At the turn of the twentieth century, as the automotive age dawned, three power-generating
technologies competed for dominance: steam, gasoline, and electricity. In the year 1900, steam was
well known as the power source of the industrial revolution, and electricity was widely regarded as
the power source of the future, so it was not at all obvious that internal combustion engines burning
a fractional distillate of crude petroleum would have any particular edge in this race for the
powertrains of America. Indeed, when engineer H. Piper filed the first patent application for a
gasolineelectric hybrid motor in 1905, his intention was to use the gas to give a little kick to his
perfectly serviceable electric engine. His goal: an engine that could accelerate from 0 to 25 miles per
hour in 10 seconds.

Piper achieved his goal. Electric and hybridelectric engines powered more than 35,000 vehicles sold
in 1912. These cars were perfectly adequate for the time, but over the following decade they mostly
disappeared from the market, through no fault of their own. The cause of their decline was the
spectacular improvements in the cost and performance of gasoline-powered cars. An onslaught of
fast and cheap internal combustion cars from Ford, General Motors, and Buick essentially buried the
electric and electric-hybrid motors by the 1920s.

Continuing performance improvements in internal combustion engines and inexpensive gas pretty
much kept hybrids buried until the oil crises of 1973 and 1979 gave Americans a reason to start
thinking about fuel efficiency. Engineers had the motivation to think about fuel-efficient hybrids, but
they still lacked the means to make hybrids economically competitive with gas-powered cars,
because the performance of gas-electric engines lagged far behind that of gas-powered engines in
acceleration, top speed, and cruising range. Dramatic improvements in electronics and computer
technology during the 1990s, however, finally made the hybrid a reality. Advances in battery
performance and, most importantly, computerguided electric power transfer created a car that could
drive like a regular car, but do so on half the tank of gas. As another century dawns, perhaps we are
entering into a new automotive age.

1. Based on the tone and content of the passage, the author of the passage is most
likely which of the following?
A. An automotive engineer writing to his company management
B. An enthusiast of automotive history writing to the editors of a car magazine
C. A college engineering student writing to a car manufacturer
D. A history professor writing to a television producer of historical documentaries
E. An environmental activist writing to the editors of a newspaper

Given all the coverage that the emergence of hybrid cars has received in your pages in recent
months, your readers may be interested to learn....
2. The purpose of the article could best be summarized as which of the following?
A. To correct a mistaken impression about the performance of gasoline-electric hybrid cars
B. To educate readers about the economic and technological potential of hybrid cars
C. To refute a factually inaccurate statement made previously in the publication regarding the
history of hybrid cars
D. To acquaint readers with the history of gasoline-electric hybrid cars
E. To educate readers about technological innovations at the dawn of the automotive age

3. According to the passage, electric and hybrid cars failed to capture the American
automotive market in the early twentieth century because of what factor?
A. The improvement in cost and performance of gasoline-powered cars
B. The superior fuel efficiency of hybrid cars
C. The substandard performance of steampowered cars
D. Consumer fear of being electrocuted by gasoline-electric hybrids
E. Government subsidies for gasoline- and coal-powered cars

4. According to the information given in the passage, which of the following best
characterizes the different motivations behind the earliest experiments with gasoline-
electric hybrids and the experiments going on in modern times?
A. The earliest experiments with hybrids sought to improve the fuel efficiency of electric engines,
while modern experiments seek to improve the performance of gas-burning engines.
B. The earliest experiments with hybrids sought to improve the fuel efficiency of gas-burning
engines, while modern experiments seek to improve the performance of electric engines.
C. Modern experiments with hybrids seek to improve the fuel efficiency of gas-burning
engines, while the earliest experiments sought to improve the performance of electric
engines.
D. Modern experiments with hybrids seek to improve the cruising range of gaspowered cars, while
earlier experiments sought to improve the handling and safety of electric cars.
E. The earliest experiments with hybrids sought to combine the power of steam with the efficiency
of electricity, while modern experiments seek to combine the efficiency of electricity with the power
of gas.

Engineers had the motivation to think about fuel-efficient hybrids, but they still lacked the means to
make hybrids economically competitive with gas-powered cars, because the performance of gas-
electric engines lagged far behind that of gas-powered engines in acceleration, top speed,
and cruising range.

Dramatic improvements in electronics and computer technology during the 1990s, however, finally
made the hybrid a reality. Advances in battery performance and, most importantly, computerguided
electric power transfer created a car that could drive like a regular car, but do so on half the tank
of gas. As another century dawns, perhaps we are entering into a new automotive age.

It means the gas burning engines work more efficiency

5. The passage lists which of the following as a reason for the resurgence of interest in
the gasoline-electric hybrid engine?
A. The onslaught of fast and inexpensive internal combustion cars from Ford, General Motors, and
Buick
B. Advances in battery performance and electricity transfer
C. The Iranian hostage crisis of 1979
D. Dramatic improvements in computer technology
E. The oil crisis of 1973

6. Which of the following examples of business and technology bears the most similarity
to the history of the hybrid car, as presented in the passage?
A. American aerospace companies in the 1960s created working prototypes of supersonic passenger
aircraft that could complete intercontinental flights in half the time of conventional aircraft, but these
projects were canceled because of concerns that the high-altitude craft posed too great a threat to
the integrity of the ozone layer.
B. Although oil companies first attempted deep-sea drilling in the Gulf of Mexico in the
1930s, these deep-sea projects could not compete with land-based drilling projects until
advances in drilling technology and the rising price of oil made deep-sea drilling
economically viable in the late twentieth century.
C. Automakers in the 1980s, after concluding that the average driver could not be relied on to use
seat belts consistently, chose to adopt airbags as a standard safety feature.
D. Lighter-than-air craft, such as Zeppelins, made up a substantial part of total air traffic in the early
twentieth century, but they rapidly fell out of favor after airplanes proved to be a faster and safer
form of transportation.
E. Although railroads carried more than 90 percent of all land-based commercial cargo in the United
States in 1910, by 1980 railroads had been surpassed by trucks in total cargo carried, because of
the greater speed and flexibility offered by the heavy truck as compared with the railroad.

7. Based on the information given in the passage, which of the following can be inferred
about an electric-gasoline hybrid car in 1950?

A. It would have been difficult to find a power supply for such a car.
B. Its acceleration would have been roughly comparable to that of a gasoline-burning car that cost
less than half as much as the hybrid car.
C. It would have been viewed as “the car of the future.”
D. Its performance would have been limited by contemporary battery technology.
E. Had it been produced, it would have sold more than 35,000 units per year.

Acquaint (v) : làm cho ai đó hiểu rõ vấn đề

Coverage (n) : phủ sóng

Emergence (n) : sự xuất hiện

Incarnation (n) : sự hiện thân

Internal combustion (n) : đốt trong

Distillate (n) : chưng cất

Onslaught (n) : tấn công dữ dội

Lag (v) : tụt hậu

Acceleration (n) : sự tăng tốc


Mar 16 – 2

What is the biggest lesson from the Great Depression? In my view, it is that monetary policy and the
financial sector play a crucial role in economic development. One important component of the
monetary policy is the financial market, more specifically the banking sector.

Why are financial markets and the banking sector so important? Banks fulfill a very important role in
the economy by matching borrowers and lenders. When we deposit $100 in a bank, the bank keeps,
at most, two to three dollars in its vaults (some of this is actually kept with the central bank), the
remaining $98 or so are lent to a borrower.

Most businesses require loans for their normal operations. When the banking sector does not work
properly, businesses cannot get loans and they have to curtail their production and lay off workers.
As they curtail production, they demand fewer products from their suppliers and therefore their
suppliers have to reduce their output and fire workers. If manufacturers cannot sell their goods
because the firm downstream does not need as many products as before, they cannot generate
enough revenue to repay their earlier loans. Businesses go bankrupt and banks experience further
problems as their balance sheet deteriorates due to non performing loans. At this point, banks want
to lend even less because of the uncertainty generated from bankruptcies. As they lend less, the
vicious circle continues – with producers cutting production and firing workers. On top of this,
depositors start worrying about their deposits because the non-performing loans have made some
banks go belly up – your bank has lent out your money to borrowers who cannot return it.
Depositors start withdrawing their cash and banks have even fewer possibilities for lending as they
have to hoard cash in case there is a run on the bank. If the financial sector does not work, the real
economy can go into a deadly spiral and shrink by 30 per cent as during the Great Depression.

One would have thought that this fact would be obvious to all the policy makers. However all the
lessons from the Great Depression seem to have been lost within three-quarters of a century. It
seems, to paraphrase Marc Bard, that politics (especially of the petty and partisan variety) eats
policy for lunch seven days a week.

1. What is the main purpose of the author in writing the passage?

(A) To explain how banks and other financial institutions function


(B) To discuss the lessons learnt from the Great Depression
(C) To argue that banks and manufacturing businesses are interdependent
(D) To criticise a group of people for not learning from the lessons of the Great
Depression
(E) To conclude that people give preference to politics over policies

One would have thought that this fact would be obvious to all the policy makers. However all the
lessons from the Great Depression seem to have been lost within three-quarters of a century.
2. According to the information in the passage, which of the following can be inferred?

(A) Banks are short of cash most of the time


(B) Banks do not like to keep money with the central bank
(C) Banks do not like to keep large amounts of money in their vaults
While it is true from the 2nd paragraph that banks do not like to keep a large proportion of their
funds within their vaults, the amount of money (in absolute terms) in their vaults could still be
fairly large.
(D) Banks usually keep some money with the central bank
(E) Banks actually fool the customers

3. In the last paragraph, the tone of the author is

(A) Adulatory
(B) Optimistic
(C) Critical
(D) Analytical
(E) Ridiculing

Partisan (n) : đảng viên, đảng phái

Ridicule (v) : nhạo báng

Adulatory (adj) : nịnh hót


Mar 17 – 1

On the whole, the American population has very little taste for income redistribution as economic
policy. Beginning in the 1930s, public opinion polls have rarely shown strong support for income
redistribution; during times of economic hardship, the percentage of Americans in favor of such a
system has barely crested 50 percent. Similarly, Americans have been reluctant to press for a limit
on the profits of big corporations, with less than a third of those polled in the 20 year span between
1950 and 1970 favoring such a policy.

Even during the Depression, the populace was reluctant to embrace income redistribution as a
solution to the country’s woes. In 1939, over 60 percent of respondents indicated that the
government should not increase taxes on the wealthy and an overwhelming majority—over 80
percent—rejected the idea of the government confiscating wealth. Clearly, the American spirit of
Lockean liberalism and rugged individualism runs deep. It appears that most people are content to
trust income distribution to the private market.

Of course, while overall support for income distribution remains low on average, there are some
significant differences in levels of support based on income levels. As expected, those in the lowest
income bracket demonstrate the strongest support for employment and income maintenance
programs. However, contrary to expectations, these differences in support were not largest during
the volatile economic times of the 1930s and 1940s. Rather, the documented differences in support
based on income have been relatively stable over time. On the whole, political scientists have
noticed anywhere from a 22 percent to 34 percent difference between the opinions of those
classified as “prosperous” and those classified as “poor” on the question of income redistribution.

1. The main point made by the passage is that

A. The American spirit of Lockean liberalism is primarily responsible for the population’s rejection of
income redistribution.
B. Despite some variations in support based on income levels, the American population
on the whole views income redistribution policies with disfavor, even during times of
economic hardship.
C. Although polls have indicated some variation in response level, on the whole, less than 50
percent of Americans support income redistribution.
D. Poll results on government income redistribution policies have remained stable over time, even
during periods of economic hardship.
E. Income redistribution is not viewed as the solution to economic difficulties by the majority of
American citizens.

2. The passage states that differences in support for governmental income redistribution
policies

A. defied expectations by showing that those in lower income brackets had less support for these
programs than those in higher income brackets
B. vary greatly depending the income level of respondents and the economic conditions at the time
the poll is taken
C. are not, for the most part, significantly affected by outside influences
D. are not significantly different due to the American ideal of rugged individualism
E. are based solely on differences in the income levels of respondents

This is a supporting idea question. The question asks about “differences” in support, so look in the
last paragraph. The author states “However, contrary to expectations, these differences in
support were not largest during the volatile economic times of the 1930s and 1940s.
Rather, the documented differences in support based on income have been relatively
stable over time.” If the differences are stable over time, they are not significantly affected by
outside influences,

3. Which of the following can be inferred from the passage?

I. During the Depression, a smaller percentage of Americans supported raising taxes than supported
government confiscation of wealth
II. Today, about a third of Americans support limits on corporate profits
III. Before 1930, a majority of Americans supported income distribution

A. none
B. I only
C. I and II only
D. I and III only
E. I, II, and III

4. An assumption underlying the author’s assertion in the second paragraph is that


A. the private market is the best way to distribute income in an economy
B. Americans had never experienced economic hardships as severe as those experienced during the
Great Depression
C. Lockean liberalism and rugged individualism entail a reliance on private markets for
income distribution
D. the government had no intentions of confiscating property during the Great Depression
E. income redistribution would be insufficient to solve the country’s economic woes during the Great
Depression

Poll (n) : cuộc thăm dò

In favor of (v) : ủng hộ

Crest (v) : vượt qua

Reluctant (adj) : không sẵn sàng, lưỡng lự

Embrace (v) : siết chặt

Woe (n) : tai ương

Confiscate (v) : tịch thu

Liberalism (n) : chủ nghĩa tự do

Volatile (adj) : bay hơi

Prosperous (adj) : thành công


Mar 17 – 2

The number of women directors appointed to corporate boards in the United States has increased
dramatically, but the ratio of female to male directors remains low. Although pressure to recruit
women directors, unlike that to employ women in the general work force, does not derive from
legislation, it is nevertheless real.

Although small companies were the first to have women directors, large corporations currently have
a higher percentage of women on their boards. When the chairs of these large corporations began
recruiting women to serve on boards, they initially sought women who were chief executive officers
(CEO’s) of large corporations. However, such women CEO’s are still rare. In addition, the ideal of six
CEO’s (female or male) serving on the board of each of the largest corporations is realizable only if
every CEO serves on six boards. This raises the specter of director over-commitment and the
resultant dilution of contribution. Consequently, the chairs next sought women in business who had
the equivalent of CEO experience. However, since it is only recently that large numbers of women
have begun to rise in management, the chairs began to recruit women of high achievement outside
the business world. Many such women are well known for their contributions in government,
education, and the nonprofit sector. The fact that the women from these sectors who were
appointed were often acquaintances of the boards’ chairs seems quite reasonable: chairs have
always considered it important for directors to interact comfortably in the boardroom.

Although many successful women from outside the business world are unknown to corporate
leaders, these women are particularly qualified to serve on boards because of the changing nature
of corporations. Today a company’s ability to be responsive to the concerns of the community and
the environment can influence that company’s growth and survival. Women are uniquely positioned
to be responsive to some of these concerns. Although conditions have changed, it should be
remembered that most directors of both sexes are over fifty years old. Women of that generation
were often encouraged to direct their attention toward efforts to improve the community. This fact
is reflected in the career development of most of the outstandingly successful women of the
generation now in their fifties, who currently serve on corporate boards: 25 percent are in education
and 22 percent are in government, law, and the nonprofit sector.

One organization of women directors is helping business become more responsive to the changing
needs of society by raising the level of corporate awareness about social issues, such as problems
with the economy, government regulation, the aging population, and the environment. This
organization also serves as a resource center of information on accomplished women who are
potential candidates for corporate boards.

1. The author of the passage would be most likely to agree with which of the following
statements about achievement of the “ideal” (Highlighted)?

(A) It has only recently become a possibility.


(B) It would be easier to meet if more CEO’s were women.
(C) It is very close to being a reality for most corporate boards.
(D) It might affect the quality of directors’ service to corporations.
(E) It would be more realizable if CEO’s had a more extensive range of business experience.
2. According to the passage, the pressure to appoint women to corporate boards differs
from the pressure to employ women in the work force in which of the following ways?

(A) Corporate boards are under less pressure because they have such a small number of openings.
(B) Corporate boards have received less pressure from stockholders, consumers, and workers within
companies to include women on their boards.
(C) Corporate boards have received less pressure from the media and the public to include women
on their boards.
(D) Corporations have only recently been pressured to include women on their boards.
(E) Corporations are not subject to statutory penalty for failing to include women on
their boards.

3. All of the following are examples of issues that the organization described in the last
paragraph would be likely to advise corporations on EXCEPT

(A) long-term inflation


(B) health and safety regulations
(C) retirement and pension programs
(D) the energy shortage
(E) how to develop new markets

4. It can be inferred from the passage that, when seeking to appoint new members to a
corporation’s board, the chair traditionally looked for candidates who

(A) had legal and governmental experience


(B) had experience dealing with community affairs
(C) could work easily with other members of the board
(D) were already involved in establishing policy for that corporation
(E) had influential connections outside the business world

5. According to the passage, which of the following is true about women outside the
business world who are currently serving on corporate boards?

(A) Most do not serve on more than one board.


(B) A large percentage will eventually work on the staff of corporations.
(C) Most were already known to the chairs of the board to which they were appointed.
(D) A larger percentage are from government and law than are from the nonprofit sector.
(E) Most are less than fifty years old.

6. The passage suggests that corporations of the past differ from modern corporations
in which of the following ways?

(A) Corporations had greater input on government policies affecting the business community.
(B) Corporations were less responsive to the financial needs of their employees.
(C) The ability of a corporation to keep up with changing markets was not a crucial factor in its
success.
(D) A corporation’s effectiveness in coping with community needs was less likely to
affect its growth and prosperity.
(E) Corporations were subject to more stringent government regulations.
7. Which of the following best describes the organization of the passage?

(A) A problem is described, and then reasons why various proposed solutions succeeded or failed
are discussed.
(B) A problem is described, and then an advantage of resolving it is offered.
(C) A problem is described, and then reasons for its continuing existence are summarized.
(D) The historical origins of a problem are described, and then various measures that have
successfully resolved it are discussed.
(E) The causes of a problem are described, and then its effects are discussed.

8. It can be inferred from the passage that factors making women uniquely valuable
members of modern corporate boards would include which of the following?

I. The nature of modern corporations


II. The increased number of women CEO’s
III. The careers pursued by women currently available to serve on corporate boards

(A) I only
(B) II only
(C) III only
(D) I and III only
(E) I, II, and III

Appoint (v) : bổ nhiệm

Corporate board (n) : hội đồng quản trị

Nevertheless (adv) : tuy nhiên

It’s nevertheless : nó vẫn là

Dilution (n) : pha loãng

Resultant (adj) : kết quả

Acquaintance (n) : người quen

Statutory (n) : theo luật quy định


Mar 18 – 1

Lee Bollinger, rejecting traditional models of the defense of free speech as inadequate, defends it
with a model designed to take into account changes in the function of speech attributable to the
emergence of a society marked by stability and widespread consensus on essential values. This new,
―self-restraint model justifies free speech from a different perspective. Although staunchly
supporting free speech, the self-restraint model inverts the relationship between speech and
tolerance. Under traditional models, the value of tolerance is subordinated to the value of speech.
The self-restraint model, however, often subordinates the value of speech to that of tolerance.
Traditional justifications of the free speech principle originated in the belief that speech is entitled to
greater tolerance than other kinds of activity.

A review of the traditional justifications reveals two distinct models of explanation. Although both
these models link the need to protect speech to its inherent value, they agree on little else.
According to the classical model, freedom of speech serves an indispensable function in democratic
self-government. Meiklejohn uses the traditional New England town meeting as a paradigm for a
self-governing society.

From this perspective, the free speech principle need only protect political speech—the facts,
theories, and opinions relating to any issue on which the citizens must vote. Meiklejohn insists that
even extremist views cannot be withheld from voting citizens, if these views bear on any public
issue. Protection of free speech, including extremist political speech, serves the collective interests
of a self-governing society, made up of all rational, equal, and fully participating citizens. Predicated
on the belief that speech itself is valuable, this theory ascribes positive value to a very broad range
of speech, including any that may be offensive to many people.

In contrast to the serene and optimistic, the fortress model is built on a foundation of pessimism,
individualism, relativism, and self-doubt. According to Holmes, speech represents not so much a
free marketplace of ideas as a kind of ―counsel of despair.‖ Freedom of speech is necessary to the
discovery of truth; but, although any belief held by an individual is ultimately likely to prove false,
individuals tend to feel certain about their beliefs and consequently justified in requiring others to
conform. From Holmes‘ perspective, the government and any majority of the people pose a great
danger of intolerance. In order to protect speech from the natural tendency to censor
nonconforming views, the fortress model prescribes overprotection of speech. This strategy
establishes a broad "buffer zone" that encompasses extremist speech because its protection
substantially diminishes the probability that inherently valuable speech will be suppressed. Even if
speech is so extreme that it cannot seriously be considered to contribute to the discovery of truth—
like the most extreme views propounded by the Nazis—it still ought not to be censored, for once
unleashed censorship cannot be reasonably expected to remain confined to worthless views.
1. It can be inferred from the passage that speech is viewed as a fundamental value in
the:

I. classical model.
II. fortress model.
III. self-restraint model.

A. I only
B. I and II only
C. II and III only
D. I, II and III
E. I and III only

Passage-'Under traditional models, the value of tolerance is subordinated to the value of speech.
The self-restraint model, however, often subordinates the value of speech to that of tolerance.'
i.e. in the self restraint model, tolerance is fundamental
only 1 and 2 is the answer

2. Turning an eye to the greater structure of the passage, which of the following best
describes the function of the last three paragraphs of the passage?

A. The author describes two theories and links each to the historical situation in which it was
proposed.
B. The author refers to a traditional way of viewing a question and examines two
contrasting approaches that spring from that view.
C. The author establishes contrasts between two approaches to a question and then explores their
points of agreement.
D. The author discusses two theories and the opposed conclusions that follow from them.
E. The author reconciles two competing theories for a particular phenomenon

3. The author indicates that Meiklejohn‘s and Holmes‘ understanding of free speech is
similar in that both:

A. believe that free speech ultimately leads to the discovery of truth.


B. favour extending the right of speech to those who express extremist doctrines.
C. consider that censorship involves the suppression of valuable speech.
D. justify free speech by referring to the citizen‘s right to be informed of all views relevant to public
issues.
E. are against blindly supporting free speech

The defense of free speech (n) : bảo vệ tự do ngôn luận

Attributable (adj) : quy kết

Consesus (n) : sự đồng thuận

Self-restraint (adj) : tự kiềm chế

Staunchly (adv) : trung thành

Tolerance (n) : lòng khoan dung


Subordinate (v) : cấp dưới, ở dưới, hạ cấp

Justification (n) : biện minh

Indispensable (adj) : cần thiết, bắt buộc

Paradigm (n) : mô hình

Extremist (adj) : cực đoan

Withheld (v) : từ chối

Predicate (v) : dựa trên

Ascribe (v) : mô tả

Serene (n) : thanh thản

Counsel of despair (n) : lời khuyên tuyệt vọng

Censor (v) : kiểm duyệt

Nonconforming (adj) : không phù hợp

Encompass (v) : bao gồm

Suppress (v) : đàn áp, bãi bỏ

Prepound (v) : đưa ra

Confined (adj) : hạn chế


Mar 20 – 1

Because we have so deeply interiorized writing, we find it difficult to consider writing to be an alien
technology, as we commonly assume printing and the computer to be. Most people are surprised to
learn that essentially the same objections commonly urged today against computers were urged by
Plato in the Phaedrus, against writing.

Writing, Plato has Socrates say, is inhuman, pretending to establish outside the mind what in reality
can be only in the mind. Secondly, Plato‘s Socrates urges, writing destroys memory. Those who use
writing will become forgetful, relying on external resource for what they lack in internal resources.
Thirdly, a written text is basically unresponsive, whereas real speech and thought always exist
essentially in a context of give-and-take between real persons.

Without writing, words as such have no visual presence, even when the objects they represent are
visual. Thus, for most literates, to think of words as totally disassociated from writing is
psychologically threatening, for literates‘ sense of control over language is closely tied to the visual
transformations of language. Writing makes ―words‖ appear similar to things because we think of
words as the visible marks signalling words to decoders, and we have an inability to represent to our
minds a heritage of verbally organized materials except as some variant of writing. A literate person,
asked to think of the word ―nevertheless‖ will normally have some image of the spelled-out word
and be quite unable to think of the word without adverting to the lettering. Thus the thought
processes of functionally literate human beings do not grow out of simply natural powers but out of
these powers as structured by the technology of writing.

Without writing, human consciousness cannot achieve its fuller potentials, cannot produce other
beautiful and powerful creations. Literacy is absolutely necessary for the development not only of
science, but also of history, philosophy, explicative understanding of literature and of any art, and
indeed for the explanation of language (including oral speech) itself. Literate users of a grapholect
such as standard English have access to vocabularies hundreds of times larger than any oral
language can manage. Thus, in many ways, writing heightens consciousness. Technology, properly
interiorized, does not degrade human life but enhances it.

In the total absence of any writing, there is nothing outside the writer, no text, to enable him or her
to produce the same line of thought again or even verify whether he has done so or not. In primary
oral culture, to solve effectively the problem of retaining and retrieving carefully articulated thought,
you have to do your thinking in mnemonic patterns, shaped for ready oral recurrence. A judge in an
oral culture is often called upon to articulate sets of relevant proverbs out of which he can produce
equitable decisions in the cases under formal litigation under him. The more sophisticated orally
patterned thought is, the more it is likely to be marked by set expressions skilfully used. Among the
ancient Greeks, Hesiod, who was intermediate between oral Homeric Greece and fully developed
Greek literacy, delivered quasiphilosophic material in the formulaic verse forms from which he had
emerged.
1). In paragraph 5 of the passage, the author mentions Hesiod (Highlighted) in order to:

A. prove that oral poets were more creative than those who put their verses in written words.
B. show that some sophisticated expressions can be found among the preliterate
ancient Greeks.
C. demonstrate that a culture that is partially oral and partially literate forms the basis of an ideal
society.
D. thinking in mnemonic patterns is an unsuccessful memory device.
E. no sophisticated expressions could be found among the pre-literate ancient Greeks.

2). According to the author, an important difference between oral and literate cultures
can be expressed in terms of:

A. extensive versus limited reliance on memory.


B. chaotic versus structured modes of thought.
C. simple versus complex use of language.
D. barbaric versus civilized forms of communication.
E. presence and absence of books

3). The author refers to Plato in the first and second paragraphs. He brings the
philosopher up primarily in order to:

A. provide an example of literate Greek philosophy.


B. suggest the possible disadvantages of writing.
C. illustrate common misconceptions about writing.
D. define the differences between writing and computer technology.
E. suggest possible benefits of writing

4). The passage is primarily concerned with

A. criticising those who speak against ‗writing‘


B. emphasising the importance of writing
C. assert that writing and consciousness are independent of each other
D. documenting the negative effects of writing
E. discussing how writing has influenced human consciousness

Interiorize (v) : nội tâm hóa

Urge (v) : thúc giục

Socrates (n) : xã hội

Inhuman (adj) : bất nhân, vô nhân đạo

Forgetful (adj) : hay quên

Unresponsive (adj) : không phản hồi

Literate (n) : người biết chữ

Psychologically (adv) : tâm lý


Signalling (n) : báo hiệu

Decoder (n) : bộ giải mã

Inability (n) : không có khả năng

Heritage (n) : di sản, gia tài

Verbally (adv) : bằng lời nói

Organized (adj) : sắp đặt

Material (n) : tài liệu

Degrade (v) : giảm bớt, xuống cấp

Mnemonic (adj) : ghi nhớ

Recurrence (n) : sự nhớ lại

Articulate (adj) : khớp nối

Proverb (n) : tục ngữ

Equitable (adj) : công bằng

Litigation (n) : kiện tụng

Intermediate (adj) : trung gian

Quasi-philosophic (adj) : gần như triết học

Verse (n) : thơ

Formulaic (adj) : công thức

Civilized (adj) : văn minh

Barbaric (adj) : chưa khai hóa

Preliterate (adj) : ưu tiên


Mar 20 – 2

Because of the proximity and likeness of Mars to Earth, scientists have long speculated about the
possibility of life on Mars. As early as the mid 17th century, astronomers observed polar ice caps on
Mars, and by the mid-19th century, scientists discovered other similarities to Earth, including the
length of day and axial tilt. But in 1965, photos taken by the Mariner 4 probe revealed a Mars
without rivers, oceans or signs of life. And in the 1990s, it was discovered that Mars, unlike Earth,
no longer possessed a substantial global magnetic field, allowing celestial radiation to reach the
planet's surface and solar wind to eliminate much of Mars's atmosphere over the course of several
billion years.

More recent probes have focused on whether there was once water on Mars. Some scientists believe
that this question is definitively answered by the presence of certain geological landforms. Others
posit that different explanations, such as wind erosion or carbon dioxide oceans, may be responsible
for these formations. Mars rovers Opportunity and Spirit, which have been exploring the surface of
Mars since 2004, have both discovered geological evidence of past water activity. These findings
substantially bolster claims that there was once life on Mars.

1. The author’s stance on the possibility of life on Mars can best be described as

(A) optimistic
(B) disinterested
(C) skeptical
(D) simplistic
(E) cynical

2. The passage is primarily concerned with which of the following?

(A) disproving a widely accepted theory


(B) initiating a debate about the possibility of life on Mars
(C) presenting evidence in support of a controversial claim
(D) describing the various discoveries made concerning the possibility of life on Mars
(E) detailing the findings of the Mars rovers Opportunity and Spirit

3. Each of the following discoveries is mentioned in the passage EXCEPT

(A) Wind erosion and carbon dioxide oceans are responsible for certain geological
landforms on Mars.
(B) Mars does not have a substantial global magnetic field.
(C) Mars does not currently have water activity.
(D) The length of day on Mars is similar to that on Earth.
(E) The axial tilt of Mars is similar to that of Earth.

Wind erosion and carbon dioxide oceans are responsible for the formation of water

More recent probes have focused on whether there was once water on Mars. Some scientists
believe that this question is definitively answered by the presence of certain geological landforms.
Others posit that different explanations, such as wind erosion or carbon dioxide oceans, may
be responsible for these formations.
4. In the first paragraph, the author most likely mentions the discovery of polar ice caps
to suggest that

(A) until recently Mars’ polar ice caps were thought to consist largely of carbon dioxide
(B) Martian polar ice caps are made almost entirely of water ice
(C) Mars has many similarities to Earth, including the existence of polar ice caps
(D) Mars has only a small fraction of the carbon dioxide found on Earth and Venus
(E) conditions on the planet Mars were once very different from what they are at present

5. Each of the following can be inferred from the passage EXCEPT

(A) The presence of certain geological landforms is not definitive proof that there was once life on
Mars.
(B) It is likely that there were few significant discoveries related to the possibility of life on Mars
prior to the mid-17th century.
(C) The absence of a substantial global magnetic field on Mars suggests that it would be difficult to
sustain life on Mars.
(D) The presence of water activity on Mars is related to the possibility of life on Mars.
(E) The claim that there was once water on Mars has only limited and indirect support
from recent discoveries.

The first para says " scientists have long speculated about the possibility of life on Mars. As
early as the mid-17th century "

This means that scientists have been working before the 17th century and got some healthy results
in 17th and dr was some lack of evidence before that.

Disinterested (adj) : vô tư

Cynical (adj) : hoài nghi, thô lỗ

Bolster (v) : ủng hộ, tăng cường


Mar 21 – 1

Nanotechnology -- the use of structures that have unusual properties because of their atomically
tiny size – is already in use in more than 300 consumer products, from computers to cosmetics, but
like atomic power itself, this new technology holds great promise as well as alarming possibilities.
Determining the safety level of products that utilize nanotechnology is the daunting task of the
Environmental Protection Agency (EPA). One reason that regulating nanotech products is so
challenging is that nanoparticles, though they consist of the same chemicals as traditional products,
have physical structures that give them very different properties, and they can enter the body by
means other than those used by other small particles.

The actual method by which nanoparticles are created is relevant as well. For example, carbon can
be used to create fullerenes, but there is more than one way to manufacture these nanoparticles,
and each way leads to a fullerene with different characteristics. \ Because of this, some research has
identified fullerenes as anti-oxidants, which are useful for treating cells by neutralizing free radicals,
while other research shows fullerenes to be oxidants, which can damage cell membranes. Similarly,
nanoparticles show great potential in cleaning up environmental contaminants, but these particles
themselves might become nearly intractable long-term pollutants.

Some recent research has been truly alarming. Carbon nanotubes placed directly into the lungs of
mice stimulated an immune response that left scar tissue. A similar experiment with a single-walled
carbon nanotube soot mixture caused granulomas and lesions in the rats’ lungs, and proved fatal for
15% of the rats. (It should be noted that these experiments involved instilling the nanomaterials
into the lungs; more experiments are needed involving inhalation.) Another consideration is that
when studying the effects of nanomaterials, more than with typical chemicals, it is not enough to
study a nanoparticle in a vacuum; all physical and chemical characteristics have to be microanalyzed
by studying a material’s reactivity, solubility, and biological persistence under real world conditions.

Because the costs and benefits of nanotech research are so high, international cooperation among
governments and businesses is essential to success. Currently, the U.S. is underfunding research in
this field, and this could have calamitous results. The Toxic Substances Control Act is insufficient for
covering many of the dangers unique to nanotech materials. Furthermore, not all nations have even
created a set of regulations for nanotechnology, also a serious mistake. A product that reaches the
market before it has been thoroughly understood could prove harmful to individual users and whole
populations that have had only minimal exposure. This could so turn public opinion against
nanotechnology that its tantalizing benefits would be indefinitely postponed.

1) Which of the following can be inferred from the passage regarding recent research on
nanoparticles?

A) It has been done mostly in a vacuum without consideration for what happens under real world
conditions.
B) Further experiments are required to adequately judge particular risks that have been
identified.
C) It has been too narrowly focused on older nanoparticles, rather than the newer nanoparticles that
will be used in the future.
D) It has created a generally negative sentiment in the general public regarding nanoparticles.
E) It has shown that carbon nanotubes are particularly dangerous in comparison to other types of
nanoparticles.

2) Which of the following can be properly inferred from the passage?


A) A majority of nations lack the adequate regulations for making nanotechnology safe.
B) Nanotechnology should be banned in the U.S. unless it can be properly regulated by the EPA.
C) Successful research in nanotechnology depends on cooperation among governments
and businesses.
D) The way nanoparticles are formed is the most important element in determining its safety.
E) The potential benefits of nanotechnology outweigh the potential risks.

Because the costs and benefits of nanotech research are so high, international
cooperation among governments and businesses is essential to success

3) According to the passage, which of the following contributes to the complexity of


regulating nanomaterials?
A) The EPA does not have the proper funding with which to create these materials.
B) More than one method can be used to create a nanoparticle.
C) Nations have conflicting standards concerning the safety of nanotech products.
D) Public opinion is currently divided regarding the desirability of products made with nanomaterials.
E) Products that contain nanomaterials are already being used by individuals and businesses.

Unusual (adj) : bất thường

Property (n) : tính chất, đặc tính

Atomically (adv) : nguyên tử

Daunting (adj) : nản chí, khó khăn

Particle (n) : hạt

Anti-oxidant (n) : chất chống oxy hóa

Free radical (n) : các gốc tự do

Membrane (n) : màng

Intractable (adj) : khó khăn, không trị được

Granuloma (n) : u hạt

Lesion (n) : tổn thương

Inhalation (n) : sự hít vào, thở vào

Solubility (n) : độ hòa tan

Calamitous (adj) : tai họa

Tantalize (v) : trêu ngươi

Indefinitely (adv) : vô thời hạn

Postpone (v) : hoãn lại


Mar 21 – 2

Of Homer's two epic poems, the Odyssey has always been more popular than the Iliad, perhaps
because it includes more features of mythology that are accessible to readers. Its subject (to use
Maynard Mack's categories) is "life-as-spectacle," for readers, diverted by its various incidents,
observe its hero Odysseus primarily from without; the tragic Iliad, however, presents "life-
experience": readers are asked to identify with the mind of Achilles, whose motivations render him a
not particularly likable hero. In addition, the Iliad, more than the Odyssey, suggests the complexity
of the gods' involvement in human actions, and to the extent that modern readers find this
complexity a needless complication, the Iliad is less satisfying than the Odyssey, with its simpler
scheme of divine justice. Finally, since the Iliad presents a historically verifiable action, Troy's siege,
the poem raises historical questions that are absent from the Odyssey's blithely imaginative world.

1. The author uses Mack's "categories" most probably in order to

(A) argue that the Iliad should replace the Odyssey as the more popular poem
(B) indicate Mack's importance as a commentator on the Iliad and the Odyssey
(C) suggest one way in which the Iliad and the Odyssey can be distinguished
(D) point out some of the difficulties faced by readers of the Iliad and the Odyssey
(E) demonstrate that the Iliad and the Odyssey can best be distinguished by comparing their
respective heroes

2. The author suggests that the variety of incidents in the Odyssey is likely to deter the
reader from.

(A) concentrating on the poem's mythological features


(B) concentrating on the psychological states of the poem's central character
(C) accepting the explanations that have been offered for the poem's popularity
(D) accepting the poem's scheme of divine justice
(E) accepting Maynard Mack's theory that the poem's subject is "life-as-spectacle"

Its subject (to use Maynard Mack's categories) is "life-as-spectacle," for readers, diverted by its
various incidents, observe its hero Odysseus primarily from without; the tragic Iliad, however,
presents "life-experience": readers are asked to identify with the mind of Achilles, whose
motivations render him a not particularly likable hero. ( readers are not asked to identify with
mind of Odysseus )

3. The passage is primarily concerned with

(A) distinguishing arguments


(B) applying classifications
(C) initiating a debate
(D) resolving a dispute
(E) developing a contrast

4. It can be inferred from the passage that a reader of the Iliad is likely to have trouble
identifying with the poem's hero for which of the following reasons?

(A) The hero is eventually revealed to be unheroic.


(B) The hero can be observed by the reader only from without.
(C) The hero's psychology is not historically verifiable.
(D) The hero's emotions often do not seem appealing to the reader.
(E) The hero's emotions are not sufficiently various to engage the reader's attention

Unheroic (adj) : phi đạo đức

Epic poem (n) : thơ sử thi

Mythology (n) : thần thoại

Spectable (n) : cảnh tượng

Divert (v) : chuyển hướng

Incident (n) : sự cố, bất ngờ

Particularly (adv) : đặc biệt

Likable (adj) : được yêu thích

Needless (adj) : vô ích

Complication (n) : sự khó chịu, phiền phức, biến chứng

Scheme of (n) : sơ đồ

Divine justice (n) : sự công bằng của thánh thần

Verifiable (adj) : có thể kiểm chứng

Siege (n) : sự bao vây

Blithely (adv) : chân thành

Deter (v) : ngăn cản

Psychological state (adj/n) : trạng thái tâm lý


Mar 22 – 1

The question of when the first people populated the American subcontinents is hotly debated. Until
recently, the Clovis people, based on evidence found in New Mexico, were thought to have been the
first to have arrived, some thirteen thousand years ago. Yet evidence gathered from other sites
suggests the Americas had been settled at least one thousand years prior to the Clovis people's
arrival. The "Clovis-first" idea, nevertheless, was treated as gospel, backed by supporters who, at
least initially, outright discounted any claims that suggested precedence by non-Clovis people. While
such a stance smacked of fanaticism, proponents did have a solid claim: if the Clovis crossed the
Bering Strait thirteen thousand years ago, only after it had become ice-free, how would others have
been able to make a similar trip but over ice?

A recent school of thought, backed by Weber, provides the following answer: pre-Clovis people
reached the Americas by relying on a sophisticated maritime culture, which allowed them to take
advantage of refugia, or small areas in which aquatic life flourished. Thus, they were able to make
the long journey by hugging the coast as far south as what is today British Columbia. Additionally,
they were believed to have fashioned a primitive form of crampon so that they would be able to
dock in these refugia and avail themselves of the microfauna. Still, such a theory raises the question
as to how such a culture developed.

The Solutrean theory has been influential in answering this question, a fact that may seem
paradoxical—and startling—to those familiar with its line of reasoning: the Clovis people were
actually Solutreans, an ancient seafaring culture along the Iberian peninsula, who had—
astoundingly, given the time period—crossed into the Americas via the Atlantic Ocean. Could a
similar Siberian culture, if not the pre-Clovis people themselves, not have displayed equal nautical
sophistication?

Even if one subscribes to this line of reasoning, the "Clovis-first" school still has an objection:
proponents of a pre-Clovis people rely solely on the Monte Verde site in Chile, a site so far south
that its location raises the question: what of the six thousand miles of coastline between the ice
corridor and Monte Verde? Besides remains found in a network of caves in Oregon, there has been
scant evidence of a pre-Clovis people.

Nevertheless, Meade and Pizinsky claim that a propitious geologic accident could account for this
discrepancy: Monte Verde was located near a peat bog that essentially fossilized the village.
Archaeologists uncovered two of the wooden stakes, which, at one time, were used in twelve huts.
Furthermore, plant species associated with areas one hundred and fifty miles away were found,
suggesting a trade network. These findings indicate that the Clovis may not have been the first to
populate the Americas, yet more excavation, both in Monte Verde and along the coast, must be
conducted in order to determine the extent of pre-Clovis settlements in the Americas.

1. In the context in which it appears, the phrase "avail themselves of" most nearly means

A. locate
B. exploit
C. regard
D. fathom
E. distribute
2. It can be inferred from the passage that the reason the author finds the Solutrean
hypothesis both startling and paradoxical is that

A. ancient cultures were most likely unable to develop such a sophisticated form of maritime
transport that they were able to cross the Atlantic
B. it supports the Clovis school of thought and posits the existence of a capacity not
commonly associated with ancient people
C. the Clovis people had crossed from Siberia navigating across a difficult ice corridor, whereas the
pre-Clovis people had sailed, with far less difficulty, across the Atlantic Ocean
D. it suggests that the pre-Clovis people had a way to circumvent the ice corridor, yet were unlikely
to have traveled as far south as modern day Chile
E. it runs counter to one of the chief tenets of the "Clovis-first" school of thought

3. It can be most reasonably inferred from the passage that, in regard to the manner in
which the Monte Verde village was preserved,

A. unless evidence of other pre-Clovis people was fossilized the same way it was in Monte Verde,
archaeologists will be unable to determine the extent of the settlement of pre-Clovis people
B. major discoveries can sometimes result from random processes in the environment
C. plant species can offer valuable clues into the origin of other pre-Clovis settlements
D. sites dated from slightly after the period of the Clovis people did not offer archaeologists such a
trove of information
E. archaeologists are unlikely to find any other significant evidence of pre-Clovis people unless they
venture as much as one hundred and fifty miles from the site

4. If it is true that a trade network between pre-Clovis people had been established,
then which of the following could be expected to be found at settlements near Monte
Verde?

I. other villages that have been preserved in a peat bog


II. plants species similar to those uncovered at Monte Verde
III. the same number of wooden stakes for supporting dwellings

A. I only
B. II only
C. III only
D. I and III only
E. II and III only

Gospel (n) : sách phúc âm, sự thuyết giáo

Discount (v) : không để ý đến

Outright (adv) : ngay

Precedence (n) : ưu tiên, quyền ở trước

Smack of (v) : đánh vào

Fanaticism (n) : sự cuồng tín


Refugia (n) : một loài thực vật

Flourish (v) : mọc lên, khởi sắc, hưng thịnh

Fashion (v) : hình dáng

Primitive (adj) : nguyên thủy

Crampon (n) : móc sắt

Astoundingly (adv) : đáng kinh ngạc

Nautical (adj) : hải lý, thuộc về sự đi biển

Corridor (n) : hành lang

Scant (adj) : hiếm, ít ỏi, không đủ

Propitious (adj) : thuận tiện

Geologic (n) : địa chất

Peat bog (n) : mỏ than bùn

Wooden stake (n) : cọc gỗ

Hut (n) : chòi, lều, trại bằng ván

Manner (n) : cách thức

Preserve (v) : bảo quản, bảo tồn

Exploit (v) : khai thác

Paradoxical (adj) : nghịch lý

Startling (adj) : giật mình


Mar 22 – 2

Theorists are divided concerning the origin of the Moon. Some hypothesize that the Moon was
formed in the same way as were the planets in the inner solar system (Mercury, Venus, Mars, and
Earth)—from planet-forming materials in the presolar nebula. But, unlike the cores of the inner
planets, the Moon's core contains little or no iron, while the typical planet-forming materials were
quite rich in iron. Other theorists propose that the Moon was ripped out of the Earth's rocky mantle
by the Earth's collision with another large celestial body after much of the Earth's iron fell to its core.
One problem with the collision hypothesis is the question of how a satellite formed in this way could
have settled into the nearly circular orbit that the Moon has today. Fortunately, the collision
hypothesis is testable. If it is true, the mantle rocks of the Moon and the Earth should be the same
geochemically.

1. The primary purpose of the passage is to

(A) present two hypotheses concerning the origin of the Moon


(B) discuss the strengths and weaknesses of the collision hypothesis concerning the origin of the
Moon
(C) propose that hypotheses concerning the Moon's origin be tested
(D) argue that the Moon could not have been formed out of the typical planet-forming materials of
the presolar nebula
(E) describe one reason why the Moon's geochemical makeup should resemble that of the Earth

2. According to the passage, Mars and the Earth are similar in which of the following
ways?

I. Their satellites were formed by collisions with other celestial bodies.


II. Their cores contain iron.
III. They were formed from the presolar nebula.

(A) III only


(B) I and II only
(C) I and III only
(D) II and III only
(E) I, II, and III

3. The author implies that a nearly circular orbit is unlikely for a satellite that

(A) circles one of the inner planets


(B) is deficient in iron
(C) is different from its planet geochemically
(D) was formed by a collision between two celestial bodies
(E) was formed out of' the planet-forming materials in the presolar nebula

Other theorists propose that the Moon was ripped out of the Earth's rocky mantle by the Earth's
collision with another large celestial body after much of the Earth's iron fell to its core. One problem
with the collision hypothesis is the question of how a satellite formed in this way could have
settled into the nearly circular orbit that the Moon has today
Tác giả cho rằng nếu một vệ tinh được hình thành bằng một vụ va chạm giữa hai thiên thể giống
như giả thuyết giả định, thì vệ tinh đấy cũng không thể có quỹ đạo tròn giống như mặt trăng bây
giờ  Tác giả ngầm ẩn ý rằng nếu mặt trăng thật sự hình thành từ vỏ trái đất tách ra sau một vụ
va chạm, thì mặt trăng không thể xoay quanh trái đất như bây giờ

4. Which of the following, if true, would be most likely to make it difficult to verify the
collision hypothesis in the manner suggested by the author?

(A) The Moon's core and mantle rock are almost inactive geologically.
(B) The mantle rock of the Earth has changed in composition since the formation of the
Moon, while the mantle rock of the Moon has remained chemically inert.
(C) Much of the Earth's iron fell to the Earth's core long before the formation of the Moon, after
which the Earth's mantle rock remained unchanged.
(D) Certain of the Earth's elements, such as platinum, gold, and iridium, followed iron to the Earth's
core.
(E) The mantle rock of the Moon contains elements such as platinum, gold, and iridium.

Nebula (n) : tinh vân

Mantle (n) : lớp phủ

Collision (n) : va chạm

Celestial body (adj/n) : thiên thể

Circular orbit (n) : quỹ đạo tròn

Unlikely (adj) : không có khả năng xảy ra

Form (v) : hình thành


Mar 23 – 1

When speaking of Romare Bearden, one is tempted to say, "A great Black American artist?' The
subject matter of Bearden's collages is certainly Black. Portrayals of the folk of Mecklenburg County,
North Carolina, whom he remembers from early childhood, of the jazz musicians and tenement roofs
of his Harlem days, of Pittsburgh steelworkers, and his reconstruction of classical Greek myths in the
guise of the ancient Black kingdom of Benin, attest to this. In natural harmony with this choice of
subject matter are the social sensibilities of the artist, who remains active today with the Cinque
Gallery in Manhattan, which he helped found and which is devoted to showing the work of minority
artists. Then why not call Bearden a Black American artist? Because ultimately this categorization is
too narrow. "What stands up, in the end, is structure," Bearden says. "What I try to do is amplify. If
I were just creating a picture of a farm woman from back home, it would have meaning to her and
people there. But art amplifies itself to something universal."

1. According to the passage, all of the following are depicted in Bearden's


collages EXCEPT

(A) workers in Pittsburgh's steel mills


(B) scenes set in the ancient kingdom of Benin
(C) people Bearden knew as a child
(D) traditional representations of the classical heroes of Greek mythology
(E) the jazz musicians of the Harlem Bearden used to know

2. The author suggests that Bearden should not be called a Black American artist
because

(A) there are many collages by Bearden in which the subject matter is not Black
(B) Bearden's work reflects the Black American experience in a highly individual style
(C) through the structure of Bearden's art his Black subjects come to represent all of
humankind
(D) Bearden's true significance lies not so much in his own work as in his efforts to help other
minority artists
(E) much of Bearden's work uses the ancient Black kingdom of Benin for its setting

3. Bearden's social sensibilities and the subject matter of his collages are mentioned by
the author in order to explain

(A) why one might be tempted to call Bearden a Black American artist
(B) why Bearden cannot be readily categorized
(C) why Bearden's appeal is thought by many to be ultimately universal
(D) how deeply an artist's artistic creations are influenced by the artist's social conscience
(E) what makes Bearden unique among contemporary Black American artists

4. The author of the passage is chiefly concerned with

(A) discussing Bearden's philosophy of art


(B) assessing the significance of the ethnic element in Bearden's work
(C) acknowledging Bearden's success in giving artistic expression to the Black American experience
(D) pointing out Bearden's helpfulness to other minority artists
(E) tracing Bearden's progress toward artistic maturity

Collage (n) : cắt dán ( tác phẩm nghệ thuật được tạo ra bằng cách dán nhiều chất liệu khác khau
như ảnh, mảnh giấy hoặc vải )

Portrayal (n) : chân dung

Guise (n) : hình thức bên ngoài, cách trình bày, thường che giấu bản chất thực sự của một cái gì đó

Attest (v) : chứng thực

Amplify (v) : khoa trương, khếch đại


Mar 23 – 2

Objects in the inner solar system - the planets and asteroid belt - follow ellipsoid orbits that are
predictable in nature. While they may shift over billions of years, on a million-year timescale these
objects are relatively easy to model. In contrast, scientists have long debated why small objects
beyond Neptune and Pluto tend to have orbits that are circular in nature and irregular on the million
year timescale. Most theorized that there must be an elusive “ninth planet” beyond Neptune that
had enough mass to deform the orbits of smaller objects in close proximity. However, as telescope
technology has increased in complexity, there has been no sign of an additional planet of sufficient
size to meet this requirement.

To investigate the cause of these irregularities and find an alternate explanation, Madigan et al.
used computer simulations to model the way different objects in the outer solar system moved
around the sun. They saw that within the simulation the smaller objects move more quickly than do
larger ones. This allowed them to notice a peculiar phenomenon. As the objects move around the
sun, pile ups of objects in orbits close to one another occur as smaller objects catch up to bigger
ones, causing a gravitational “jostling” effect. This effect causes the usually ellipsoid orbits to deform
into the circular ones that scientists have tried to explain for so long. It also leads to the temporary
creation of a bigger object that moves more slowly and moves further away from the sun. As the
conglomeration breaks up, the now-smaller objects begin to move more quickly are pulled closer to
the sun, creating the hypothesized irregularities in the objects’ orbits.

1) Which of the following best describes the structure of this passage?

A) A scientific norm is explained and an interpretation of that norm is presented and then rejected.
B) Two theories about a scientific observation are presented and one is rejected as impractical.
C) Two methods for predicting a scientific phenomenon are presented and one is deemed more
suitable.
D) A former scientific theory is presented and then rejected before an alternative theory is revealed
and confirmed.
E) A scientific problem is introduced, a potential answer is discarded, and an alternative
explanation is posed.

2) It can be inferred that a large planet as described in paragraph 1 would

A) move more quickly around the sun than would smaller planetoids.
B) move more slowly around the sun than would smaller planetoids.
C) orbit closer to the sun than would smaller planetoids.
D) explain all deviations in other objects’ orbits.
E) suggest an explanation for gravitational jostling.

3) Within the context of the second paragraph, why is it important that large objects
move around the sun more slowly than do smaller ones?

A) If there was no difference in the speed at which the objects move, jostling may not
occur.
B) Relative speeds directly cause changes in the orbits of objects in the outer solar system.
C) The fact that larger objects move so slowly definitively proves that an additional planet beyond
Neptune cannot exist.
D) The differences in speed directly explain the irregularities in the distance between objects in the
outer solar system and the sun.
E) Larger objects have more inertia and could distort other objects’ orbits more if they moved more
quickly.

Asteroid belt (n) : vành đai tiểu hình tinh

Ellipsoid (n) : hình elip

Elusive (adj) : khó nắm bắt

Deform (v) : biến dạng

Proximity (n) : trạng thái ở gần

Peculiar (adj) : dị thường

Pile ups of : chồng chất, đống

Jostle (v) : chen lấn

Conglomeration (n) : sự dính lại, sự kết lại

Distort (v) : làm siêu vẹo

Inertia (n) : quán tính


Mar 24 – 1

One scientific discipline, during its early stages of development, is often related to another as an
antithesis to its thesis. The thesis discipline tend to concern itself with discovery and classification of
phenomena, to offer holistic explanations emphasizing pattern and form, and to use existing theory
to explain the widest possible range of phenomena. The paired or antidiscipline, on the other hand,
can be characterized by a more focused approach, concentrating on the units of construction, and
by a belief that the discipline can be reformulated in terms of the issues and explanations of the
antidiscipline.

The relationship of cytology (cell biology) to biochemistry in the late nineteenth century, when both
disciplines were growing at a rapid pace, exemplifies such a pattern. Researchers in cell biology
found mounting evidence of an intricate cell architecture. They also deduced the mysterious
choreography of the chromosomes during cell division. Many biochemists, on the other hand,
remained skeptical of the idea that so much structure existed, arguing that the chemical reactions
that occur in cytological preparations might create the appearance of such structures. Also, they
stood apart from the debate then raging over whether protoplasm, the complex of living material
within a cell, is homogeneous, networklike, granular, or foamlike. Their interest lay in the core
“fundamental” issues of the chemical nature of protoplasm, especially the newly formulated enzyme
theory of life.

In general, biochemists judged cytologists to be too ignorant of chemistry to grasp the basic
processes, whereas cytologists considered the methods of biochemists inadequate to characterize
the structures of the living cell. The renewal of Mendelian genetics and, later, progress in
chromosome mapping did little at first to effect a synthesis.

Both sides were essentially correct. Biochemistry has more than justified its extravagant early claims
by explaining so much of the cellular machinery. But in achieving this feat (mostly since 1950) it has
been partially transformed into the new discipline of molecular biology—biochemistry that deals with
spatial arrangements and movements of large molecules. At the same time cytology has
metamorphosed into modern cellular biology. Aided by electron microscopy, it has become more
similar in language and outlook to molecular biology. The interaction of a discipline and its
antidiscipline has moved both sciences toward a synthesis, namely molecular genetics.

This interaction between paired disciplines can have important results. In the case of late
nineteenth-century cell research, progress was fueled by competition among the various attitudes
and issues derived from cell biology and biochemistry. Joseph Fruton, a biochemist, has suggested
that such competition and the resulting tensions among researchers are a principal source of vitality
and “are likely to lead to unexpected and exciting novelties in the future, as they have in the past.”

Discipline (n) : môn học

Antithesis (n) : phản đề

Holistic (adj) : toàn diện

Reformulate (v) : cải cách

Cytology (n) : tế bào học


Exemplify (v) : làm ví dụ mẫu mực

Mount (v) : gắn kết

Intricate (adj) : phức tạp, lộn xộn

Choreography (n) : vũ đạo

Chromosome (n) : nhiễm sắc thể

Cell division (n) : phân chia tế bào

Protoplasm (n) : nguyên sinh chất

Granular (adj) : dạng hạt

Foamlike (adj) : giống như bọt

Grasp (v) : cầm lấy, chụp lấy, cướp lấy

Systhesis (n) : sự tổng hợp

Extravagant (adj) : ngông cuồng, vô lý

Cellular (adj) : thuộc về tế báo

Molecular (adj) : phân tử

Spatial (adj) : không gian

Arrangement (n) : sắp xếp

Metamorphose (v) : biến đổi

Electron microscopy (n) : kính hiển vi điện tử

Novelty (n) : mới lạ

Vitality (n) : sức sống


Mar 24 - 2

There are two major systems of criminal procedure in the modern world—the adversarial and the
inquisitorial. Both systems were historically preceded by the system of private vengeance in which
the victim of a crime fashioned a remedy and administered it privately, either personally or through
an agent.

The modern adversarial system is only one historical step removed from the private vengeance
system and still retains some of its characteristic features. For example, even though the right to
initiate legal action against a criminal has now been extended to all members of society (as
represented by the office of the public prosecutor), and even though the police department has
effectively assumed the pretrial investigative functions on behalf of the prosecution, the adversarial
system still leaves the defendant to conduct his or her own pretrial investigation. The trial is viewed
as a forensic duel between two adversaries, presided over by a judge who, at the start, has no
knowledge of the investigative background of the case. In the final analysis the adversarial system
of criminal procedure symbolizes and regularizes punitive combat.

By contrast, the inquisitorial system begins historically where the adversarial system stopped its
development. It is two historical steps removed from the system of private vengeance. From the
standpoint of legal anthropology, then, it is historically superior to the adversarial system. Under the
inquisitorial system, the public prosecutor has the duty to investigate not just on behalf of society
but also on behalf of the defendant. Additionally, the public prosecutor has the duty to present the
court not only evidence that would convict the defendant, but also evidence that could prove the
defendant’s innocence. The system mandates that both parties permit full pretrial discovery of the
evidence in their possession. Finally, an aspect of the system that makes the trial less like a duel
between two adversarial parties is that the inquisitorial system mandates that the judge take an
active part in the conduct of the trial, with a role that is both directive and protective.

Fact-finding is at the heart of the inquisitorial system. This system operates on the philosophical
premise that in a criminal action the crucial factor is the body of facts, not the legal rule (in contrast
to the adversarial system), and the goal of the entire procedure is to attempt to recreate, in the
mind of the court, the commission of the alleged crime.

Because of the inquisitorial system’s thoroughness in conducting its pretrial investigation, it can be
concluded that, if given the choice, a defendant who is innocent would prefer to be tried under the
inquisitorial system, whereas a defendant who is guilty would prefer to be tried under the
adversarial system.

1. It can be inferred from the passage that the crucial factor in a trial under the
adversarial system is

(A) rules of legality


(B) dramatic reenactment of the crime
(C) the search for relevant facts
(D) the victim’s personal pursuit of revenge
(E) police testimony about the crime
2. The author sees the judge’s primary role in a trial under the inquisitorial system as
that of

(A) passive observer


(B) biased referee
(C) uninvolved administrator
(D) aggressive investigator
(E) involved manager

3. According to the passage, a central distinction between the system of private


vengeance and the two modern criminal procedure systems was the shift in
responsibility for initiating legal action against a criminal from the

(A) defendant to the courts


(B) victim to society
(C) defendant to the prosecutor
(D) courts to a law enforcement agency
(E) victim to the judge

4. All of the following are characteristics of the inquisitorial system that the author cites
EXCEPT:

(A) It is based on cooperation rather than conflict.


(B) It encourages full disclosure of evidence.
(C) It requires that the judge play an active role in the conduct of the trial.
(D) It places the defendant in charge of his or her defense.
(E) It favors the innocent.

5. The author’s attitude toward the inquisitorial system can best be described as

(A) doubtful that its judges can be both directive and protective
(B) satisfied that it has potential for uncovering the relevant facts in a case
(C) optimistic that it will replace the adversarial system
(D) wary about its down playing of legal rules
(E) critical of its close relationship with the private vengeance system

Criminal procedure (n) : tố tụng hình sự

Adversarial (adj) : nghịch cảnh, hệ thống đối nghịch

Inquisitorial (adj) : điều tra

Precede (v) : đi trước

Vengeance (n) : báo thù

Fashion (v) : làm cho thích hợp

Initiate (v) : khởi xướng

Public prosector (n) : công tố viên


Pretrial investigation (n) : điều tra trước

Prosecution (n) : truy tố

Forensic (adj) : tòa án, thuộc về tư pháp

Duel (n) : đấu tay đôi

Adversary (n) : đối thủ

Preside over (v) : chủ trì

Symbolize (v) : tượng trưng

Regularize (v) : làm cho đúng luật

Punitive (adj) : trừng phạt

Standpoint (n) : quan điểm

Convict (v) : kết án

Mandate (v) : uỷ nhiệm, ủy thác

Directive (adj) : chỉ thị

Commission (n) : ủy ban

Allege (v) : báo buộc

Testimony (n) : lời khai, sự làm chứng


Mar 25 – 1

The Taft-Hartley Act, passed by the United States Congress in 1947, gave states the power to enact
“right-to-work” legislation that prohibits union shop agreements. According to such an agreement, a
labor union negotiates wages and working conditions for all workers in a business, and all workers
are required to belong to the union. Since 1947, 20 states have adopted right-to-work laws. Much of
the literature concerning right-to-work laws implies that such legislation has not actually had a
significant impact. This point of view, however, has not gone uncriticized. Thomas V Carroll has
proposed that the conclusions drawn by previous researchers are attributable to their myopic focus
on the premise that, unless right-to-work laws significantly reduce union membership within a state,
they have no effect. Carroll argues that the right-to-work laws “do matter” in that such laws
generate differences in real wages across states. Specifically, Carroll indicates that while right-to-
work laws may not “destroy” unions by reducing the absolute number of unionized workers, they do
impede the spread of unions and thereby reduce wages within right-to-work states. Because the
countervailing power of unions is weakened in right-to-work states, manufacturers and their
suppliers can act cohesively in competitive labor markers, thus lowering wages in the affected
industries.

Such a finding has important implications regarding the demographics of employment and wages in
right-to-work states. Specifically, if right-to-work laws lower wages by weakening union power,
minority workers can be expected to suffer a relatively greater economic disadvantage in right-to-
work states than in union shop states. This is so because, contrary to what was once thought, union
tend to have a significant positive impact on the economic position of minority workers, especially
Black workers, relative to White workers. Most studies concerned with the impact of unionism on the
Black worker’s economic position relative to the White worker’s have concentrated on the changes in
Black wages due to union membership. That is, they have concentrated on union versus nonunion
groups. In a pioneering study, however, Ashenfelter finds that these studies overlook an important
fact: although craft unionism increase the differential between the wages of White workers and
Black workers due to the traditional exclusion of minority workers from unions in the craft sectors of
the labor market, strong positive wage gains are made by Black workers within industrial unions. In
fact, Ashenfelter estimates that industrial unionism decreases the differential between the wages of
Black workers and White workers by about 3 percent. If state right-to-work laws weaken the
economic power of unions to raise wages, Black workers will experience a disproportionate decline
in their relative wage positions. Black workers in right-to-work states would therefore experience a
decline in their relative economic positions unless there is strong economic growth in right-to-work
states, creating labor shortages and thereby driving up wages.
1. The reasoning behind the “literature” (Highlighted), as that reasoning is presented in
the passage, is most analogous to the reasoning behind which one of the following
situations?

(A) A law is proposed that benefits many but disadvantages a few: those advocating passage of the
law argue that the disadvantages to few are not so serious that the benefits should be denied to
many.
(B) A new tax on certain categories of consumer items is proposed: those in favor of the tax argue
that those affected by the tax are well able to pay it, since the items taxed are luxury items.
(C) A college sets strict course requirements that every student must complete before graduating;
students already enrolled argue that it is unfair for the new requirements to apply to those enrolled
before the change.
(D) The personnel office of a company designs a promotions become effective on January 1: the
managers protest that such a policy means that they cannot respond fast enough to changes in
staffing needs.
(E) A fare increase in a public transportation system does not significantly reduce the
number of fares sold: the management of the public transportation system asserts,
therefore, that the fare hike has had no negative effects.

2. According to the passage, which one of the following is true of Carroll’s study?

(A) It implies that right-to-work laws have had a negligible effect on workers in right-to-work states.
(B) It demonstrates that right-to-work laws have significantly decreased union membership from
what it once was in right-to-work states.
(C) It argues that right-to-work laws have affected wages in right-to-work states.
(D) It supports the findings of most earlier researchers.
(E) It explains the mechanisms by which collusion between manufacturers and suppliers is
accomplished.

3. It can be inferred from the passage that the author believes which one of following
about craft unions?

(A) Craft unions have been successful in ensuring that the wages of their members
remain higher than the wages of nonunion workers in the same occupational groups.
(B) The number of minority workers joining craft unions has increased sharply in states that have
not adopted right-to-work legislation.
(C) Wages for workers belonging to craft unions have generally risen faster and more steadily than
wages for workers belonging to industrial unions.
(D) The wages of workers belonging to craft unions have not been significantly affected by right-to-
work legislation, although the wages of workers belonging to industrial unions have been negatively
affected.
(E) The wages of workers belonging to craft unions are more likely to be driven up in the event of
labor shortages than are the wages of workers belonging to industrial unions.
4. Which one of the following best describes the effect industrial unionism has had on
the wages of Black workers relative to those of White workers, as that effect is
presented in the passage?

(A) Prior to 1947, industrial unionism had little effect on the wages of Black workers relative to those
of White workers: since 1947, it has had a slight positive effect.
(B) Prior to 1947, industrial unionism had a strong positive effect on the wages of Black workers
relative to those of White workers: since 1947, it has had little effect.
(C) Prior to 1947, industrial unionism had a negative effect on the wages of Black workers relative to
those of White workers: since 1947, it has had a significant positive effect.
(D) Industrial unionism has contributed moderately to an increase in the wage differential between
Black workers and White workers.
(E) Industrial unionism has contributed strongly to a 3 percent decrease in the wage
differential between Black workers and White workers.

5. According to the passage, which one of the following could counteract the effects of a
decrease in unions’ economic power to raise wages in right-to-work states?

(A) a decease in the number of union shop agreements


(B) strong economic growth that creates labor shortages
(C) a decrease in membership in craft unions
(D) the merging of large industrial unions
(E) a decline in the craft sectors of the labor market

6. Which one of the following best describes the passage as a whole?

(A) an overview of a problem in research methodology and a recommended solution to that problem
(B) a comparison of two competing theories and a suggestion for reconciling them
(C) a critique of certain legislation and a proposal for modification of that legislation
(D) a review of research that challenges the conclusions of earlier researchers
(E) a presentation of a specific case that confirms the findings of an earlier study

Countervail (v) : đối kháng

Demographic (n) : nhân khẩu học

Unionism (n) : đoàn thể

Pioneering (adj) : tiên phong

Craft (adj) : hội tương trợ


Mar 25 – 2

Keynesian economics is a theory that in the short run, especially during recessions, economic output
is strongly influenced by aggregate demand (total spending in the economy). In the Keynesian view,
aggregate demand does not necessarily equal the productive capacity of the economy; instead, it is
influenced by a host of factors and sometimes behaves erratically, affecting production,
employment, and inflation.

The theories forming the basis of Keynesian economics were first presented by the
British economist John Maynard Keynes in his book, The General Theory of Employment, Interest
and Money, published in 1936 during the Great Depression. Keynes contrasted his new approach to
the aggregate supply-focused 'classical' economics that preceded his book. The interpretations of
Keynes that followed are contentious and several schools of economic thought claim his legacy.

Keynesian economists often argue that private sector decisions lead to inefficient macroeconomic
outcomes and as a result active policy responses by the public sector are required. In particular,
they call for monetary policy actions by the central bank and fiscal policy actions by the government
to stabilize output over the business cycle. Keynesian economic theory advocates a mixed economy
– predominantly private sector, but with some role for government intervention during recessions.

Keynesian economics served as the standard economic model in the developed nations during the
later part of the Great Depression, World War II, and the post-war economic expansion (1945–
1973), though it lost some influence following the oil shock and resulting stagflation of the 1970s.
The advent of the global financial crisis in 2008 has caused a resurgence in Keynesian thought, and
the theory remains as important today as it was nearly 100 years ago.

1)A Keynesian economist would most likely agree with which of the following statements:

A)Government intervention should be the primary means for fixing the economy in a recession.
B)Private sector decisions lead to an efficient market.
C)The private sector should play the primary role in the economy.
D)Stagflation resulted from private sector behavior.
E)The supply side of the economy is more important than the demand side.

Keynesian economic theory advocates a mixed economy – predominantly private sector, but
with some role for government intervention during recessions

2)Which of the following can be properly inferred from the passage?

A)In the Keynesian view, the aggregate demand represents the productive capacity of the economy.
B)The Keynesian approach is best used during periods of recession.
C)The Keynesian approach was developed as a way to end the Great Depression.
D)Keynesian economists believe that the government should sometimes influence fiscal
policy.
E)Keynesian economists believe in focusing on the supply side of the economy.

In particular, they call for monetary policy actions by the central bank and fiscal policy actions by the
government to stabilize output over the business cycle
3)The primary purpose of the passage is to:

A)advocate John Maynard Keyes’ economic theory


B)challenge classical economic theory
C)discuss Keynes’ The General Theory of Employment, Interest and Money
D)present an important historical theory
E)discuss the Keynesian view on monetary policy actions

Predominantly (adv) : chủ yếu

Aggregate (adj) : tổng

Contentious (adj) : gây tranh cãi

Fiscal policy (n) : chính sách tài khóa

Stagflation (n) : lạm phát

Advent (n) : đột ngột, xuất hiện

Resurgence (n) : sự hồi sinh


Mar 26 – 1

Saving the rainforests has been a classic environmental cause for decades. Rainforests absorb large
quantities of carbon dioxide from the air and make it suitable for human use. However, recent
developments have started a debate that questions the endangered status of the rainforests. New
“secondary” forests have been cropping up in tropical regions around the world. These new forests
are growing in land that had been used for farming or logging but has lain fallow in recent years as
farmers and rural dwellers migrate to the cities in search of better work and opportunities. Scientists
are now debating the newfound benefits of the “secondary” forests and deciding whether they can
be valid replacements for the old rainforest that is being destroyed daily.

The rainforests play a vital role in removing carbon dioxide, one of the main heat-trapping gasses
and a major cause of global warming, from the air and turning it into oxygen by a process called
photosynthesis. Photosynthesis occurs in two separate reactions. During the light reaction,
chlorophyll and other pigments such as beta-carotene absorb slightly different lengths of light waves
and transmit the energy to the central chlorophyll molecule. The energy collected from the light
reaction is stored in molecules of ATP, which is similar in structure to the building blocks of human
DNA. The ATP is used in the Calvin Cycle in combination with carbon dioxide to form complex
sugars, giving off oxygen as a byproduct.

Some scientists believe that the new forests can supplement the waning number of rainforest foliage
and absorb carbon dioxide in their place. One statistic says the new forests are replacing the
destroyed areas at a rate of 50 to 1. Because of the heat and intense rainfall in these tropical areas,
the new growth increases at an astounding rate and makes these new forests appear to the
untrained eye as if they had flourished as rainforest for hundreds of years. Some scientists
concerned with carbon emissions look at the emerging forests and conclude that they are suitable
replacements. After all, the Mayans and other indigenous peoples cleared lands in the areas that are
now clearly visible as lush rainforest.

Not all scientists agree that the “secondary” forests are a perfect solution. Although they agree that
the new forestry can reduce carbon emissions, they argue that new forests do not possess all the
same qualities as the primary rainforests. “Secondary” forests suffer from a notable difference in
floristics and species diversity. Since the unused farmland is often situated hundreds of miles away
from the native rainforests, species endangered by their destruction cannot benefit from the new
growth in those areas. To this group of scientists, the ecosystems in the new forests are just not the
same as those of a true rainforest.

1. Which of the following does the author suggest about species endangered by the
destruction of the native rainforests?

A) They cannot reach the native rainforests from the new growth in unused farmland.
B) They cannot survive the journey from the native rainforests to the new forests.
C) They are not suited to the conditions in the new growth.
D) They cannot cope with human settlements that lie in their path to the new growth.
E) Animals must travel great distances in open farmland in order to move between native
rainforests.
2. The third paragraph performs which of the following functions in the passage?

A) It refutes the evidence presented in the fourth paragraph.


B) It establishes the author's knowledgeability and, therefore, authority in this field of study.
C) It presents one side of a debate.
D) It supports the previous paragraphs by elaborating on the phenomenon described in the second
paragraph.
E) It presents the new forests as a comprehensive solution to the rainforests problem.

3. According to the passage, what is one function of ATP?

A) It is part of the DNA molecule.


B) It creates complex sugars.
C) It stores the energy created in the light reaction.
D) It gives off oxygen as a byproduct.
E) It is used in the light reaction in combination with carbon dioxide to form complex sugars.

Lain fallow (v) : bỏ hoang

Dweller (n) : cư dân

Photosynthesis (n) : quang hợp

Pigment (n) : sắc tố

Chlorophyll (n) : diệp lục

Molecule (n) : phân tử

Wane (v) : suy tàn

Foliage (n) : tán lá

Floristic (n) : trồng hoa

Indigenuos (adj) : bản địa

Lush (adj) : tươi tốt


Mar 26 – 2

For better or for worse, race places a very large part in people‘s perception of others in the world
and in their own society. The notion of one‘s own race often influences the actions and interests of
an individual (either towards or away from activities typically associated with a group) and, in many
cases, the perception of another individual‘s race influences the perceiver‘s actions toward that
individual. Races are inaccurate as biological categories. The existence of racism, and the genesis of
our racial taxonomies themselves in the history of colonialism and slavery, argue for abandoning
racial categories altogether.

Few would deny the importance of racial categories in our everydaylives, nor the social problems
and conflict race has caused. Less well known are the scientific problems with race: racial categories
cannot be reconciled with what scientists know about human biological diversity. Biological races are
branches of a species that have been unable to reproduce with each other for a significant period of
time. Their separation may be due to geographic or other barriers, but anatomically, members of
different races can interbreed, since they are of the same species.

Breeds of domesticated dogs are an example of races cultivated by humans. In contrast, human
groups have interbred for our entire history as a species, and none have been isolated long enough
to be considered true races. The American racial classification system is no more scientifically valid
than are other racial taxonomies, local conceptions of race affirmed in other societies or countries.
Racial taxonomies in different countries are not biological races, but rather what anthropologist
Charles Wagley calls “social races."

Racial classification presupposes that people with certain phenotypes share a common recent
ancestry that others do not share. However, physical traits are not a reliable indicator of recent
shared descent. There are no sharp borders between human groups, as there are between so-
called races, because physical traits change gradually. Anatomical features in human populations
represent adaptations to evolutionary forces: skin colour is an adaptation to latitude, facial shape to
climate or altitude, and blood type to endemic diseases. Any particular trait is shared by groups of
people of varied heritages, people who adapted to similar conditions in different parts of the world.
Since different features do not vary together, no assortment of traits can accurately delineate any
group as a true race.

If race were biological, different societies would understand race in similar ways. In fact, societies
use widely varying criteria to determine race. Nor are these criteria all internally consistent. Although
most Americans believe that appearance or genetics form the basis of race, in the United States, a
person's race, legally, is determined by his ancestry, the race of his parents. Further, some state
laws, legacies of slavery, place biracial individuals into the race of the minority parent, without
regard to chromosomes or physical appearance. In Brazil, on the other hand, people do not consider
ancestry when identifying a person's race, and there exist many more racial categories than in the
US. Race in Brazil derives solely from appearance. One's race can change from day to day, and may
differ from the race of one's family, including that of full siblings. The racial taxonomies in Brazil and
the United States differ, but neither one is based on scientific principles.
1. The passage cites Brazil as a country in which racial definitions have a meaning that
might not otherwise be readily apparent. The author suggests that a person who is
Brazilian might change his race by:

A. altering his birth certificate.


B. marrying a person of a different race.
C. having his DNA tested.
D. getting a sun tan.
E. surrendering his passport

2. The overall purpose of this passage is to:

A. present a hypothesis that may explain a recent discovery.


B. compare and contrast two methods of classification.
C. criticize the basis of a popular belief.
D. describe worldwide variations in a cultural phenomenon.
E. praise a widely accepted belief

3. The author of this passage would be most likely to agree with which of the following
statements about abandoning racial classification?

A. We can improve our society through conscious and concerted effort.


B. The United States' racial classification system should be replaced with that of Brazil.
C. It would be disastrous for scientists to strip people of their valued beliefs.
D. All beliefs that are not scientifically sound should be abandoned.
E. The society has reached the point of no return and nothing can be done to change it

4. The author presents the example of racial classification in the United States in the
passage most probably in order to show that:

A. racial taxonomies may be logically inconsistent and widely misunderstood.


B. the system of racial classification in North America is grounded in scientific research.
C. individuals should be allowed to choose and to change their own racial identification.
D. racial classifications are most accurate when they take all factors (appearance , ancestry, and
DNA) into account.
E. there is not enough being done by the authorities to control this problem

Taxonomy (n) : phân loại học

Racial (adj) : chủng tộc


Mar 27 – 1

How does ritual affect relationships between groups and entities external to them? According to
traditional cultural anthropology, aggregates of individuals who regard their collective well-being as
dependent upon a common body of ritual performances use such rituals to give their members
confidence, to dispel their anxieties, and to discipline their social organization. Conventional theories
hold that rituals come into play when people feel they are unable to control events and processes in
their environment that are of crucial importance to them. However, recent studies of the Tsembaga,
a society of nomadic agriculturalists in New Guinea, suggest that rituals do more than just give
symbolic expression to the relationships between a cultural group and components of its
environments; they influence those relationships in measurable ways.

Perhaps the most significant finding of the studies was that , among the Tsembaga, ritual operates
as a regulating mechanism in a system of a set of interlocking systems that include such variables as
the area of available land,necessary length of fallow periods, size of the human and pig populations,
nutritional requirements of pigs and people, energy expended in various activities, and frequency of
misfortune. In one sense, the Tsembaga constitute an ecological population in an ecosystem that
also includes the other living organisms and nonliving substances found within the Tsembaga
territory. By collating measurable data (such as average monthly rainfall, average garden yield,
energy expenditure per cultivated acre, and nutritive values of common foods) with the collective
decision to celebrate certain rituals, anthropologists have been able to show how Tsembaga rituals
allocate energy and important materials. Studies have described how Tsembaga rituals regulate
those relationships among people, their pigs, and their gardens that are critical to survival; control
meat consumption; conserve marsupial fauna; redistribute land among territorial groups; and limit
the frequency of warfare. These studies have important methodological and theoretical implications,
for they enable cultural anthropologists to see that rituals can in fact produce measurable results in
an external world.

By focusing on Tsembaga rituals as part of the interaction within an ecosystem, newer quantitative
studies permit anthropologists to analyze how ritual operates as a mechanism regulating survival. In
the language of sociology, regulation is a “latent function” of Tsembaga ritual, since the Tsembaga
themselves see their rituals as pertaining less to their material relations with the ecosystem than to
their spiritual relations with their ancestors. In the past, cultural anthropologists might have
centered on the Tsembaga’s own interpretations of their rituals in order to elucidate those rituals;
but since tools now exist for examining the adaptive aspects of rituals, these anthropologists are in a
far better position to appreciate fully the ecological sophistication of rituals, both among the
Tsembaga and in other societies.
1. The primary purpose of the passage is to

A) Propose that the complex functions of ritual have been best analyzed when anthropologists and
ecologists have collaborated in order to study human populations as measurable units.
B) Criticize anthropologists’ use of an ecological approach that ignores the symbolic, psychological,
and socially cohesive effects of ritual.
C) Evaluate theories of culture that view ritual as an expression of a society’s understanding of its
relationship to its environment.
D) Point out the ecological sophistication of Tsembaga ritual and suggest the value of
quantitative methods in assessing this sophistication.
E) Argue that the studies showing that the effects of Tsembaga ritual on the environment can be
measured prove that the effects of ritual on other environments can also be measured.

2. On the basis of the information in the passage, one might expect to find all of the
following in the recent anthropological studies of the Tsembaga except

A) An examination of the caloric and nutritive value of the Tsembaga diet.


B) A study of the relationship between the number of Tsembaga rituals and the number of pigs
owned by the Tsembaga.
C) An analysis of the influence of Tsembaga forms of worship on the traditions of
neighboring populations.
D) A catalog of the ways in which Tsembaga rituals influence planting and harvest cycles.
E) A matrix summarizing the seasonality of Tsembaga rituals and the type and function of weapons
made.

3. Which of the following best expresses the author’s view of ritual?

A) Rituals symbolize the relationships between cultural groups and their environments.
B) As a cultural phenomenon, ritual is multifaceted and performs diverse functions.
C) Rituals imbue the events of the material world with spiritual significance.
D) A society’s view of its rituals yields the most useful information concerning the rituals’ functions.
E) The spiritual significance of ritual is deemed greater than the material benefits of ritual.

4. The author of the passage uses the term “latent function” (Highlighted) in order to
suggest that

A) The ability of ritual to regulate the environment is more a matter of study for sociologists than for
anthropologists.
B) Sociological terms describe ritual as precisely as anthropological terms.
C) Anthropologists and sociologists should work together to understand the symbolic or
psychological importance of rituals.
D) Anthropologists are more interested in the regulatory function of rituals of the Tsembaga than
they are the psychological function of rituals.
E) The Tsembaga are primarily interested in the spiritual values that are embodied in
their rituals.
Ritual (n) : nghi lễ

Entity (n) : thực thể

External (n) : bên ngoài

Collective well-being (n) : hạnh phúc tập thể

Dispel (v) : xua tan

Nomadic (adj) : du mục

Agriculturalist (n) : nhà nông

Fallow (adj) : bỏ hoang

Misfortune (n) : bất hạnh

Ecological (adj) : sinh thái

Constitute (v) : cấu tạo

Collate (v) : đối chiếu

Conserve (v) : bảo tồn

Marsupial (adj) : thú có túi

Fauna (n) : hệ động vật

Methodological (adj) : phương pháp luận

Regulation (n) : quy định

Latent (adj) : ngầm

Pertain (v) : liên quan

Elucidate (v) : làm rõ


Mar 27 – 2

Although the legal systems of England and the United States are superficially similar, they differ
profoundly in their approaches to and uses of legal reasons: substantive reasons in the United
States, whereas in England the reverse is true. This distinction reflects a difference in the visions of
law that prevail in the two counties. In England the law has traditionally been viewed as a system of
rules; the United States favors a vision of law as an outward expression of the community’s sense of
right and justice.

Substantive reasons, as applied to law, are based on moral, economic, political, and other
considerations. These reasons are found both “in the law” and “outside the law,” so to speak.
Substantive reasons inform (to give character or essence to “the principles which inform modern
teaching”) the content of a large part of the law: constitutions, statutes, contracts, verdicts, and the
like. Consider, for example, a statute providing (to make a proviso or stipulation) that “no vehicles
shall be taken into public parks”. Suppose that no specific rationales or purposes were explicitly
written into this statute, but that it was clear (from its legislative history) that the substantive
purpose of the statute was to ensure quiet and safety in the park. Now suppose that a veterans’
group mounts a World War II jeep (in running order but without a battery) as a war memorial on a
concrete slab in the park, and charges are brought against its members. Most judges in the United
States would find the defendants not guilty because what they did had no adverse effect on park
quiet and safety.

Formal reasons are different in that they frequently prevent substantive reasons from coming into
play, even when substantive reasons are explicitly incorporated into the law at hand. For example,
when a document fails to comply with stipulated requirements, the court may render the document
legally ineffective. A will requiring written witness may be declared null and void and therefore,
unenforceable for the formal reason that the requirement was not observed. Once the legal rule—
that a will is invalid for lack of proper witnessing—has been clearly established, and the legality of
the rule is not in question, application of that rule precludes from consideration substantive
arguments in favor of the will’s validity or enforcement.

Legal scholars in England and the United States have long bemused themselves with extreme
examples of formal and substantive reasoning. On the one hand, formal reasoning in England has
led to wooden interpretations of statutes and an unwillingness to develop the common law through
judicial activism. On the other hand, freewheeling substantive reasoning in the United States has
resulted in statutory interpretations so liberal that the texts of some statutes have been ignored
altogether.

1. Which one of the following best describes the content of the passage as a whole?

(A) an analysis of similarities and differences between the legal systems of England and the United
States
(B) a reevaluation of two legal systems with the use of examples
(C) a contrast between the types of reasons embodied in the United States and England
legal systems
(D) an explanation of how two distinct visions of the law shaped the development of legal reasoning
(E) a presentation of two types of legal reasons that shows the characteristics they have in common
2. It can be inferred from the passage that English judges would be likely to find the
veterans’ group discussed in the second paragraph guilty of violating the statute
because

(A) not to do so would encourage others to act as the group did


(B) not to do so would be to violate the substantive reasons underlying the law
(C) the veterans failed to comply with the substantive purpose of the statute
(D) the veterans failed to demonstrate that their activities had no adverse effect on the public
(E) the veterans failed to comply with the stipulated requirements of the statute

3. From the discussion of wills in the third paragraph it can be inferred that substantive
arguments as to the validity of a will might be considered under which one of the
following circumstances?

(A) The legal rule requiring that a will be witnessed in writing does not stipulate the format of the
will.
(B) The legal rule requiring that a will be witnessed stipulates that the will must be witnessed in
writing by two people.
(C) The legal rule requiring that a will be witnessed in writing stipulates that the witnessing must be
done in the presence of a judge.
(D) A judge rules that the law requires a will to be witnessed in writing regardless of extenuating
circumstances
(E) A judge rules that the law can be interpreted to allow for a verbal witness to a will in
a case involving a medical emergency.

4. The author of the passage makes use of all of the following in presenting the
discussion of the English and the United States legal systems EXCEPT

(A) comparison and contrast


(B) generalization
(C) explication of term
(D) a chronology of historical developments
(E) a hypothetical case

5. Which one of the following best describes the function of the last paragraph of the
passage?

(A) It presents the consequences of extreme interpretations of the two types of legal
reasons discussed by the author.
(B) It shows how legal scholars can incorrectly use extreme examples to support their views.
(C) It corrects inaccuracies in legal scholars’ views of the nature of the two types of legal systems.
(D) It suggests how characterizations of the two types of legal reasons can become convoluted and
inaccurate.
(E) It presents scholars’ characterizations of both legal systems that are only partially correct
6. The author of the passage suggests that in English law a substantive interpretation of
a legal rule might be warranted under which one of the following circumstances?

(A) Social conditions have changed to the extent that to continue to enforce the rule would be to
decide contrary to present-day social norms.
(B) The composition of the legislature has changed to the extent that to enforce the rule would be
contrary to the views of the majority in the present legislative assembly.
(C) The legality of the rule is in question and its enforcement is open to judicial
interpretation.
(D) Individuals who have violated the legal rule argue that application of the rule would lead to
unfair judicial interpretations.
(E) Superior court judges have consistently ruled in decisions regarding the interpretation of the
legal rule

7. According to the passage, which one of the following statements about substantive
reasons is true?

(A) They may be written into laws, but they may also exert an external influence on the
law.
(B) They must be explicitly written into the law in order to be relevant to the application of the law.
(C) They are legal in nature and determine particular applications of most laws.
(D) They often provide judges with specific rationales for disregarding the laws of the land.
(E) They are peripheral to the law, whereas formal reasons are central to the law.

Profoundly (adv) : sâu sắc

Substantive (adj) : thực chất

Prevail (v) : chiếm ưu thế

Outward (adj) : bề ngoài

Essence (n) : bản chất

Statute (n) : đạo luật, điều lệ

Proviso (n) : điều kiện

Stipulation (n) : sự quy định

Explicitly (adv) : rõ ràng

Rationale (n) : lý luận

Slab (n) : phiến

Mount (v) : trèo lên

Incorporate (v) : hợp nhất

Comply (v) : tuân theo

Null and void (adj) : vô hiệu


Preclude (v) : ngăn chặn

Bemuse (v) : lưỡng lự

Wooden (adj) : vụng về

Judicial (adj) : tư pháp

Freewheeling (adj) : tự do
Mar 28 – 1

Before Laura Gilpin (1891-1979), few women in the history of photography had so devoted
themselves to chronicling the landscape. Other women had photographed the land, but none can be
regarded as a landscape photographer with a sustained body of work documenting the physical
terrain. Anne Brigman often photographed woodlands and coastal areas, but they were generally
settings for her artfully placed subjects. Dorothea Lange’s landscapes were always conceived of as
counterparts to her portraits of rural women.

At the same time that Gilpin’s interest in landscape work distinguished her from most other women
photographers, her approach to landscape photography set her apart from men photographers who,
like Gilpin, documented the western United States. Western American landscape photography grew
out of a male tradition, pioneered by photographers attached to government and commercial
survey teams that went west in the 1860’s and 1870’s. These explorer-photographers documented
the West that their employers wanted to see: an exotic and majestic land shaped by awesome
natural forces, unpopulated and ready for American settlement. The next generation of male
photographers, represented by Ansel Adams and Eliot Porter, often worked with conservationist
groups rather than government agencies or commercial companies, but they nonetheless preserved
the “heroic” style and maintained the role of respectful outsider peering in with reverence at a
fragile natural world.

For Gilpin, by contrast, the landscape was neither an empty vista awaiting human settlement nor a
jewel-like scene resisting human intrusion, but a peopled landscape with a rich history and tradition
of its own, an environment that shaped and molded the lives of its inhabitants. Her photographs of
the Rio Grande, for example, consistently depict the river in terms of its significance to human
culture: as a source of irrigation water, a source of food for livestock, and a provider of town sites.
Also instructive is Gilpin’s general avoidance of extreme close-ups of her natural subjects: for her,
emblematic details could never suggest the intricacies of the interrelationship between people and
nature that made the landscape a compelling subject. While it is dangerous to draw conclusions
about a “feminine” way of seeing from the work of one woman, it can nonetheless be argued that
Gilpin’s unique approach to landscape photography was analogous to the work of many women
writers who, far more than their male counterparts, described the landscape in terms of its potential
to sustain human life.

Gilpin never spoke of herself as a photographer with a feminine perspective: she eschewed any
discussion of gender as it related to her work and maintained little interest in interpretations that
relied on the concept of a “woman’s eye.” Thus it is ironic that her photographic evocation of a
historical landscape should so clearly present a distinctively feminine approach to landscape
photography.
1. Which of the following best expresses the main idea of the passage?

(A) Gilpin’s landscape photographs more accurately documented the Southwest than did the
photographs of explorers and conservationists.
(B) Gilpin’s style of landscape photography substantially influenced the heroic style practiced by her
male counterparts.
(C) The labeling of Gilpin’s style of landscape photography as feminine ignores important ties
between it and the heroic style.
(D) Gilpin’s work exemplifies an arguably feminine style of landscape photography that
contrasts with the style used by her male predecessors.
(E) Gilpin’s style was strongly influenced by the work of women writers who described the landscape
in terms of its relationship to people.

2. It can be inferred from the passage that the teams (Highlighted) were most interested
in which of the following aspects of the land in the western United States?

(A) Its fragility in the face of increased human intrusion


(B) Its role in shaping the lives of indigenous peoples
(C) Its potential for sustaining future settlements
(D) Its importance as an environment for rare plants and animals
(E) Its unusual vulnerability to extreme natural forces

3. The author of the passage claims that which of the following is the primary reason
why Gilpin generally avoided extreme close-ups of natural subjects?

(A) Gilpin believed that pictures of natural details could not depict the interrelationship
between the land and humans.
(B) Gilpin considered close-up photography to be too closely associated with her predecessors.
(C) Gilpin believed that all of her photographs should include people in them.
(D) Gilpin associated close-up techniques with photography used for commercial purposes.
(E) Gilpin feared that pictures of small details would suggest an indifference to the fragility of the
land as a whole.

4. The passage suggests that a photographer who practiced the heroic style would be
most likely to emphasize which of the following in a photographic series focusing on the
Rio Grande?

(A) Indigenous people and their ancient customs relating to the river
(B) The exploits of navigators and explorers
(C) Unpopulated, pristine parts of the river and its surroundings
(D) Existing commercial ventures that relied heavily on the river
(E) The dams and other monumental engineering structures built on the river
5. It can be inferred from the passage that the first two generations of landscape
photographers in the western United States had which of the following in common?

(A) They photographed the land as an entity that had little interaction with human
culture.
(B) They advanced the philosophy that photographers should resist alliances with political or
commercial groups.
(C) They were convinced that the pristine condition of the land needed to be preserved by
government action.
(D) They photographed the land as a place ready for increased settlement.
(E) They photographed only those locations where humans had settled.

6. Based on the description of her works in the passage, which of the following would
most likely be a subject for a photograph taken by Gilpin?

(A) A vista of a canyon still untouched by human culture


(B) A portrait of a visitor to the West against a desert backdrop
(C) A view of historic Native American dwellings carved into the side of a natural cliff
(D) A picture of artifacts from the West being transported to the eastern United States for retail sale
(E) An abstract pattern created by the shadows of clouds on the desert

Her photographs of the Rio Grande, for example, consistently depict the river in terms of its
significance to human culture: as a source of irrigation water, a source of food for livestock, and a
provider of town sites.

7. The author of the passage mentions women writers in line 50 most likely in order to

(A) counter a widely held criticism of her argument


(B) bolster her argument that Gilpin’s style can be characterized as a feminine style
(C) suggest that Gilpin took some of her ideas for photographs from landscape descriptions by
women writers
(D) clarify the interrelationship between human culture and the land that Gilpin was attempting to
capture
(E) offer an analogy between photographic close-ups and literary descriptions of small details

Pristine (adj) : nguyên sơ

Intrusion (n) : xâm nhập

Jewel-like scene (n) : cảnh như trang sức quý

Resist (v) : chống cự

Dwelling (n) : nhà ở, trú ngụ

Eschew (v) : tránh

Interpretation (n) : diễn giải

Ironic (adj) : mỉa mai

Distinctively (adv) : đặc biệt


Mar 28 – 2

Historians have long accepted the notion that women of English descent who lived in the English
colonies of North America during the seventeenth and eighteenth centuries were better off than
either the contemporary women in England or the colonists’ own nineteenth-century daughters and
granddaughters. The “golden age” theory originated in the 1920s with the work of Elizabeth Dexter,
who argued that there were relatively few women among the colonists, and that all hands—male
and female—were needed to sustain the growing settlements. Rigid sex-role distinctions could not
exist under such circumstances; female colonists could accordingly engage in whatever occupations
they wished, encountering few legal or social constraints if they sought employment outside the
home. The surplus of male colonists also gave women crucial bargaining power in the marriage
market since women’s contributions were vital to the survival of colonial households.

Dexter’s portrait of female colonists living under conditions of rough equality with their male
counterparts was eventually incorporated into studies of nineteenth-century middle-class women.
The contrast between the self-sufficient colonial woman and the oppressed nineteenth-century
woman, confined to her home by stultifying ideologies of domesticity and by the fact that
industrialization eliminated employment opportunities for middle-class women, gained an
extraordinarily tenacious hold on historians. Even scholars who have questioned the “golden age”
view of colonial women’s status have continued to accept the paradigm of a nineteenth-century
decline from a more desirable past. For example, Joan Hoff-Wilson asserted that there was no
“golden age” and yet emphasized that the nineteenth century brought “increased loss of function
and authentic status for” middle-class women.

Recent publications about colonial women have exposed the concept of a decline in status as
simplistic and unsophisticated, a theory that based its assessment of colonial women’s status solely
on one factor (their economic function in society) and assumed all too readily that a relatively simple
social system automatically brought higher standing to colonial women. The new scholarship
presents a far more complicated picture, one in which definitions of gender roles, the colonial
economy, demographic patterns, religion, the law, and household organization all contributed to
defining the circumstances of colonial women’s lives. Indeed, the primary concern of modern
scholarship is not to generalize about women’s status but to identify the specific changes and
continuities in women’s lives during the colonial period. For example, whereas earlier historians
suggested that there was little change for colonial women before 1800, the new scholarship
suggests that a three-part chronological division more accurately reflects colonial women’s
experiences. First was the initial period of English colonization (from the 1620s to about 1660); then
a period during which patterns of family and community were challenged and reshaped (roughly
from 1660 to 1750); and finally the era of revolution (approximately 1750 to 1815), which brought
other changes to women’s lives.
1. Which one of the following best expresses the main idea of the passage?

(A) An earlier theory about the status of middle-class women in the nineteenth century has been
supported by recent scholarship.
(B) Recent studies of middle-class nineteenth-century women have altered an earlier theory about
the status of colonial women.
(C) Recent scholarship has exposed an earlier theory about the status of colonial women
as too narrowly based and oversimplified.
(D) An earlier theory about colonial women has greatly influenced recent studies on middle-class
women in the nineteenth century.
(E) An earlier study of middle-class women was based on insufficient research on the status of
women in the nineteenth century.

2. The author discusses Hoff-Wilson (Highlighted) primarily in order to

(A) describe how Dexter’s theory was refuted by historians of nineteenth-century North America
(B) describe how the theory of middle-class women’s nineteenth-century decline in status was
developed
(C) describe an important influence on recent scholarship about the colonial period
(D) demonstrate the persistent influence of the “golden age” theory
(E) provide an example of current research one the colonial period

3. It can be inferred from the passage that the author would be most likely to describe
the views of the scholars (Highlighted) as

(A) unassailable
(B) innovative
(C) paradoxical
(D) overly sophisticated
(E) without merit

4. It can be inferred from the passage that in proposing the “three-part chronological
division” (Highlighted), scholars recognized which one of the following?

(A) The circumstances of colonial women’s lives were defined by a broad variety of
social and economic factors.
(B) Women’s lives in the English colonies of North America were similar to women’s lives in
seventeenth-and eighteenth-century England.
(C) Colonial women’s status was adversely affected when patterns of family and community were
established in the late seventeenth century.
(D) Colonial women’s status should be assessed primarily on the basis of their economic function in
society.
(E) Colonial women’s status was low when the colonies were settled but changed significantly during
the era of revolution.
5. According to the author, the publications about colonial women mentioned in the
third paragraph had which one of the following effects?

(A) They undermined Dexter’s argument on the status of women colonists during the
colonial period.
(B) They revealed the tenacity of the “golden age” theory in American history.
(C) They provided support for historians, such as Hoff-Wilson. Who study the nineteenth century.
(D) They established that women’s status did not change significantly from the colonial period to the
nineteenth century.
(E) They provided support for earlier theories about women colonists in the English colonies of North
America.

6. Practitioners of the new scholarship discussed in the last paragraph would be most
likely to agree with which one of the following statements about Dexter’s argument?

(A) It makes the assumption that women’s status is determined primarily by their political power in
society.
(B) It makes the assumption that a less complex social system necessarily confers
higher status on women.
(C) It is based on inadequate research on women’s economic role in the colonies.
(D) It places too much emphasis on the way definitions of gender roles affected women colonists in
the colonial period.
(E) It accurately describes the way women’s status declined in the nineteenth century.

Recent publications about colonial women have exposed the concept of a decline in status as
simplistic and unsophisticated, a theory that based its assessment of colonial women’s status solely
on one factor (their economic function in society) and assumed all too readily that a relatively
simple social system automatically brought higher standing to colonial women.

Paradoxical (adj) : nghịch lý, trái nghịch

Contemporary (adj) : đồng thời

Constraint (n) : hạn chế

Unassailable (adj) : không thể công kích


Mar 29 – 1

Outside the medical profession, there are various efforts to cut medicine down to size: not only
widespread malpractice litigation and massive governmental regulation, but also attempts by
consumer groups and others to redefine medicine as a trade rather than as a profession, and the
physician as merely a technician for hire under contract. Why should physicians (or indeed all
sensible people) resist such efforts to give the practice of medicine a new meaning? We can gain
some illumination from etymology. “Trade,” from Germanic and AngloSaxon roots meaning “a
course or pathway,” has come to mean derivatively a habitual occupation and has been related to
certain skills and crafts. On the other hand, while “profession” today also entails a habit of work, the
word “profession” itself traces to an act of selfconscious and public—even confessional—speech. “To
profess”preserves the meaning of its Latin source, “to declare publicly; to announce, affirm, avow.”
A profession is an activity or occupation to which its practitioner publicly professes, that is,
confesses, devotion. But public announcement seems insufficient; publicly declaring devotion to
plumbing or auto repair would not turn these trades into professions.

Some believe that learning and knowledge are the diagnostic signs of a profession. For reasons
probably linked to the medieval university, the term “profession” has been applied to the so-called
learned professions—medicine, law, and theology—the practices of which are founded upon inquiry
and knowledge rather than mere “knowhow.” Yet it is not only the pursuit and acquisition of
knowledge that makes one a professional. The knowledge involved makes the profession one of the
learned variety, but its professional quality is rooted in something else.

Some mistakenly seek to locate that something else in the prestige and honor accorded
professionals by society, evidenced in their special titles and the special deference and privileges
they receive. But externalities do not constitute medicine a profession. Physicians are not
professionals because they are honored; rather, they are honored because of their profession. Their
titles and the respect they are shown superficially signify and acknowledge something deeper, that
physicians are persons of the professional sort, knowingly and freely devoting themselves to a way
of life worthy of such devotion. Just as lawyers devote themselves to rectifying injustices, looking up
to what is lawful and right; just as teachers devote themselves to the education of the young,
looking up to truth and wisdom; so physicians heal the sick, looking up to health and
wholesomeness. Being a professional is thus rooted in our moral nature and in that which warrants
and impels making a public confession to a way of life.

Professing oneself a professional is an ethical act because it is not a silent and private act, but an
articulated and public one; because it promises continuing devotion to a way of life, not merely
announces a present preference or a way to a livelihood; because it is an activity in service to some
high good that insists on devotion; because it is difficult and demanding. A profession engages one’s
character and heart, not merely one’s mind and hands.
1. According to the author, which one of the following is required in order that one be a
professional?

(A) significant prestige and a title


(B) “know-how” in a particular field
(C) a long and difficult educational endeavor
(D) a commitment to political justice
(E) a public confession of devotion to a way of life

2. Which one of the following best expresses the main point made by the author in the
passage?

(A) Medicine is defined as a profession because of the etymology of the word “profession.”
(B) It is a mistake to pay special honor to the knowledge and skills of physicians.
(C) The work of physicians is under attack only because it is widely misunderstood.
(D) The correct reason that physicians are professionals is that their work involves
public commitment to a high good.
(E) Physicians have been encouraged to think of themselves as technicians and need to reorient
themselves toward ethical concerns.

3. The question posed by the author in first para (Highlighted) of the passage introduces
which one of the following?

(A) the author’s belief that it is futile to resist the trend toward defining the physician’s work as a
trade
(B) the author’s dislike of governmental regulation and consumer advocacy
(C) the author’s inquiry into the nature of the practice of medicine
(D) the author’s suggestions for rallying sensible people to a concentrated defense of physicians
(E) the author’s fascination with the origins of words

4. In the passage, the author mentions or suggests all of the following EXCEPT

(A) how society generally treats physicians


(B) that the practice of medicine is analogous to teaching
(C) that being a professional is in part a public act
(D) the specific knowledge on which trades are based
(E) how a livelihood is different from a profession

5. The author’s attitude towards professionals is best described as

(A) eager that the work of one group of professionals, physicians, be viewed from a new perspective
(B) sympathetic toward professionals who have become demoralized by public opinion
(C) surprised that professionals have been balked by governmental regulations and threats of
litigation
(D) dismayed that most professionals have come to be considered technicians
(E) certain that professionals confess a commitment to ethical ideals
6. Based on the information in the passage, it can be inferred that which one of the
following would most logically begin a paragraph immediately following the passage?

(A) A skilled handicraft is a manual art acquired by habituation that enables tradespeople to tread
regularly and reliably along the same path.
(B) Critics might argue that being a doctor, for example, requires no ethical or public
act; thus medicine, as such, is morally neutral, does not bind character, and can be used
for good or ill.
(C) Sometimes the pursuit of personal health competes with the pursuit of other goods, and it has
always been the task of the community to order and define the competing ends.
(D) Not least among the myriad confusions and uncertainties of our time are those attending efforts
to discern and articulate the essential characteristics of the medical profession.
(E) When, in contrast, we come to physicians of the whole body, we come tacitly acknowledging the
meaning of illness and its potential threat to all that we hold dear.

7. Which one of the following best describes the author’s purpose in lines 14–33 of the
passage?
“To profess” ..........medicine a profession.

(A) The author locates the “something else” that truly constitutes a profession.
(B) The author dismisses efforts to redefine the meaning of the term “profession.”
(C) The author considers, and largely criticizes, several definitions of what constitutes a
profession.
(D) The author clarifies the meaning of the term “profession” by advocating a return to its linguistic
and historical roots.
(E) The author distinguishes trades such as plumbing and auto repair from professions such as
medicine, law, and theology.

Profession (n) : nghề nghiệp

Litigation (n) : kiện tụng

Malpractice (n) : sơ suất

Physician (n) : bác sĩ, thầy thuốc

Illumination (n) : giải thích

Etymology (n) : ngữ nguyên học

Derivatively (adv) : phái sinh

Craft (n) : đồ thủ công

Confessional (adj) : thú tội

Avow (v) : chấp nhận, thú nhận

Diagnostic (adj) : chuẩn đoán

Inquiry (n) : điều tra

Privilege (n) : đặc quyền


Externality (n) : ngoại cảnh

Signify (v) : biểu thị, tuyên bố

Superficially (adv) : hời hợt

Impel (v) : thúc đẩy

Articulate (v) : khớp nối

Livelihood (n) : sinh kế


Mar 29 – 2

Until recently most astronomers believed that the space between the galaxies in our universe was a
near-perfect vacuum. This orthodox view of the universe is now being challenged by astronomers
who believe that a heavy “rain” of gas is falling into many galaxies from the supposedly empty space
around them. The gas apparently condenses into a collection of small stars, each a little larger than
the planet Jupiter. These stars vastly outnumber the other stars in a given galaxy. The amount of
“intergalactic rainfall” into some of these galaxies has been enough to double their mass in the time
since they formed. Scientists have begun to suspect that this intergalactic gas is probably a mixture
of gases left over from the “big bang” when the galaxies were formed and gas was forced out of
galaxies by supernova explosions.

It is well known that when gas is cooled at a constant pressure its volume decreases. Thus, the
physicist Fabian reasoned that as intergalactic gas cools, the cooler gas shrinks inward toward the
center of the galaxy. Meanwhile its place is taken by hotter intergalactic gas from farther out on the
edge of the galaxy, which cools as it is compressed and flows into the galaxy. The net result is a
continuous flow of gas, starting as hot gases in intergalactic space and ending as a drizzle of cool
gas called a “cooling flow,” falling into the central galaxy.

A fairly heretical idea in the 1970’s, the cooling-flow theory gained support when Fabian observed a
cluster of galaxies in the constellation Perseus and found the central galaxy, NGC 1275, to be a
strange-looking object with irregular, thin strands of gas radiating from it. According to previous
speculation, these strands were gases that had been blown out by an explosion in the galaxy.
Fabian, however, disagreed. Because the strands of gas radiating from NGC 1275 are visible in
optical photographs, Fabian suggested that such strands consisted not of gas blown out of the
galaxy but of cooling flows of gas streaming inward. He noted that the wavelengths of the radiation
emitted by a gas would changes as the gas cooled, so that as the gas flowed into the galaxy and
became cooler, it would emit not x-rays, but visible light, like that which was captured in the
photographs. Fabian’s hypothesis was supported by Canizares’ determination in 1982 that most of
the gas in the Perseus cluster was at a temperature of 80 million degrees Kelvin, whereas the gas
immediately surrounding NGC 1275 (the subject of the photographs) was at one-tenth this
temperature.

1. The primary purpose of the passage is to

(A) illustrate a hypothesis about the origin of galaxies


(B) provide evidence to dispute an accepted theory about the evolution of galaxies
(C) summarize the state of and prospects for research in intergalactic astronomy
(D) report new data on the origins of intergalactic gas
(E) reconcile opposing views on the formation of intergalactic gas
2. The author uses the phrase “orthodox view of the universe” (Highlighted) to refer to
the belief that

(A) the space between the galaxies is devoid of matter


(B) the space between galaxies is occupied by stars that cannot be detected by optical photographs
(C) galaxies have decreased in mass by half since their formation
(D) galaxies contain stars, each the size of Jupiter, which form clusters
(E) galaxies are being penetrated by gas forced out of other galaxies by supernova explosions

3. It can be inferred from the passage that, if Fabian is correct, gas in the peripheral
regions of a galaxy cluster

(A) streams outward into intergalactic space


(B) is hotter than gas in the central regions of the galaxy
(C) is composed primarily of gas left over from the big bang
(D) results in the creation of unusually large stars
(E) expands to increase the size of the galaxy

Meanwhile its place is taken by hotter intergalactic gas from farther out on the edge of
the galaxy, which cools as it is compressed and flows into the galaxy. The net result is a
continuous flow of gas, starting as hot gases in intergalactic space and ending as a drizzle of cool
gas called a “cooling flow,” falling into the central galaxy.

4. The author of the passage probably mentions Canizares’ determination in order to

(A) clarify an ambiguity in Fabian’s research findings


(B) illustrate a generalization about the temperature of gas in a galaxy cluster
(C) introduce a new argument in support of the orthodox view of galaxies
(D) provide support for Fabian’s assertions about the Perseus galaxies
(E) provide an alternate point of view concerning the movement of gas within a galaxy cluster

5. According to the passage, Fabian believes that gas flowing into a central galaxy has
which of the following characteristics?

(A) It is one-tenth hotter than it was in the outer regions of the galaxy cluster.
(B) It emits radiation with wavelengths that change as the gas moves toward the center
of the galaxy.
(C) The total amount of radiation emitted diminishes as the gas cools.
(D) It loses 90 percent of its energy as it moves to the center of the galaxy.
(E) It condenses at a rate much slower than the rate of decrease in temperature as the gas flows
inward.

6. According to the passage, Fabian’s theory makes use of which of the following
principles?

(A) Gas emanating from an explosion will be hotter the more distant it is from the origin.
(B) The wavelength of radiation emitted by a gas as it cools remains constant.
(C) If pressure remains constant, the volume of a gas will decrease as it is cooled.
(D) The volume of a gas will increase as the pressure increases.
(E) As gas cools, its density decreases.

It is well known that when gas is cooled at a constant pressure its volume decreases

7. It can be inferred from the passage that which of the following is true of Fabian’s
theory?

(A) It did not receive approval until Canizares’ work was published.
(B) It was not widely accepted in the 1970’s.
(C) It did not receive support initially because technology was not available to confirm its tenets.
(D) It supports earlier speculation that intergalactic gas was largely the result of explosions outside
the galaxy.
(E) It was widely challenged until x-ray evidence of gas temperatures in NGC 1275 had been
presented.

Vacuum (n) : khoảng chân không

Orthodox (adj) : chính thống

Condense (v) : ngưng tụ

Supernova (n) : siêu tân tinh

Heretical (adj) : dị giáo

Cluster (n) : cụm, thành bầy

Constellation (n) : chòm sao

Strand (n) : sợi

Radiate (v) : tỏa ra

Radiation (n) : sự bức xạ

Peripheral (adj) : ngoại vi, chu vi, ngoài biên

Devoid (adj) : không có

Drizzle (n) : mưa phùn


Mar 30 – 1

Amsden has divided Navajo weaving into four distinct styles. He argues that three of them can be
identified by the type of design used to form horizontal bands: colored strips, zigzags, or diamonds.
The fourth, or bordered, style he identifies by a distinct border surrounding centrally placed,
dominating figures.

Amsden believes that the diamond style appeared after 1869 when, under Anglo influence and
encouragement, the blanket became a rug with larger designs and bolder lines. The bordered style
appeared about 1890, and, Amsden argues, it reflects the greatest number of Anglo influences on
the newly emerging rug business. The Anglo desire that anything with a graphic designs have a top,
bottom, and border is a cultural preference that the Navajo abhorred, as evidenced, he suggests, by
the fact that in early bordered specimens strips of color unexpectedly break through the enclosing
pattern.

Amsden argues that the bordered rug represents a radical break with previous styles. He asserts
that the border changed the artistic problem facing weavers: a blank area suggests the use of
isolated figures, while traditional, banded Navajo designs were continuous and did not use isolated
figures. The old patterns alternated horizontal decorative zones in a regular order.

Amsden’s view raises several questions. First, what is involved in altering artistic styles? Some
studies suggest that artisans’ motor habits and thought processes must be revised when a style
changes precipitously. In the evolution of Navajo weaving, however, no radical revisions in the way
articles are produced need be assumed. After all, all weaving subordinates design to the physical
limitations created by the process of weaving, which includes creating an edge or border. The habits
required to make decorative borders are, therefore, latent and easily brought to the surface.

Second, is the relationship between the banded and bordered styles as simple as Amsden suggests?
He assumes that a break in style is a break in psychology. But if style results from constant quests
for invention, such stylistic breaks are inevitable. When a style has exhausted the possibilities
inherent in its principles, artists cast about for new, but not necessarily alien, principles. Navajo
weaving may have reached this turning point prior to 1890.

Third, is there really a significant stylistic gap? Two other styles lie between the banded styles and
the bordered styles. They suggest that disintegration of the bands may have altered visual and
motor habits and prepared the way for a border filled with separate units. In the Chief White
Antelope blanket, dated prior to 1865, ten years before the first Anglo trading post on the Navajo
reservation, whole and partial diamonds interrupt the flowing design and become separate forms.
Parts of diamonds arranged vertically at each side may be seen to anticipate the border.

1. The author’s central thesis is that

(A) the Navajo rejected the stylistic influences of Anglo culture


(B) Navajo weaving cannot be classified by Amsden’s categories
(C) the Navajo changed their style of weaving because they sought the challenge of new artistic
problems
(D) original motor habits and thought processes limit the extent to which a style can be revised
(E) the casual factors leading to the emergence of the bordered style are not as clear-
cut as Amsden suggests

2. It can be inferred from the passage that Amsden views the use of “strips of color”
(Highlighted) in the early bordered style as

(A) a sign of resistance to a change in style


(B) an echo of the diamond style
(C) a feature derived from Anglo culture
(D) an attempt to disintegrate the rigid form of the banded style
(E) a means of differentiating the top of the weaving from the bottom

3. The author’s view of Navajo weaving suggests which one of the following?

(A) The appearance of the first trading post on the Navajo reservation coincided with the
appearance of the diamond style.
(B) Traces of thought processes and motor habits of one culture can generally be found in the art of
another culture occupying the same period and region.
(C) The bordered style may have developed gradually from the banded style as a result
of Navajo experiencing with design.
(D) The influence of Anglo culture was not the only non-Native American influence on Navajo
weaving.
(E) Horizontal and vertical rows of diamond forms were transformed by the Navajos into solid lines
to create the bordered style.

4. According to the passage, Navajo weavings made prior to 1890 typically were
characterized by all of the following EXCEPT

(A) repetition of forms


(B) overall patterns
(C) horizontal bands
(D) isolated figures
(E) use of color

5. The author would most probably agree with which one of the following conclusions
about the stylistic development of Navajo weaving?

(A) The styles of Navajo weaving changed in response to changes in Navajo motor habits and
thought processes.
(B) The zigzag style was the result of stylistic influences from Anglo culture.
(C) Navajo weaving used isolated figures in the beginning, but combined naturalistic and abstract
designs in later styles.
(D) Navajo weaving changed gradually from a style in which the entire surface was
covered by horizontal bands to one in which central figures dominated the surface.
(E) The styles of Navajo weaving always contained some type of isolated figure.

6. The author suggests that Amsden’s claim that borders in Navajo weaving were
inspired by Anglo culture could be
(A) conceived as a response to imagined correspondences between Anglo and Navajo art
(B) biased by Amsden’s feelings about Anglo culture
(C) a result of Amsden’s failing to take into account certain aspects of Navajo weaving
(D) based on a limited number of specimens of the styles of Navajo weaving
(E) based on a confusion between the stylistic features of the zigzag and diamond styles

7. The author most probably mentions the Chief White Antelope blanket in order to

(A) establish the credit influence of Anglo culture on the bordered style
(B) cast doubts on the claim that the bordered style arose primarily from Anglo
influence
(C) cite an example of a blanket with a central design and no border
(D) suggest that the Anglo influence produced significant changes in the two earliest styles of
Navajo weaving
(E) illustrate how the Navajo had exhausted the stylistic possibilities of the diamond style

8. The passage is primarily concerned with

(A) comparing and contrasting different styles


(B) questioning a view of how a style came into being
(C) proposing alternative methods of investigating the evolution of styles
(D) discussing the influence of one culture on another
(E) analyzing the effect of the interaction between two different cultures

Weaving (n) : cách dệt

Figure (n) : dung mạo, hình dáng, nhân vật

Rug (n) : tấm thảm, mền

Abhor (v) : không thích làm

Speciment (n) : mẫu vật

Alternate (v) : xen kẽ

Latent (adj) : kín đáo

Inevitable (adj) : chắc chắn xảy ra

Alien (adj) : khác với


Mar 30 – 2

Historians attempting to explain how scientific work was done in the laboratory of the seventeenth-
century chemist and natural philosopher Robert Boyle must address a fundamental discrepancy
between how such experimentation was actually performed and the seventeenth-century
rhetoric describing it. Leaders of the new Royal Society of London in the 1660s insisted that
authentic science depended upon actual experiments performed, observed, and recorded by the
scientists themselves. Rejecting the traditional contempt for manual operations, these scientists, all
members of the English upper class, were not to think themselves demeaned by the mucking about
with chemicals, furnaces, and pumps; rather, the willingness of each of them to become, as Boyle
himself said, a mere “drudge” and “under-builder” in the search for God’s truth in nature was taken
as a sign of their nobility and Christian piety.

This rhetoric has been so effective that one modern historian assures us that Boyle himself actually
performed all of the thousand or more experiments he reported. In fact, due to poor eyesight,
fragile health, and frequent absences from his laboratory, Boyle turned over much of the labor of
obtaining and recording experimental results to paid technicians, although published accounts of the
experiments rarely, if ever, acknowledged the technicians’ contributions. Nor was Boyle unique in
relying on technicians without publicly crediting their work.

Why were the contributions of these technicians not recognized by their employers? One reason is
the historical tendency, which has persisted into the twentieth century, to view scientific discovery
as resulting from momentary flashes of individual insight rather than from extended periods of
cooperative work by individuals with varying levels of knowledge and skill. Moreover, despite the
clamor of seventeenth-century scientific rhetoric commending a hands-on approach, science was still
overwhelmingly an activity of the English upper class, and the traditional contempt that genteel
society maintained for manual labor was pervasive and deeply rooted. Finally, all of Boyle’s
technicians were “servants,” which in seventeenth-century usage meant anyone who worked for
pay. To seventeenth-century sensibilities, the wage relationship was charged with political
significance. Servants, meaning wage earners, were excluded from the franchise because they were
perceived as ultimately dependent on their wages and thus controlled by the will of their employers.
Technicians remained invisible in the political economy of science for the same reasons that
underlay servants’ general political exclusion. The technicians’ contribution, their observations and
judgment, if acknowledged, would not have been perceived in the larger scientific community as
objective because the technicians were dependent on the wages paid to them by their employers.
Servants might have made the apparatus work, but their contributions to the making of scientific
knowledge were largely—and conveniently—ignored by their employers.

1. Which one of the following best summarizes the main idea of the passage?

(A) Seventeenth-century scientific experimentation would have been impossible without the work of
paid laboratory technicians.
(B) Seventeenth-century social conventions prohibited upper-class laboratory workers from taking
public credit for their work.
(C) Seventeenth-century views of scientific discovery combined with social class
distinctions to ensure that laboratory technicians’ scientific work was never publicly
acknowledged.
(D) Seventeenth-century scientists were far more dependent on their laboratory technicians than are
scientists today, yet far less willing to acknowledge technicians’ scientific contributions.
(E) Seventeenth-century scientists liberated themselves from the stigma attached to manual labor
by relying heavily on the work of laboratory technicians.

2. It can be inferred from the passage that the “seventeenth-century rhetoric”


(Highlighted) would have more accurately described the experimentation performed in
Boyle’s laboratory if which one of the following were true?

(A) Unlike many seventeenth-century scientists, Boyle recognized that most scientific discoveries
resulted from the cooperative efforts of many individuals.
(B) Unlike many seventeenth-century scientists, Boyle maintained a deeply rooted and pervasive
contempt for manual labor.
(C) Unlike many seventeenth-century scientists, Boyle was a member of the Royal Society of
London.
(D) Boyle generously acknowledged the contribution of the technicians who worked in his
laboratory.
(E) Boyle himself performed the actual labor of obtaining and recording experimental
results.

3. According to the author, servants of seventeenth-century England were excluded


from the franchised because of the belief that

(A) their interests were adequately represented by their employers


(B) their education was inadequate to make informed political decisions
(C) the independence of their political judgment would be compromised by their
economic dependence on their employers
(D) their participation in the elections would be a polarizing influence on the political process
(E) the manual labor that they performed did not constitute a contribution to the society that was
sufficient to justify their participation in elections

4. According to the author, the Royal Society of London insisted that scientists abandon
the

(A) belief that the primary purpose of scientific discovery was to reveal the divine truth that could be
found in nature
(B) view that scientific knowledge results largely from the insights of a few brilliant individuals rather
than from the cooperative efforts of many workers
(C) seventeenth-century belief that servants should be denied the right to vote because they were
dependent on wages paid to them by their employers
(D) traditional disdain for manual labor that was maintained by most members of the
English upper class during the seventeenth-century
(E) idea that the search for scientific truth was a sign of piety

5. The author implies that which one of the following beliefs was held in both the
seventeenth and the twentieth centuries?

(A) Individual insights rather than cooperative endeavors produce most scientific
discoveries.
(B) How science is practiced is significantly influenced by the political beliefs and assumption of
scientists.
(C) Scientific research undertaken for pay cannot be considered objective.
(D) Scientific discovery can reveal divine truth in nature.
(E) Scientific discovery often relies on the unacknowledged contributions of laboratory technicians.

6. Which one of the following best describes the organization of the last paragraph?

(A) Several alternative answers are presented to a question posed in the previous paragraph, and
the last is adopted as the most plausible.
(B) A question regarding the cause of the phenomenon described in the previous paragraph is
posed, two possible explanations are rejected, and evidence is provided in support of a third.
(C) A question regarding the phenomenon described in the previous paragraph is posed, and several
incompatible views are presented.
(D) A question regarding the cause of the phenomenon described in the previous
paragraph is posed, and several contributing factors are then discussed.
(E) Several answers to a question are evaluated in light of recent discoveries cited earlier in the
passage.

7. The author’s discussion of the political significance of the “wage relationship”


(Highlighted) serves to

(A) place the failure of seventeenth-century scientists to acknowledge the contributions


of their technicians in the large context of relations between workers and their
employers in seventeenth-century England
(B) provide evidence in support of the author’s more general thesis regarding the relationship of
scientific discovery to the economic conditions of societies in which it takes place
(C) provide evidence in support of the author’s explanation of why scientists in seventeenth-century
England were reluctant to rely on their technicians for the performance of anything but the most
menial tasks
(D) illustrate political and economic changes in the society of seventeenth-century England that had
a profound impact on how scientific research was conduced
(E) undermine the view that scientific discovery results from individual enterprise rather than from
the collective endeavor of many workers

8. It can be inferred from the passage that “the clamor of seventeenth-century scientific
rhetoric” (Highlighted) refers to

(A) the claim that scientific discovery results largely from the insights of brilliant individuals working
alone
(B) ridicule of scientists who were members of the English upper class and who were thought to
demean themselves by engaging in the manual labor required by their experiments
(C) criticism of scientists who publicly acknowledged the contributions of their technicians
(D) assertions by members of the Royal Society of London that scientists themselves
should be responsible for obtaining and recording experimental results
(E) the claim by Boyle and his colleagues that the primary reason for scientific research is to
discover evidence of divine truth in the natural world

Rhetoric (n) : hùng biện


Authentic (adj) : xác thực

Comtempt (n) : khinh thường

Demean (v) : hạ mình

Muck (v) : làm dơ, vấy bẩn

Furnace (n) : lò nung

Drudge (n) : người làm vất vả

Nobility (n) : tính cao thượng, thanh cao

Piety (n) : lòng thành kính, hiếu thảo

Fragile (adj) : yếu ớt

Tendency (n) : khuynh hướng, xu hướng

Persist (v) : kiên trì

Momentary (adj) : nhất thời

Insight (n) : sự sáng suốt, minh mẫn

Clamor (n) : kêu la

Hands-on approach (n) : phương pháp thực hành

Overwhelmingly (adv) : áp đảo

Pervasive (adj) : phổ biến, thẩm thấu, xâm nhập

Deeply root (v) : bắt nguồn sâu sắc

Franchise (n) : quyền công dân, quyền bầu cử

Apparatus (n) : bộ máy

Ridicule (n) : chế giễu

Disdain (n) : khinh bỉ


Mar 31 – 1

The extent of a nation’s power over its coastal ecosystems and the natural resources in its coastal
waters has been defined by two international law doctrines: freedom of the seas and adjacent state
sovereignty. Until the mid-twentieth century, most nations favored application of broad open-seas
freedoms and limited sovereign rights over coastal waters. A nation had the right to include within
its territorial dominion only a very narrow band of coastal waters (generally extending three miles
from the shoreline), within which it had the authority but not the responsibility, to regulate all
activities. But, because this area of territorial dominion was so limited, most nations did not establish
rules for management or protection of their territorial waters.

Regardless of whether or not nations enforced regulations in their territorial waters, large ocean
areas remained free of controls or restrictions. The citizens of all nations had the right to use these
unrestricted ocean areas for any innocent purpose, including navigation and fishing. Except for
controls over its own citizens, no nation had the responsibility, let alone the unilateral authority, to
control such activities in international waters. And, since there were few standards of conduct that
applied on the “open seas”, there were few jurisdictional conflicts between nations.

The lack of standards is traceable to popular perceptions held before the middle of this century. By
and large, marine pollution was not perceived as a significant problem, in part because the adverse
effect of coastal activities on ocean ecosystems was not widely recognized, and pollution caused by
human activities was generally believed to be limited to that caused by navigation. Moreover, the
freedom to fish, or overfish, was an essential element of the traditional legal doctrine of freedom of
the seas that no maritime (of, relating to, or bordering on the sea “a maritime province”) country
wished to see limited. And finally, the technology that later allowed exploitation of other ocean
resources, such as oil, did not yet exist.

To date, controlling pollution and regulating ocean resources have still not been comprehensively
addressed by law, but international law—established through the customs and practices of nations—
does not preclude such efforts. And two recent developments may actually lead to future
international rules providing for ecosystem management. First, the establishment of extensive
fishery zones extending territorial authority as far as 200 miles out from a country’s coast, has
provided the opportunity for nations individually to manage larger ecosystems. This opportunity,
combined with national self-interest in maintaining fish populations, could lead nations to reevaluate
policies for management of their fisheries and to address the problem of pollution in territorial
waters. Second, the international community is beginning to understand the importance of
preserving the resources and ecology of international waters and to show signs of accepting
responsibility for doing so. As an international consensus regarding the need for comprehensive
management of ocean resources develops, it will become more likely that international standards
and policies for broader regulation of human activities that affect ocean ecosystems will be adopted
and implemented.
1. According to the passage, until the mid-twentieth century there were few
jurisdictional disputes over international waters because.

(A) the nearest coastal nation regulated activities


(B) few controls or restrictions applied to ocean areas
(C) the ocean areas were used for only innocent purposes
(D) the freedom of the seas doctrine settled all claims concerning navigation and fishing
(E) broad authority over international waters was shared equally among all nations

Regardless of whether or not nations enforced regulations in their territorial waters, large ocean
areas remained free of controls or restrictions.

Except for controls over its own citizens, no nation had the responsibility, let alone the unilateral
authority, to control such activities in international waters. And, since there were few
standards of conduct that applied on the “open seas”, there were few jurisdictional conflicts
between nations.

2. According to the international law doctrines applicable before the mid-twentieth


century, if commercial activity within a particular nation’s territorial waters threatened
all marine life in those waters, the nation would have been

(A) formally censured by an international organization for not properly regulating marine activities
(B) called upon by other nations to establish rules to protect its territorial waters
(C) able but not required to place legal limits on such commercial activities
(D) allowed to resolve the problem at it own discretion providing it could contain the threat to its
own territorial waters
(E) permitted to hold the commercial offenders liable only if they were citizens of that particular
nation

3. The author suggests that, before the mid-twentieth century, most nations’ actions
with respect to territorial and international waters indicated that

(A) managing ecosystems in either territorial or international waters was given low
priority
(B) unlimited resources in international waters resulted in little interest in territorial waters
(C) nations considered it their responsibility to protect territorial but not international waters
(D) a nation’s authority over its citizenry ended at territorial lines
(E) although nations could extend their territorial dominion beyond three miles from their shoreline,
most chose not to do so

4. The author cites which one of the following as an effect of the extension of territorial
waters beyond the three-mile limit?

(A) increased political pressure on individual nations to establish comprehensive laws regulating
ocean resources
(B) a greater number of jurisdictional disputes among nations over the regulation of fishing on the
open seas
(C) the opportunity for some nations to manage large ocean ecosystems
(D) a new awareness of the need to minimize pollution caused by navigation
(E) a political incentive for smaller nations to solve the problems of pollution in their coastal waters

5. According to the passage, before the middle of the twentieth century, nations failed
to establish rules protecting their territorial waters because

(A) the waters appeared to be unpolluted and to contain unlimited resources


(B) the fishing industry would be adversely affected by such rules
(C) the size of the area that would be subject to such rules was insignificant
(D) the technology needed for pollution control and resource management did not exist
(E) there were few jurisdictional conflicts over nations’ territorial waters

6. The passage as a whole can best be described as

(A) a chronology of the events that have led up to present-day crisis


(B) a legal inquiry into the abuse of existing laws and the likelihood of reform
(C) a political analysis of the problems inherent in directing national attention to an international
issue
(D) a historical analysis of a problem that requires international attention
(E) a proposal for adopting and implementing international standards to solve an ecological problem

Inquiry (n) : điều tra

Navigation (n) : sự đi biển

Jurisdictional (adj) : quyền hạn tài phán


Mar 31 – 2

Although bacteria are unicellular and among the simplest autonomous forms of life, they show a
remarkable ability to sense their environment. They are attracted to materials they need and are
repelled by harmful substances. Most types of bacteria swim very erratically: short smooth runs in
relatively straight lines are followed by brief tumbles, after which the bacteria shoot off in random
directions. This leaves researchers with the question of how such bacteria find their way to an
attractant such as food or, in the case of photosynthetic bacteria, light, if their swimming pattern
consists only of smooth runs and tumbles, the latter resulting in random changes in direction.

One clue comes from the observation that when a chemical attractant is added to a suspension of
such bacteria, the bacteria swim along a gradient of the attractant, from an area where the
concentration of the attractant is weaker to an area where it is stronger. As they do so, their
swimming is characterized by a decrease in tumbling and an increase in straight runs over relatively
longer distances. As the bacteria encounter increasing concentrations of the attractant, their
tendency to tumble is suppressed, whereas tumbling increases whenever they move away from the
attractant. The net effect is that runs in the direction of higher concentrations of the attractant
become longer and straighter as a result of the suppression of tumbling, whereas runs away from it
are shortened by an increased tendency of the bacteria to tumble and change direction.

Biologists have proposed two mechanisms that bacteria might use in detecting changes in the
concentration of a chemical attractant. First, a bacterium might compare the concentration of a
chemical at the front and back of its cell body simultaneously. If the concentration is higher at the
front of the cell, then it knows it is moving up the concentration gradient, from an area where the
concentration is lower to an area where it is higher. Alternatively, it might measure the
concentration at one instant and again after a brief interval, in which case the bacterium must retain
a memory of the initial concentration. Researchers reasoned that if bacteria do compare
concentrations at different times, then when suddenly exposed to a uniformly high concentration of
an attractant, the cells would behave as if they were swimming up a concentration gradient, with
long, smooth runs and relatively few tumbles. If, on the other hand, bacteria detect a chemical
gradient by measuring it simultaneously at two distinct points, front and back, on the cell body, they
would not respond to the jump in concentration because the concentration of the attractant in front
and back of the cells, though high, would be uniform. Experimental evidence suggests that bacteria
compare concentrations at different times.

1.It can be inferred from the passage that which one of the following experimental
results would suggest that bacteria detect changes in the concentration of an attractant
by measuring its concentration in front and back of the cell body simultaneously?

(A) When suddenly transferred from a medium in which the concentration of an attractant was
uniformly low to one in which the concentration was uniformly high, the tendency of the bacteria to
tumble and undergo random changes in direction increased.
(B) When suddenly transferred from a medium in which the concentration of an
attractant was uniformly low to one in which the concentration was uniformly high, the
bacteria’s exhibited no change in the pattern of their motion.
(C) When suddenly transferred from a medium in which the concentration of an attractant was
uniformly low to one in which the concentration was uniformly high, the bacteria’s movement was
characterized by a complete absence of tumbling.
(D) When placed in a medium in which the concentration of an attractant was in some areas low
and in others high, the bacteria exhibited an increased tendency to tumble in those areas where the
concentration of the attractant was high.
(E) When suddenly transferred from a medium in which the concentration of an attractant was
uniformly low to one that was completely free of attractants, the bacteria exhibited a tendency to
suppress tumbling and move in longer, straighter lines.

Because bacteria can feel the change of the concentration so if it recognize it is already
in the high concentration of attractant, it won’t need to move, so it stop their motion

If, on the other hand, bacteria detect a chemical gradient by measuring it simultaneously at two
distinct points, front and back, on the cell body, they would not respond to the jump in
concentration because the concentration of the attractant in front and back of the cells,
though high, would be uniform
2. It can be inferred from the passage that a bacterium would increase the likelihood of
its moving away from an area where the concentration of a harmful substance is high if
it did which one of the following?

(A) Increased the speed at which it swam immediately after undergoing the random changes in
direction that result from tumbling.
(B) Detected the concentration gradient of an attractant toward which it could begin to swim.
(C) Relied on the simultaneous measurement of the concentration of the substance in front and
back of its body, rather than on the comparison of the concentration at different points in time.
(D) Exhibited a complete cessation of tumbling when it detected increases in the concentration of
substance.
(E) Exhibited an increased tendency to tumble as it encountered increasing
concentrations of the substance, and suppressed tumbling as it detected decreases in
the concentration of the substance.

3. It can be inferred from the passage that when describing bacteria as “swimming up a
concentration gradient” (Highlighted), the author means that they were behaving as if
they were swimming

(A) Against a resistant medium that makes their swimming less efficient.
(B) Away from a substance to which they are normally attracted.
(C) Away from a substance that is normally harmful to them.
(D) From an area where the concentration of a repellent is weaker to an area where it is completely
absent.
(E) From an area where the concentration of a substance is weaker to an area where it
is stronger.

If the concentration is higher at the front of the cell, then it knows it is moving up the concentration
gradient, from an area where the concentration is lower to an area where it is higher.

4. The passage indicates that the pattern that characterizes a bacterium’s motion
changes in response to

(A) The kinds of chemical attractants present in different concentration gradients.


(B) The mechanism that the bacterium adopts in determining the presence of an attractant.
(C) The bacterium’s detection of changes in the concentration of an attractant.
(D) The extent to which neighboring bacteria are engaged in tumbling.
(E) Changes in the intervals of time that occur between the bacterium’s measurement of the
concentration of an attractant.

5. Which one of the following best describes the organization of the third paragraph of
the passage?

(A) Two approaches to a problem are discussed, a test that would determine which is more efficient
is described, and a conclusion is made, based on experimental evidence.
(B) Two hypotheses are described, a way of determining which of them is more likely to
be true is discussed, and one said to be more accurate on the basis of experimental
evidence.
(C) Two hypotheses are described, the flaws inherent in one of them are elaborated, and
experimental evidence confirming the other is cited.
(D) An assertion that a species has adopted two different mechanisms to solve a particular problem
is made, and evidence is then provided in support of that assertion.
(E) An assertion that one mechanism for solving a particular problem is more efficient than another
is made, and evidence is then provided in support of that assertion.

6. The passage provides information in support of which one of the following


assertions?

(A) The seemingly erratic motion exhibited by a microorganism can in fact reflect a
mechanism by which it is able to control its movement.
(B) Biologists often overstate the complexity of simple organisms such as bacteria.
(C) A bacterium cannot normally retain a memory of a measurement of the concentration of an
attractant.
(D) Bacteria now appear to have less control over their movement than biologists had previously
hypothesized.
(E) Photosynthetic bacteria appear to have more control over their movement than do bacteria that
are not photosynthetic.

Autonomous (adj) : tự chủ

Repel (v) : từ chối, cự tuyệt

Erratically (adv) : thất thường

Tumble (v) : té nhào

Photosynthetic (n) : quang hợp

Suppress (v) : bãi bỏ

Attractant (n) : chất hấp dẫn

Cessation (n) : đình chỉ, sự thôi


Apr 01 – 1

In recent years the early music movement, which advocates performing a work as it was performed
at the time of its composition, has taken on the character of a crusade, particularly as it has moved
beyond the sphere of medieval and baroque music and into music from the late eighteenth and
early nineteenth centuries by composers such as Mozart and Beethoven. Granted, knowledge about
the experience of playing old music on now-obsolete instruments has been of inestimable value to
scholars. Nevertheless, the early music approach to performance raises profound and troubling
questions.

Early music advocates assume that composers write only for the instruments available to them, but
evidence suggests that composers of Beethoven’s stature imagined extraordinarily high and low
notes as part of their compositions, even when they recognized that such notes could not be played
on instruments available at the time. In the score of Beethoven’s first piano concerto, there is a
“wrong” note, a high F-natural where the melody obviously calls for a high F-sharp, but pianos did
not have this high an F-sharp when Beethoven composed the concerto. Because Beethoven once
expressed a desire to revise his early works to exploit the extended range of pianos that became
available to him some years later, it seems likely that he would have played the F-sharp if given the
opportunity. To use a piano exactly contemporary with the work’s composition would require playing
a note that was probably frustrating for Beethoven himself to have had to play.

In addition, early music advocates often inadvertently divorce music and its performance from the
life of which they were, and are, a part. The discovery that Haydn’s and Mozart’s symphonies were
conducted during their lifetimes by a pianist who played the chords to keep the orchestra together
has given rise to early music recordings in which a piano can be heard obtrusively in the foreground,
despite evidence indicating that the orchestral piano was virtually inaudible to audiences at
eighteenth-century concerts and was dropped as musically unnecessary when a better way to beat
time was found. And although in the early nineteenth century the first three movements (sections)
of Mozart’s and Beethoven’s symphonies were often played faster, and the last movement slower,
than today, this difference can readily be explained by the fact that at that time audiences
applauded at the end of each movement, rather than withholding applause until the end of the
entire work. As a result, musicians were not forced into extra brilliance in the finale in order to
generate applause, as they are now. To restore the original tempo of these symphonies represents
an irrational denial of the fact that our concepts of musical intensity and excitement have, quite
simply, changed.

1. It can be inferred from the passage that by “a piano exactly contemporary”


(Highlighted) with the composition of Beethoven’s first piano concerto, the author means
the kind of piano that was

(A) designed to be inaudible to the audience when used by conductors of orchestras


(B) incapable of playing the high F-natural that is in the score of Beethoven’s original version of the
concerto
(C) unavailable to Mozart and Haydn
(D) incapable of playing the high F-sharp that the melody of the concerto calls for
(E) influential in Beethoven’s decision to revise his early compositions
2. Which one of the following best expresses the main idea of the passage?

(A) The early music movement has yet to resolve a number of troubling questions
regarding its approach to the performance of music.
(B) The early music movement, while largely successful in its approach to the performance of
medieval and baroque music, has yet to justify its use of obsolete instruments in the performance of
music by Beethoven and Mozart.
(C) The early music approach to performance often assumes that composers write music that is
perfectly tailored to the limitations of the instruments on which it will be performed during their
lifetimes.
(D) Although advocates of early music know much about the instruments used to perform music at
the time it was composed, they lack information regarding how the style of such performances has
changed since such music was written.
(E) The early music movement has not yet fully exploited the knowledge that it has gained from
playing music on instruments available at the time such music was composed.

3. In the second paragraph, the author discusses Beethoven’s first piano concerto
primarily in order to

(A) illustrate how piano music began to change in response to the extended range of pianos that
became available during Beethoven’s lifetime
(B) illustrate how Beethoven’s work failed to anticipate the changes in the design of instruments that
were about to be made during his lifetime
(C) suggest that early music advocates commonly perform music using scores that do not reflect
revisions made to the music years after it was originally composed
(D) illustrate how composers like Beethoven sometimes composed music that called for
notes that could not be played on instruments that were currently available
(E) provide an example of a piano composition that is especially amenable to being played on pianos
available at the time the music was composed

4. The author suggests that the final movements of symphonies by Mozart and
Beethoven might be played more slowly by today’s orchestras if which one of the
following were to occur?

(A) orchestras were to use instruments no more advanced in design than those used by orchestras
at the time Mozart and Beethoven composed their symphonies
(B) audiences were to return to the custom of applauding at the end of each movement
of a symphony
(C) audiences were to reserve their most enthusiastic applause for the most brilliantly played finales
(D) conductors were to return to the practice of playing the chords on an orchestral piano to keep
the orchestra together
(E) conductors were to conduct the symphonies in the manner in which Beethoven and Mozart had
conducted them

5. Which one of the following best describes the organization of the last paragraph?

(A) A generalization is made, evidence undermining it is presented, and a conclusion rejecting it is


then drawn.
(B) A criticism is stated and then elaborated with two supporting examples.
(C) An assumption is identified and then evidence undermining its validity is presented.
(D) An assertion is made and evidence frequently provided in support of it is then
critically evaluated.
(E) Two specific cases are presented and then a conclusion regarding their significance is drawn.

6. It can be inferred from the passage that the author’s explanation in lines (this
difference can readily be explained by the fact that at that time audiences applauded at the end of each
movement, rather than withholding applause until the end of the entire work.) would be most
weakened if which one of the following were true?

(A) Musicians who perform in modern orchestras generally receive more extensive training than did
their nineteenth-century counterparts.
(B) Breaks between the movements of symphonies performed during the early
nineteenth century often lasted longer than they do today because nineteenth-century
musicians needed to retune their instruments between each movement.
(C) Early nineteenth-century orchestral musicians were generally as concerned with the audience’s
response to their music as are the musicians who perform today in modern orchestras.
(D) Early nineteenth-century audiences applauded only perfunctorily after the first three movements
of symphonies and conventionally withheld their most enthusiastic applause until the final
movement was completed.
(E) Early nineteenth-century audiences were generally more knowledgeable about music than are
their modern counterparts.

7. It can be inferred from the passage that the author would be most likely to agree
with which one of the following assertions regarding the early music recordings
mentioned in the third paragraph?

(A) These recordings fail to recognize that the last movements of Haydn’s and Mozart’s symphonies
were often played slower in the eighteenth century than they are played today.
(B) These recordings betray the influence of baroque musical styles on those early music advocates
who have recently turned their attention to the music of Haydn and Mozart.
(C) By making audible the sound of an orchestral piano that was inaudible in eighteenth-century
performances, these recordings attempt to achieve aesthetic integrity at the expense of historical
authenticity.
(D) By making audible the sound of an orchestral piano that was inaudible in
eighteenth-century performances, these recordings unwittingly create music that is
unlike what eighteenth-century audiences heard.
(E) These recordings suggest that at least some advocates of early music recognize that concepts of
musical intensity and excitement have changed since Haydn and Mozart composed their
symphonies.

8. The author suggests that the modern audience’s tendency to withhold applause until
the end of a symphony’s performance is primarily related to which one of the following?

(A) the replacement of the orchestral piano as a method of keeping the orchestra together
(B) a gradual increase since the time of Mozart and Beethoven in audiences’ expectations regarding
the ability of orchestral musicians
(C) a change since the early nineteenth century in audiences’ concepts of musical
excitement and intensity
(D) a more sophisticated appreciation of the structural integrity of the symphony as a piece of music
(E) the tendency of orchestral musicians to employ their most brilliant effects in the early
movements of symphonies composed by Mozart and Beethoven

Crusade (n) : thập tự chinh

Baroque (adj) : kỳ quái, lạ lùng

Inestimable (adj) : vô giá

Advocate (n) : người bênh vực

Score (n) : bản nhạc

Concerto (n) : nhạc hợp tấu

Inadvertently (adv) : vô tình

Symphony (n) : giao hưởng

Conduct (v) : tiến hành

Chord (n) : hợp âm

Orchestra (n) : dàn nhạc

Obstrusive (adj) : gây khó chịu

Virtually (adv) : hầu như

Inaudible (adj) : không nghe được

Applaud (v) : vỗ tay

Finale (n) : bản nhạc sau cùng

Irrational (adj) : phi lý

Intensity (n) : cường độ

Unwittingly (adv) : vô tình


Apt 02 – 2

Modern art challenges our understanding of art by adding a new and crucial element, the concept,
to its definition. With the brazen beginnings of Marcel Duchamp, modern art began to uproot our
society's limited conceptions of art and question its very nature. The difference between modern and
traditional art could not be more plain. Previously, aesthetic principles and specialized artistic skill
were the hallmarks of "good" artwork. Now, unlike the Old Masters, modern artists use their
considerable artistic talents to beg the questions of the role of the viewer, the function of the artist
in society and what can be deemed art. Some modern artists posit that any object may be
considered art as long as it is presented as art. The suggestion that everyday objects or intangible
ideas may be considered as artistic as Leonardo da Vinci's Mona Lisa often leads to shock, dismay
and disgust. However, in eliciting those responses, modern artists achieve their intended goal: to
provoke debate as to the nature of art and make the viewer expand his or her conception of art.

Robert Rauschenberg's White Paintings are a series of monochromatic works whose underlying
concept exemplifies modern art. As one can expect from their name, the paintings consist of multi-
paneled canvases painted white. They represent the antithesis of traditional artistic standards: their
creation requires no special artistic skill and their value lies not in their appearance but in the
abstract concept they invoke. The color white is the key element to the concept behind their
creation. The white paint on the canvas emphasizes its flat, two-dimensional nature. Historically,
painting has tried to represent three-dimensional topics on canvas, drawing attention away from the
canvas and towards the topic of the painting. The White Paintings do not just acknowledge the flat
surface on which they are painted, they make the canvas itself the topic of the work. By drawing
attention to the canvas, Rauschenberg succeeds in giving his viewer a radically creative alternative
view of art.

1. The author is most likely to agree that which of the following objects would not be
considered art by some modern artists?

A) the saliva of the artist contained in jars on display in a gallery


B) realistic paintings by the Old Masters on display in a gallery
C) a goat painted green as part of a highly publicized art show
D) the dust on the floor of an art gallery
E) an empty room which the artist has entitled "Invisible Paintings"

2. Which of the following can be inferred from the passage about Leonardo da Vinci's
Mona Lisa?

A) Leonardo da Vinci is considered an Old Master.


B) Viewing the Mona Lisa cannot lead to shock, dismay and disgust.
C) It is a work of undisputed high artistic value.
D) It is more artistic than everyday objects or intangible ideas.
E) Artists who consider everyday objects or intangible ideas to be as artistic as the Mona Lisa do not
appreciate its true value.
3. Which of the following best describes the relation of the second paragraph to the
passage as a whole?

A) It summarizes the arguments made in the first paragraph.


B) It presents a contrasting point of view to the one presented in the first paragraph.
C) It illustrates the argument made in the first paragraph.
D) It aims to refute the argument of the first paragraph by showing an example
E) It further develops the argument made in the first paragraph.

It does not further develops the argument

Saliva (n) : nước bọt

Jar (n) : lọ

Dismay (v) : mất tinh thần

Disgust (v) : ghê tởm

Uproot (v) : nhổ, bứng rễ

Provoke (v) : kích động

Monochromatic (adj) : đơn sắc

Canvase (n) : tranh sơn dầu

Antithesis (n) : phản đề, phản đối, đối chiếu

Radically (adv) : hoàn toàn

Undisputed (adj) : không thể tranh cãi

Elicit (v) : khơi gợi

Hallmark (n) : dấu ấn

Plain (adj) : hiển nhiên, rõ ràng, minh bạch, phân minh


Apr 02 – 1

The labor force is often organized as if workers had no family responsibilities. Preschool-age children
need full-time care; children in primary school need care after school and during school vacations.
Although day-care services can resolve some scheduling conflicts between home and office, workers
cannot always find or afford suitable care. Even when they obtain such care, parents must still cope
with emergencies, such as illnesses, that keep children at home. Moreover, children need more than
tending; they also need meaningful time with their parents. Conventional full-time workdays,
especially when combined with unavoidable household duties, are too inflexible for parents with
primary child-care responsibility.

Although a small but increasing number of working men are single parents, those barriers against
successful participation in the labor market that are related to primary child-care responsibilities
mainly disadvantage women. Even in families where both parents work, cultural pressures are
traditionally much greater on mothers than on fathers to bear the primary child-rearing
responsibilities.

In reconciling child-rearing responsibilities with participation in the labor market, many working
mothers are forced to make compromises. For example, approximately one-third of all working
mothers are employed only part-time, even though part-time jobs are dramatically underpaid and
often less desirable in comparison to full-time employment. Even though part-time work is usually
available only in occupations offering minimal employee responsibility and little opportunity for
advancement or self-enrichment, such employment does allow many women the time and flexibility
to fulfill their family duties, but only at the expense of the advantages associated with full-time
employment.

Moreover, even mothers with full-time employment must compromise opportunities in order to
adjust to barriers against parents in the labor market. Many choose jobs entailing little challenge or
responsibility or those offering flexible scheduling, often available only in poorly paid positions, while
other working mothers, although willing and able to assume as much responsibility as people
without children, find that their need to spend regular and predictable time with their children
inevitably causes them to lose career opportunities to those without such demands. Thus, women in
education are more likely to become teachers than school administrators, whose more conventional
full-time work schedules do not correspond to the schedules of school-age children, while female
lawyers are more likely to practice law in trusts and estates, where they can control their work
schedules, than in litigation, where they cannot. Nonprofessional women are concentrated in
secretarial work and department store sales, where their absences can be covered easily by
substitutes and where they can enter and leave the work force with little loss, since the jobs offer so
little personal gain. Indeed, as long as the labor market remains hostile to parents, and family roles
continue to be allocated on the basis of gender, women will be seriously disadvantaged in that labor
market.
1. Which one of the following best summarizes the main idea of the passage?

(A) Current trends in the labor force indicate that working parents, especially women, may not
always need to choose between occupational and child-care responsibilities.
(B) In order for mothers to have an equal opportunity for advancement in the labor force, traditional
family roles have to be reexamined and revised.
(C) Although single parents who work have to balance parental and career demands, single mothers
suffer resulting employment disadvantages that single fathers can almost always avoid.
(D) Although child-care responsibilities disadvantage many women in the labor force, professional
women (such as teachers and lawyers) are better able to overcome this problem than are
nonprofessional women.
(E) Traditional work schedules are too inflexible to accommodate the child-care
responsibilities of many parents, a fact that severely disadvantages women in the labor
force.

2. Which one of the following statements about part-time work can be inferred from the
information presented in the passage?

(A) One-third of all part-time workers are working mothers.


(B) Part-time work generally offers fewer opportunities for advancement to working mothers than to
women generally.
(C) Part-time work, in addition to having relatively poor wages, often requires that employees work
during holidays, when their children are out of school.
(D) Part-time employment, despite its disadvantages, provides working mothers with an
opportunity to address some of the demands of caring for children.
(E) Many mothers with primary child-care responsibility choose part-time jobs in order to better
exploit full-time career opportunities after their children are grown.

3. It can be inferred from the passage that the author would be most likely to agree
with which one of the following statements about working fathers in two-parent
families?

(A) They are equally burdened by the employment disadvantages placed upon all parents-male and
female-in the labor market.
(B) They are so absorbed in their jobs that they often do not see the injustice going on around
them.
(C) They are shielded by the traditional allocation of family roles from many of the
pressures associated with child-rearing responsibilities.
(D) They help compound the inequities in the labor market by keeping women form competing with
men for career opportunities.
(E) They are responsible for many of the problems of working mothers because of their insistence
on traditional roles in the family.

4. Of the following, which one would the author most likely say is the most troublesome
barrier facing working parents with primary child-care responsibility?

(A) the lack of full-time jobs open to women


(B) the inflexibility of work schedules
(C) the low wages of part-time employment
(D) the limited advancement opportunities for nonprofessional employees
(E) the practice of allocating responsibilities in the workplace on the basis of gender

5. The passage suggests that day care is at best a limited solution to the pressures
associated with child rearing for all of the following reasons EXCEPT:
(A) Even the best day care available cannot guarantee that children will have meaningful time with
their parents.
(B) Some parents cannot afford day-care services.
(C) Working parents sometimes have difficulty finding suitable day care for their children.
(D) Parents who send their children to day care still need to provide care for their
children during vacations.
(E) Even children who are in day care may have to stay home when they are sick.

6. According to the passage, many working parents may be forced to make any of the
following types of career decisions EXCEPT
(A) declining professional positions for nonprofessional ones, which typically have less
conventional work schedules
(B) accepting part-time employment rather than full-time employment
(C) taking jobs with limited responsibility, and thus more limited career opportunities, in order to
have a more flexible schedule
(D) pursuing career specializations that allow them to control their work schedules instead of
pursuing a more desirable specialization in the same field
(E) limiting the career potential of one parent, often the mother, who assumes greater child-care
responsibility

7. Which one of the following statements would most appropriately continue the
discussion at the end of the passage?
(A) At the same time, most men will remain better able to enjoy the career and salary
opportunities offered by the labor market.
(B) Of course, men who are married to working mothers know of these employment barriers but
seem unwilling to do anything about them.
(C) On the other hand, salary levels may become more equitable between men and women even if
the other career opportunities remain more accessible to men than to women.
(D) On the contrary, men with primary child-rearing responsibilities will continue to enjoy more
advantages in the workplace than their female counterparts.
(E) Thus, institutions in society that favor men over women will continue to widen the gap between
the career opportunities available for men and for women.

Equitable (adj) : công bằng, không thiên vị

Engross (v) : hăng say

Absorb (v) : hấp thụ

Hostile (adj) : thù nghịch

Litigation (n) : kiện tụng

Compromise (v) : thỏa hiệp


Apr 02 – 2

Although the United States steel industry faces widely publicized economic problems that have
eroded its steel production capacity, not all branches of the industry have been equally affected. The
steel industry is not monolithic: it includes integrated producers, minimills, and specialty-steel mills.
The integrated producers start with iron ore and coal and produce a wide assortment of shaped
steels. The minimills reprocess scrap steel into a limited range of low-quality products, such as
reinforcing rods for concrete. The specialty-steel mills are similar to minimills in that they tend to be
smaller than the integrated producers and are based on scrap, but they manufacture much more
expensive products than minimills do and commonly have an active in-house research-and-
development effort.

Both minimills and specialty-steel mills have succeeded in avoiding the worst of the economic
difficulties that are afflicting integrated steel producers, and some of the mills are quite profitable.
Both take advantage of new technology for refining and casting steel, such as continuous casting, as
soon as it becomes available. The minimills concentrate on producing a narrow range of products for
sale in their immediate geographic area, whereas specialty-steel mills preserve flexibility in their
operations in order to fulfill a customer’s particular specifications.

Among the factors that constrain the competitiveness of integrated producers are excessive labor,
energy, and capital costs, as well as manufacturing inflexibility. Their equipment is old and less
automated, and does not incorporate many of the latest refinement in steelmaking technology. (For
example, only about half of the United States integrated producers have continuous casters, which
combine pouring and rolling into one operation and thus save the cost of separate rolling
equipment.) One might conclude that the older labor-intensive machinery still operating in United
States integrated plants is at fault for the poor performance of the United States industry, but this
cannot explain why Japanese integrated producers, who produce a higher-quality product using less
energy and labor, are also experiencing economic trouble. The fact is that the common technological
denominator of integrated producers is an inherently inefficient process that is still rooted in the
nineteenth century.

Integrated producers have been unable to compete successfully with minimills because the
minimills, like specialty-steel mills, have dispensed almost entirely with the archaic energy and
capital-intensive front end of integrated steelmaking: the iron-smelting process, including the mining
and preparation of the raw materials and the blast-furnace operation. In addition, minimills have
found a profitable way to market steel products: as indicated above, they sell their finished products
locally, thereby reducing transportation costs, and concentrate on a limited range of shapes and
sizes within a narrow group of products that can be manufactured economically. For these reasons,
minimills have been able to avoid the economic decline affecting integrated steel producers.
1. Which one of the following best expresses the main idea of the passage?

(A) United States steel producers face economic problems that are shared by producers in other
nations.
(B) Minimills are the most successful steel producers because they best meet market demands for
cheap steel.
(C) Minimills and specialty-steel mills are more economically competitive than
integrated producers because they use new technology and avoid the costs of the iron
smelting process.
(D) United States steel producers are experiencing an economic decline that can be traced back to
the nineteenth century.
(E) New steelmaking technologies such as continuous casting will replace blast-furnace operations to
reverse the decline in United States steel production.

2. The author mentions all of the following as features of minimills EXCEPT

(A) flexibility in their operations


(B) local sale of their products
(C) avoidance of mining operations
(D) use of new steel-refining technology
(E) a limited range of low-quality products

The minimills concentrate on producing a narrow range of products for sale in their immediate
geographic area, whereas specialty-steel mills preserve flexibility in their operations in order to fulfill
a customer’s particular specifications.

3. The author of the passage refers to “Japanese integrated producers” (Highlighted)


primarily in order to support the view that

(A) different economic difficulties face the steel industries of different nations
(B) not all integrated producers share a common technological denominator
(C) labor-intensive machinery cannot be blamed for the economic condition of United
States integrated steel producers
(D) modern steelmaking technology is generally labor-and energy-efficient
(E) labor-intensive machinery is an economic burden on United States integrated steel producers

4. Which one of the following best describes the organization of the third paragraph?

(A) A hypothesis is proposed and supported; then an opposing view is presented and criticized.
(B) A debate is described and illustrated: then a contrast is made and the debate is resolved.
(C) A dilemma is described and cited as evidence for a broader criticism.
(D) A proposition is stated and argued, then rejected in favor of a more general statement, which is
supported with additional evidence.
(E) General statements are made and details given; then an explanation is proposed and
rejected, and an alternative is offered.

5. It can be inferred from the passage that United States specialty-steel mills generally
differ from integrated steel producers in that the specialty-steel mills
(A) sell products in a restricted geographical area
(B) share the economic troubles of the minimills
(C) resemble specialty-steel mills found in Japan
(D) concentrate on producing a narrow range of products
(E) do not operate blast furnaces

6. Each of the following describes an industry facing a problem also experienced by


United Stated integrated steel producers EXCEPT
(A) a paper-manufacturing company that experiences difficulty in obtaining enough
timber and other raw materials to meet its orders
(B) a food-canning plant whose canning machines must constantly be tended by human operators
(C) a textile firm that spends heavily on capital equipment and energy to process raw cotton before
it is turned into fabric
(D) a window-glass manufacturer that is unable to produce quickly different varieties of glass with
special features required by certain customers
(E) a leather-goods company whose hand-operated cutting and stitching machines were
manufactured in Italy in the 1920s

7. Which one of the following, if true, would best serve as supporting evidence for the
author’s explanation of the economic condition of integrated steel producers?

(A) Those nations that derive a larger percentage of their annual steel production from minimills
than the United States does also have a smaller per capita trade deficit.
(B) Many integrated steel producers are as adept as the specialty-steel mills at producing high-
quality products to meet customer specifications.
(C) Integrated steel producers in the United States are rapidly adopting the production methods of
Japanese integrated producers.
(D) Integrated steel producers in the United States are now attempting to develop a worldwide
market by advertising heavily.
(E) Those nations in which iron-smelting operations are carried out independently of
steel production must heavily subsidize those operations in order to make them
profitable.

Publicize (v) : công khai

Erode (v) : xói mòn

Monolithic (adj) : nguyên khối

Assertment (n) : các loại

Refinement (n) : sự tinh luyện

Casters (n) : bánh đúc

Casting (n) : vật đúc

Dispense (v) : phân chia, thoát khỏi, loại bỏ

Archaic (adj) : cổ xưa

Blast-furnace (n) : lò luyện sắt


Apr 03 – 1

For visitors unaccustomed to this system of hereditary social divisions, the complex and mostly
unwritten rules governing whom a person can marry, what kind of work she can do, and even what
kind of food she can eat may seem puzzling and mysterious. One reason for this confusion is that
the concept of caste is actually divided into two separate but related concepts in Indian
culture: varna and jati.

Varna, which literally means “color,” is the most basic social division. There are four varna: the
Brahmans, the traditional priest class; the Kshatriya, the warrior class; the Vaishya, the skilled
workers and merchants; and the Sudra, laborers whose role is to serve the three higher classes.
Below the Sudra are a class known as the Untouchables, who technically fall outside of
the varnasystem because they are supposedly “unclean” in a ritual sense. The Untouchables are the
lowest class in India, but they make life possible for everyone else because they take care of the
jobs that would “pollute” the higher classes, such as working with dead animals or cleaning sewage.
The Indian statesman Mohandas Gandhi, in an effort to promote social equality, encouraged people
to refer to Untouchables as the Harijan, which means “Children of God.”

Each varna is then divided into hundreds or thousands of jati, a term that literally means “birth.”
The jati are kinship groups with hereditary roles and professions, such as leatherworker or
brickmaker. Observant Hindus have traditionally married within their varna and jati.

The origins of the caste system are obscure. The prevailing theory among anthropologists is that
the Varna system emerged shortly after the so-called Aryan Invasion of the second millennium B.C.
According to this theory, a population of Indo-European invaders conquered northern India around
1500 B.C. The Indo-Europeans placed themselves in the three highest rungs of society (Brahman,
Kshatriya, and Vaishya), corresponding to the traditional division of Indo-European societies into
priests, warriors, and commoners, while placing the conquered local populations into the worker
classes of the Sudra and the Untouchables. This theory does not account for the jati system,
however, which has parallels in no other Indo-European society. Most anthropologists suggest that
the jati system predates the varna system,
and that it might have originated in the Harappan civilization that prevailed in northern India prior to
the Aryan Invasion.

1. What is the primary purpose of the passage?

A. To compare and contrast two theories on the origin of the caste system in India
B. To shed some light on an aspect of Indian society that can be confusing to outsiders
C. To highlight the social injustices still faced by the Untouchable caste in India today
D. To discredit the theory that the “Aryan Invasion” brought an end to the Harappan civilization
E. To explain to the reader the distinction between the Brahman and Kshatriya castes

2. The final paragraph plays what role in relation to the rest of the passage?

A. It offers information that raises doubts about the conclusions expressed in the previous
paragraph.
B. It discusses an alternative view to points stated as facts in the second paragraph.
C. It restates the question raised in the opening paragraph, using information offered in the second
and third paragraphs.
D. It offers background information for the system described in the two preceding
paragraphs.
E. It summarizes points raised elsewhere in the passage.

3. What can we infer about the intentions of Mohandas Gandhi from the mention of him
in the passage?

A. He wished to switch the respective socioeconomic positions of the highest and the lowest castes
in Indian society.
B. He desired the reconciliation of India with Pakistan.
C. He rejected the so-called Aryan Invasion in favor of an autochthonous explanation for the roots of
Indian culture.
D. He supported the cultural dominance of the Brahman “priest” caste over the Kshatriya “warrior”
caste.
E. He wished to free a class of people from the stigma of an unfavorable name.

4. The relationship of jati to varna is most comparable to which of the following


relationships?

A. The relationship of individual playing cards to a fifty-two-card deck


B. The relationship of a group of related species of animals to the genus to which the
species belong
C. The relationship of a compact disk to the songs contained on that disk
D. The relationship of a transmission to the car in which the transmission has been installed
E. The relationship of an updated version of software to the original version of that software

5. What inference could reasonably be drawn from the second to last sentence of the
passage?

A. Unlike the situation of the jati, parallels can be found between the varna system and
the social divisions found in other Indo-European societies.
B. The jati system is actually a product of the Dravidian cultures of central and southern India.
C. Although more anthropologists support the Aryan Invasion theory than support any other
explanation for the origins of the caste system, a majority of anthropologists do not believe this
theory.
D. The jati system, unlike the varna system, developed after the influence of IndoEuropeans in India
had already been established.
E. The Sudra did not willingly accept the low-caste jati to which they were assigned, but these social
roles were forced upon them anyway.

6. According to the passage, the caste system would affect all except which of the
following scenarios in a caste-observant Indian person’s life?

A. A Brahman is having dinner with foreign visitors and is offered either a hamburger or a bowl of
rice.
B. A Vaishya mother is considering which of the young women in the local town would make suitable
marriage prospects for her son.
C. Two Sudra teenagers engage in a foot race, and the younger of the two wins the race.
D. A 40-year-old man from the Untouchable caste has moved from the countryside to a large city
and must look for work in order to support his family.
E. The home of a Kshatriya family is located near a city sewer line, which bursts one day and floods
the family’s garage.

And the Sudra, laborers whose role is to serve the three higher classes.

The Untouchables are the lowest class in India, but they make life possible for everyone
else because they take care of the jobs that would “pollute” the higher classes, such as
working with dead animals or cleaning sewage.

Unaccustomed (adj) : không quen

Hereditary (adj) : di truyền

Social division (n) : phân chia xã hội

Sewage (n) : nước thải

Observant (adj) : người quan sát, lưu ý

Caste (n) : đẳng cấp

Prevailing (adj) : thịnh hàn

Commoner (n) : thường dân

Predate (v) : tiền định

Prevail (v) : chiếm ưu thế


Apr 03 – 2

Cultivation of a single crop on a given tract of land leads eventually to decreased yields. One reason
for this is that harmful bacterial phytopathogens, organisms parasitic on plant hosts, increase in the
soil surrounding plant roots. The problem can be cured by crop rotation, denying the pathogens a
suitable host for a period of time. However, even if crops are not rotated, the severity of diseases
brought on by such phytopathogens often decreases after a number of years as the microbial
population of the soil changes and the soil becomes “suppressive” to those diseases. While there
may be many reasons for this phenomenon, it is clear that levels of certain bacteria, such as
Pseudomonas fluorescens, a bacterium antagonistic to a number of harmful phytopathogens, are
greater in suppressive than in nonsuppressive soil. This suggests that the presence of such bacteria
suppresses phytopathogens. There is now considerable experimental support for this view. Wheat
yield increases of 27 percent have been obtained in field trials by treatment of wheat seeds with
fluorescent pseudomonads. Similar treatment of sugar beets, cotton, and potatoes has had similar
results.

These improvements in crop yields through the application of Pseudomonas fluorescens suggest that
agriculture could benefit from the use of bacteria genetically altered for specific purposes. For
example, a form of phytopathogen altered to remove its harmful properties could be released into
the environment in quantities favorable to its competing with and eventually excluding the harmful
normal strain. Some experiments suggest that deliberately releasing altered nonpathogenic
Pseudomonas syringae could crowd out the nonaltered variety that causes frost damage. Opponents
of such research have objected that the deliberate and large-scale release of genetically altered
bacteria might have deleterious results. Proponents, on the other hand, argue that this particular
strain is altered only by the removal of the gene responsible for the strain’s propensity to cause frost
damage, thereby rendering it safer than the phytopathogen from which it was derived.

Some proponents have gone further and suggest that genetic alteration techniques could create
organisms with totally new combinations of desirable traits not found in nature. For example, genes
responsible for production of insecticidal compounds have been transposed from other bacteria into
pseudomonads that colonize corn roots. Experiments of this kind are difficult and require great care:
such bacteria are developed in highly artificial environments and may not compete well with natural
soil bacteria. Nevertheless, proponents contend that the prospects for improved agriculture through
such methods seem excellent. These prospects lead many to hope that current efforts to assess the
risks of deliberate release of altered microorganisms will successfully answer the concerns of
opponents and create a climate in which such research can go forward without undue impediment.
1. Which one of the following best summarizes the main idea of the passage?

(A) Recent field experiments with genetically altered Pseudomonas bacteria have shown that
releasing genetically altered bacteria into the environment would not involve any significant danger.
(B) Encouraged by current research, advocates of agricultural use of genetically altered
bacteria are optimistic that such use will eventually result in improved agriculture,
though opponents remain wary.
(C) Current research indicates that adding genetically altered Pseudomonas syringae bacteria to the
soil surrounding crop plant roots will have many beneficial effects, such as the prevention of frost
damage in certain crops.
(D) Genetic alteration of a number of harmful phytopathogens has been advocated by many
researchers who contend that these techniques will eventually replace such outdated methods as
crop rotation.
(E) Genetic alteration of bacteria has been successful in highly artificial laboratory conditions, but
opponents of such research have argued that these techniques are unlikely to produce organisms
that are able to survive in natural environments.

2. The author discusses naturally occurring Pseudomonas fluorescens bacteria in the


first paragraph primarily in order to do which one of the following?

(A) prove that increases in the level of such bacteria in the soil are the sole cause of soil
suppressivity
(B) explain why yields increased after wheat fields were sprayed with altered Pseudomonas
fluorescens bacteria
(C) detail the chemical processes that such bacteria use to suppress organisms parasitic to crop
plants, such as wheat, sugar beets, and potatoes
(D) provide background information to support the argument that research into the
agricultural use of genetically altered bacteria would be fruitful
(E) argue that crop rotation is unnecessary, since diseases brought on by phytopathogens diminish
in severity and eventually disappear on their own

3. It can be inferred from the author’s discussion of Pseudomonas fluorescens bacteria


that which one of the following would be true of crops impervious to parasitical
organisms?

(A) Pseudomonas fluorescens bacteria would be absent from the soil surrounding their roots.
(B) They would crowd out and eventually exclude other crop plants if their growth were not carefully
regulated.
(C) Their yield would not be likely to be improved by adding Pseudomonas fluorescens
bacteria to the soil.
(D) They would mature more quickly than crop plants that were susceptible to parasitical organisms.
(E) Levels of phytopathogenic bacteria in the soil surrounding their roots would be higher compared
with other crop plants.
4. It can be inferred from the passage that crop rotation can increase yields in part
because

(A) moving crop plants around makes them hardier and more resistant to disease
(B) the number of Pseudomonas fluorescens bacteria in the soil usually increases when crops are
rotated
(C) the roots of many crop plants produce compounds that are antagonistic to phytopathogens
harmful to other crop plants
(D) the presence of phytopathogenic bacteria is responsible for the majority of plant diseases
(E) phytopathogens typically attack some plant species but find other species to be
unsuitable hosts

5. According to the passage, proponents of the use of genetically altered bacteria in


agriculture argue that which one of the following is true of the altered bacteria used in
the frost-damage experiments?

(A) The altered bacteria had a genetic constitution differing from that of the normal
strain only in that the altered variety had one less gene.
(B) Although the altered bacteria competed effectively with the nonaltered strain in the laboratory,
they were not as viable in natural environments.
(C) The altered bacteria were much safer and more effective than the naturally occurring
Pseudomonas fluorescens bacteria used in earlier experiments.
(D) The altered bacteria were antagonistic to several types of naturally occurring phytopathogens in
the soil surrounding the roots of frost-damaged crops.
(E) The altered bacteria were released into the environment in numbers sufficient to guarantee the
validity of experimental results.

6. Which one of the following, if true, would most seriously weaken the proponents’
argument regarding the safety of using altered Pseudomonas syringae bacteria to
control frost damage?

(A) Pseudomonas syringae bacteria are primitive and have a simple genetic constitution.
(B) The altered bacteria are derived from a strain that is parasitic to plants and can cause damage
to crops.
(C) Current genetic-engineering techniques permit the large-scale commercial production of such
bacteria.
(D) Often genes whose presence is responsible for one harmful characteristic must be
present in order to prevent other harmful characteristics.
(E) The frost-damage experiments with Pseudomonas syringae bacteria indicate that the altered
variety would only replace the normal strain if released in sufficient numbers.

Cần điền vocabularies


Apr 4 – 1

Gray marketing, the selling of trademarked products through channels of distribution not authorized
by the trademark holder, can involve distribution of goods either within a market region or across
market boundaries. Gray marketing within a market region (“channel flow diversion”) occurs when
manufacturer-authorized distributors sell trademarked goods to unauthorized distributors who then
sell the goods to consumers within the same region. For example, quantity discounts from
manufacturers may motivate authorized dealers to enter the gray market because they can
purchase larger quantities of a product than they themselves intend to stock if they can sell the
extra units through gray marketing channels.

When gray marketing occurs across market boundaries, it is typically in an international setting and
may be called “parallel importing.” Manufacturers often produce and sell products in more than one
country and establish a network of authorized dealers in each country. Parallel importing occurs
when trademarked goods intended for one country are diverted from proper channels (channel flow
diversion) and then exported to unauthorized distributors in another country.

Trademark owners justifiably argue against gray marketing practices since such practices clearly
jeopardize the goodwill established by trademark owners: consumers who purchase trademarked
goods in the gray market do not get the same “extended product,” which typically includes pre- and
postsale service. Equally important, authorized distributors may cease to promote the product if it
becomes available for much lower prices through unauthorized channels.

Current debate over regulation of gray marketing focuses on three disparate theories in trademark
law that have been variously and confusingly applied to parallel importation cases: universality,
exhaustion, and territoriality. The theory of universality holds that a trademark is only an indication
of the source or origin of the product. This theory does not recognize the goodwill functions of a
trademark. When the courts apply this theory, gray marketing practices are allowed to continue
because the origin of the product remains the same regardless of the specific route of the product
through the channel of distribution. The exhaustion theory holds that a trademark owner
relinquishes all rights once a product has been sold. When this theory is applied, gray marketing
practices are allowed to continue because the trademark owners’ rights cease as soon as their
products are sold to a distributor. The theory of territoriality holds that a trademark is effective in
the country in which it is registered. Under the theory of territoriality, trademark owners can stop
gray marketing practices in the registering countries on products bearing their trademarks. Since
only the territoriality theory affords trademark owners any real legal protection against gray
marketing practices, I believe it is inevitable as well as desirable that it will come to be consistently
applied in gray marketing cases.

1. Which one of the following best expresses the main point of the passage?

(A) Gray marketing is unfair to trademark owners and should be legally controlled.
(B) Gray marketing is practiced in many different forms and places, and legislators should recognize
the futility of trying to regulate it.
(C) The mechanisms used to control gray marketing across markets are different from those most
effective in controlling gray marketing within markets.
(D) The three trademark law theories that have been applied in gray marketing cases lead to
different case outcomes.
(E) Current theories used to interpret trademark laws have resulted in increased gray marketing
activity.

2. The function of the passage as a whole is to

(A) criticize the motives and methods of those who practice gray marketing
(B) evaluate the effects of both channel flow diversion and parallel importation
(C) discuss the methods that have been used to regulate gray marketing and evaluate such
methods’ degrees of success
(D) describe a controversial marketing practice and evaluate several legal views
regarding it
(E) discuss situations in which certain marketing practices are common and analyze the economic
factors responsible for their development

3. Which one of the following does the author offer as an argument against gray
marketing?

(A) Manufacturers find it difficult to monitor the effectiveness of promotional efforts made on behalf
of products that are gray marketed.
(B) Gray marketing can discourage product promotion by authorized distributors.
(C) Gray marketing forces manufacturers to accept the low profit margins that result from quantity
discounting.
(D) Gray marketing discourages competition among unauthorized dealers.
(E) Quality standards in the manufacture of products likely to be gray marketed may decline.

Equally important, authorized distributors may cease to promote the product if it becomes
available for much lower prices through unauthorized channels.

4. The information in the passage suggests that proponents of the theory of territoriality
would probably differ from proponents of the theory of exhaustion on which one of the
following issues?

(A) the right of trademark owners to enforce, in countries in which the trademarks are
registered, distribution agreements intended to restrict distribution to authorized
channels
(B) the right of trademark owners to sell trademarked goods only to those distributors who agree to
abide by distribution agreements
(C) the legality of channel flow diversion that occurs in a country other than the one in which a
trademark is registered
(D) the significance consumers attach to a trademark
(E) the usefulness of trademarks as marketing tools

5. The author discusses the impact of gray marketing on goodwill in order to

(A) fault trademark owners for their unwillingness to offer a solution to a major consumer complaint
against gray marketing
(B) indicate a way in which manufacturers sustain damage against which they ought to
be protected
(C) highlight one way in which gray marketing across markets is more problematic than gray
marketing within a market
(D) demonstrate that gray marketing does not always benefit the interests of unauthorized
distributors
(E) argue that consumers are unwilling to accept a reduction in price in exchange for elimination of
service

6. The author’s attitude toward the possibility that the courts will come to exercise
consistent control over gray marketing practices can best be characterized as one of

(A) resigned tolerance


(B) utter dismay
(C) reasoned optimism
(D) unbridled fervor
(E) cynical indifference

7. It can be inferred from the passage that some channel flow diversion might be
eliminated if

(A) profit margins on authorized distribution of goods were less than those on goods marketed
through parallel importing
(B) manufacturers relieved authorized channels of all responsibility for product promotion
(C) manufacturers charged all authorized distributors the same unit price for products
regardless of quantity purchased
(D) the postsale service policies of authorized channels were controlled by manufacturers
(E) manufacturers refused to provide the “extended product” to consumers who purchase goods in
the gray market

For example, quantity discounts from manufacturers may motivate authorized dealers to enter the
gray market because they can purchase larger quantities of a product than they themselves intend
to stock if they can sell the extra units through gray marketing channels.
Apr 4 – 2

Any study of autobiographical narratives that appeared under the ostensible authorship of African
American writers between 1760 and 1865 inevitably raises concerns about authenticity and
interpretation. Should an autobiography whose written composition was literally out of the hands of
its narrator be considered as the literary equivalent of those autobiographies that were authored
independently by their subjects?

In many cases, the so-called edited narrative of an ex-slave ought to be treated as a ghostwritten
account insofar as literary analysis is concerned, especially when it was composed by its editor from
“a statement of facts” provided by an African American subject. Blassingame has taken pains to
show that the editors of several of the more famous antebellum slave narratives were “noted for
their integrity” and thus were unlikely to distort the facts given them by slave narrators. From a
literary standpoint, however, it is not the moral integrity of these editors that is at issue but the
linguistic, structural, and tonal integrity of the narratives they produced. Even if an editor faithfully
reproduced the facts of a narrator’s life, it was still the editor who decided what to make of these
facts, how they should be emphasized, in what order they ought to be presented, and what
was extraneous or germane. Readers of African American autobiography then and now have too
readily accepted the presumption of these eighteenth- and nineteenth-century editors that
experiential facts recounted orally could be recorded and sorted by an amanuensis-editor, taken out
of their original contexts, and then published with editorial prefaces, footnotes, and appended
commentary, all without compromising the validity of the narrative as a product of an African
American consciousness.

Transcribed narratives in which an editor explicitly delimits his or her role undoubtedly may be
regarded as more authentic and reflective of the narrator’s thought in action than those edited
works that flesh out a statement of facts in ways unaccounted for. Still, it would be naive to accord
dictated oral narratives the same status as autobiographies composed and written by the subjects of
the stories themselves. This point is illustrated by an analysis of Works Progress Administration
interviews with ex-slaves in the 1930s that suggests that narrators often told interviewers what they
seemed to want to hear. If it seemed impolitic for former slaves to tell all they knew and thought
about the past to interviewers in the 1930s, the same could be said of escaped slaves on the run in
the antebellum era. Dictated narratives, therefore, are literary texts whose authenticity is difficult to
determine. Analysts should reserve close analytic readings for independently authored texts.
Discussion of collaborative texts should take into account the conditions that governed their
production.
1. Which one of the following best summarizes the main point of the passage?

(A) The personal integrity of an autobiography’s editor has little relevance to its value as a literary
work.
(B) Autobiographies dictated to editors are less valuable as literature than are autobiographies
authored by their subjects.
(C) The facts that are recorded in an autobiography are less important than the personal
impressions of its author.
(D) The circumstances under which an autobiography was written should affect the way
it is interpreted as literature.
(E) The autobiographies of African Americans written between 1760 and 1865 deserve more careful
study than they have so far received.

2. The information in the passage suggests that the role of the “editor” (Highlighted) is
most like that of

(A) an artist who wishes to invent a unique method of conveying the emotional impact of a scene in
a painting
(B) a worker who must interpret the instructions of an employer
(C) a critic who must provide evidence to support opinions about a play being reviewed
(D) an architect who must make the best use of a natural setting in designing a public building
(E) a historian who must decide how to direct the reenactment of a historical event

3. Which one of the following best describes the author’s opinion about applying literary
analysis to edited autobiographies?

(A) The author is adamantly opposed to the application of literary analysis to edited
autobiographies.
(B) The author is skeptical of the value of close analytical reading in the case of edited
autobiographies.
(C) The author believes that literary analysis of the prefaces, footnotes, and commentaries that
accompany edited autobiographies would be more useful than an analysis of the text of the
autobiographies.
(D) The author believes that an exclusively literary analysis of edited autobiographies is more
valuable than a reading that emphasizes their historical import.
(E) The author believes that the literary analysis of edited autobiographies would enhance their
linguistic, structural, and tonal integrity.

4. The passage supports which one of the following statements about the readers of
autobiographies of African Americans that were published between 1760 and 1865?

(A) They were more concerned with the personal details in the autobiographies than with their
historical significance.
(B) They were unable to distinguish between ghostwritten and edited autobiographies.
(C) They were less naive about the facts of slave life than are readers today.
(D) They presumed that the editing of the autobiographies did not affect their
authenticity.
(E) They had little interest in the moral integrity of the editors of the autobiographies.
5. Which one of the following words, as it is used in the passage, best serves to
underscore the author’s concerns about the authenticity of the autobiographies
discussed?

(A) “ostensible” (line 2)


(B) “integrity” (line 14)
(C) “extraneous” (line 21)
(D) “delimits” (line 30)
(E) “impolitic” (line 39)

6. According to the passage, close analytic reading of an autobiography is appropriate


only when the

(A) autobiography has been dictated to an experienced amanuensis-editor


(B) autobiography attempts to reflect the narrator’s thought in action
(C) autobiography was authored independently by its subject
(D) moral integrity of the autobiography’s editor is well established
(E) editor of the autobiography collaborated closely with its subject in its editing

7. It can be inferred that the discussion in the passage of Blassingame’s work primarily
serves which one of the following purposes?

(A) It adds an authority’s endorsement to the author’s view that edited narratives ought to be
treated as ghostwritten accounts.
(B) It provides an example of a mistaken emphasis in the study of autobiography.
(C) It presents an account of a new method of literary analysis to be applied to autobiography.
(D) It illustrates the inadequacy of traditional approaches to the analysis of autobiography.
(E) It emphasizes the importance of the relationship between editor and narrator.
Apr 5 – 1

Direct observation of contemporary societies at the threshold of widespread literacy has not assisted
our understanding of how such literacy altered ancient Greek society, in particular its political
culture. The discovery of what Goody has called the “enabling effects” of literacy in contemporary
societies tends to seduce the observer into confusing often rudimentary knowledge of how to read
with popular access to important books and documents; this confusion is then projected onto
ancient societies. “In ancient Greece,” Goody writes, “alphabetic reading and writing was important
for the development of political democracy.”

An examination of the ancient Greek city Athens exemplifies how this sort of confusion is
detrimental to understanding ancient politics. In Athens, the early development of a written law
code was retrospectively mythologized as the critical factor in breaking the power monopoly of the
old aristocracy: hence the Greek tradition of the “law-giver,” which has captured the imaginations of
scholars like Goody. But the application and efficacy of all law codes depend on their interpretation
by magistrates and courts, and unless the right of interpretation is “democratized,” the mere
existence of written laws changes little.

In fact, never in antiquity did any but the elite consult documents and books. Even in Greek courts
the juries heard only the relevant statutes read out during the proceedings, as they heard verbal
testimony, and they then rendered their verdict on the spot, without the benefit of any discussion
among themselves. True, in Athens the juries were representative of a broad spectrum of the
population, and these juries, drawn from diverse social classes, both interpreted what they had
heard and determined matters of fact. However, they were guided solely by the speeches prepared
for the parties by professional pleaders and by the quotations of laws or decrees within the
speeches, rather than by their own access to any kind of document or book.

Granted, people today also rely heavily on a truly knowledgeable minority for information and its
interpretation, often transmitted orally. Yet this is still fundamentally different from an ancient
society in which there was no “popular literature,” i.e., no newspapers, magazines, or other media
that dealt with sociopolitical issues. An ancient law code would have been analogous to the Latin
Bible, a venerated document but a closed book. The resistance of the medieval Church to vernacular
translations of the Bible, in the West at least, is therefore a pointer to the realities of ancient
literacy. When fundamental documents are accessible for study only to an elite, the rest of the
society is subject to the elite’s interpretation of the rules of behavior, including right political
behavior. Athens, insofar as it functioned as a democracy, did so not because of widespread literacy,
but because the elite had chosen to accept democratic institutions.
1. Which one of the following statements best expresses the main idea of the passage?

(A) Democratic political institutions grow organically from the traditions and conventions of a
society.
(B) Democratic political institutions are not necessarily the outcome of literacy in a
society.
(C) Religious authority, like political authority, can determine who in a given society will have access
to important books and documents.
(D) Those who are best educated are most often those who control the institutions of authority in a
society.
(E) Those in authority have a vested interest in ensuring that those under their control remain
illiterate.

2. It can be inferred from the passage that the author assumes which one of the
following about societies in which the people possess a rudimentary reading ability?

(A) They are more politically advanced than societies without rudimentary reading ability.
(B) They are unlikely to exhibit the positive effects of literacy.
(C) They are rapidly evolving toward widespread literacy.
(D) Many of their people might not have access to important documents and books.
(E) Most of their people would not participate in political decision-making.

3. The author refers to the truly knowledgeable minority in contemporary societies in


the context of the fourth paragraph in order to imply which one of the following?

(A) Because they have a popular literature that closes the gap between the elite and the majority,
contemporary societies rely far less on the knowledge of experts than did ancient societies.
(B) Contemporary societies rely on the knowledge of experts, as did ancient societies, because
contemporary popular literature so frequently conveys specious information.
(C) Although contemporary societies rely heavily on the knowledge of experts, access to
popular literature makes contemporary societies less dependent on experts for
information about rules of behavior than were ancient societies.
(D) While only some members of the elite can become experts, popular literature gives the majority
in contemporary society an opportunity to become members of such an elite.
(E) Access to popular literature distinguishes ancient from contemporary societies because it relies
on a level of educational achievement attainable only by a contemporary elite.

4. According to the passage, each of the following statements concerning ancient Greek
juries is true EXCEPT:

(A) They were somewhat democratic insofar as they were composed largely of people
from the lowest social classes.
(B) They were exposed to the law only insofar as they heard relevant statutes read out during legal
proceedings.
(C) They ascertained the facts of a case and interpreted the laws.
(D) They did not have direct access to important books and documents that were available to the
elite.
(E) They rendered verdicts without benefit of private discussion among themselves.
5. The author characterizes the Greek tradition of the “law-giver” (Highlighted) as an
effect of mythologizing most probably in order to

(A) illustrate the ancient Greek tendency to memorialize historical events by transforming them into
myths
(B) convey the historical importance of the development of the early Athenian written law code
(C) convey the high regard in which the Athenians held their legal tradition
(D) suggest that the development of a written law code was not primarily responsible
for diminishing the power of the Athenian aristocracy
(E) suggest that the Greek tradition of the “law-giver” should be understood in the larger context of
Greek mythology

6. The author draws an analogy between the Latin Bible and an early law code
(Highlighted) in order to make which one of the following points?

(A) Documents were considered authoritative in premodern society in proportion to their


inaccessibility to the majority.
(B) Documents that were perceived as highly influential in premodern societies were
not necessarily accessible to the society’s majority.
(C) What is most revered in a nondemocratic society is what is most frequently misunderstood.
(D) Political documents in premodern societies exerted a social influence similar to that exerted by
religious documents.
(E) Political documents in premodern societies were inaccessible to the majority of the population
because of the language in which they were written.

7. The primary purpose of the passage is to

(A) argue that a particular method of observing contemporary societies is inconsistent


(B) point out the weaknesses in a particular approach to understanding ancient
societies
(C) present the disadvantages of a particular approach to understanding the relationship between
ancient and contemporary societies
(D) examine the importance of developing an appropriate method for understanding ancient
societies
(E) convey the difficulty of accurately understanding attitudes in ancient societies

Contemporary society (n) : xã hội đương đại

Threhold (n) : ngưỡng

Seduce (v) : quyến rũ

Rudimentary (adj) : thô sơ

Confusion (n) : gây nhầm lẫn

Project onto (v) : chiếu lên

Detrimental (adj) : bất lợi, có hại

Retrospectively (adv) : hồi tưởng


Mythologize (v) : thần thoại hóa

Aristocracy (n) : quý tộc

Capture (v) : chiếm được, quyến rũ người nào

Interpretation (n) : diễn dịch, giải thích

Magistrate (n) : quan tòa

Antiquity (n) : đời xưa, cổ xưa

Statute (n) : đạo luật

Render (v) : báo cáo, kết xuất

Verdict (n) : bản án, phán quyết

On spot (n) : tại chỗ

Juries (n) : bồi thẩm đoàn

Pleader (n) : người biện hộ, luật sư

Decree (n) : nghị định

Granted (adv) : thừa nhận rằng

Bible (n) : kinh thánh

Venerate (v) : tôn kính, sùng bái

Close book (n) : sách kín

Vernacular (adj/n) : bản địa,thuộc về bản xứ/thổ ngữ, tiếng bình dân

Subject (v) : bắt buộc

Insofar (adv) : trong chừng mực

Institution (n) : thể chế, sự bắt đầu


Apr 5 – 2

The English who in the seventeenth and eighteenth centuries inhabited those colonies that would
later become the United States shared a common political vocabulary with the English in England.
Steeped as they were in the English political language, these colonials failed to observe that their
experience in America had given the words a significance quite different from that accepted by the
English with whom they debated; in fact, they claimed that they were more loyal to the English
political tradition than were the English in England.

In many respects the political institutions of England were reproduced in these American colonies.
By the middle of the eighteenth century, all of these colonies except four were headed by Royal
Governors appointed by the King and perceived as bearing a relation to the people of the colony
similar to that of the King to the English people. Moreover, each of these colonies enjoyed a
representative assembly, which was consciously modeled, in powers and practices, after the English
Parliament. In both England and these colonies, only property holders could vote.

Nevertheless, though English and colonial institutions were structurally similar, attitudes toward
those institutions differed. For example, English legal development from the early seventeenth
century had been moving steadily toward the absolute power of Parliament. The most unmistakable
sign of this tendency was the legal assertion that the King was subject to the law. Together with this
resolute denial of the absolute right of kings went the assertion that Parliament was unlimited in its
power: it could change even the Constitution by its ordinary acts of legislation. By the eighteenth
century the English had accepted the idea that the parliamentary representatives of the people were
omnipotent.

The citizens of these colonies did not look upon the English Parliament with such fond eyes, nor did
they concede that their own assemblies possessed such wide powers. There were good historical
reasons for this. To the English the word “constitution” meant the whole body of law and legal
custom formulated since the beginning of the kingdom, whereas to these colonials a constitution
was a specific written document, enumerating specific powers. This distinction in meaning can be
traced to the fact that the foundations of government in the various colonies were written charters
granted by the Crown. These express authorizations to govern were tangible, definite things. Over
the years these colonials had often repaired to the charters to justify themselves in the struggle
against tyrannical governors or officials of the Crown. More than a century of government under
written constitutions convinced these colonists of the necessity for and efficacy of protecting their
liberties against governmental encroachment by explicitly defining all governmental powers in a
document.

1. Which one of the following best expresses the main idea of the passage?

(A) The colonials and the English mistakenly thought that they shared a common political
vocabulary.
(B) The colonials and the English shared a variety of institutions.
(C) The colonials and the English had conflicting interpretations of the language and
institutional structures that they shared.
(D) Colonial attitudes toward English institutions grew increasingly hostile in the eighteenth century.
(E) Seventeenth-century English legal development accounted for colonial attitudes toward
constitutions.
2. The passage supports all of the following statements about the political conditions
present by the middle of the eighteenth century in the American colonies discussed in
the passage EXCEPT:

(A) Colonials who did not own property could not vote.
(B) All of these colonies had representative assemblies modeled after the British Parliament.
(C) Some of these colonies had Royal Governors.
(D) Royal Governors could be removed from office by colonial assemblies.
(E) In these colonies, Royal Governors were regarded as serving a function like that of a king.

3. The passage implies which one of the following about English kings prior to the early
seventeenth century?

(A) They were the source of all law.


(B) They frequently flouted laws made by Parliament.
(C) Their power relative to that of Parliament was considerably greater than it was in
the eighteenth century.
(D) They were more often the sources of legal reform than they were in the eighteenth century.
(E) They had to combat those who believed that the power of Parliament was absolute.

4. The author mentions which one of the following as evidence for the eighteenth-
century English attitude toward Parliament?

(A) The English had become uncomfortable with institutions that could claim absolute authority.
(B) The English realized that their interests were better guarded by Parliament than by the King.
(C) The English allowed Parliament to make constitutional changes by legislative
enactment.
(D) The English felt that the King did not possess the knowledge that would enable him to rule
responsibly.
(E) The English had decided that it was time to reform their representative government.

5. The passage implies that the colonials discussed in the passage would have
considered which one of the following to be a source of their debates with England?

(A) their changed use of the English political vocabulary


(B) English commitment to parliamentary representation
(C) their uniquely English experience
(D) their refusal to adopt any English political institutions
(E) their greater loyalty to the English political traditions

6. According to the passage, the English attitude toward the English Constitution
differed from the colonial attitude toward constitutions in that the English regarded
their Constitution as

(A) the legal foundation of the kingdom


(B) a document containing a collection of customs
(C) a cumulative corpus of legislation and legal traditions
(D) a record alterable by royal authority
(E) an unchangeable body of governmental powers
7. The primary purpose of the passage is to

(A) expose the misunderstanding that has characterized descriptions of the relationship between
seventeenth- and eighteenth-century England and certain of its American colonies
(B) suggest a reason for England’s treatment of certain of its American colonies in the seventeenth
and eighteenth centuries
(C) settle an ongoing debate about the relationship between England and certain of its American
colonies in the seventeenth and eighteenth centuries
(D) interpret the events leading up to the independence of certain of England’s American colonies in
the eighteenth century
(E) explain an aspect of the relationship between England and certain of its American
colonies in the seventeenth and eighteenth centuries

Inhabit (v) : trú ngụ

Steep (v) : đắm chìm

Political institution (n) : thể chế chính trị

Governor (n) : thống đốc

Bear (v) : chịu trách nhiệm

Assembly (n) : hội đồng quốc gia

Consciously (adv) : có ý thức

Parliament (n) : nghị viện, quốc hội

Unmistakable (adj) : không thể nhầm lẫn

Legal assertion (n) : khẳng định pháp lý

Constitution (n) : hiến pháp

Legislation (n) : pháp luật

Ordinary (adj) : thông thường

Omnipotent (adj/n) : toàn năng / thượng đế

Concede (v) : công nhận

Legal custom (n) : phong tục pháp lý

Enumerate (v) : liệt kê

Charter (n) : điều lệ, hiến pháp, hiến chương

Authorization (n) : ủy quyền

Justify (v) : biện minh, bào chữa

Tyrannical (adj) : chuyên chế, tàn bạo

Encroachment (n) : lấn chiếm, lấn quyền


Explicity (adv) : rõ ràng

Corpus (n) : văn thể, bộ các văn bằng,…

Cumulative (adj) : tích lũy

Enactment (n) : ban hành, điều luật, sắc lệnh, ban bố

Flout (v) : bỏ qua, coi thường


Apr 6 – 1

Common sense suggests that we know our own thoughts directly, but that we infer the thoughts of
other people. The former process is noninferential and infallible, while the latter is based on others'
behavior and can always be wrong. But this assumption is challenged by experiments in psychology
demonstrating that in certain circumstances young children tend to misdescribe their own thoughts
regarding simple phenomena while nonetheless correctly describing those phenomena. It seems
that these children have the same thoughts that adults have regarding the phenomena but are
much less capable of identifying these thoughts. Some psychologists argue that this indicates that
one's awareness of one's own thoughts is every bit as inferential as one's awareness of another
person's thoughts. According to their interpretation of the experiments, thoughts are unobservable
entities that, among other things, help to explain why we act as we do. It follows from this that we
are wrong to think of ourselves as having noninferential and infallible access to our own thoughts.

Recognizing an obligation to explain why we cling so tenaciously to an illusory belief in


noninferential and infallible knowledge of our own thoughts, these psychologists suggest that this
illusion is analogous to what happens to us when we become experts in a particular area. Greater
expertise appears to change not only our knowledge of the area as a whole, but our very perception
of entities in that area. It appears to us that we become able to see and to grasp these entities and
their relations directly, whereas before we could only make inferences about them. For instance,
chess experts claim the ability to see without calculation whether a position is weak or strong. From
a psychological perspective, we become so expert in making incredibly fast introspective inferences
about our thinking that we fail to notice that we are making them. This failure leads naturally to the
supposition that there is no way for us to be wrong in our identification of what we ourselves think
because we believe we are perceiving it directly.

In claiming that we have only inferential access to our thoughts, the psychologists come perilously
close to claiming that we base our inferences about what we ourselves are thinking solely on
observations of our own external behavior. But, in fact, their arguments do not commit them to this
claim; the psychologists suggest that we are somehow able to base our inferences about what we
are thinking on internal cognitive activity that is not itself thought—e.g., fleeting and instantaneous
sensations and emotions. The frequent occurrence of such internal activities explains why we
develop the capacity to make quick and reliable inferences. Their internality makes it impossible for
anyone else to make an inference based on them that contradicts our own. Thus, they are crucial in
creating the illusion of noninferentiality and infallibility.

1. Which one of the following most accurately expresses the main point of the passage?

(A) Only experts within a given domain have noninferential and infallible access to their own
thoughts; other people must infer their own thoughts as they do others' thoughts.
(B) In opposition to the common belief that thoughts are directly perceived, some
psychologists argue that people infer what their own thoughts are.
(C) In response to the common belief that thoughts are directly perceived, some psychologists claim
that this belief is an illusion resulting from our inability to make quick and reliable inferences.
(D) Some psychologists have recently attributed children's failure to give an accurate description of
their own thoughts to their lack of expertise.
(E) Some psychologists hold that people are able to make inferences about what they are thinking
that are based solely on observing their own external behavior.
2. Which one of the following, if true, would most call into question the psychologists'
interpretation of the experiments with children (fourth and fifth sentences of the
passage)?

(A) Some children who took part in the experiments were no less capable than some adults at
identifying their own thoughts.
(B) Experiments with older children found that they were as accurate as adults in identifying their
thoughts.
(C) The limited language skills possessed by young children make it difficult for them to
accurately communicate their thoughts.
(D) Most young children cannot be expected to know the difference between direct and indirect
access to one's thoughts.
(E) The psychologists who conducted the experiments with children were concerned with
psychological issues other than the nature of people's access to their own thoughts.

3. Based on the passage, the author is most likely to believe which one of the following
about the view that "we base our inferences about what we ourselves are thinking
solely on observations of our own external behavior" (first sentence of the last
paragraph)?

(A) It constitutes a denial of the possibility of scientifically studying thinking processes.


(B) It has often been misunderstood by psychologists.
(C) It was the prevailing view until undermined by recent psychology experiments.
(D) It seems to contradict common sense but is basically sound.
(E) It is not considered to be an intellectually defensible position.

4. Which one of the following is most closely analogous to the explanation in the
passage of how persons fail to notice that they are making inferences about their
thoughts?

(A) An anthropologist cannot describe his own culture accurately because he has
become too familiar with its workings and therefore takes them for granted.
(B) Science is limited with regard to studying the human mind because science necessarily depends
on human reasoning.
(C) As they develop, children become increasingly comfortable with formal abstraction and therefore
become vulnerable to failures to learn from concrete experiences.
(D) Judges are barred from trying cases involving their family members because of a potential
conflict of interest.
(E) A ship's commander must delegate certain duties and decisions to other officers on her ship
because she is too busy to attend to those duties and decisions.
5. According to the passage, one's gaining greater expertise in a field appears to result
in

(A) an altered way of expressing one's judgments about issues in that field
(B) a more detail-oriented approach to questions in that field
(C) an increased tendency to ignore one's own errors in judgment within that field
(D) a substantively different way of understanding relations within that field
(E) a reduced reliance on sensations and emotions when inferring one's thoughts regarding that
field

6. According to the psychologists cited in the passage, the illusion of direct knowledge
of our own thoughts arises from the fact that

(A) we ignore the feedback that we receive regarding the inaccuracy of the inferences we make
about our thought processes
(B) knowledge of our own thoughts is usually unmediated due to our expertise, and we simply
overlook instances where this is not the case
(C) we are unaware of the inferential processes that allow us to become aware of our
thoughts
(D) our inferences regarding our own thoughts are generally extremely accurate, as are our
perceptions of the world
(E) our inferences regarding our own thoughts are sometimes clouded and uncertain, as are our
perceptions of the world

7. It can most reasonably be inferred that the choice of children as the subjects of the
psychology experiments discussed in the passage was advantageous to the
experimenters for which one of the following reasons?

(A) Experiments involving children are more likely to give interesting results because children are
more creative than adults.
(B) Adults are more likely than children to give inaccurate reports of their thought processes.
(C) Since adults are infallible in their access to their own thoughts, only the thought processes of
children shed light on the nature of inference.
(D) Mental processes are sometimes easier to study in children because children are
more likely than adults to make certain cognitive errors.
(E) Children are less experienced than adults in inferring the thoughts of others from observations of
their behavior.

You might also like